You are on page 1of 283

Notes for edX/MITx 8.

02x Electricity and Magnetism,


Spring 2013
Thomas Backman, exscape@gmail.com
June 18, 2013

Contents
0 Preface/disclaimer
1 Week 1
1.1 Coulombs law . . . . . . . .
1.2 Electric fields . . . . . . . .
1.2.1 Multiple charges . .
1.2.2 Electric field lines . .
1.2.3 Dipoles . . . . . . . .
1.3 Electric flux and Gausss law
1.3.1 Quick facts . . . . .
1.3.2 Slower this time... . .
1.3.3 Gausss law . . . . .

.
.
.
.
.
.
.
.
.

.
.
.
.
.
.
.
.
.

.
.
.
.
.
.
.
.
.

.
.
.
.
.
.
.
.
.

.
.
.
.
.
.
.
.
.

.
.
.
.
.
.
.
.
.

.
.
.
.
.
.
.
.
.

.
.
.
.
.
.
.
.
.

2 Week 2
2.1 Electrostatic Potential Energy and Electric
2.1.1 Electrostatic Potential Energy . . .
2.1.2 Electric potential . . . . . . . . . .
2.1.3 More on electric potential . . . . .
2.2 Electric fields inside hollow conductors . .
2.3 High-voltage breakdown and lightning . .
2.3.1 Electric breakdown . . . . . . . . .
3 Week 3
3.1 Capacitance and Field Energy . . . . . .
3.1.1 Field Energy . . . . . . . . . . .
3.1.2 Capacitance . . . . . . . . . . . .
3.2 Dielectrics . . . . . . . . . . . . . . . . .
3.2.1 More on capacitors . . . . . . . .
3.2.2 A bit on van de Graaff generators
3.3 Current, resistivity and Ohms law . . .
4 Week 4
4.1 Batteries and EMF . . . . .
4.1.1 Kirchhoffs rules . . .
4.1.2 Basic circuit analysis
4.2 Magnetic Field and Torques
4.2.1 The B Field . . . . .
4.3 Review for Exam 1 . . . . .
4.3.1 Thats it . . . . . . .

.
.
.
.
.
.
.

.
.
.
.
.
.
.

.
.
.
.
.
.
.

.
.
.
.
.
.
.

.
.
.
.
.
.
.

.
.
.
.
.
.
.

.
.
.
.
.
.
.

.
.
.
.
.
.
.

.
.
.
.
.
.
.

.
.
.
.
.
.
.
.
.

.
.
.
.
.
.
.
.
.

.
.
.
.
.
.
.
.
.

.
.
.
.
.
.
.
.
.

.
.
.
.
.
.
.
.
.

.
.
.
.
.
.
.
.
.

.
.
.
.
.
.
.
.
.

.
.
.
.
.
.
.
.
.

.
.
.
.
.
.
.
.
.

.
.
.
.
.
.
.
.
.

.
.
.
.
.
.
.
.
.

.
.
.
.
.
.
.
.
.

.
.
.
.
.
.
.
.
.

.
.
.
.
.
.
.
.
.

.
.
.
.
.
.
.
.
.

.
.
.
.
.
.
.
.
.

.
.
.
.
.
.
.
.
.

.
.
.
.
.
.
.
.
.

.
.
.
.
.
.
.
.
.

.
.
.
.
.
.
.
.
.

.
.
.
.
.
.
.
.
.

.
.
.
.
.
.
.
.
.

7
7
7
8
9
9
9
9
10
11

Potential
. . . . . .
. . . . . .
. . . . . .
. . . . . .
. . . . . .
. . . . . .

.
.
.
.
.
.
.

.
.
.
.
.
.
.

.
.
.
.
.
.
.

.
.
.
.
.
.
.

.
.
.
.
.
.
.

.
.
.
.
.
.
.

.
.
.
.
.
.
.

.
.
.
.
.
.
.

.
.
.
.
.
.
.

.
.
.
.
.
.
.

.
.
.
.
.
.
.

.
.
.
.
.
.
.

.
.
.
.
.
.
.

.
.
.
.
.
.
.

.
.
.
.
.
.
.

.
.
.
.
.
.
.

.
.
.
.
.
.
.

.
.
.
.
.
.
.

.
.
.
.
.
.
.

.
.
.
.
.
.
.

.
.
.
.
.
.
.

14
14
14
15
18
19
20
21

.
.
.
.
.
.
.

23
23
23
24
26
30
30
31

.
.
.
.
.
.
.

34
34
35
36
36
37
38
39

.
.
.
.
.
.
.
.
.

.
.
.
.
.
.
.

.
.
.
.
.
.
.

.
.
.
.
.
.
.
.
.

.
.
.
.
.
.
.

.
.
.
.
.
.
.

.
.
.
.
.
.
.
.
.

.
.
.
.
.
.
.

.
.
.
.
.
.
.

.
.
.
.
.
.
.
.
.

.
.
.
.
.
.
.

.
.
.
.
.
.
.

.
.
.
.
.
.
.
.
.

.
.
.
.
.
.
.

.
.
.
.
.
.
.

.
.
.
.
.
.
.

.
.
.
.
.
.
.

.
.
.
.
.
.
.

.
.
.
.
.
.
.

.
.
.
.
.
.
.

.
.
.
.
.
.
.

.
.
.
.
.
.
.

.
.
.
.
.
.
.

.
.
.
.
.
.
.

.
.
.
.
.
.
.

.
.
.
.
.
.
.

.
.
.
.
.
.
.

.
.
.
.
.
.
.

.
.
.
.
.
.
.

.
.
.
.
.
.
.

.
.
.
.
.
.
.

.
.
.
.
.
.
.

.
.
.
.
.
.
.

.
.
.
.
.
.
.

.
.
.
.
.
.
.

.
.
.
.
.
.
.

.
.
.
.
.
.
.

.
.
.
.
.
.
.

.
.
.
.
.
.
.

.
.
.
.
.
.
.

.
.
.
.
.
.
.

.
.
.
.
.
.
.

.
.
.
.
.
.
.

.
.
.
.
.
.
.

.
.
.
.
.
.
.

.
.
.
.
.
.
.

.
.
.
.
.
.
.

.
.
.
.
.
.
.

.
.
.
.
.
.
.

.
.
.
.
.
.
.

.
.
.
.
.
.
.

.
.
.
.
.
.
.

.
.
.
.
.
.
.

.
.
.
.
.
.
.

.
.
.
.
.
.
.

.
.
.
.
.
.
.

.
.
.
.
.
.
.

5 Week 5
5.1 Moving Charges in Magnetic Fields . . . . . . . .
5.1.1 Moving charges, radii and special relativity
5.1.2 Isotope separation . . . . . . . . . . . . .
5.1.3 Particle accelerators/cyclotrons . . . . . .
5.1.4 Cloud chambers and bubble chambers . .
5.2 Biot-Savart Law . . . . . . . . . . . . . . . . . . .
5.3 Amperes law . . . . . . . . . . . . . . . . . . . .
5.3.1 The magnetic field of a solenoid . . . . . .
6 Week 6
6.1 Electromagnetic induction . . . . . . .
6.1.1 Lenzs law . . . . . . . . . . . .
6.1.2 Magnetic flux . . . . . . . . . .
6.1.3 Faradays law . . . . . . . . . .
6.1.4 The breakdown of intuition . .
6.2 Motional EMF and dynamos . . . . . .
6.2.1 Changing the area . . . . . . .
6.2.2 Eddy currents . . . . . . . . . .
6.3 Displacement currents and synchronous
6.3.1 The amended Amperes law . .
6.3.2 Displacement current . . . . . .
6.3.3 Synchronous motors . . . . . .

.
.
.
.
.
.
.
.

.
.
.
.
.
.
.
.

.
.
.
.
.
.
.
.

.
.
.
.
.
.
.
.

.
.
.
.
.
.
.
.

.
.
.
.
.
.
.
.

.
.
.
.
.
.
.
.

.
.
.
.
.
.
.
.

.
.
.
.
.
.
.
.

.
.
.
.
.
.
.
.

.
.
.
.
.
.
.
.

.
.
.
.
.
.
.
.

.
.
.
.
.
.
.
.

.
.
.
.
.
.
.
.

.
.
.
.
.
.
.
.

.
.
.
.
.
.
.
.

.
.
.
.
.
.
.
.

.
.
.
.
.
.
.
.

.
.
.
.
.
.
.
.

.
.
.
.
.
.
.
.

.
.
.
.
.
.
.
.

.
.
.
.
.
.
.
.

.
.
.
.
.
.
.
.

40
40
40
42
43
45
46
48
49

.
.
.
.
.
.
.
.
.
.
.
.

.
.
.
.
.
.
.
.
.
.
.
.

.
.
.
.
.
.
.
.
.
.
.
.

.
.
.
.
.
.
.
.
.
.
.
.

.
.
.
.
.
.
.
.
.
.
.
.

.
.
.
.
.
.
.
.
.
.
.
.

.
.
.
.
.
.
.
.
.
.
.
.

.
.
.
.
.
.
.
.
.
.
.
.

.
.
.
.
.
.
.
.
.
.
.
.

.
.
.
.
.
.
.
.
.
.
.
.

.
.
.
.
.
.
.
.
.
.
.
.

.
.
.
.
.
.
.
.
.
.
.
.

.
.
.
.
.
.
.
.
.
.
.
.

.
.
.
.
.
.
.
.
.
.
.
.

.
.
.
.
.
.
.
.
.
.
.
.

.
.
.
.
.
.
.
.
.
.
.
.

.
.
.
.
.
.
.
.
.
.
.
.

.
.
.
.
.
.
.
.
.
.
.
.

.
.
.
.
.
.
.
.
.
.
.
.

.
.
.
.
.
.
.
.
.
.
.
.

.
.
.
.
.
.
.
.
.
.
.
.

.
.
.
.
.
.
.
.
.
.
.
.

.
.
.
.
.
.
.
.
.
.
.
.

.
.
.
.
.
.
.
.
.
.
.
.

51
51
51
52
53
54
56
57
59
61
62
64
65

7 Week 7
7.1 How do magicians levitate women? . . . . . . . .
7.1.1 The human heart . . . . . . . . . . . . . .
7.1.2 Aurora borealis . . . . . . . . . . . . . . .
7.1.3 Superconductivity and magnetic levitation
7.2 Inductance and RL circuits . . . . . . . . . . . . .
7.2.1 Direct-current RL circuits . . . . . . . . .
7.2.2 Alternating-current RL circuits . . . . . .
7.2.3 More on magnetic levitation . . . . . . . .
7.3 Magnetic materials . . . . . . . . . . . . . . . . .
7.3.1 A short note on motors . . . . . . . . . . .
7.3.2 Magnetic dipole moment . . . . . . . . . .
7.3.3 The source of magnetism in matter . . . .
7.3.4 Magnetization . . . . . . . . . . . . . . . .
7.3.5 Paramagnetism . . . . . . . . . . . . . . .
7.3.6 Diamagnetism . . . . . . . . . . . . . . . .
7.3.7 Ferromagnetism . . . . . . . . . . . . . . .

.
.
.
.
.
.
.
.
.
.
.
.
.
.
.
.

.
.
.
.
.
.
.
.
.
.
.
.
.
.
.
.

.
.
.
.
.
.
.
.
.
.
.
.
.
.
.
.

.
.
.
.
.
.
.
.
.
.
.
.
.
.
.
.

.
.
.
.
.
.
.
.
.
.
.
.
.
.
.
.

.
.
.
.
.
.
.
.
.
.
.
.
.
.
.
.

.
.
.
.
.
.
.
.
.
.
.
.
.
.
.
.

.
.
.
.
.
.
.
.
.
.
.
.
.
.
.
.

.
.
.
.
.
.
.
.
.
.
.
.
.
.
.
.

.
.
.
.
.
.
.
.
.
.
.
.
.
.
.
.

.
.
.
.
.
.
.
.
.
.
.
.
.
.
.
.

.
.
.
.
.
.
.
.
.
.
.
.
.
.
.
.

.
.
.
.
.
.
.
.
.
.
.
.
.
.
.
.

.
.
.
.
.
.
.
.
.
.
.
.
.
.
.
.

.
.
.
.
.
.
.
.
.
.
.
.
.
.
.
.

.
.
.
.
.
.
.
.
.
.
.
.
.
.
.
.

.
.
.
.
.
.
.
.
.
.
.
.
.
.
.
.

.
.
.
.
.
.
.
.
.
.
.
.
.
.
.
.

.
.
.
.
.
.
.
.
.
.
.
.
.
.
.
.

.
.
.
.
.
.
.
.
.
.
.
.
.
.
.
.

.
.
.
.
.
.
.
.
.
.
.
.
.
.
.
.

.
.
.
.
.
.
.
.
.
.
.
.
.
.
.
.

.
.
.
.
.
.
.
.
.
.
.
.
.
.
.
.

68
68
68
70
71
73
75
80
82
83
83
83
84
85
86
87
88

.
.
.
.

90
90
92
95
96

.
.
.
.

97
97
97
100
101

8 Week 8
8.1 Hysteresis and electromagnets . . . . .
8.1.1 Ferromagnetism and hysteresis .
8.1.2 Maxwells equations . . . . . .
8.2 Review for Exam 2 . . . . . . . . . . .
9 Week 9
9.1 Transformers, Car Coils and RC
9.1.1 RC circuits . . . . . . .
9.1.2 Transformers . . . . . .
9.1.3 Spark plugs / car coils

. . . . .
. . . . .
. . . . .
. . . . .
. . . . .
. . . . .
. . . . .
. . . . .
motors
. . . . .
. . . . .
. . . . .

.
.
.
.

circuits
. . . . .
. . . . .
. . . . .

.
.
.
.

.
.
.
.

.
.
.
.

.
.
.
.
2

.
.
.
.

.
.
.
.

.
.
.
.

.
.
.
.

.
.
.
.

.
.
.
.

.
.
.
.

.
.
.
.

.
.
.
.

.
.
.
.

.
.
.
.

.
.
.
.

.
.
.
.

.
.
.
.

.
.
.
.

.
.
.
.

.
.
.
.

.
.
.
.

.
.
.
.

.
.
.
.

.
.
.
.

.
.
.
.

.
.
.
.

.
.
.
.

.
.
.
.

.
.
.
.

.
.
.
.

.
.
.
.

.
.
.
.

.
.
.
.

.
.
.
.

.
.
.
.

.
.
.
.

.
.
.
.

.
.
.
.

.
.
.
.

.
.
.
.

.
.
.
.

.
.
.
.

.
.
.
.

.
.
.
.

.
.
.
.

.
.
.
.

.
.
.
.

.
.
.
.

.
.
.
.

.
.
.
.

.
.
.
.

.
.
.
.

.
.
.
.

9.2
9.3

Driven RLC circuits and resonance


Traveling waves and standing waves
9.3.1 Traveling waves . . . . . . .
9.3.2 Standing waves . . . . . . .
9.3.3 Musical instruments . . . .

.
.
.
.
.

.
.
.
.
.

.
.
.
.
.

.
.
.
.
.

.
.
.
.
.

.
.
.
.
.

.
.
.
.
.

.
.
.
.
.

.
.
.
.
.

.
.
.
.
.

.
.
.
.
.

.
.
.
.
.

.
.
.
.
.

.
.
.
.
.

.
.
.
.
.

.
.
.
.
.

.
.
.
.
.

.
.
.
.
.

.
.
.
.
.

.
.
.
.
.

.
.
.
.
.

.
.
.
.
.

.
.
.
.
.

.
.
.
.
.

.
.
.
.
.

.
.
.
.
.

.
.
.
.
.

.
.
.
.
.

.
.
.
.
.

.
.
.
.
.

102
105
105
106
107

10 Week 10
10.1 Resonance, electromagnetic waves . . . . . . . .
10.1.1 Radar and measuring distance . . . . . .
10.1.2 Radio . . . . . . . . . . . . . . . . . . .
10.2 Index of refraction and Poynting vector . . . . .
10.2.1 Poynting vector . . . . . . . . . . . . . .
10.2.2 Waves due to accelerating charges . . . .
10.2.3 Spherical waves and the Poynting vector
10.2.4 Photons and radiation pressure . . . . .
10.2.5 Polarization . . . . . . . . . . . . . . . .
10.3 Snells law, refraction and total reflection . . . .
10.3.1 Total internal reflection . . . . . . . . . .
10.3.2 Frequency and wavelength in refraction .
10.3.3 Dispersion, prisms and white light . . . .
10.3.4 Primary colors . . . . . . . . . . . . . .

.
.
.
.
.
.
.
.
.
.
.
.
.
.

.
.
.
.
.
.
.
.
.
.
.
.
.
.

.
.
.
.
.
.
.
.
.
.
.
.
.
.

.
.
.
.
.
.
.
.
.
.
.
.
.
.

.
.
.
.
.
.
.
.
.
.
.
.
.
.

.
.
.
.
.
.
.
.
.
.
.
.
.
.

.
.
.
.
.
.
.
.
.
.
.
.
.
.

.
.
.
.
.
.
.
.
.
.
.
.
.
.

.
.
.
.
.
.
.
.
.
.
.
.
.
.

.
.
.
.
.
.
.
.
.
.
.
.
.
.

.
.
.
.
.
.
.
.
.
.
.
.
.
.

.
.
.
.
.
.
.
.
.
.
.
.
.
.

.
.
.
.
.
.
.
.
.
.
.
.
.
.

.
.
.
.
.
.
.
.
.
.
.
.
.
.

.
.
.
.
.
.
.
.
.
.
.
.
.
.

.
.
.
.
.
.
.
.
.
.
.
.
.
.

.
.
.
.
.
.
.
.
.
.
.
.
.
.

.
.
.
.
.
.
.
.
.
.
.
.
.
.

.
.
.
.
.
.
.
.
.
.
.
.
.
.

.
.
.
.
.
.
.
.
.
.
.
.
.
.

.
.
.
.
.
.
.
.
.
.
.
.
.
.

.
.
.
.
.
.
.
.
.
.
.
.
.
.

.
.
.
.
.
.
.
.
.
.
.
.
.
.

.
.
.
.
.
.
.
.
.
.
.
.
.
.

109
109
112
113
113
113
114
115
116
118
120
121
122
123
124

11 Week 11
11.1 Polarizers and and Maluss law . . . . . . . . . . . . . .
11.1.1 Polarization by reflection . . . . . . . . . . . . . .
11.1.2 Polarization by scattering . . . . . . . . . . . . .
11.1.3 Scattering demonstration and Rayleigh scattering
11.2 Rainbows . . . . . . . . . . . . . . . . . . . . . . . . . .
11.2.1 Other atmospherical optical phenomena . . . . .
11.2.2 Polarization of rainbows . . . . . . . . . . . . . .
11.3 Review for Exam 3 . . . . . . . . . . . . . . . . . . . . .

.
.
.
.
.
.
.
.

.
.
.
.
.
.
.
.

.
.
.
.
.
.
.
.

.
.
.
.
.
.
.
.

.
.
.
.
.
.
.
.

.
.
.
.
.
.
.
.

.
.
.
.
.
.
.
.

.
.
.
.
.
.
.
.

.
.
.
.
.
.
.
.

.
.
.
.
.
.
.
.

.
.
.
.
.
.
.
.

.
.
.
.
.
.
.
.

.
.
.
.
.
.
.
.

.
.
.
.
.
.
.
.

.
.
.
.
.
.
.
.

.
.
.
.
.
.
.
.

.
.
.
.
.
.
.
.

.
.
.
.
.
.
.
.

.
.
.
.
.
.
.
.

126
126
127
129
131
132
138
138
139

.
.
.
.
.
.
.

140
140
145
147
149
150
151
152

.
.
.
.
.
.
.
.
.
.
.

153
153
153
154
154
155
158
159
161
162
162
163

12 Week 12
12.1 Double slit interference and interferometers
12.2 Gratings and resolving power . . . . . . .
12.3 Single-slit diffraction . . . . . . . . . . . .
12.4 Doppler effect and the Big Bang . . . . . .
12.4.1 The Doppler effect and light . . . .
12.4.2 Big Bang cosmology . . . . . . . .
12.5 Farewell special . . . . . . . . . . . . . . .

.
.
.
.
.
.
.

.
.
.
.
.

.
.
.
.
.
.
.

.
.
.
.
.
.
.

.
.
.
.
.
.
.

.
.
.
.
.
.
.

.
.
.
.
.
.
.

.
.
.
.
.
.
.

.
.
.
.
.
.
.

.
.
.
.
.
.
.

.
.
.
.
.
.
.

.
.
.
.
.
.
.

.
.
.
.
.
.
.

.
.
.
.
.
.
.

.
.
.
.
.
.
.

.
.
.
.
.
.
.

.
.
.
.
.
.
.

.
.
.
.
.
.
.

.
.
.
.
.
.
.

.
.
.
.
.
.
.

13 Homework problems
13.1 Week 2 . . . . . . . . . . . . . . . . . . . . . . . . . . . . . . . . . . . . . . .
13.1.1 Problem 1: Motion of charged particle in electric field . . . . . . . . .
13.1.2 Problem 2: Three plates capacitor . . . . . . . . . . . . . . . . . . . .
13.1.3 Problem 3: Electric field, potential, and electrostatic potential energy
13.1.4 Problem 4: Electric field of a charged ring . . . . . . . . . . . . . . .
13.1.5 Problem 5: Two spherical conductors . . . . . . . . . . . . . . . . . .
13.1.6 Problem 6: Two conducting hollow cylinders . . . . . . . . . . . . . .
13.1.7 Problem 7: Speed of an electron . . . . . . . . . . . . . . . . . . . . .
13.2 Week 3 . . . . . . . . . . . . . . . . . . . . . . . . . . . . . . . . . . . . . . .
13.2.1 Problem 1: Spherical capacitor . . . . . . . . . . . . . . . . . . . . .
13.2.2 Problem 2: Coaxial cylinders . . . . . . . . . . . . . . . . . . . . . .
3

.
.
.
.
.
.
.

.
.
.
.
.
.
.
.
.
.
.

.
.
.
.
.
.
.

.
.
.
.
.
.
.
.
.
.
.

.
.
.
.
.
.
.

.
.
.
.
.
.
.
.
.
.
.

.
.
.
.
.
.
.

.
.
.
.
.
.
.
.
.
.
.

.
.
.
.
.
.
.

.
.
.
.
.
.
.
.
.
.
.

.
.
.
.
.
.
.

.
.
.
.
.
.
.
.
.
.
.

.
.
.
.
.
.
.

.
.
.
.
.
.
.
.
.
.
.

13.2.3 Problem 3: The effect of a dielectric medium on capacitance .


13.2.4 Problem 4: Coaxial cable with dielectric . . . . . . . . . . . .
13.2.5 Problem 5: Capacitor network . . . . . . . . . . . . . . . . . .
13.2.6 Problem 6: Resistances of conducting wires . . . . . . . . . .
13.2.7 Problem 7: Resistor network . . . . . . . . . . . . . . . . . . .
13.3 Week 4 . . . . . . . . . . . . . . . . . . . . . . . . . . . . . . . . . . .
13.4 Week 5 . . . . . . . . . . . . . . . . . . . . . . . . . . . . . . . . . . .
13.4.1 Problem 1: Lorentz Force . . . . . . . . . . . . . . . . . . . .
13.4.2 Problem 2: Motion of a charged particle in magnetic field . . .
13.4.3 Problem 3: Cyclotron . . . . . . . . . . . . . . . . . . . . . . .
13.4.4 Problem 4: Rectangular current loop . . . . . . . . . . . . . .
13.4.5 Problem 5: Resistor network . . . . . . . . . . . . . . . . . . .
13.4.6 Problem 6: Coaxial current loops . . . . . . . . . . . . . . . .
13.4.7 Problem 7: Parallel plate capacitor . . . . . . . . . . . . . . .
13.5 Week 6 . . . . . . . . . . . . . . . . . . . . . . . . . . . . . . . . . . .
13.5.1 Problem 1: Amperes law in action . . . . . . . . . . . . . . .
13.5.2 Problem 2: Intuition breaks down . . . . . . . . . . . . . . . .
13.5.3 Problem 3: Helmholtz coils . . . . . . . . . . . . . . . . . . .
13.5.4 Problem 4: Spinning loop in a magnetic field . . . . . . . . . .
13.5.5 Problem 5: Loop in a magnetic field . . . . . . . . . . . . . .
13.5.6 Problem 6: Electrodynamic tether . . . . . . . . . . . . . . . .
13.5.7 Problem 7: Motor . . . . . . . . . . . . . . . . . . . . . . . . .
13.5.8 Problem 8: Auroral zone . . . . . . . . . . . . . . . . . . . . .
13.6 Week 7 . . . . . . . . . . . . . . . . . . . . . . . . . . . . . . . . . . .
13.6.1 Problem 1: Magnetic energy of a solenoid . . . . . . . . . . .
13.6.2 Problem 2: Displacement current . . . . . . . . . . . . . . . .
13.6.3 Problem 3: RL circuit . . . . . . . . . . . . . . . . . . . . . .
13.6.4 Problem 4: RL circuit . . . . . . . . . . . . . . . . . . . . . .
13.6.5 Problem 5: Opening a switch on an RL circuit . . . . . . . . .
13.6.6 Problem 6: Self-inductance of a toroid . . . . . . . . . . . . .
13.6.7 Problem 7: RL circuit . . . . . . . . . . . . . . . . . . . . . .
13.7 Week 8 . . . . . . . . . . . . . . . . . . . . . . . . . . . . . . . . . . .
13.8 Week 9 . . . . . . . . . . . . . . . . . . . . . . . . . . . . . . . . . . .
13.8.1 Problem 1: RC circuit . . . . . . . . . . . . . . . . . . . . . .
13.8.2 Problem 2: RC circuit . . . . . . . . . . . . . . . . . . . . . .
13.8.3 Problem 3: RLC circuit . . . . . . . . . . . . . . . . . . . . .
13.8.4 Problem 4: An LRC circuit . . . . . . . . . . . . . . . . . . .
13.8.5 Problem 5: Design a flute . . . . . . . . . . . . . . . . . . . .
13.8.6 Problem 6: Width of resonance peak . . . . . . . . . . . . . .
13.8.7 Problem 7: Standing wave . . . . . . . . . . . . . . . . . . . .
13.8.8 Problem 8: Traveling wave . . . . . . . . . . . . . . . . . . . .
13.8.9 Problem 9: Lightly damped undriven circuit . . . . . . . . . .
13.9 Week 10 . . . . . . . . . . . . . . . . . . . . . . . . . . . . . . . . . .
13.9.1 Problem 1: Traveling electromagnetic waves . . . . . . . . . .
13.9.2 Problem 2: A standing electromagnetic wave . . . . . . . . . .
13.9.3 Problem 3: E-M waves - Maxwells equations, and the speed of
13.9.4 Problem 4: Polarized radiation . . . . . . . . . . . . . . . . .
13.9.5 Problem 5: Polarization of electromagnetic radiation . . . . .
13.9.6 Problem 6: Poynting vector . . . . . . . . . . . . . . . . . . .
13.9.7 Problem 7: Intensity of the sun . . . . . . . . . . . . . . . . .
13.9.8 Problem 8: Snells law in action: fiber optics! . . . . . . . . .
13.10Week 11 . . . . . . . . . . . . . . . . . . . . . . . . . . . . . . . . . .
4

. . .
. . .
. . .
. . .
. . .
. . .
. . .
. . .
. . .
. . .
. . .
. . .
. . .
. . .
. . .
. . .
. . .
. . .
. . .
. . .
. . .
. . .
. . .
. . .
. . .
. . .
. . .
. . .
. . .
. . .
. . .
. . .
. . .
. . .
. . .
. . .
. . .
. . .
. . .
. . .
. . .
. . .
. . .
. . .
. . .
light
. . .
. . .
. . .
. . .
. . .
. . .

.
.
.
.
.
.
.
.
.
.
.
.
.
.
.
.
.
.
.
.
.
.
.
.
.
.
.
.
.
.
.
.
.
.
.
.
.
.
.
.
.
.
.
.
.
.
.
.
.
.
.
.

.
.
.
.
.
.
.
.
.
.
.
.
.
.
.
.
.
.
.
.
.
.
.
.
.
.
.
.
.
.
.
.
.
.
.
.
.
.
.
.
.
.
.
.
.
.
.
.
.
.
.
.

.
.
.
.
.
.
.
.
.
.
.
.
.
.
.
.
.
.
.
.
.
.
.
.
.
.
.
.
.
.
.
.
.
.
.
.
.
.
.
.
.
.
.
.
.
.
.
.
.
.
.
.

.
.
.
.
.
.
.
.
.
.
.
.
.
.
.
.
.
.
.
.
.
.
.
.
.
.
.
.
.
.
.
.
.
.
.
.
.
.
.
.
.
.
.
.
.
.
.
.
.
.
.
.

.
.
.
.
.
.
.
.
.
.
.
.
.
.
.
.
.
.
.
.
.
.
.
.
.
.
.
.
.
.
.
.
.
.
.
.
.
.
.
.
.
.
.
.
.
.
.
.
.
.
.
.

.
.
.
.
.
.
.
.
.
.
.
.
.
.
.
.
.
.
.
.
.
.
.
.
.
.
.
.
.
.
.
.
.
.
.
.
.
.
.
.
.
.
.
.
.
.
.
.
.
.
.
.

.
.
.
.
.
.
.
.
.
.
.
.
.
.
.
.
.
.
.
.
.
.
.
.
.
.
.
.
.
.
.
.
.
.
.
.
.
.
.
.
.
.
.
.
.
.
.
.
.
.
.
.

.
.
.
.
.
.
.
.
.
.
.
.
.
.
.
.
.
.
.
.
.
.
.
.
.
.
.
.
.
.
.
.
.
.
.
.
.
.
.
.
.
.
.
.
.
.
.
.
.
.
.
.

.
.
.
.
.
.
.
.
.
.
.
.
.
.
.
.
.
.
.
.
.
.
.
.
.
.
.
.
.
.
.
.
.
.
.
.
.
.
.
.
.
.
.
.
.
.
.
.
.
.
.
.

165
166
168
169
170
171
171
171
172
173
173
175
176
176
179
179
181
182
184
185
186
186
188
188
188
190
190
192
193
196
197
198
199
199
200
201
201
203
203
204
205
206
208
208
209
211
213
216
217
218
219
221

13.11Week 12 . . . . . . . . . . . . . . . . . . . . . . . . . .
13.11.1 Problem 1: Primary rainbow . . . . . . . . . . .
13.11.2 Problem 2: Polarization of primary rainbow . .
13.11.3 Problem 3: Glassbow . . . . . . . . . . . . . . .
13.11.4 Problem 4: Secondary rainbow . . . . . . . . .
13.11.5 Problem 5: Diffraction pattern . . . . . . . . . .
13.11.6 Problem 6: Optical resolution of the human eye

.
.
.
.
.
.
.

.
.
.
.
.
.
.

.
.
.
.
.
.
.

.
.
.
.
.
.
.

.
.
.
.
.
.
.

.
.
.
.
.
.
.

.
.
.
.
.
.
.

.
.
.
.
.
.
.

.
.
.
.
.
.
.

.
.
.
.
.
.
.

.
.
.
.
.
.
.

221
221
223
223
224
225
226

14 Exam problems
14.1 Midterm 1 . . . . . . . . . . . . . . . . . . . . . . . . . . . . . . . . . . .
14.1.1 Problem 1: Electric field on the surface of a conductor . . . . . .
14.1.2 Problem 2: Non-conducting charged planes . . . . . . . . . . . . .
14.1.3 Problem 3: Incandescent bulbs . . . . . . . . . . . . . . . . . . .
14.1.4 Problem 4: Circuit . . . . . . . . . . . . . . . . . . . . . . . . . .
14.1.5 Problem 5: Point charge in a hollow conducting sphere . . . . . .
14.1.6 Problem 6: Charges on an equilateral triangle . . . . . . . . . . .
14.1.7 Problem 7: Capacitor network . . . . . . . . . . . . . . . . . . . .
14.2 Midterm 2 . . . . . . . . . . . . . . . . . . . . . . . . . . . . . . . . . . .
14.2.1 Problem 1: RL circuit . . . . . . . . . . . . . . . . . . . . . . . .
14.2.2 Problem 2: Non-conservative fields . . . . . . . . . . . . . . . . .
14.2.3 Problem 3: Bainbridge mass spectrometer . . . . . . . . . . . . .
14.2.4 Problem 4: RL circuit . . . . . . . . . . . . . . . . . . . . . . . .
14.2.5 Problem 5: Magnetic field of a loop . . . . . . . . . . . . . . . . .
14.2.6 Problem 6: Magnetic field of a current-carrying ribbon . . . . . .
14.2.7 Problem 7: Magnetic field of a rotating charged sphere . . . . . .
14.3 Midterm 3 . . . . . . . . . . . . . . . . . . . . . . . . . . . . . . . . . . .
14.3.1 Problem 1: RC Circuit . . . . . . . . . . . . . . . . . . . . . . . .
14.3.2 Problem 2: RLC circuit . . . . . . . . . . . . . . . . . . . . . . .
14.3.3 Problem 3: Energy flow of a capacitor . . . . . . . . . . . . . . .
14.3.4 Problem 4: Electromagnet with small air gap . . . . . . . . . . .
14.3.5 Problem 5: RLC Circuit . . . . . . . . . . . . . . . . . . . . . . .
14.3.6 Problem 6: Electromagnetic wave . . . . . . . . . . . . . . . . . .
14.3.7 Problem 7: Radiation pressure on the Earth . . . . . . . . . . . .
14.4 Final Exam . . . . . . . . . . . . . . . . . . . . . . . . . . . . . . . . . .
14.4.1 Problem 1: Loop in a Magnetic Field . . . . . . . . . . . . . . . .
14.4.2 Problem 2: Non-conducting infinite sheet and infinite parallel slab
14.4.3 Problem 3: RLC circuit . . . . . . . . . . . . . . . . . . . . . . .
14.4.4 Problem 4: Plane electromagnetic wave . . . . . . . . . . . . . . .
14.4.5 Problem 5: Loop in a magnetic field . . . . . . . . . . . . . . . .
14.4.6 Problem 6: Charged particles in a magnetic field . . . . . . . . . .
14.4.7 Problem 7: Diffraction pattern . . . . . . . . . . . . . . . . . . . .
14.4.8 Problem 8: Capacitor washers . . . . . . . . . . . . . . . . . . . .
14.4.9 Problem 9: Circuit . . . . . . . . . . . . . . . . . . . . . . . . . .
14.4.10 Problem 10: An LC circuit . . . . . . . . . . . . . . . . . . . . . .
14.4.11 Problem 11: Dielectric sphere . . . . . . . . . . . . . . . . . . . .
14.4.12 Problem 12: Parallel RLC . . . . . . . . . . . . . . . . . . . . . .

.
.
.
.
.
.
.
.
.
.
.
.
.
.
.
.
.
.
.
.
.
.
.
.
.
.
.
.
.
.
.
.
.
.
.
.
.

.
.
.
.
.
.
.
.
.
.
.
.
.
.
.
.
.
.
.
.
.
.
.
.
.
.
.
.
.
.
.
.
.
.
.
.
.

.
.
.
.
.
.
.
.
.
.
.
.
.
.
.
.
.
.
.
.
.
.
.
.
.
.
.
.
.
.
.
.
.
.
.
.
.

.
.
.
.
.
.
.
.
.
.
.
.
.
.
.
.
.
.
.
.
.
.
.
.
.
.
.
.
.
.
.
.
.
.
.
.
.

.
.
.
.
.
.
.
.
.
.
.
.
.
.
.
.
.
.
.
.
.
.
.
.
.
.
.
.
.
.
.
.
.
.
.
.
.

.
.
.
.
.
.
.
.
.
.
.
.
.
.
.
.
.
.
.
.
.
.
.
.
.
.
.
.
.
.
.
.
.
.
.
.
.

.
.
.
.
.
.
.
.
.
.
.
.
.
.
.
.
.
.
.
.
.
.
.
.
.
.
.
.
.
.
.
.
.
.
.
.
.

.
.
.
.
.
.
.
.
.
.
.
.
.
.
.
.
.
.
.
.
.
.
.
.
.
.
.
.
.
.
.
.
.
.
.
.
.

.
.
.
.
.
.
.
.
.
.
.
.
.
.
.
.
.
.
.
.
.
.
.
.
.
.
.
.
.
.
.
.
.
.
.
.
.

.
.
.
.
.
.
.
.
.
.
.
.
.
.
.
.
.
.
.
.
.
.
.
.
.
.
.
.
.
.
.
.
.
.
.
.
.

227
227
227
228
229
230
231
232
233
235
235
236
236
238
238
239
240
243
243
244
247
250
251
252
253
255
255
256
260
262
263
265
266
268
272
275
278
279

.
.
.
.
.
.
.

.
.
.
.
.
.
.

.
.
.
.
.
.
.

.
.
.
.
.
.
.

.
.
.
.
.
.
.

.
.
.
.
.
.
.

.
.
.
.
.
.
.

.
.
.
.
.
.
.

.
.
.
.
.
.
.

Chapter 0
Preface/disclaimer
The most important thing to keep in mind where reading these notes is that Im just another student,
taking this course. I have almost zero previous knowledge of electromagnetism, though I previously took
the MITx 6.002x Circuits and Electronics class. That knowledge helped only in small parts of this course,
however, and Ive learned almost everything except circuit analysis on a week-to-week basis, while writing
down all this.
In other words: there may well be errors in here!
In part because I make typos and other stupid mistakes, but also because I might have misunderstood
some concept somewhere. I usually point out whenever I feel less than certain, but that doesnt guarantee
anything, of course.
These notes are made up of three parts: lecture notes, homework solutions, and exam solutions.
The lecture notes are in part from the lectures as-is (essentially a transcription), but I do add my own
comments and extra information here and there.
They will lack some information from the lectures, as I dont write down everything (for example, I rarely
write down anything during demonstrations, only their results if necessary), but also have some tiny extra
bits (from the book, things I knew to begin with, or things I look up elsewhere).
I write the homework solutions as I solve them, so the text and even the answers are usually written before
Ive even clicked check. Still, I look at the staffs solutions after the fact and at least if I remember using
a different method, I try to check that I wasnt just lucky in getting the correct answer for myself.
The exact same thing applies to the exam solutions as well.
There are no notes/solutions for homework 1, as I started writing the solutions down the second week.
By the way: I certainly welcome all feedback, corrections, comments and whatnot! Too small a fraction
of my daily e-mail is written by actual human beings (rather than being spam, mass-emails/mailing lists,
forum notices, ...), anyway. :-)

Chapter 1
Week 1
1.1

Coulombs law

Two point charges q1 and q2 , separated by distance r, equal in magnitude and of equal polarity, will repel
one another. The force on q2 due to q1 , in the direction 180 degrees opposite to to the direction of q1 , is
q1 q2 K
r1,2

F~1,2 =
r2
... where K is Coulombs constant, roughly 8.987551787 109 N m2 C2 , and F~1,2 means the force on
charge 2 due to charge 1. r1,2
is the unit vector in the direction from charge 1 to charge 2. And, since
each force has an equal and opposite reaction, we also have a F~2,1 (F~2,1 = F~1,2 )
Note how the equation is sign-sensitive: if both changes have the same sign, the force forces the particles
apart. On the other hand, if they differ in sign, the result will be negative, and so the force vector will
point towards q1 from q2 (and vice versa) instead of away from it.
Also:
K=

1
40

where 0 is the permittivity of free space, also (more recently) known as the electric constant.
0 8.854187 1012 farad/meter

1.2

Electric fields

Imagine a setup very similar to the one above.


We have a charge, Q+ , and a test charge q a distance r away. As before, the force on the test charge is
QqK
F~ = 2 r
r
If we take that force vector, and divide out q, the test charge, we get the electric field at the point in
space where q is located. Lets call that point p; in that case, we have the electric field E~p :
F~
QK
E~p =
= 2 r
q
r
where r is again the unit vector pointing away from Q in the direction of the test charge.
By convention, we choose the electric field to be such that the field vector is in the direction of the force
on a positive test charge. If the test charge were negative, the electric force would attract the two charges
7

toward each other, and the electric field would be pointing towards Q.
Put more simply: for a positive charge, we draw the field as a set of arrows pointing from the charge and
away. This field is spherically symmetric, and as we saw above, decays in a inverse-r-squared fashion. If
the charge is instead negative, we draw it as a set of arrows pointing towards the charge, again with more
force (longer arrows, to represent larger vector magnitudes) closer to the charge.
The electric field can be thought of as the force per unit charge (as we have divided out the charge of the
equation).

1.2.1

Multiple charges

To calculate the net electric field for multiple charges, we use the superposition principle: very simply, we
calculate the electric field due to each charge alone, at a certain point p; the net electric field a that point
is then simply the vector sum of all these individual electric fields.
For example, for a set of three charges, Q1+ , Q2 and Q3 , we get three separate electric fields: one that
repels from Q1 , and two that attract towards Q2 and Q3 respectively. The net electric field is then
E~p = E~1 + E~2 + E~3
and more generally, for a set of i charges, the same sum holds:
X
~i
E~p =
E
i

So depending on how much change each of these points hold, the resulting vector may be in any direction,
with any magnitude; we simply cannot say until we do a quantitative calculation of the force.
Now, then, what is the point of calculating this electric field? Well, now we know what the electric field
at that point is, we can very easily calculate the force vector for any charge we place at the location, like
this:
~
F~ = q E
So, in other words, we simply multiply the charge by the electric field vector, and we get the force. q can
be large or small, positive or negative, and the result will still come out right.
If the charge is large, the force will be large, and vice versa.
~ If the charge is negative, it
If the charge is positive, the force vector will be in the same direction as E.
~
will be in the opposite direction of E.
In a system of multiple charges, there may be a point in space where the electric fields magnitude is
exactly zero, because the forces from all the charges in the system perfectly cancel out.
One of the simplest such cases is when you have a set of three charges, equal in polarity and magnitude,
arranged at the vertices of an equilateral triangle. In such a case, the electric field strength will be exactly
zero in the middle of the three charges.
Charges of differing polarity can also produce points with no net force.
Imagine a one-dimensional line, measuared in meters. At the exact middle, 0 m, we have a 1C charge.
At 4 m, we have a 0.5C charge.
For a positive test charge located in between these two, i.e. on the interval (0, 4), the forces will be
additive; the positive charge at 0 will push towards the right, and the negative charge at 4 m will pull
towards the right.
If we go past 4 m, however, there will come a point where the force cancels out exactly: the stronger, but
8

more distant positive charge repels, and


the weaker, but closer negative charge attracts.
In this example, that spot is at 8 + 4 2 meters, which can be found by solving the equation relating the
two electric fields:
1K
E~0 = 2 x
r
0.5K
E~4 =
x
(r 4)2
E~0 + E~4 = 0
where E~0 is the electric field due to the particle a 0 m alone, and E~4 the field due to the negative particle
at 4 m alone.
If we solve the equation (on the third line above), we get 13.65 meters.

1.2.2

Electric field lines

Theres a second common way of visually representing electric fields, besides the vector field: electric field
lines. Here, we lose some information about the field strength at any particular point, but can instead see
the curvature much more clearly.
Such a representation would have a ton of lines coming from (or going to, for negative charges) each charge,
curving around depending on other, nearby charges.
The further away you get from a charge, the further apart the lines will become. This represents the
inverse-r-squared weakening of the field.
One crucial aspect about these lines is that if you place a positive charge anywhere on a field line, the
force vector on that charge will always be tangential to the field line at that point. If the placed charge is
negative, the force vector will simply be the same but flipped 180 degrees around.
It is also important to note that because of this, the trajectory of a charge (say, released a 0 velocity) will
not follow any one field line exactly; the trajectory can be quite complex. Any time you reach a field line,
however, the force vector will again be tangential to that field line.

1.2.3

Dipoles

As we saw above, with two charges of equal polarity, there is a point between the two where the force
cancels out, and the electric field is zero.
If we instead have two charges, equal in magnitude but opposite in sign, we have a dipole. In this case,
there is no point where the electric field is zero.
Two charges of unequal charge and opposite polarity will, as observed from far away, behave as a single
charge whose value is the difference of the two. For example, a +3 change next to a -1 will look like a
+2 charge from far away, and the electric field falls off with a inverse-r-squared relationship.
With dipoles, that is no longer the case: the field falls off with an inverse-r-cubed relationship, i.e. it
weakens much faster as the distance increases. This does make intuitive sense: from afar, it should act
like a single point charge with zero charge!

1.3
1.3.1

Electric flux and Gausss law


Quick facts

According to Wikipedia, electric flux is the rate of flow per unit charge of the electric field. It can be
thought of as the number of electric field lines going through a surface.
For a uniform electric field, the flux passing through a surface of vector area S is
9

~ A
~ = EA cos
E = E
where is the angle between the normal of the surface (the vector perpendicular to the surface) and the
electric field lines.
For a non-uniform electric field, the flux can be calculated by using a surface integral:
x
~
~ dA
E =
E
S

~ is a vector area infinitesimal.


where dA

1.3.2

Slower this time...

OK, lets start over. Say we have an open surface, e.g. a paper (unlike a closed surface, e.g. a bag or a
balloon) placed in an electric field. We can divide the field into infinitesimals with area dA. Each of these
will have a normal unit vector n
that is perpendicular to it.
Each of these infinitesimal areas will have a flux d associated with it, which can be calculated:
~ = E dA cos
~ n
~ dA
dE = E
dA = E
Here, is, as above, the angle between the normal and the electric field lines. The notation with and
without the n
are two ways of writing the same thing. One considers dA as an area, with a normal unit
~
vector to give it direction, while the second combines the to into the vector area dA.
The flux can be larger than, smaller than, or equal to zero.
One can make a useful parallel between electric flux and air flow. Consider a hollow rectangle position
perpendicular to a uniform air flow. In other words, the normal to the rectangle is parallel with the air
flow. How much air goes through it? Well, its simply the rate of air flow multiplied by the rectangles
area.
If we rotate the rectangle 90 degrees, such that the normal is now perpendicular to to the air flow, no air
goes through the rectangle - it is parallel to the air flow.
If the angle is anything in between the two, we simply multiply the v A we already have by the cosine of
the angle. That is, the air flow through the rectangle is given by vA cos .
The exact same way can be used to calculate the electric flux through the rectangle - multiply the electric
field vector and the vector area using the dot product, such that you get EA cos .
Example with a point charge
Say we have a point charge + Q, and a spherical surface with radius R surrounding it.
For any chosen infinitesimal area dA on the surface, the normal will be radially outwards - as will the
electric field, since we have a positive point charge. In fact, the two will be parallel at all locations on the
surface, so the cosine of the angle between them is always 1.
Not only that, but since we chose to do this with a spherical surface, the electric field has exactly the
same magnitude at all points along the surface. These facts, when combined, makes for an extremely easy
calculation of the total flux - it is simply the magnitude of the electric field, times the surface area - EA,
or in the case of a sphere, 4R2 E.
QK
Well, E is 2 r, as we have seen previously, with r being the unit vector in the direction from the charge
R
to the chosen surface area infinitesimal. K, the Coulomb constant, has an exact definition (but we often
simply use 9 109 instead, which about 0.14% off from the real value):
K=

1
40

10

If we substitute that in, we have


Q
r
40 R2
... which looks more complex, until we finalize the flux calculation by multiplying the electric fields
magnitude with the surface area of 4R2 :
~ =
E

Q
0
Spectacular! Note not only how simple it is, but indeed that the radius term disappeared. And, since 0 is
a physical constant, we clearly see that the net charge is the only thing that determines the flux through
this closed surface.
Indeed, this also means that the result holds regardless of the shape of the surface used to confine the
charge. Whether it is a sphere, a cylinder, a box or just something entirely different, the amount of flux
is exactly the same.
This can be intuitively understood if we go back to the air flow analogy: regardless of the bags size,
assuming air could travel through it, it would have to get out of there somewhere regardless of the bags
size.
E 4R2 =

Now, since electric fields from multiple point charges are added using vector addition, it is clear that any
collection of charges inside this bag must behave in the same way. That leads us to:

1.3.3

Gausss law

Gausss law states that the electric flux through a closed surface, with nonzero net charge inside, is,
regardless of the shape and size of the surface:
P
{
Qins
~ =
~ dA
=
E
0
P
where (also E ) is the total electric flux through the surface,
Qins is the total electric charge inside
the surface, and 0 as always the permittivity of free space.
The ring in the integral symbol is to indicate that the surface we are integrating over is a closed surface.
Gausss law always holds, no matter how the charge is arranged inside the bag, or how it is shaped.
It is, however, not very useful unless we have some form of symmetry. The main use of Gausss law is
to calculate electric fields, and to do so, we need to take advantage of one of three forms of symmetry:
spherical, to encapsulate a point charge or such; cylindrical, to encapsulate an infinite wire of charge;
and planar, to calculate electric fields around an infinite plane. In all three cases, the charge distribution
must be uniform.
Gausss law application: a charged, hollow sphere
Lets attempt an application of Gausss law, on a hollow, conductive sphere, with charge Q uniformly
distributed on the surface. The sphere is of radius R.
First, we need to choose one of the gaussian surfaces: the spherical one, the cylinder (for the infinite wire)
or the pillbox, also a cylinder, used for planar surfaces.
Clearly, we want the spherical one in this case; we choose a concentric sphere (i.e. it is perfectly centered
on the charged sphere) such that the radius r of the Gauss sphere touches the point where we want to
calculate the electric field.
Then, we note a few things: the problem is spherically symmetric, so the electric fields magnitude at any
point along the surface of our Gauss sphere must be exactly the same as for any other point on it. That
follows from the symmetry of the problem.
Also, we know that the electric field is radially outwards (or inwards, if the charge is negative) - either

11

way, it is either parallel or antiparallel between any chosen dA on the surface.


Now that we have the same situation as before - the direction of E is always parallel with the surface
normals, and we know how the surface area of a sphere - we can calculate the (again simply EA), and
then solve for the electric field.
Lets begin with the case of r < R, i.e. inside the hollow sphere:
P
Qins
E4r2 =
0
P
Qins
E=
4r2 0
However, we know from the problem definition that there is no charge inside, so the numerator is zero!
Therefore, the electric field at every point inside the sphere is zero, since the result clearly holds for any
r inside the sphere - we didnt specify it.
Next, lets look at the case where r > R, i.e. outside the hollow sphere.
The same arguments hold here, as well: the problem is spherically symmetric, so the magnitude must
be equal at all points at the distance r from the sphere; also, we know that the electric field is radially
outwards (or inwards). Therefore, we can use the same calculation:
P
Qins
2
E4r =
0
P
Qins
E=
40 r2
This, too, is an incredible result! The result is exactly what you get for a point charge, when using
1
Coulombs law (only that we use
= K here instead of the symbol K). In other words, this charged
40
hollow sphere behaves exactly as a point charge to everyone outside the sphere (while, as we saw just
above, the electric field is exactly zero inside the sphere).
Second Gausss law application: infinite charged plane
Next, consider a large plane - infinite, even, for the calculation - again with a uniform charge density .
That is, we have a set number of coulombs per square meter .
As before, our first task is to choose a Gaussian surface. For planes, we use the pillbox, which is a
cylinder. Our goal is to find the electric field at any point in space. We choose a point d, and place
our cylinder such that the plane is exactly centered along its height. That is, d of the cylinders height is
above the plane, and d is also below the plane.
Now, for the symmetry arguments to work, we need a few things to be true: the circular area at the top
of the cylinder must be parallel to the charged plane, and same for the bottom. Second, the vertical walls
of the cylinder must be perpendicular to the plane. Lastly, as stated previously, the two side caps must
both be exactly d away from the plane, i.e. be at the same distance from the plane.
Since the surface charge distribution is uniform, we can find the enclosed charge in our cylinder easily: it
is simply the charge density multiplied by the area of the cylinders end caps:
Qenc = r2
Keep in mind that since the charge isnt in the end caps, the fact that there are two of them is irrelevant.
We are only interested in the volume they enclose, namely the infinitesimally thin plane in the center.
The total flux through the end caps, however, is doubled because of this. The flux through each cap is
E A, with E being the electric field magnitude, and A the end cap area. Therefore, the total flux is (with
A = r2 ):
12

= 2Er2
Now that we know the enclosed charge and the total flux, we can use Gausss law to find the electric field.
Qenc
To do so, we set
equal to the total flux :
0
r2
2
2Er =
0

2E =
0

E=
20
The direction of this force is z above the plane, and
z below it, so we have:

z for z > 0

20
~ =
E

z for z < 0
20

13

Chapter 2
Week 2
2.1

Electrostatic Potential Energy and Electric Potential

Before we get started, lets introduce the two terms we will work with. The first is electrostatic potential
J
energy, U , measured in joules. The second is electric potential, V , measured in volts (1 V = 1 ).
C
There two are, as the dimensions suggest, not interchangable, but they are related.

2.1.1

Electrostatic Potential Energy

Now then. Lets say we have two charges, q1 and q2 . They are both positive, and are separated by a
distance R. Because they are both positive, they repel each other, and therefore, some external agent
must have done work to move the two together. The work we put in to moving the charges together (by
convention, we count all the way from infinity to their final position) is the electrostatic potential energy.
It can be thought of as the electrical analogue of gravitational potential energy - lift an object upwards,
against the Earths gravity, and you have increased its gravitational potential energy. If you then let it
go, that potential energy will be turned into kinetic energy as the object falls back down.
In the same manner, if you bring the positive charges together, and then let them go, the electric force will
repel them and cause them to move, thus converting their electrostatic potential energy into kinetic energy.
The work we will have to do to push the charges together is of course exactly equal to the negative work
the electric force will do - they are opposite in direction (i.e. the vectors are antiparallel), but equal in
magnitude. If we use F~we do denote our force, we have that F~we = F~el , where the latter is the electric
force.
The work required to bring the charge in from infinity, then, is:
Z R
Z
~
~
~
Fwe dr =
F~el dr
Wwe =

The latter half is, as stated above, exactly the same as the force we provide; only the sign differs between
the two forces, so we swap the integration limits to in effect multiply the integral by 1.
We know that the force and r are in the same direction, so the angle between them is 0, and therefore the
cosine of the angle is 1. Therefore we can ignore the vectors, and instead use Coulombs law for much of
the integral. Continuing:
Z
Z
q1 q2 dr
~
~
Wwe =
Fel dr =
40 R r2
R
Z
dr
1
1
1
= = 0 ( ) =
2
r R
R
R
R r
So, when we combine the two, we just need to add that R to the denominator, and were done:

14

q1 q2
40 R
Since the quantity of this is work, this is a scalar, and it is as stated above in joules.
Note how the equation is sign-sensitive: if the two charges are of the same sign (whether positive or negative), the resulting number is positive (since nothing in the denominator can ever be negative), while if
they differ in sign, the result is negative. This is of course exactly right: we need to do positive work if
they repel each other, but will do negative work if they attract.
U = Wwe =

The electric force is conservative. This means that regardless of which path you take from infinity to the
point in question, the work done is exactly the same, no matter how convoluted the path. This is also
true of gravity, which is another example (perhaps the most common one) of a conservative force.
As a counterpoint, an example of a non-conservative force is friction. Moving an object along some short,
straight line between points will cause some amount of work, while moving it along a very long path will
cause more work.

2.1.2

Electric potential

The electric potential is closely related to the electrostatic potential energy.


W take a similar setup as in the section above, with a positive charge Q, a test charge q at point P , a
distance R away from Q.
The electrostatic potential energy of this system we already know, as we calculated it just above:
Qq
40 R
The electric potential is defined as the work per unit charge we have to do in order to bring the charge to
that location. That is, we divide out the test charge q, and so we have that the electric potential V and
the point P is:
U=

VP =

U
Q
=
q
40 R

And, as stated in the introduction to this section, this is measured in joules per coulomb, also known as
volts (V).
If Q is positive, the potential everywhere in space (due to that charge alone, of course!) will be positive.
If the charge is negative, the potential will be negative everywhere.
Lets calculate an example, while adding some new information still.
Take a hollow sphere with radius R = 0.3 meters, with 10C of charge on it, evenly distributed (as it will
be automatically for a conductor).
We want to calculate the electric potential everywhere in space. We divide it into two cases: r < R (inside
the sphere) and r > R (outside). Lets begin with the calculation for the potential outside the sphere, at
a point P .
U
, where V is the electric potential and U is the electrostatic potential energy, and
qZ

~ Therefore, the electric potential is the same integral, except we divide


how the latter is U =
F~elec dr.

Remember how V =

out the test charge q:

Z
VP =
r

F~elec ~
dr
q

Aha! Force divided by charge is nothing other than the definition of an electric field! Therefore this is
equal to:
15

Z
VP =
r

Z
F~elec ~
~ = Q
~ dr
E
dr =
q
40 r
r

... which is the same result we got from a point charge. In other words, just like how an electric field is
the same for a sphere with uniform charge density and a point charge of the same magnitude, the same
thing applies to the electric potential.
Now that we can calculate the electric potential, how do we use it? Well, we can now calculate the work
required to bring a charge to a point very easily. We know from above that U = V q (except we solved
for V last time), so we simply multiply the charge we are interested in by the potential, and we have our
answer!
As an example, lets say we calculate/measure a potential of 10 kV near a charged sphere. We want to
bring 1 millicoulomb to the point in space where the potential is 10 kV. The work we must do is
W = 103 C 104 V = 10 J
So the work required is 10 joules. We can now solve the same question for a charge of 2 C instead, without
having to solve a new integral - we simply multiply it by the 104 volts and get our answer of 20 kilojoules.
What happens, then, if we move this charge inside this hollow, charged sphere? Well, as we saw in the
previous week, there is no electric field whatsoever inside, so the charges would experience no force. No
force means no work, which implies the electric potential must be the same everywhere inside. Note that
it only implies that the potential is the same everywhere, and not that it is zero! It will, in fact, be 10
kilovolts everywhere inside the sphere (same as on the surface).
Equipotentials
Now, lets look at another phenomenon outside the charge. Since the electric potential for a point charge
/ charged sphere depends only on a set of constants, plus the amount of charge and the distance r, it is
clear that all points that are r away from the charge will have the exact same potential, by symmetry.
Each such surface, where all points have the same electric potential, is known as an equipotential surface.
There will be an infinite amount of such surfaces at different distances away from the charge.
Electric fields lines are always perpendicular to equipotential surfaces: if they were not, we could decompose the electric field vector at a point on the surface, into parallel and perpendicular dimensions. That
implies that there is an electric field along the surface, which means the surface is not an equipotential !
Therefore, field lines are always perpendicular to equipotential surfaces.
Now, back to the example. To make things more interesting, lets add a second charge; both charges will
now be positive, and separated by a fairly small distance.
The electric potential at any point in space will be the sum of the potential due to each charge alone, so
potentials act exactly as electric fields do in this case. (A notable difference is that the potential is a scalar
field, however, while the electric fields is a vector field.)
Because both of our charges are positive, the potential due to these charges will always be either positive
or zero. There is no negative charge anywhere, and so the potential cannot be negative, either.
Lets say one charge is +4, while the other is +1.
Very close to to +4 charge, the equipotential surface will be a near-perfect sphere, concentric with the
charge. Slightly further out, but still closer to the +4 charge than the +1, the surface will be a distorted
sphere, with is being stretched out towards the second charge.
Further out yet, there will be an (infinite number of) equipotential surface(s) that enclose both charges,
and is shaped sort of like the symbol, except asymmetrically: the +4 charge is much stronger, and so
its equipotential is larger.
Further away yet - far away that the +4 and the +1 are essentially a point charge of +5, the equipotentials
16

will again start looking like more and more perfect spheres.
If we change the +1 charge to a 1 charge, what happens? Well, first off, because we now have a mix of
positive and negative charges, we will now have both positive and negative equipotentials. Also, since we
have a mix, and there will be a transition between the two, there will now be points in space where the
potential is zero.
Other than that, the situation is similar to what you would expect. Close to the 1 charge, the equipotentials are again spherical, but they are of negative potential. That is, if you bring a positive test charge
from infinity up to that negative equipotential, you will have done net negative work. Despite having to
work your way through the positive potential of the stronger +4 charge, the attracting force of the 1
charge wins out in the end. And, since the electric force is conservative, bringing such a test charge to a
point in space where the potential is zero means you have done a net zero amount of work.
Because we now have a mix of positive and negative charges, there will both be a point where the electric
field is zero, and a point where the potential is zero. These point are not one and the same, however!
Positive charges will always travel from high potentials to low potentials, which is analogous to gravity:
object always fall downwards (towards lower gravitational potential), and never upwards.
In electricity, unlike gravity, we also have negative charges. Negative charges always travel from low
potentials to high potential, so they essentially go in reverse.
Electric potentials: example
Lets define two points, A and B, with electric potentials VA and VB , respectively. They are separated by
a distance R.
We know that
Z
~
~ dr
VA =
E
A

~
~ dr
E

VB =
B

And therefore,
Z

~
~ dr
E

VA VB =
A

~
~ dr
E

VB VA =
A

~ implies (by convention) that we go in a straight line between the two points.
The infinitesimal vector dr
However, as stated previously, the electric force in conservative, and so it doesnt matter which path we
~ with d`
~ to imply this:
take. We can replace dr
Z B
~
~ d`
VA VB =
E
A

VB VA =

~
~ d`
E

Now, then. Lets calculate an example. Say VA = 150 volts, and VB = 50 volts. Therefore, VB VA = 100
volts. If we release a free electron at point B, it will go towards VA - electrons travel towards higher
potentials, while a proton would travel away from it.
In this calculation, we will use an electron. The electron has a mass and a charge, of course:
me = 9.10938 1031 kg
17

Qe = 1.602 1019 C
We will need those values momentarily.
Via the Work-Energy theorem, the kinetic energy of the electron will, at the end of its journey, have used
up all of its electrostatic potential energy. We know how to calculate the latter - its U = V q, and we
1
know both the potential and the charge. The kinetic energy of a particle is (in classical physics) mv 2 .
2
We can now set the sum of the two equal to zero, substitute in the values for the situation, and solve for
v:
1 2
mv + V q = 0
2
mv 2 = 2V q
r
2V q
v=+
m
r
v=+

2 (100) (1.602 1019 )


9.109378 1031
v 5.93 106 m/s

Thats roughly 2% of the speed of light, despite the relatively small potential difference (compared to
multi-kilovolt Van de Graaff generators, at least)! Also note how the distance between the two points does
not matter - the ending velocity of the particle is the same either way, as can be seen by the fact that
there is no distance specified anywhere in the equation.

2.1.3

More on electric potential

Lets say that we are at a point P in space. At that point, there is a potential VP , as well as an electric
field (it may be zero, of course).
We now take a very small step in the x direction, and the x direction only - y and z are kept exactly
the same. We measure the electric potential at this new point. If the potential is the same, then the x
component of the electric field here is, by definition, zero. If it is not, the magnitude of that component is
|V |
x
The same holds, of course, for the y and z directions.
V
N
Note how the fraction above has units , while describing an electric field. We have previously used
m
C
kg m
to describe electric field magnitudes, but the two are in fact equivalent: they are both
in base SI
A s3
units.
Because volts per meter are more intuitive for visualising potential difference per unit length, we will
mostly use those units from here on.
Ex =

We can calculate the electric field from a known potential, by taking the negative gradient of the potential:


V
V
V
~ = V = grad V =
E
x +
y +
z
x
y
z
The partial derivatives shown make up the components of the vector field:
V
x
V
Ey =
y

Ex =

18

V
z
Lets try an example, in one dimension. We have the electric potential
Ez =

V = 105 x
which is valid over a small range of x, from 0 to 102 , so a distance of 1 cm.
At x = 0, we have a large plate A, and at x = 102 m, we have another plate B. B has a positive charge,
while A has a negative charge.
Knowing the potential, we can calculate the electric field using the formula above. Since the y and z
components are both zero, we just take the negative derivative, and multiply it by x, as above, and get:
~ = 105 x
E
We know since before that
Z

VA VB =

~
~ d`
E

~ instead of
... but in this case, we know that the movement is strictly in the x direction, so we can use dx
~ Lets also substitute in the value for E,
~ so that we get:
d`.
Z B
~
105 x dx
VA VB =
A
5

Well, now what? Well, first off, 10 is a constant, so we can move it outside the integral. Second, x and
~ are by definition in the same direction, so the cosine of the angle between them is one. Therefore, we
dx
can get rid of all the vector stuff and reduce the integral to
Z B
5
VA VB = 10
dx = 105 (xB xA )
A

If we plug in xB = 102 and xA = 0, we get 1000 volts, which is our answer. That is, A is 1000 volts
lower than B. The electric field between the two plates point in the
x direction, i.e. the left, while
the potential grows towards the right. Thats the physical meaning of having the electric field being the
negative of the derivative of the electric potential.

2.2

Electric fields inside hollow conductors

The electric field is always zero everywhere inside a solid conductor. The same is not necessarily true in
a non-conductor. In a conductor, it is true because if there is an electric field, the free electrons (that
define a conductor) would move around until the field is cancelled out, and they no longer experience a force.
Imagine a solid (non-spherical, to show that spherical symmetry is not required) conductor. We add a
positive charge to this conductor. The question is, now: how does this charge distribute itself?
We know that it does so in a manner that ensures there is no electric field inside the conductor (once it
has settled down), but that doesnt answer the question of exactly where the charge is.
Lets first imagine that it spreads itself out evenly everywhere inside the solid block. Could this be?
No, it could not. We know that the electric field must be zero inside the conductor. Now, lets choose a
Gaussian surface, e.g. a small sphere, placed at any point inside this solid conductor. Recall Gausss law:
P
{
Qins
~
~
=
E dA =
0

19

~ is zero, since this is inside the conductor. Since the left side (the integral) is zero, we
We know that |E|
P
can only conclude that
Qins must be zero. There can be no charge anywhere inside the conductor only on the surface - or this could not be true!
So, the charge is on the surface. It is not uniformly distributed, however! That would only be the case if
the conductor were a sphere.
Now, lets see what happens if we make it hollow. That is, there is an inner surface and an outer surface
(with a nonzero amount of the conducting material between them), but the shape remains the same (remember that it is non-spherical!).
We now have two surfaces - one outer and one inner. Will the situation change? That is, will some of the
charge now be on the inside surface as well?
The answer is no. We can use the same argument: imagine a Gaussian surface that is located just inside
the outer surface, such that it envelops the entire shape, minus the outer surface. The Gaussian surface
is inside the conductor at all points, which means the electric field must be zero everywhere on it. That,
~ integral will be zero, and so there cannot be any charge enclosed
~ dA
in turn (just as above) means the E
by the surface.
The charge is still exclusively on the outer surface!
Lets take it another step. We take a similar hollow conductor - a sphere, now - except we bring a charge
+Q inside the (closed) conductor... somehow.
The same rules still apply: the electric field inside the conductor must be zero.
Using the same Gausss law argument, we can show that there must, in fact, be negative charge on the
inside surface now!
~ integral along the Gaussian surface - again chosen to enclose the conductor minus
~ dA
Once again, the E
the outer surface - must be zero, since the electric field is everywhere
zero in the conductor.
P
Qins
- must be zero! That can only happen
This means also that the right-hand side of Gausss law 0
when the net charge inside is zero. In other words, if we bring a positive charge to the inside of the hollow
conductor, nature will cause negative charge to accumulate on the inside surface to cancel it out.
Do note that the charge will be on the inside surface only - never in the middle of the conductor. The same
argument with the small Gaussian surface as before holds: if you enclose only a tiny bit of the conductor,
~ must be zero, which
~ dA
the electric field must be zero everywhere, which means the surface integral of E
means there can be no charge inside the conductor itself.
Now, because this conductor was neutral when we began this experiment, and brought charge inside the
hollow cavity, it must still be overall neutral. Since negative charge has gathered on the inside surface,
the same amount of positive charge must have gathered on the outside surface now!
Whats more, in order to obey the laws of physics, this charge must be uniformly distributed, since the
conductor is spherical. This would not be the case for a non-sphere.
The net effect of this is, then, that since the charge distribution on the surface is uniform, what happens
on the inside of the sphere is completely un-observable to the outside world. We can move the charge inside
the hollow cavity around, and the external electric field will be unaffected.
This effect is known as electrostatic shielding, and the conductor will often be known as a Faraday cage.

2.3

High-voltage breakdown and lightning

We now know that charge distribution is non-uniform on all non-spherical surfaces (surfaces, since we also
know that all charge will reside at the surface for solid conductors).
More specifically, the charge density will be higher at regions of higher curvature, which we will now show.
Imagine two conducting spheres, far apart, but connected with conductive wire. The two spheres are
20

together an equipotential, because of the connection between them.


Lets call one A, with radius RA and charge QA , with the other being B with radius RB and charge QB .
Because of the large separation between the two, the potential around A is not dependent upon B. That
is, the work required to bring a charge from infinity to A is essentially the same with or without B.
Therefore, we can calculate the potential of A to be:
QA
40 r
... but since the two are an equipotential, that must also be the potential of B, which also must follow the
same formula itself:
VA =

VB =

QA
QB
=
40 RB
40 RA

Therefore,
QA
QB
=
RA
RB
This must be true regardless of the radii involved, so imagine B having a radius that is 5 times larger than
A. This implies that there is also 5 times more charge on B than there is on A. However, the surface area
of B is 25 times larger than that of A.
The surface charge density is the total charge on the sphere divided by the surface area, so
Q
4R2
For these facts to be able to co-exist - that the surface area is 25 times more, but the charge is only 5
times more, we get that
=

1
B = A
5
merely because of the larger radius of B. Therefore, the smaller the radius, the higher the local charge
density. And, via Gausss law, we can show that this also implies that the local electric field will be
stronger at such points of high curvature.
There results apply to all shapes. An otherwise spherical surface with a small, pointy outwards tip will
have a stronger electric field at that tip, due to the higher charge density.

2.3.1

Electric breakdown

When electric fields become too strong, electric breakdown will occur. This is what happens when you
have two points with vastly different electric potentials, and a spark forms and transfers charge, while also
creating sound and light (the spark).
It is fairly easy to see why this happens. Imagine an electron traveling in an electric field. It will travel in
the opposite direction of the fields lines (since it is negatively charged).
It will accelerate in the field, and it will collide with molecules (nitrogen, oxygen etc.) in the air in travels
through.
When the kinetic energy of the electron becomes high enough to ionize molecules in the air, there will now
be multiple electrons traveling. They, too, accelerate in the electric field, ionize further molecules, etc.
If this keeps up, as it will if the electric field remains, a conductor is formed from the ionized molecules and
electrons (which now make up a plasma). Charge is transferred, and as the molecules recombine to form

21

neutral atoms/molecules again, they will emit photons, which is why we see a spark. They also produce
a pressure wave, which is why we can also hear the sparks.
Because of what we discussed above, this will typically occur at sharp points, where the electric field is
the strongest.
Very roughly, this breakdown voltage - at room temperature, with dry air, at standard pressure - is about
3 MV/m. So to produce a spark 10 cm long, for example, you would need a tenth of those 3 million volts
per meter, i.e. 300 kV.

22

Chapter 3
Week 3
3.1
3.1.1

Capacitance and Field Energy


Field Energy

Say we have a system with two large plates, separated by a distance h. One has charge +Q = +A, where
A is the surface area, and the other the exact opposite charge Q = A (A is the same for both plates).
Now suppose we move one of the the plate a distance x, so that they are further apart. The plates attract
each other, so its clear we must do work in order to separate the two. The work required to separate them
is the force required times the distance the force acts,
W = |F~ | x
since the electric field is constant.
Finding the force required is simple, yet a bit tricky. We know that the electric field strength between the
plates is

0
but we have to keep in mind that while the plate will be very thin, it does have a nonzero thickness.
We know that charge will gather on the surface, but in reality, that too cannot be of zero thickness.

, half of which is due to the bottom plate. However, the electric


0
field inside the plate is zero - it is a conductor, so that must be the case.
Because of this, the electric field we need to use in our calculation is only half of what we might expect:

So, the electric field below the plate is

1
F = QE
2
Therefore, the work is
1
QEx
2
We know that Q = A, so lets substitute that in there:
1
AxE
2
If we multiply by 00 , i.e. 1, we find another interesting result:
1
0
AxE
2
0

23

We now have another

0

= E in there, so we can simplify the expression to:


1
AxE 2 0
2

Now we note that Ax is the new volume that we have created an electric field in. Remember that A is the
plate area, and x the distance we moved it.
Therefore, we can find that the work required per unit volume is
W
1
= 0 E 2
volume
2
... which is known as field energy density.
J
Clearly, the units for this would then be joules per cubic meter, 3 .
m
This result is valid for all charge configurations, not just the one system we just calculated. Therefore, if
it is more convenient for us, we can now use this instead of calculating the electric potential energy due
to each point charge alone, by integrating over all space:
Z
1
0 E 2 dV
U=
2
... where V is for volume, not the electric potential!
Lets calculate the electric potential energy for the plates above - which by the way is a parallel plate

in there:
capacitor. We have the integral we require just above; lets substitute in E =
0
Z
1 2
U=
0 ( ) dV
2 0
Because we know the volume of the box to be Ah, and we know the electric field outside the plates to
be zero (the plates have exactly opposite fields, so they cancel out outside of the plates), we dont need to
integrate, but get simply
1 2
1
Ah
U = 0 ( )2 Ah =
2 0
2 0
We know since before that
Q = A
V = Eh
... where V is now the electric potential again. Sadly, we use the same symbol for potential and volume.
We can use the first equivalency to get Q in there:
U=
And since E =

3.1.2

1
Qh
2 0

, we can get rid of that and the h by using V instead:


0
1
U = QV
2

Capacitance

Lets now introduce the concept of capacitance, or for short, C (not to be confused with the unit coulomb
- just as W means both work and watt, despite work being measured in joules (watt-seconds)).

C=
24

Q
V

The unit of capacitance is the farad, F, named after Michael Faraday.


If we charge two objects up to have the same electric potential, then the one with the most charge on
it has the greatest capacitance. Thus the name - capacitance is the capacity to hold charge for a given
electric potential.
Using the definition of capacitance above, we can easily calculate the capacitance of a conducting sphere:
C=

Q
=
V

Q
Q
40 R

= 40 R

Since 40 1011 (order of magnitude-wise), clearly capacitances will be extremely low even if the radius
is very high. For example, for a sphere of radius 1 meter, the capacitance is only on the order of 10
picofarads. An electrolytic capacitor, cylinder-shaped (the outside, but thats not how it works - more on
that later) with radius 1 cm and height 4 cm can have a capacitance about ten million times more than
that!
Lets now look at the case of two spheres A and B, side by side. Sphere A has charge QA and potential
VA , while sphere B has charge QB and potential VB .
The spheres are close to one another in space. What is the capacitance of B?
Well, we have the definition:
QB
VB
However, keep in note what the potential VB really is. It is the work per unit charge you have to do to
bring a charge from infinity up to the sphere. However, since A is nearby, that will charge the situation!
Carrying a positive test charge moving towards B, you will have to do work to overcome the electric force.
However, the negative sphere A will be attracting you at the same time, so the work you have to do
becomes less! Therefore, by definition, the potential of B has gone down - and from the equation above,
clearly, the capacitance of B has gone up!
CB =

Calling it the capacitance of B is a bit of a misnomer, its really the capacitance of B in the presence of
A.
Therefore, we will change the definition of capacitance to be inversely proportial to the potential difference
between two conductors:
Q
V
... where +Q is the charge on one conductor, while Q is the charge on the other one.
Lets now calculate the capacitance of two concentric hollow spheres.
C=

We know the electric field due to the inner sphere:


~ =
E

Q
r
40 r2

... and also know that the contribution to the electric field inside the larger sphere, from the larger sphere,
is zero.
Therefore, we use the integral for a potential difference over this electric field:



Z R2
Z R2
Q
1 R2
Q
1
1
dr
~
~
VR1 VR2 =
E dr =
= =

40 R1 r2
r R1
40
R2 R1
R1
Flip the order to get rid of the minus sign:

25

VR1 VR2

Q
=
40

1
1

R1 R2

We can now get find the capacitance C by dividing Q with the potential difference above:
C=

Q
= 40 
V

1
1
R1

1
R2

 = 40

R1 R2
R2 R1

The last step is simply a simplification to make the expression easier to read, i.e. without nested fractions.
Lets now go back to our parallel plate capacitor, and calculate its capacitance - now we know to use the
potential difference between the two plates.
We had that
Q
V
Q = A

E=
0
... and V will as usual be the integral of the electric field between the two plates. However, the field is
constant, and so we can multiply the field times the distance instead of a hairy dot-product integration.
Lets call the separation between the two plates d:
C=

C=

Q
A
=
V
Ed

Lets substitute the value for E in there:


C=

Q
A0
=
V
d

So now, the sigmas cancel:


A0
d
The capacitance is linearly proportial to area, and inversely proportial to d. The last part goes together
nicely with the spheres - the closer they are, the more one will affect the work required by the other. Closer
together means lower potential difference, which means higher capacitance - since capacitance is inversily
proportional to the potential difference.
C=

3.2

Dielectrics

Lets now go back to our good old parallel plate capacitor. We will add a new twist to it shortly, but to
begin with, its the same, only with some new namings.
We have two large plates, each with area A, separated by a distance d. We put charge +Q on the top
plate, and thus get a charge Q on the bottom plate (if we do this the easy way, by connecting a voltage
source across the plates).
f ree
We call the charge density on the plates f ree , and the electric field between the plates Ef~ree =
.
0
The electric field is in the downwards direction, since the top plate is positive, and the bottom plate is
negative.
Now that we have changed the plates with a voltage source, we disconnect the voltage source, thus trapping the charge on the plates. Q is now a constant, no matter what we do - except connect the plates to
26

a conductor, of course. Things that we still can modify, however, include the electric field strength, the
capacitance, and the potential difference between the plates.
Now, lets insert a linear dielectric material between the two plates - remember, with the voltage source
disconnected, and the charge trapped. For fairly long-winded reasons I wont reproduce here (see the book
on polarization and dielectrics), this will cause a negative induced layer of charge on the bottom on the
top plate, and a positive induced layer of charge on the top of the bottom plate. That is, the charge will
be on the inside of the plates, close to the empty space between them.
We can call this charge density ind , for induced; thus the top plate, with charge +f ree , will also have a
charged ind , while the bottom plate will have f ree and +ind .
This induced charge will produce an electric field, of course. Because the induced charge on the bottom is
positive, and the induced charge on the top is negative, the electric field Eind will be in opposite direction
of Ef ree , which was due to the charge we put on the plates. Therefore, the net electric field between will
be lowered, since this induced-charge electric field cancels out part of the original field.
The net electric field will as usual be the vectorial sum of the two; since they are exactly opposite in
direction, and its a fact that the induced field will be smaller in magnitude than the original one, we get
that
E~net = Ef~ree E~ind
The induced charge will be some fraction of f ree . If we call it b, we have that
b f ree = ind

(b < 1)

Therefore, we also have that


b Ef~ree = E~ind
It follows that
E~net = (1 b)Ef~ree
1
1
We call that 1 b = (kappa) or . is known as the dielectric constant. It is a dimensionless number,

K
and is 1 for a vacuum. It is greater than one for non-vacuums.
will be greater for materials which are themselves dipoles (even in the absence of electric fields), than
for materials which will contain induced dipoles from the electric field.
Another way of writing the previous equation, then, is
~
~ = E~net = Ef ree
E

~ will now refer to the net electric field, i.e. the original field minus the
From here on, in this section, E
induced field, to keep things simple (as these are mostly lecture notes, and thats how Walter Lewin did
in lecture).
All in all, we see that by inserting a dielectric material between the plates, with the power supply disconnected and the charge trapped, the electric field strength has gone down by a factor of . Since V = Ed
in this case of a constant electric field, and the distance d between the plates has remained constant, then
the potential difference must also have decreased by a factor of .
What about Gausss law, then? Well, we have that

27

P
Qins
~
~
E dA =
0
P
How does this change in our situation now? Well,
Qins is the net charge inside, such that a proton and
an electron would come out to 0, as the cancel each others charge out exactly.
The charge that is relevant in this case, then, is Qnet = Qf ree + Qind , where Qf ree is positive and Qind is
negative.
Theres a very easy way to take this into account: the factor perfectly captures the amount of charge
that is cancelled out or induced, so:
P
{
Qf ree
~ =
~ dA
E
0
is all we need to calculate.
We can also write it as
{

~ =
~ dA
E

~ =
~ dA
D

Qf ree

where  = 0 is called the dielectric permittivity (compared to 0 which is the permittivity of free space,
also known as the permittivity of the vacuum).
Lastly, we can write it as
X

Qf ree

~ = 0 E
~ is called the electric displacement vector.
where D
We will now do four experiments, each with a notable difference from the previous, while still being very
similar.
First, lets reproduce a few equations that will be necessary here:
Q
V
V = Ed
C=

Experiment one: disconnected power supply, no dielectric


We charge two circular plates, with an initial separation of 1 mm, to a potential difference of 1500 volts.
Once charged, we disconnect the power supply, so that the charge is trapped on the plates.
We then increase the separation from 1 mm to 7 mm. The question is: what will happen to the electric
field, and what will happen to the potential difference?
The electric field must remain unchanged, because E =

, and is constant since the charge is trapped


0

on the plates!
For the same reason, however, V must increase by a factor of seven! Note that V = Ed; with E held
constant, and d increasing, V must clearly increase!
This is also exactly what the experiment shows when the plates are separated: the potential increases, but
that is the only change.

28

Experiment two: disconnected power supply, dielectric inserted


We start this experiment where the previous one ended; at d 7 mm and V 10500 volts (7 times higher
than the initial 1500).
Now, we insert a dielectric - a glass plate, with a of roughly 5.
What happens with the free charge, the E-field, and the potential difference between the plates?
Back to the equations!
First off, Qf ree is still trapped and so cannot change this time around, either. However, there will be an
induced E-field from the induced charge, and so the E-field will decrease - by a factor of .
With V = Ed, and E decreased with d held constant, clearly V will also decrease by a factor of .
Q
The most interesting, and useful, change is with the capacitance, however! Since C = , and Q is held
V
constant while V decreases, C must increase by a factor ! Thus, capacitance as related to the geometry
of the parallel plate capacitor must be amended by this factor:
C=

0 A

Experiment three: power supply remains connected, no dielectric


We start over like experiment one: 1 mm separation, 1500 volts, no dielectric; however we leave the power
supply in there the entire time.
Because the power supply is now connected, with a fixed potential difference of 1500 volts, the results of
this experiment will be very different from the first one. With V fixed, we expect that other parameters
will have to change than ones that did last time.
Again, V = Ed, so when V is fixed and d is increased, E must clearly decrease! As for the capacitance,
the only variable that changes in the above equation is d, so the capacitance must also decrease by the
Q
or equivalently Q = CV , with V fixed and C decreasing, Q must
same factor. Lastly, because C =
V
decrease! This is now possible, since the power supply is connected and allows for charge to move.
Experiment four: power supply remains connected, dielectric inserted
We now move to the fourth experiment. We start where we left off in experiment three. 7 mm separation,
1500 volts with the power supply connected, and the glass plate is inserted between the plates.
We take a final look at the equations to predict what will happen. The capacitance will clearly increase
by a factor , as capacitance is only related to plate area, separation between plates and . The latter is
the only which changes, from roughly 1 (the of vacuum, and air is extremely near 1 as well), to roughly
5, the of glass.
V
What about the free charge? Well, first, lets look at the electric field strength. V = Ed, and so E = .
d
V is held constant by the power supply, d is constant, and so E must remain constant.
However, we know that inserting a dielectric will cause induced charge to appear on the plates, which
lowers the net electric field by a factor !
For these two facts to be able to coincide, the free electric field must increase in order to compensate
for the induced, antiparallel electric field, in order to keep the net field strength exactly the same as before.
Therefore, the free charge will increase by a factor of , to balance out the reduction in field strength that
would otherwise occur.

29

3.2.1

More on capacitors

If we want to design a capacitor with a very high capacitance, how should we proceed?
Well, clearly, from the parallel plate capacitor equation, we want to maximize the area A, minimize the
distance d between the plates and maximize the dielectric constant . Those are all the variables in the
equation, so we cant do much more!
Lets compare two different 100 microfarad capacitors. One is rated for 40 volts, and the other for 4000
volts(!). We use polyethylene as a dielectric; it has a of roughly 3, and can tolerate roughly 18 MV/m
before breaking down.
Because of this breakdown limit, the only way to make very high-voltage capacitors is to increase the
dielectric thickness d (which is then also the plate separation distance). However, doing so reduces the
capacitance, so you need to increase the plate area to compensate! For this reason, high-voltage, highcapacitance capacitors are physically very large. The 4 kV, 100 F capacitor mentioned looks like a car
battery, while the 40 V one is roughly 1 cm3 in volume!
Lets do a back-of-the-envelope calculation. Say we have a plate area of 1 m2 (for easy numbers sake, but
since the plates can be rolled up, its not an extreme value as it sounds, either), with a separation of 0.01
mm, with air between the plates. The capacitance is, according to the capacitance equation, roughly 0.88
F, and the maximum voltage before breakdown, at 3 MV/m for air, about 30 volts. In practice it will
probably be less due to imperfections etc, but this is a back-of-the-envelope calculation!
Lets now insert a dielectric of = 3 between the plates, which can also withstand about 18 MV/m before
breakdown. First, multiplies the capacitance by three, so if we are happy with the capacitance, we can
1
immediately reduce the plate area by a factor of to retain our 0.88 F. Thus the area is now m2 vs
3
the old 1.
Second, note that the dielectric can take a lot more than the 30 volts at the same distance now. If were
happy with the 30 volts maximum, we can reduce d by a factor of 18
(new / old breakdown limit), from
3
0.01 mm (10 m) to 1.666 m. The new maximum voltage will still be 30 volts, since we reduced the
distance by the same factor.
However, now that d is 6 times smaller, we can reduce A by a further factor of 6, getting it down from
1 initally to just 0.055 m2 now, without any sacrifice in capacitance or voltage tolerance! All we did was
insert a dielectric and reduce the distance - which we couldnt have done without the dielectric. Clearly,
dielectrics are very important for capacitors - especially when you want higher values of capacitance,
perhaps in the millifarad range and above, without making truly enormous (car battery-sized or even
larger) capacitors.

3.2.2

A bit on van de Graaff generators

Imagine a fairly big, hollow, conducting sphere. We can use a high-voltage power supply, say 3 kV, to
bring charge on the sphere: we charge a small conductor, bring the two in contact, and repeat the process
until no charge is transferred any longer.
Clearly, that point will occur when the large sphere has the same electric potential as the smaller conductor. If we want to charge the big sphere, the van de Graaff, to say 200 kV, this method is useless.
What we can do instead is to charge the small conductor in the same way, but instead bring it inside the
large, hollow sphere, through an opening at the bottom.
30

The electric field inside will always be zero - not quite, since its not a perfect sphere, but close enough.
Even when the potential at the outside of the sphere and the small conductor are equal, we can still
transfer charge to the inside of the large sphere!
We can use this method to charge the van de Graaff to any voltage we want, until it goes into electric
breakdown, which we cant really prevent. Breakdown is what will ultimately limit the amount of charge
we can get on the sphere, and thus ultimately the voltage.
So, how does a real van de Graaff generator work? Clearly, it is not through a person manually moving
charge!
First, there is a motor that drives a belt. The belt goes from the outside of the sphere all the way inside.
We put charge on this belt, using two sharp tips that cause corona discharge. The charged belt travels up
inside the sphere, where another two sharp points through corona discharge move the charge to the inside
of the hollow sphere. Of course, the charge will move to the outer surface almost instantly, thus charging
the sphere a bit more.

3.3

Current, resistivity and Ohms law

Current is a flow of electric charge. By convention, we say that if a current goes towards the right in a
conductor, positive charges would move towards the right. Negative charges would then move towards the
left, in the opposite direction of the current flow.
While this causes some confusion, electron flow is the most common form of current. Thus, when we
talk about a current flowing towards the right in e.g. a circuit diagram, the electrons are in fact moving
towards the left. However, most of the time, it doesnt really matter in which direction the electrons flow,
as long as we are consistent about this.
This convention where the electrons flow opposite to the current is called conventional current. The other
convention, where they flow in the same direction, is called electron flow.
We will stick to convential current in these notes, however.
Electrons are the main charge carrier since there are free electrons in conductors, which will move when
we apply a potential difference over a conductor. Such a potential difference will cause an electric field
to appear inside the conductor, and the electrons will attempt to move in order to neutralize the electric
field (since a force will be applied to them).
Now, lets do some dirty math to try to derive Ohms law. This will be a quick and likely hard-to-fullygrasp explanation, as its overly simplified and rushed through. Apparently quantum mechanics is required
for a proper derivation.
In copper, at room temperature, the average speed of the free electrons is the metal is on the order of 106
meters per second; this is a random, thermal motion due to the temperature.
The time between collisions between the atoms and the free elecrons is roughly = 3 1014 ( is the
Greek letter tau).
The number of free electrons is roughly n = 1029 per m3 ; roughly one per atom.
When we apply a potential difference to the conductor, the free electrons will experience a force of magnitude F = eE, where e is the magnitude of the electrons charge (the elementary charge e), and E is the
electric fields magnitude. Via Newton, we know that the electrons will now experience an acceleration of
eE
F
=
, where me is the mass of the electron. Because of the accelerations, they will pick up an
a=
me
me
average speed between the collisions, of vd = a , where vd is known as the drift velocity.
If we combine the two equations, we have
31

eE

me
If we use this equation to calculate the drift velocity in a 10 meter long copper conductor, with a potential
difference of 10 volts (and thus an average electric field of 1 volt/meter), we get a drift velocity of roughly
vd =

(1.6 1019 )(1)


(3 1014 ) 0.0053 m/s
9.1 1031
That is in fact not a miscalculation - the drift velocity will actually be on the order of millimeters per
second! The random thermal motion causes the electrons to move at a million meter per second, and our
1
m/s!
electric field of 1 volt/meter doesnt even break
100
Of course, without the electric field, the net movement of the electrons would be zero - the motion is
random, and so you would expect that on average, they return to where they were.
vd =

Lets imagine now a wire, shaped like a cylinder. It has cross-sectional area A, length `, and we apply a
potential difference V such that the electric potential is higher on the right side.
This will cause a current I (the symbol for current is I) to travel in the leftwards direction, which is also
V
the direction of the electric field E = . Of course, the electrons will travel towards the right, opposite
`
to the electric field vector.
In one second, they will travel a distance of vd meters to the right, by our definition of the drift velocity.
Lets look at how we can calculate the current in terms of the drift velocity and such things.
If we take a cross-section of the conductor, with area A, the volume that moves through per second wil
be vd A, since vd is the length the electrons move per second. We multiply that by n (free electrons per
cubic meter) to get the number of free electrons that pass through per second. We then multiply that by
e, the magnitude of the electrons charge, to get the total charge that passes per second.
Therefore, the current that flows is the product of the above terms:
I = vd A n e
We found vd previously, so we can substitute that value for it in there:


e2 n
eE
Ane=
AE
I=
me
me
The first term there, in the fraction, depends only on the conductor (for a given temperature). We denote
that by , and call it conductivity.
=

e2 n
me

A
S
we be roughly 108 for copper at room temperature, and the units are
, or equivalently,
(siemens
V m
m
1A
per meter), as 1 S =
.
1V
V
Because E = , we can also write the current as
`
e2 n
AV
I=
AE = AE =
me
`
We can solve that for V , as well:
V =

`
I
A

32

`
... which is, in fact, Ohms law, since R =
.
A
1
We will often use = , which is then known as the resistivity of a material. ( is the Greek letter rho.)

Note from the above equations that the current (for a given potential difference) is proportional to the
conductors area, and inversely proportional to its length. This makes a lot of sense if we use the common
analogy of current being water, electric potential being water pressure, and resistance being, well, the
resistance a pipe causes. Clearly, a longer pipe will require more pressure to maintain the same current
(current is inversely proportional to `). Also, a wider pipe will allow more current to flow (current is
proportional to A).
Lets move away a bit from all these equations and their symbols, and put some numbers in there, instead.
Say we have a chunk of copper 1 mm x 1 mm x 1 m in size. The cross-sectional area is then 106 m.
1
= 102 .
What is the resistance? Well, we ` and A. is roughly 108 , so the resistance R is about 8
10 106
What about insulators? A good insulator - example include glass, quartz, rubber and porcelain - can have
resistivities as high as 1012 to 1016 , with conductivities () of one over those numbers.
For example, if the material had = 1014 (equivalent to = 1014 ), the resistance R would be about
1020 ! With such high resistances, clearly only extremely small currents will flow even for very high
voltages.
Ohms law has many downsides. Its only valid for a subset of circuits elements, and only works in the
ideal case.
The conductivity (and thus the resistivity ) is a strong function of temperature. If the temperature
increases, then the random, thermal motion of the electrons will increase. In turn, , the time between
collisions, will go down. If we have a look at the definition of :
e2 n
me
its clear that this means that the conductivity will go down if the temperature goes up. This has some
very big ramifications.
As a real-world example, we can take a simple light bulb. Consider a small light bulb; it has a 50 ohm
resistance - when hot. When cold, the resistance is closer to just 7 ohm. What happens now? Well, the
V
instant we switch the circuit on, the bulb is cold, and the current will be I = . Say V = 12 volts. In that
7
case, the power dissipation (which we havent discussed yet in this course) will be 20.5 watts. Considering
the tiny size of the light bulb in the example, that is a lot of power. It cant possibly keep that up, as it
will get too hot.
Instead, what happens is that the power dissipation will heat the bulb up extremely quickly, and so the
V
resistance will increase to the 50 ohms in about a second, if not even less. At that point, I = , and
50
for 12 V, the power dissipation is now 2.88 watts, a much more reasonable figure.
=

Clearly then, the voltage-current relation of this light bulb is far from linear! It will be fairly linear when
the bulb is hot, but if the voltage were to increase or decrease, then the bulb would either heat up or cool
down, and so the curve would yet again be a function of temperature and non-linear.

33

Chapter 4
Week 4
4.1

Batteries and EMF

If we draw up a very simple circuit with a voltage source connected to a resistor, we find that the current
(conventional current) from the + side of the supply, through the resistor - in through the positive side of
the resistor and out the negative, and back into the power supply.
The electric field always points from a higher potential to a lower potential, so in the resistor it points
along with the current.
What about in the power supply? Well, we just said that the electric field always points from a higher
potential to a lower, so we find that the electric field opposes the current! The current must enter the
terminal and flow out of the + terminal, while the electric field must point from + to !
Therefore, there must be a mechanism which forces the current to run despite the opposing electric field;
a mechanism which does work to overcome the electric force.
Lets consider a chemical battery. We have two plates, one zinc (Zn) and one copper (Cu) in a solution of
sulfuric acid (H2 SO4 ).
In this solution, ions will form: Zn++ , Cu++ and SO
4 .
We connect this battery to a resistor R, and a current flows. If we now look at the current direction
and the electric field direction, we will find the same result as before: the align in the resistor, but are in
opposite directions inside the battery. So, again, the charge carriers in the solution must flow in a direction
that would require work to be done.
Why does this happen? The answer is that in doing so, they participate in a chemical reaction which will
net more energy than is spent on fighting the electric force.
When measuring the potential difference of an open circuit, such as an otherwise unconnected battery, we
call that voltage the electromotive force, or EMF (sometimes in lowercase, so emf), for which we use the
symbol E, so a curly E, a bit like a large lowercase epsilon, but not quite.
Since all materials (except superconductors, which we may encounter later on) have a nonzero resistance,
the battery will have an internal resistance. Therefore, if we short-circuit a battery - even with a superconductor of 0 resistance, the current that flows will be limited by the internal resistance, such that
E
where ISC is the short-circuit current.
ISC =
Rint
When we draw a current from any power source, the voltage as measured across its terminals will drop,
due to the internal resistance. Via Ohms law,
E = IR = I(Rext + Rint )
Via Ohms law, there will be a voltage drop across the internal resistance,
34

VRint = IRint
and so the potential difference that reaches the external resistor (note that voltage doesnt flow) is
VRext = E IRint
The higher the current, the lower the potential difference. For this reason, if you buy a 9 volt battery, the
open circuit voltage will likely be a bit higher than that (when it is new). Under relatively heavy load, the
batterys internal resistance will get a voltage drop, and so the voltage seen by the external circuit may
well be less than 9 volts.
In a similar manner, if we short-circuit such a battery, then clearly the entire 9 volts will be across the
internal resistance, causing a large current, and lots of heat production.
V2
, so for V = 9 volts and R perhaps on the order of 2 , the
The power is given by P = V I = I 2 R =
R
current will be large, as will the power dissipation and thus heat generation be.
Power is given by energy per unit time, and so the units are joules per second, or watts (W).
From experience, I can recommend not shorting 9 volt batteries for a long time - they get hot, the outside
starts melting, and they smell awful! Other than that, its relatively safe, however, unlike other battery
chemistries - lithium-ion batteries can cause fires and explosions in the worst case scenario.
Now lets look at a car battery. Such a battery would usually be a lead-acid type; they have lead and
lead-oxide plates in a solution of sulfuric acid. They are nominally 12 volts, but usually a bit higher in
practice, especially when the car is running and the battery is being charged.
Such a battery has a very low internal resistance, perhaps on the order of 20 milliohms. Thus the maximum
V2
7200
current, if short-circuited, is on the order of 600 amperes, which would cause a power output of
R
watts inside the battery!
Thats several times more power than a space heater, which might be 2 kilowatts or so. Needless to say,
short-circuiting a car battery may be a very bad idea.

4.1.1

Kirchhoffs rules

A set of very useful rules - sometimes called laws, but like Ohms law they are not always valid - are called
Kirchhoffs rules. The first, KVL for Kirchoffs Voltage Law, states that
I
~ =0
~ d`
E
Or, in plain English, that the potential difference as measured when starting at a point, moving around a
loop, and coming back to the same point, must be zero. This is always true for a conservative field, but
is not always true - time-varying magnetic fields are one thing that will make this untrue, as we will see
later on.
We can also state this in summation notation:
n
X

Vk = 0

k=1

This rule is very useful for analysis of DC circuits, where we can pretend that it is always valid. It holds
for all loops that you can choose whereby you end up where you started.
Kirchhoffs Current Law, or KCL, states that the current that flows into a junction/node must flow out,
i.e. there cannot be a pile-up of charge. Thus if you have a T-junction in a circuit, the net sum of the
35

currents must be zero - either one goes in and two out, or two goes in and one out. There are no other
possibilities.
We can state this using summation notation as well:
n
X

Ik = 0

k=1

4.1.2

Basic circuit analysis

Because this is such a pain to describe without circuit diagrams, and since adding circuit diagrams are an
even bigger pain, I will refrain from documenting this right now. If the course continues on with anything
more than the very basic analysis done in lecture 10, I will change this. If not, see the lecture videos.

4.2

Magnetic Field and Torques

In ancient greece, in the 5th century BC, it was already known that certain rocks attract iron. One of
these minerals is now called magnetite, named after Magnesia, a district where the rocks where plentiful.
This is of course also where the names magnet and magnetism come from.
Much later, these minerals were used to create compasses, 1100 AD, in China. Yet a century later, it was
found that magnets have two points of maximum attraction, known as poles. All magnets found have two
poles; if you split a magnet in two, you get two smaller magnets with two poles each.
We name these poles north and south; the north pole will point (approximately) towards Earths geographic north pole, while the magnets south pole points (roughly) towards Earths geographic south pole.
Opposite poles attract, and like poles repel each other, as with electric charge.
Because of this, the magnets north pole will point towards the Earth south magnetic pole, which somewhat confusingly is located near the Earths geographic north pole.
(The Earths magnetic field shifts with time, and so it is not fixed at the Earths geographic poles.)
Magnetic monopoles - magnets with only one pole - could exist, but despite much research, such a magnet
has never been observed to exist.
As with electric fields, we can visualize magnetic field by using field lines. Unlike electric fields, however,
all known magnets are dipoles. Therefore, all field lines will begin at the magnets north pole and end at
its south pole.
With electric field lines, the meaning of the field lines are that at each point, the lines are tangent to the
force a positively chargd particle would experience.
For magnetic field lines, every point is tangent to the direction in which a compass needle would point.
Thus we can trace out magnetic fields with compass needles, if they are large enough. At the very least
its a useful way to think about them.
In 1819, Danish physicist Oersted discovered that magnetic needles move in response to electric current,
thus linking electricity with magnetism, in the first step towards a theory of electromagnetism.
N
If a current moves into this page (denoted by , which looks like the rear end of an arrow), the magnetic
field created J
will be in a clockwise circle around the wire. If a current were to move out of the page
(denoted by , which looks like an arrow coming towards you), the magnetic field created would then
be counter-clockwise. As we will soon see, magnetic fields are always perpendicular to the current that
creates them.
Well, if a running current causes a magnet to move by exerting a force, as Oersted showed, then by Newtons third law, the opposite must also be the case. If we run a current through a wire that is placed in
36

an external field, the wire will experince a force. The direction of this force is given by the cross product
where B
~ is how we (always) denote the magnetic field. (In this instance, we used the unit
F = I B,
~
= B .)
vector B
~
|B|
If we run a current through a wire towards the right, like so: I1
then the magnetic field produced would, below the wire, be going into the page, and above, be coming out
of the page.
We can use the right-hand rule, used for determining the direction of cross products, here. Or, rather, a
related rule. If you point your right thumb in the direction of the current, with your remaining fingers
slighly curled, then the other fingers will show the direction of the magnetic field. Try it out with your
right thumb pointing towards the right, and youll find that indeed above the wire, your fingers point
towards you (out of the page), while the hand below the wire points into the page, and away from you.
If we add a second current I2 in the same direction, in a wire just below I1 , the magnetic force between
the two wires will attract the wires. We can convince ourselves of this by using the above finger curl rule
again.

4.2.1

The B Field

So how do we define a magnetic field? Unfortunately, since we have no magnetic monopoles, we cant use
~ as we might like to.
a definition such as F~ = q B
Instead, the magnetic field is described in terms of how it acts on moving charges:
~
F~B = q(~v B)
where is the cross product (vector product). Due to the nature of the cross product, this result is not
only always perpendicular to te velocity of the particle, but also always perpendicular to the magnetic
field, which is at least to me is unintuitive.
The above result is sometimes considered describing the Lorentz force, but that same term often refers to
the sum of electric and magnetic effects on a charged particle.
The equation is sign-sensitive: if any of the three terms is negative, then the force flips over 180 degrees.
(Of course, if exactly two are negative, the minus signs would cancel out.)
Via the above equation we can derive the units of the magnetic field strength:
N = C(m/s B)
where B is a temporary name for the magnetic field units. We can simply solve for B:
N s
= tesla
C m
This interesting-looking unit is called the tesla, and has the symbol T. The tesla is a very large unit;
for example, the Earths magnetic field is on the order of 50 T. The gauss (symbol G) is also often used,
despite being a non-SI unit. They are related such that 1 T = 104 G, so 1 G = 104 T.
B=

Because the magnetic fields force is always perpendicular to a particles velocity vector, the magnetic
field can never do any work on a moving particle! Work is defined as the dot product of the force and
the displacement, and with an angle of 90 degrees between them, the cosine term in the dot product will
always be zero.
This means that the magnetic force cannot change the kinetic energy of a particle - it cant increase it,
and it cant decrease it. It can and will force the particle to change directions, however. This fact is
exploited in CRT monitors and television sets - which are obsolete since about a decade. Such monitors
work by firing an electron beam towards a set of phosphors. The beam is scanned over the whole image
37

multiple times a second, and oriented by using magnetic fields to force the particles to where you want them.
The total electromagnetic force on a moving, charged particle is given by
~ + ~v B)
~
F~ = q(E
... which is, as stated above, also known as the Lorentz force. These notes will either follow the convention
of the lectures, if there is one used often enough to be noteworthy, or it will call this total force the Lorentz
force.
Lets now return to looking at the force on a wire with a current through it.
Say that we have a charge dq moving in a wire. It moves forward with the drift velocity, vd (rather, the
electrons move backwards with that velocity, but we can pretend its positive charge moving forwards for
mathematical simplicity, as it all works out the same). We call this current I, and it runs through an exter~ which may vary at different points of the wire. We call the angle between v~d and B
~ .
nal magnetic field B,
Well, if we keep working with the differential charge dq, we get that the magnetic force on the charge is
~
dF~ B = dq(v~d B)
We also know that, by definition
dq
dt
Therefore, we can substitute that into the previous equation, and get
I=

~
dF~ B = Idt(v~d B)
Whether we multiply I by dt, or do so to v~d doesnt matter; mathematically, it is the exact same thing.
We can therefore think of this as
~
dF~ B = I(v~d dt B)
We now have velocity multiplied by time, which is simply a distance. Therefore, we can substitute the
~
drift velocity for a chunk of the wire, d`:
~ B)
~
dF~ B = I(d`
Of course, to get the total force on the wire, we need to sum up all of these tiny segments, which means
integrating the above:
Z
~ B)
~
F~B =
I(d`
wire

In the special case that the magnetic field is constant, at least over a portion of the wire, we can do the
above calculation with simple multiplication of the three terms instead:
|F~B | I`B

4.3

Review for Exam 1

Say we have a solid cylinder, with uniform charge distribution (in 3 dimensions!) coulomb per cubic
meter, and radius R.
What is the E-field outside the cylinder (r > R)?
Lets see. We use Gausss law, as always:
38

Q
0
A would be the surface area of the sides, so 2r`. Q meanwhile we need to find by multiplying the volume
with the charge density:
EA =

Q = R2 `
If we merge the two equations, we have
R2 `
0

2r`E =

R2 `
E=
0 2r`
E=

R2
20 r

Next up: what is the electric field inside (r < R)?


Since the charge distribitions is uniform, and not just all on the surface, well treat this as a non-conductor.
Therefore the answer isnt immediately zero.
Well, well want to apply Gausss law again, of course. The left-hand side will be the same, only that
r < R this time. The right side must change, however: the amount of enclosed charge will change. The
enclosed charge will now be given by the radius of the Gaussian cylinder, rather than that of the real,
physical cylinder:
2r`E =
E=

r2 `
0 2r`

E=

4.3.1

r2 `
0

r
20

Thats it

Everything else covered this lecture has already been covered in these notes. While watching it again is
helpful, I dont see how reading two sets of notes, with the same content, is useful. Read the rest of the
notes twice, instead!

39

Chapter 5
Week 5
5.1
5.1.1

Moving Charges in Magnetic Fields


Moving charges, radii and special relativity

As we saw in lecture last week, the magnetic force will act to charge the direction of a moving charged
particle. The magnetic force will be in a direction perpendicular to the particles velocity, and also perpendicular to the magnetic field itself.
If we repeat the calculation of the new velocity vector over and over, as the charge moves small distances,
we will find that the particle travels in a perfect circle - given a uniform magnetic field, and no other forces
being involved.
From classical mechanics, uniform circular motion is described by a centripetal force - a force acting on
the particle towards the center of a circle. We can equate the magnetic force on the particle, given by
q(vB), with the centripetal force.
If the magnetic field is uniform, and we choose it such that it is perpendicular to v, the magnetic force is
mv 2
, we can solve for the circles radius R:
given by F~B = qvB. Equating that with the centripetal force
R
mv 2
qvB =
R
qvBR = mv 2
mv
R=
qB
The equation behaves as we would expect: the higher the mass, or the velocity, the greater the radius:
the particles inertia will make it harder for the magnetic force to chance its direction very rapidly.
Likewise, if the charge is high or the magnetic field is strong, the magnetic force will be able to act more
strongly, and so the radius will be low.
We can also write this radius in terms of the potential difference used to accelerate the particle.
Since electrostatic potential energy is given by qV , where V is potential difference, and all such potential
energy will turn into kinetic energy after having accelerated it, we can equate that with the calculation of
kinetic energy:
1 2
mv = qV
2
We can then solve for v, the velocity:
v 2 = (2qV )/m
r
2qV
v=
m
40

... and then substitute that into the equation for the radius:
q
m 2qV
m
R=
qB
Lets simplify that a bit.
Instead of dividing by q and B outside, we can move q 2 and B 2 inside; same with m in front:
s
2m2 qV
R=
mq 2 B 2
After cancelling stuff out, were left with:
s
R=

2mV
qB 2

Of course, since we got this result by using the classical equation for the kinetic energy, this will only
be valid when the particles velocity is much lower than the speed of light. When it approaches perhaps
10% of the speed of light, the result is going to be a bit off. Greater speeds mean much greater inaccuracies.
Within special relativity, a common term is , the Lorentz factor. It is used, for example, to calculate
time dilation, length contraction, and kinetic energy.
It is defined as
1
=p
1 v 2 /c2
... where c is the speed of light in a vacuum, approximately 3 108 m/s. v is as usual the particles velocity.
If 1, then classical physics and special relativity will agree. The farther it is from 1, the greater the
inaccuracy of classical physics.
We can amend the formulae for the particles radius to account for relativistic effects:
mv
R=
qB
s
( + 1)mV
R=
qB 2
In the limit as 1, the above are identical with what we had before, as we would expect.
In special relativity, kinetic energy as related to velocity and mass is given by
!
1
Ke = ( 1)mc2 = p
1 mc2
1 v 2 /c2
In order to find the velocity for a 1 MeV electron, we set the above equal to the kinetic energy of qV , and
solve for v, not a pretty affair:
!
1
p
1 mc2 = qV
2
2
1 v /c
1
qV
p
=1+
2
2
mc2
1 v /c
p
1
1 v 2 /c2 =
qV
1 + mc
2
41

1 v 2 /c2 =
1+


qV 2
2
mc

v 2 /c2 =
1+


qV 2
2
mc
1

v 2 /c2 = 1
1+


qV 2
2
mc

v 2 = c2 1
1+
v
u
u
v = ct 1


qV 2
2
mc
1

1+


qV 2
mc2

I also solved that in Mathematica, which gave me an answer with fewer levels, but equal ugliness:
p
c q V 2c2 m + qV
v=
c2 m + qV
... yeah. The good news, however, is that they both produce correct results.
If we stick the numbers in for a 1 MeV electron, we find that its speed is roughly 2.8 108 m/s, about 93%
of the speed of light. If we do the same calculation using classical physics, we end up with a velocity of
5.93 108 m/s, or about twice the speed of light! Relativity is clearly a must for these speeds.

5.1.2

Isotope separation

We can use what we now know to separate isotopes of chemical elements. Because the chemical properties
of an element is based on the element itself, all isotopes are essentially identical chemically. However, we
can use other means to separate isotopes.
An an example, we will use uranium: separating uranium isotopes was a necessary step to build the atomic
bomb during World War II. (It is still a required step for a uranium-based bomb, of course.)
92
In nature, about 99.7% of all uranium is uranium-238 (238 U196
), meaning it has 238 protons plus neutrons
(92 protons, which makes it uranium, and 196 neutrons). The much more rare isotope of uranium-235
92
(235 U193
, about 0.7% in nature) is the one used for building bombs. When the isotope purity is high
enough, we call that weapons-grade uranium.

If we ionize uranium atoms, and accelerate them over a potential difference, and then send them through
a magnetic field, the radius of the bend depends on each uranium ions mass. Therefore, U-235, which is
lighter, will have a slighly smaller radius than the heavier U-238! How much smaller? Well, they differ in
238.05
1.28%. When accelerated over a constant potential difference, in the formula above we
weight by
235.04

can see that the radius is proportional to m, so we get roughly a 0.6% difference is radius between the
two.
A device like this is known as a mass spectrometer ; it is now one of many ways used to separate isotopes.
This technique, and many others used for isotope separation, also have many peaceful uses. For example,
they are used in medicine, to separate isotopes used for radiation treatment, PET scans, etc.

42

5.1.3

Particle accelerators/cyclotrons

Lets now look at how we can accelerate protons (and other charged particles) to nearly the speed of light.
One way to do this is to use a cyclotron. A cyclotron chamber is made up of two dees, shaped like
the letter D (one of which is mirrored). Together, the dees make a circle. Seen from above, we have a
circle. We add a static magnetic field, coming upwards (out of the page, if seen from above), and inject
a moving, charged particle into the chamber.
We then introduce a potential difference between the two dees, of say 20 kV. There will now be an electric
field between the two dees, and as the particle moves between them, it will get accelerated, and gain 20
keV in kinetic energy. Because of the added velocity, the radius of the particle is now higher.
When the particle reaches the end of that dee, the potential difference is reversed, such that the electric
field is again in the same direction as the particle, and so it is again accelerated and gains 20 keV of energy,
which again increases its radius.
We repeat this process until the proton reaches the outer edge of the dees, after having gained 40 keV per
loop, many times over.
In this process, then, the magnetic field cause the protons to travel in a circle (spiral, rather, as they are
accelerated - but not by the magnetic force), while the electric field accelerates the particle. Both fields
are necessary, but they perform different functions, and only the electric force actually does physical work.
Lets try an example:
A proton is being accelerated in a cyclotron. The radius of the cyclotron is 2 m. The potential difference
between the gaps between the Ds is 50 kV. The uniform magnetic field has a magnitude of 1 T. What is
the maximum energy that the proton can achieve? Ignore relativistic effects.
Since we are to ignore relativistic effects, we use this equation:
s
2mV
R=
qB 2
... and solve it for V:
B 2 qR2
2m
8
If we put the values in, we get that V 1.916 10 volts, or about 192 MV.
By the definition of the electronvolt, the answer is then that 192 MeV is the maximum energy the particle
can achieve in this cyclotron.
V =

Approximately how many rotations of the proton are needed to achieve this maximum energy?
The potential difference is 50 kV, and the proton gains energy twice per rotation, so
192 MV
1920 rotations
2 50 kV
Next question...
How much time does it take a proton with kinetic energy of 5 MeV to go around the cyclotron once? For
simplicity, you may ignore relativistic effects.
1
As usual, we solve mv 2 = Ke for v to find the velocity. The kinetic energy is given as 5 MeV, which is
2
5 106 times the elementary charge 1.602 1019 :
r
r
2Ke
2 5 106 1.602 1019
v=
=
3.095 107 m/s
m
1.672 1031
We then use our old radius equation:
43

R=

mv
1.674 1027 3.095 107
=
0.32341 m
qB
1.602 1019 1

And finally, to calculate the time it takes, we divide the circumference of the full circle with the velocity:
T =

2R
2 0.32341
=
6.566 108 s
7
v
3.095 10

... or about 66 nanoseconds.


How much extra time will it take a proton with energy of 5 MeV to go around the cyclotron once compared
to a proton with energy of 10 MeV? For simplicity, you may ignore relativistic effects.
If we do the above calculation again but for 10 MeV, we find the exact same result. The reason can be
shown with a bit of algebra:
2 mv
2m
2R
qB
=
=
T =
v
v
qB
The velocity cancels. Physically, this is clearly because as the velocity of the particle increases (and as
such the time per loop would go down), the radius goes up by the same factor, so the velocity terms cancel
each other out. This only holds true as long as we ignore relativistic effects, however. With relativity
taken into account, the time taken is
T =

2m

qB

Keep in mind, however, that we need to switch the direction of the electric fields, every time the particle
moves around half the circle! Therefore, the switching needs to be done roughly every 33 nanoseconds in
this case, or at 30 MHz, since
1
1
=
30.3 106 Hz
T
33 109
Also, since the time per loop depends on , at relativistic speeds we need to adjust the switching frequency
as the particle is gaining speed. Such a device is called a synchrocyclotron, so named as it synchronizes
the switching.
f=

Modern particle accelerators are rings, i.e they have a constant radius. Since the particles need to take
essentially the same narrow path both at low and at high energies, the magnetic field is gradually increased
as the particles are accelerated (by the electric fields).
Modern accelerators, as used for cutting-edge physics research (as opposed to e.g. medical use, which use
relatively small accelerators) are emph extremely big. The Large Hadron Collider in Geneva, Switzerland
is so large that it indeed crosses the border to France: its circumference is 27 kilometers! The LHC accelerates protons to energies as high as 7 TeV - 7 1012 eV. While that is just under 1 J of energy, which
sounds tiny, keep the protons incredibly tiny size in mind. The energy is enough to make them move at
99.9999991% the speed of light, or about 3 m/s slower than the speed of light!
The LHC accelerates bunches of protons, with one set going clockwise and the other counter-clockwise;
they are then made to collide with each other - thus the name Large Hadron Collider. (A hadron is a
particle made up of quarks, such as protons.)
E
Via E = mc2 , and thus m = 2 , the extreme kinetic energies of these particles is converted to other
c
particles, some of which are of greater (rest) mass than the protons that supplied the energy. Because of
this equivalency, particle physicists often use eV/c2 as their preferred unit of mass, rather than the unwieldy
keV
kilogram. In such units, an electron has a rest mass of 511 2 . That is, if an electron is converted to
c
44

pure energy, the amount of energy would be 511 keV, or about 8.19 1014 J. Protons have a rest mass of
MeV
938 2 , or about 1.5 1010 J in terms of energy equivalence.
c

5.1.4

Cloud chambers and bubble chambers

Now that we have high energy particles, how can we see the results of their movement and their collisions?
There are many techniques these days, most of which are electronic in nature, but there are very simple
solutions to this problem, as well. Cloud chambers were the first such solution; cloud chambers are in
fact responsible for the discovery of the position (anti-electron), muon and kaon particles, in the 1930-1940s.
We can make the tracks of charged particles visible using a cloud chamber. When a charged particle
moves through air, it creates ions in its path, until it finally loses all of its kinetic energy and comes to a
halt.
A 10 MeV electron can travel about 40 meters in air; a 10 MeV proton just one meter. A 10 MeV alpha particle would only travel about 10 cm. (An alpha particle is a helium-4 nuclei: 2 protons, 2 neutrons.)
A cloud chamber consists of a layer of alcohol, cooled from the underside by dry ice (solid carbon dioxide).
This causes a temperature gradient through the alcohol, and there will be a layer where the alcohol is supercooled/undercooled: that is, the alcohol vapor is cold enough that it ought to be liquid, but something
is needed to trigger the phase change.
The same behavior can be seen with water, going from liquid to solid: if you cool water in an extremely
smooth container, you may be able to get the water to a sub-zero temperature without turning into ice.
All thats needed for it to turn solid almost instantly (a second or so for a bottle of water) is a trigger.
Digressing further, water can also be super-heated, e.g. in a microwave, also in a smooth container: it
can then turn from a non-boiling liquid to boiling/vapor when you insert a spoon or such, and explode in
your face, causing severe burns. This can mostly be prevented by inserting a non-metal object (a wooden
spoon or such) into the liquid before microwaving.
Back to the cloud chamber. Due to this supercooled state, all thats required for the vapor to condense is
a trigger. A charged particle which creates an ion trail is enough, and so firing charged particles into this
vapor will leave a trail of condensed alcohol vapor where the particle has travelled.
When the particles move though this vapor, they will gradually lose speed. If we have a constant magnetic
field throughout the cloud chamber, particles will move in a spiral shape: as they lose energy, the radius
of their circle due to the magnetic force will become smaller and smaller constantly, and so that makes
a spiral where they end up in the middle of the spiral.
As mentioned previously, the position was discovered using a cloud chamber. Carl D. Anderson discovered
in it 1932, by observing a particle that had an electrons mass, and the correct magnitude of charge but the wrong sign. The particle curved in the opposite way an electron would, given the magnetic field
configuration he had.
Bubble chambers are similar to cloud chambers, but in a way they are the opposite. In a bubble chamber,
there is liquid hydrogen, which is hot enough that it should really be in the form of gas. The charged
particles then act as seeds for the gas bubbles, such that the particles leave a trail of bubbles where they
have moved.
Over 30 new particles were discovered between 1958 and 1968, thanks to particle accelerators, cloud
chambers and bubble chambers, in the growing field of nuclear physics.

45

5.2

Biot-Savart Law

Lets now go back to magnetism, and more specifically, magnetic fields due to moving charges (current
in a wire). If we have a current going in an upwards direction though an infinite wire, we know (from
experiments) that the magnetic field is shaped like concentric circles around the wire, with the direction
given by the right-hand rule: curl your fingers, and then point your thumb in the direction of the current.
The four remaining fingers are now curled in the direction of the magnetic field.
Also given by experiment, is that for a current I going through a wire, the magnetic field B is proportional
to the current, and inversely proportional to the distance from the wire:
B

I
R

1
Weve seen before that for an infinite wire of uniform electric charge, the electric field falls off as . The
r
direction of the electric field differs from the magnetic field created by a current, but that is irrelevant for
making this point.
1
The electric field of electric monopoles, i.e individual charges, falls of as 2 , so when we integrate them
r
1
over a wire, the result is the falloff.
r
This suggests, then, that if we had a set of magnetic monopoles on the wire, that the field due to each
1
one would also fall off as 2 . However, magnetic monopoles dont appear to exist - one has never been
r
experimentally discovered.
1
However, it also suggests that if we were to carve this wire, where the total magnetic field falls off as , into
r
~
tiny segments d`, that each element would contribute a small amount that is indeed inversely proportional
to the square of the distance.
~ due to a such a small segment d`
~ is, where r is the unit vector in the direction of the
The contribution dB
point where we measure, is given by
~ = CI (d`
~ r)
dB
r2
... where C is a constant, which we write as
0
4
... where 0 is called the magnetic constant, previously known as the permeability of free space.
Weve seen before that in Coulombs law, electric fields are proportional to some constant as well:
1
Coulombs constant, given by
, where 0 is called the permittivity of free space (or, more recently, the
40
electric constant).
~ can also be written in terms of the displacement vector ~r instead of the unit vector
The small element dB
~r
r. In that case, because r =
, we multiply both top and bottom by the magnitude, and get
|~r|
~ = CI (d`
~ ~r)
dB
r3
0
Integrating over the entire wire, and using C =
, the total magnetic field is given by
4
Z ~

I d` ~r
0
~ =
B
4
r3
C=

46

Using this equation, we can calculate the magnetic field exactly in the center of a circular current loop.
For simplicity, we center the loop in the coordinate system.
A current of 100 A runs through a circular loop of radius 0.1 m. What is the magnitude of the magnetic
field right at the center of the circle?
~ and r will always be exactly perpendicular for a circle, since the point were
Lets look at the integral. d`
interested in is in the middle of the circle. The magnitude of the cross product is
|A B| = |A||B|sin
... where weve just established that will always be 90 degrees, so we can get rid of the vectors, and get
a regular single integral.
Z
Z
1
0 I ~
0
~
I
(d` r) =
d`
|B| =
2
4 r
4
r2
A small element d` along the circle is given by rd (via arc length), so
Z
Z 2
0
0 2
0
1
1
~
|B| =
I
I
d =
I
d` =
2
4
r
4 0 r
4 r
Simplified, and with values, we get
107 4 100
0 I
=
6.28319 104 T
2r
2 0.1
Alternatively, we could make this even easier by realizing that the integral does nothing except find the
circumference of a circle given its radius, so we can jump ahead:
Z
Z

1
0
0 I
0
0
~ r) =
~ =
|B|
(
d`
I
d`
=
I2r
=
4 r2
4
r2
4r2
2r
~ =
|B|

Power transfer and power loss


Say we have a power line - a very, very long cable - with ends A and B, A being to the left. We define
the potential at A as VA , and the potential at B as VB ; there is then also a return current wire, where we
define V = 0 at the leftmost side. (We wont calculate losses in the return cable.)
On the right side, at B, we hook up things we want to power off the electricity grid, such as computers,
fridges, and whatnot.
The cable will have a nonzero resistance, and so there will be a voltage drop over it, given by VA VB = IR,
via Ohms law. The potential at B will then be
VB = VA IR
The power extracted at B is the voltage at that point (with respect to ground) times the current, so
P = VB I. This power is, via conservation of energy you could say, the power provided by the power
station, minus the resistive losses on the way there:
VB I = VA I I 2 R
Since the I 2 R loss means energy is wasted at heat in the cable, which is clearly a waste of energy (and
money), we want to minimize that term. What can we do?
Well, we could reduce R, by making the cable thick - which is expensive. Or we can make it out of
materials that are more conductive than the copper that is most often used - even more expensive. Or we
could use superconducting cable, which requires cooling by liquid nitrogen or even colder temperatures -

47

more expensive yet again.


Lets calculate an example:
Calculate the power loss in a transmission line in an aluminum cable (resistivity= = 2.8 108 m) of
length L = 200 km and cross sectional area A = 10cm2 when VB is 100 V and when VB = 105 V. Assume
we are consuming 1 MW.
The wires resistance is given by
`
A
where ` is the length in meters and A the cross-sectional area in square meters, here 10cm2 = 0.001m2 .
Thus R 5.6.
R=

106
= 104 amperes. I 2 R is then 5.6 108 watts, or 560
We are consuming 1 MW, and VB = 100, so I =
100
times greater than the power we are consuming!!
If VB = 105 V instead, I = 10 amperes, so I 2 R is 5.6 102 = 560 watts, which is just 0.056% of the power
we are consuming.
Note that the power delivered VB I is exactly the same; we have only changes the current-to-voltage ratio
drastically.
Clearly, what we want to do is to make the potential as high as possible, to minimize resistive losses. What
limits us? Well, for one, corona discharge. If the potential at the surface of our cables in higher than 3
MV/m, there will be corona discharge to the air surrounding the cables, which will be a big power drain.

5.3

Amperes law

For a current I going into the page, the magnetic field B formed will be everywhere tangential to
concentric circles around the wire. The magnitude is given by
0 I
2r
Note that while this looks similar, except for the , to the result we got in a Biot-Savart law calculation
earlier, they were different calculations: that one was for the magnetic field inside a circular current loop,
while this is the magnetic field for a straight wire.
B=

This result is also found by integrating over the closed circle:


I
~
~ d`
B
~ is NOT the same as the one for the Biot-Savart law!!!
Note that this d`
~
In that case, d` refers to an infinitesimal wire segment, of the current-carrying wire.
In this case, for Amperes law, is a tiny movement along a magnetic field line surrounding the wire.
~ are always parallel, the result of the integral is simply B (which we see as a constant)
~ and d`
Because B
times the distance around the circle, i.e. the circumference, so
I
~ = B2r
~ d`
B
From the first equation, we can see that B2r = 0 I, so the radius doesnt matter. Therefore,
I
~ = 0 Ienclosed
~ d`
B

48

which is known as Amperes law. (This version of the law is incomplete, and will be amended twice later
in the course; once for a displacement current term, and once to take care of magnetic permeability, as
0 only holds for the classical vacuum.)
This law not only holds irrespective of the radius, but also irrespective of the shape. Any closed shape
you can draw around a current will result in the law being true.
With Amperes law, we can use the right-hand rule yet again. If we go around a surfaces edge clockwise,
and a current is flowing into the surface, we consider that current to be positive, while a current that flows
out of the surface would be negative.
Lets do an example. We have a wire with radius R, with a current I coming towards us. We assume that
the current density is uniform throughout the wire.
We choose a Amperean circle around the wire, first with r > R so that in encloses the entire wire.
We know that
I

~ = 0 I
~ d`
B

~ and the two


Because of the symmetry of the problem, the dot product will have the same value for all d`,
~ and get 2r instead. We then have
vectors are also always parallel, so we integrate d`
B2r = 0 I
0 I
2r
... which is the same result we saw earlier with Biot-Savarts law.
As for direction, we use the right-hand rule: the right thumb points along with the current, towards us,
and the remaining fingers would then curl in a counter-clockwise direction.
B=

Lets do the same calculation for r < R, so inside the wire. The same symmetry arguments still hold; the
difference here is in the enclosed current, which is now dependent on little r as well. Since the current
density is uniform, the ratio of the areas will give us the answer:
Ienclosed = I

r2
r2
=
I

R2
R2

Thus, our equation is


r2
B2r = 0 I 2
R
0 Ir
R2 2
The direction has of course not changed. Also, note that as r = R, i.e. on the surface of the conductor,
the two equations give the same result.
B=

5.3.1

The magnetic field of a solenoid

A solenoid is a long conductor, wound up into a helix. That is, it looks like a cylinder, where the cylinders
side are made up by loops of this wire.
We can use Amperes law to calculate the magnetic field inside a solenoid - that is, in the center of this
cylinder-ish shape.
49

When we have many loops, the magnetic field outside the solenoid will be almost zero, while the magnetic
field inside will be almost constant throughout the solenoid.
The shape of the magnetic field is infinitely much better described in video than in text. Even images
dont quite suffice, so Ill refer to the lecture video instead. (8.02 lecture 15.)

To calculate the magnetic field strength inside, we choose an Amperean rectangle as shown. It is (of
course) made up of four different lines. For line 2, the magnetic field is approximately zero, since it is
outside the solenoid, so we discard that part.
~ will be zero for them.
~ d`
Lines 1 and 3 are perpendicular to the magnetic field, so B
Therefore, only line 4 makes a useful contribution, and since weve assumed that the magnetic field will
be constant inside, the integral is simply B`.
We then need to calculate the current that penetrates the surface defined by our rectangle, to use on the
right side of the Amperes law equation.
If we define N as the number of loops in the solenoid, over its length L, then the number of loops through
the surface is
N`
L
and so the current is I times that.
We can now set up our Amperes law equation:
B` = 0 I = 0

N`
I
L

The little `s cancel, and we get


0 IN
L
This approximation comes close to the true value, as long as L  R, where R is the radius of the loops of
the solenoid.
With an exaggerated example, we can show intuitively that the magnetic field strength is propertional not
to the number of loops total, but the number of loops per unit length.
Imagine an extremely long solenoid - 100+ meters long. Each loop has a magnetic field that is approximately like a dipole field - which as we know fall off quite rapidly at a distance. Therefore, the magnetic
field near one end of the solenoid has a near-zero contribution from loops near the other side, and so only
relatively nearby loops matter.
If, on the other hand, we have hundreds of loops within an extremely small length, then the total magnetic
field will be almost the same as if we had one loop with N times the current through it, and so with many
loops per unit length, the magnetic field is very strong.
B=

50

Chapter 6
Week 6
6.1

Electromagnetic induction

We have previously looked at how moving electric charges (currents) create magnetic fields. Now, well
look at how magnetic field can create electric fields, and thus currents in conductors.
Unlike the case for electric magnetic fields, where a steady current creates a steady magnetic field (via
Biot-Savart and Amperes laws), a steady magnetic field does not produce a steady current. In fact, a
static magnetic field creates no current at all; only a changing magnetic field does.
The phenomenon where a changing magnetic field causes - or induces - a current, is known as electromagnetic induction. It was discovered by Michael Faraday, who did experiments to find out whether, indeed,
constant magnetic fields cause constant currents.
One experiment he did involved a battery, a solenoid, a switch, and a separate loop with a current meter
in series.
He Usedt he first three components to create a roughly constant magnetic field, inside the solenoid. He
then wound a loop of wire around the solenoid, and connected that to a current meter. If the hypothesis
was correct, he would have seen a steady current in the current meter whenever the solenoid was powered.
That was not what happened; indeed, there was no current at all.
However, he later noticed a current spike in the current meter while he was flipping the switch to the
solenoid, whether he powered it on or off, the direction of the current being dependent on which. He
concluded that a changing magnetic field induces a current.
This principle is extremely important for the modern world, as it is the basis of most of our current
power-generation: wind power, hydropower and nuclear power all depend on Faradays law, as they all
make turbines spin inside magnetic fields to produce power.

6.1.1

Lenzs law

Say we have a simple loop, i.e. a wire bent into a rectangle, a circle or such a shape. We move a bar magnet
downwards into the loop, with the magnets north pole first. The magnetic field of the bar magnetic will
point downwards, into the loop, and will be increasing as we move in closer.
There will be a induced current in the loop, such that the currents magnetic field opposes the magnetic
field of the bar magnet. Via the right-hand rule, if the currents magnetic field is field pointing upwards,
the current is flowing counterclockwise as seen from above.
If we pull the bar magnet upwards, the external (the bar magnets) B field will be going down in the loop,
and so the current will be in the opposite direction.
The fact that the currents field will always oppose the external field change is known as Lenzs law. It
states nothing more than the currents direction - to calculate the currents magnitude, we must use Faradays law, soon to be introduced.

51

Clearly, a current cannot just come about on its own - a current is usually driven by a source, that produces
a potential difference over e.g. a wire. In this case, it is an induced EMF due to the changing magnetic
field, which can be written as
Eind = Iind R
... which is of course just Ohms law. R is this case, then, is the total resistance of the entire loop where
the current flows.
In another experiment by Faraday, he had a current loop connected to a battery, that created a magnetic
field. He switched the current, such that the magnetic field was changing (since, as we now know, a static
magnetic field produces no EMF).
He then had a second loop, with no battery, independent from the first, but located nearby, such that the
changing magnetic field affects the second loop.
What he found was that the induced EMF in loop number two was proportional to the change in the
magnetic field from the first,
dB1
dt
... and also that it was proportional to the area of the second loop. This gave him the idea that perhaps
the EMF is proportional to the change in magnetic flux through the surface of the loop.
E2

6.1.2

Magnetic flux

Magnetic flux is defined in the same way as electric flux, except of course that the dot product is between
the magnetic field and an area vector.
That is, for an open surface A, and a magnetic field B, we integrate the dot product of all infinitesimal
~ on the surface with the local magnetic field at that point. This gives us the magnetic flux
area vectors dA
B (or B - the Greek letters being lowercase and uppercase phi, respectively):
Z
~
~ dA
B = B
This integral is done over the open surface where we want to know the flux.
The SI unit of magnetic flux is the weber (Wb); 1 Wb = 1 T m2 .
When the B-field is constant and through a planar surface, this simplifies down to
B = BA cos
... where is the angle between the magnetic field and the unit normal perpendicular to the surface.
For electric flux, we have Gausss law, which relates the flux through a closed surface to the amount of
charge inside:
I
~ = Qencl
~ dA
E = E
0
If we enclose a single electric charge (an electric monopole), whether positive or negative, the flux is always
nonzero.
If we enclose an electric dipole, such that we enclose the entire dipole, the flux will be zero. However, if
we choose the closed surface such that in encloses only part of the dipole, the flux will again be nonzero.
The electric field lines always begin at the positive charge(s) and end on the negative charge(s).

52

In contrast, in magnetism, we have (as stated previously) never witnessed a magnetic monopole. Magnetic
field lines are continuous, and do not end at any of the poles. Therefore, wherever in space you choose a
closed surface, exactly the same flux that enters the surface will leave it, with no known exception. There
can be no exceptions without magnetic monopoles.
Thus, unlike in the electric dipole situation, the magnetic flux through a closed surface is ALWAYS zero.
This result is usually called Gausss law for magnetism:
I
~ =0
~ dA
B
Until the day where magnetic monopoles exist, that will remain one of the laws of physics, and is one of
Maxwells four equations. (They consist of Gausss two laws, Amperes law, and Faradays law.)
We will now introduce the last of the four equations: Faradays law of induction.

6.1.3

Faradays law

Faradays law, or Faradays law of induction, relates this changing magnetic flux to the induced EMF:
dB
dt
The minus sign signifies Lenzs law, that the induced current (and thus the induced EMF) will always
oppose the change in the magnetic flux. However, if we know Lenzs law, we should never need to feel
confused by the direction of the EMF.
E=

Since we know the flux to be given by the above surface integral, we can also state the EMF as
Z
d
dB
~
~ dA
=
B
E=
dt
dt
Since a current flows in the wire, there must be an electric field inside the wire as well. We know, then,
~ integrated over
that the EMF must also equal the dot product of the electric field and the infinitesimal d`
the closed loop:
I
Z
dB
d
~
~ = E
~ d`
~ dA
E=
B
=
dt
dt
~ that we choose. The open surface
There is a convention regarding the direction of the area vector dA
attached to the loop is one chosen by us, so we follow the convention of the right-hand rule here as well.
The surface chosen does not have to be flat in the plane of the conducting loop; it can be any shape, for
example like an open bag. The result will be the same. This can be intuitively explained if we consider
the flux as a flow of air or water through the loop. If it comes into the opening of the loop, it must come
out through that surface, no matter its shape or size.
If we have a circular loop (for simplicity - it does not have to be any shape whatsoever, as long as it
~
connects back so a current can flow) in the plane of this page, and we march clockwise around it, then dA
~ will point out of the page.
will be into the page. If we march counterclockwise, dA
At this point in the lecture, the professor does an experiment, similar to the one Faraday did. He wraps
a wire, connected to nothing but an ammeter, one time around a solenoid.
If we then visualize that we attach a surface to the loop around the solenoid, the changing magnetic field
inside the solenoid will penetrate that surface, so there is magnetic flux through our surface. Furthermore,
because we are turning the solenoid off and on, the flux is changing, and so there is an induced EMF (and
thus current) in the loop weve wound around it.

53

If we increase the size of this loop, by increasing the length of wire, nothing changes. Why not? Well, the
area is larger, but the area where the magnetic flux penetrates the surface is the same: the magnetic field
outside the solenoid is roughly zero, as we saw last week.
Therefore, if we imagine an open surface formed between the edges of the wire, the part that is inside
the solenoid is exactly the same and is dictated by the size of the solenoid only. The parts that are out~ will be zero in that region.
~ dA
side the solenoid are in the region where B 0 and so they dont matter; B
However, there is one way we can increase the area that the flux penetrates: wrap the wire around the
solenoid multiple times. While its very, very hard to intuitively visualize such a surface, the wire is essentially shaped like a helix, and so there will be a surface that is curled around the center point. The main
point to realize is that with three loops, the surface area of the surface will be three times greater.
Unlike before, however, the flux will now penetrate this entire surface, so the flux will be three times
graeter. Therefore the time derivative of the flux - the magnitude of the EMF (ignoring the minus sign) will be three times greater!
Therefore, the EMF is proportional to the number of loops N around the solenoid. We can make this N
however large we wish - a thousand loops is no problem at all. This is how transformers work: the higher
the loop count, the higher the EMF.

6.1.4

The breakdown of intuition

Now comes the hard (to accept) part of this all. Weve previously used Kirchhoffs voltage law (or rule),
KVL, which states that
I
~ =0
~ d`
E
Or, in words, that the closed loop integral of the electric field in a closed loop is zero. That is, when you
walk around ANY closed loop in an electric circuit - you start at one point, walk around the circuit and
add all voltage drops - the sum will always be zero.
Well, that is NOT TRUE where there are changing magnetic fields involved! The integral is not zero any
more; in fact, weve already showed above that it is equal to the (negative of the) induced EMF, which is
certainly nonzero!
Because the electric fields are now non-conservative, and Kirchoffs rule is only valid for conservative fields,
the path we choose now matters. Previously, the voltage sum was independent of the path.

Say we have a circuit as above. The battery has an EMF of E = 1 volt. The circuit element the right
labeled V is a voltmeter.
Via very basic circuit analysis, we see that the current through the circuit is
I=

E
= 103 A
R1 + R2
54

The voltmeter will thus display the voltage drop across R2 , which is IR2 , which then is VD VA as the
points are labelled:
VD VA = IR2 = 103 900 = +0.9 V
If we instead look at the left side - imagine an identical voltmeter there, also attacked to point D (positive
side) and point A (negative side). Clearly, the voltmeter will read the same voltage - they are connected
at the same points.
We can think of this voltmeter measuring the drop across R2 as well, or we can think of it measuring the
sum of the voltage drops across R1 and the battery. The points are still VD and VA , so:
VD VA = IR1 + E = (103 100) + 1 = +0.9 V
Thus, if we subtract the two equations,
(VD VA ) (VD VA ) = VD VD = 0
The voltage drop between VD and VD is zero, as we expect. Kirchoffs law says it must be.
We now remove the battery from the circuit, and instead insert a solenoid into the middle of the current
loop.

The above is now our circuit, with the second voltmeter also visible.
The magnetic field from the solenoid is coming out of the blackboard, with the shaded area being the area
where the magnetic flux penetrates the surface of our current loop. The rest of the surface will only be
exposed to the near-zero magnetic field outside the solenoid.
The EMF will now be a function of time, as it is given by (the negative of) the rate of charge of the
magnetic flux.
Supposed that at one instant in time, the EMF is E = 1 volt, same as we had before.
However, last time, we had a battery! The battery had, by definition, a 1 volt drop over it. That is now
gone! We get our EMF from the solenoid, now. So lets do the circuit analysis.
What is the current? Well, the magnetic flux is coming out of the blackboard, and via Lenzs law, the
current will flow such that the currents magnetic field opposes that, i.e. is going into the blackboard.
That means the current will be clockwise, same as before.
If we then calculate VD VA on the right, we get the same result as before:
VD VA = IR2 = 103 900 = +0.9 V (right side)
55

What about the left side? Well, the battery is now gone, so the result is the same as before, minus the
plus one:
VD VA = IR1 = (103 100) = 0.91 V (left side)
VD VA has two different values, depending on which path we choose to measure it! The two
voltmeter are connected to the same points, but show different values! Extremely nonintuitive.
This also means, then, that Kirchoffs voltage law has been broken: the sum around the loop is not zero!
VD VA = +0.9 V (right side)
VD VA = 0.1 V (left side)
(VD VA ) (VD VA ) = VD VD = +1 V
Note that this is all at one instant in time; the result is not because the values somehow changed during
the calculation. Its just an extremely nonintuitive result that can be very hard to accept as being correct.

6.2

Motional EMF and dynamos

We have seen now that the induced EMF is related to the change in magnetic flux through a surface.
Suppose that we have a rectangular current loop of sides x and y. We assign the area vector infinitesimal
~ to it, such that dA
~ points upwards.
dA
~ pointing upwards, but at an angle towards the right.
We then have a uniform magnetic field B
~ and B
~ is .
The angle between dA
The flux through the surface is then given by BA cos . A = xy, so
B = xyB cos
The induced EMF is then given by the time derivative of this expression. However, note that there are
dA
dB
; the area, so we have
,
three things which can change over time: the magnetic field, so we have
dt
dt
d
. That is, if is changing, we are rotating the conducting
and the angle between them , so we have
dt
loop inside the uniform magnetic field.
Consider rotating this about the y axis, centered on the loop. We rotate it with angular frequency ,
which is given by 2 divided by the period T :
2
= 2f
T
... which is, as above, equal to 2 times the rotational frequency in hertz.
(The unit of is given as radians per second, rad/s; however radians are dimensionless, and so it is really
equivalent to hertz, s1 ; to not confuse values of angular frequency with frequency, we consider the latter
to be in radians/second.)
=

The angle will then become 0 + t, where 0 is the angle at t = 0. We can choose that to be 0, so that
= t. Thus, we have
B = AB cos t
The EMF is given by the negative of the derivative of that, so
dB
= AB( sin t)
dt
56

The EMF is the negative of the above, so the minus sign from the cosine derivative cancels:
E(t) = AB sin t
Keep in mind that A is the total surface area the flux penetrates! If you have a loop of N > 1 windings
in a shape of area A, the EMF is given by
E(t) = N Aone B sin t
... with Aone being the area of one winding.
The current in the loop will then also be time dependent, and it will alternate in the sinusoidal fashion
seen above; this makes it alternating current, the same stuff that comes out of your wall.
The reason AC is used there will be touched upon later in the course; one big reason is that transformers
can be used to convert AC voltages up to the high voltages in power lines, to avoid losses, and then back
down to the approximately 100 to 250 volts used in homes around the world.
Another is that power generated by generators/turbines/dynamos is sinusoidal by its nature, see below.
Lets talk about about generators, turbines or dynamos - whatever you prefer. If we have permanent
magnet, and we rotate conducting windings inside that magnetic field, we get an EMF as seen above. This
is the process used for most of our power generation - wind power, hydropower and indeed nuclear power
all drive turbines to generate electricity.
The stronger the magnetic field, the higher the EMF. The more windings, the higher the EMF. The larger
the area, the higher the EMF. And lastly, the faster you rotate it, the higher the EMF.
The last point is useless for the power grid, however: the power grid has a fixed frequency, usually of 50
Hz (in Europe, Africa, Australia and most of the world) or 60 Hz (in the US and Americas, though not
all of it).
If we were to increase , then, not only would the EMF increase, but the power would be out of sync with
the rest of the power grid, which would cause big problems.
One of the relatively smaller such problems is that many devices - some clocks, for example - use the line
frequency to keep track of time. Thus, a clock designed for 50 Hz may go 20% too fast if the line frequency
in 60 Hz. (Or it may not, depending on how it keeps time.)
V2
= I 2 R, and both V and I alone increase linearly with , the
R
power increases (or decreases) with 2 as we change it.

Because power is given by P = V I =

6.2.1

Changing the area

We have now looked at changing the magnetic field and changing . What about changing the area of the
loop, such that the flux through it changes that way?
Imagine we have a rectangular loop of sides x and `. The right side, of length `, is a crossbar; we can move
it towards the left and right, such that x changes with time. We then have a velocity vector ~v towards the
~ going upwards.
right (or left, when it is negative). Say we have a uniform magnetic field B
The flux through the surface is then
B = `xB
We then take the time derivative of this, where x is the variable that changes with time, while ` and B
are constant:

57

dB
dx
= `B
dt
dt
dx
However,
is simply the speed (or velocity, if we attach a direction of +
x to it) of the crossbar. The
dt
EMF is then, via Faradays law:
|E| = `B|~v |
The resistance of the conducting loop is R. What is the power dissipated by the circuit?
V2
, where V = E for this case. We found E above, so the answer is
R
E2
`2 B 2 v 2
=
R
R
What is the magnitude of the force experienced by the moving part of the loop?
Power is P =

We found before that the force on a current-carrying wire can be found via
Z
~ B)
~
F~B =
I(d`
wire

~ and B
~ are exactly perpendicular in this problem, the integral simplifies down to
Since d`
F~B = I`B
... where ` is the length of the wire segment.
E
`vB
=
R
R
We substitute that into the force equation and get another ` and B, so the answer is
I=

`2 vB 2
F~B =
R
~ B
~ is an infinitesimal wire
~ gives the direction, where d`
The direction of this force is to the left: d`
segment the current moves through. That vector is upwards (+
y , in our 2D plane), and the B-field is
perpendicular to that, upwards in 3D (+
z ).
How much power is needed to keep the bar moving at constant velocity?
Power is work per unit time, so the time derivative of work should give us the answer. We know the
magetic force, which is to the left. So we need to exert a force towards the right, of equal magnitude. The
work we do is then that force (that we have above) times the distance. We then need to take the time
derivative of that.
However, we already have the time derivative of the displacement - the velocity. So
P = F~B ~v
Since they are in exactly opposite directions, cos = 1, so the answer is the answer for the force above,
times another v:
P =

`2 v 2 B 2
R

58

... which happens to exactly the same as the power dissipated by the circuit! Of course, unless we find
somewhere else the power could be used, this is to be expected; energy cant be created nor destroyed.
The power we put in must go somewhere.
We can also write this in terms of I instead:
P = I`Bv
This must be equal to the EMF times the current; the current then cancels from the equation, and we get
|E| = `Bv
... which is what we found earlier, however this time we did not use Faradays law to find it, merely the
work per unit time we do to move the crossbar against the magnetic force.
Also, note that if instead of pulling towards the right, we push the crossbar towards the left. We still do
exactly the same amount of work - positive work in both cases. When ~v is flipped over, the magnetic force
is also flipped over, because the current reverses direction as the magnetic flux is now decreasing.

6.2.2

Eddy currents

Lets now move our focus to something different, but related.


Suppose we move a solid, conductive disk through a magnetic field. Say we have magnetic field pointing
upwards, and we move the disk through it sideways.
We know that the changing magnetic flux through the disk will induce a current to oppose the magnetic
flux change; that is, the currents magnetic field will be downwards, and so via the right-hand rule, the
current will be clockwise.
We call these currents eddy currents.
They produce heat in the conductor; the energy for that must come from somewhere. In this case, energy
that would otherwise be kinetic energy is used up, and so the disk slows down. This is the principle called
magnetic braking.
Eddy current calculation
A fairly hefty question was in between the lecture videos, so Ill write down how I solved it. If it werent
for the multiple-choice answers, this would make a good homework question!
A rectangular loop of wire with mass m, width w, vertical length `, and resistance R falls out of a magnetic
~ = B x) within
field under the influence of gravity. The magnetic field is uniform and out of the paper (B
the area shown (see sketch) and zero outside of that area. At the time shown in the sketch, the loop is
exiting the magnetic field at velocity ~v (t) = v(t)~z, where v(t)<0 (meaning the loop is moving downward,
not upward). Suppose at time t the distance from the top of the loop to the point where the magnetic
field goes to zero is z(t) (see sketch).

59

What is the direction of the induced current in the loop?


Right-hand rule, as always. The flux is positive through the loop, but it is decreasing, so the derivative is
negative. Via Lenzs law, if the flux is decreasing, the current will create an opposing magnetic field. If
the field lines are out of the page but decreasing, the currents magnetic field will be out of the page as
well, so the current is counterclockwise.
~ B
~ force in the top horizontal segment of the loop?
What is the direction of the I d`
(Note that there is no net force on the other segments, since they are outside the B-field, except for parts
of the left/right sides, which cancel.)
Sincet he current is counterclockwise, i.e. towards the left at the top, the cross product direction is left
cross out of the paper, which is upwards. That is, the magnetic force will be breaking the fall.
What is the EMF generated in the loop?
We use Faradays law. First, what is the flux, and the flux change? The flux is
B = wz(t)B
where wz(t) is the area, changing with time. The time derivative of the above is then
dz
dB
= w B = w~v B
dt
dt
The EMF is the negative of that, but because ~v is also negative, the signs cancel and we get the answer
E = wvB
Suppose the bar reaches terminal velocity (no longer accelerating). What will its downward speed be
then?
Not being well-versed in classical mechanics, I made a bit of a guess here: that when the forces are equal,
the above is true. That gave me the correct answer, at least.
So, I calculated the force due to gravity:
Fg = mg (since F = ma)
... and the magnetic force, which is in the opposite direction:
 
E
wvB
w2 vB 2
FB = IwB =
wB =
wB =
R
R
R

60

Set the two equal, and solve for v:


w2 vB 2
= mg
R
w2 vB 2 = mgR
v=

6.3

mgR
w2 B 2

Displacement currents and synchronous motors

Lets take another look at Amperes law. Say we have a current flowing towards the right, through
a capacitor (with circular plates of radius R, apparently). We know that the electric field inside the
capacitor is
E=

Qf ree
f ree
=
0
R2 0

The current is, per definition


dQf ree
dt
We therefore have a changing electric field between the plates, while they are being charged, given by
the time derivative of the electric field above. Since Qf ree is the only thing changing with time, the time
derivative is simply given by substituting the current for Qf ree :
I=

I
dE
=
dt
R2 0

If we now want to calculate the magnetic field at a distance r away from the wire, at point P1, what can
we do? We can try Amperes law, but keep in mind that there is an interruption in the current due to
the capacitor plates (no current flows in the air/dielectric between them), and Amperes law is only truly
valid for infinite wires (and other special cases). Lets try anyway, just to see what happens.
Amperes law, as we know it so far, states that
I
~ = 0 Ipen
~ d`
B
We choose an Amperean circle of radius r such that point P1 is part of the circle. We then attach an
open surface to the circle, such that the current penetrates that surface (we choose the circles area, so to
speak, as that is the simplest possible choice).
The closed line integral around a circle is simply 2r. As for Ipen , that is the current that penetrates the
surface, which in this case is all of it. So we find that
61

2rB = 0 Ipen
0 Ipen
B=
2r
Fair enough. What about at point P2, above the empty space in the capacitor?
Well, we use the same formula, choose an Amperean circle with radius r as before, attach the same open
surface to it... but the current penetrating the surface is now zero, so the result we get is that the magnetic
field is zero at point P2!
The choice of the open surface is up to us, so we change our choice of surface to a bag that encloses one
capacitor plate:

0 Ipen
previously.
We re-apply Amperes law, to find the magnetic field at P1 - which we found to be B =
2r
However, as we reach the point where we find the current that penetrates the surface, we now see that it
is zero! No current penetrates our surface, it just goes inside it - never through.
We had the free choice of the surface, but different choices gave us different results!
The reason behind this is that the version of Amperes law weve learned so far is incomplete. We will
now add a second term that corrects this problem.

6.3.1

The amended Amperes law

Maxwell solved this problem. There is a changing electric field between the platse, and so there is a
changing magnetic flux through our chosen surface.
He reasoned that since, via Faradays law, changing magnetic flux induces elecrtic fields, perhaps changing
electric flux induces magnetic fields, as well.
Electric flux works just as magnetic flux (as we mentioned when we introduced magnetic flux)!, so
x
~
~ dA
E =
E
S

The new version of Amperes law then relates the time derivative of that with the magnetic field:
!
I
x
d
~ = 0 Ipen + 0
~
~ d`
~ dA
B
E
dt
S

(This version of Amperes law is still incomplete - it will be adjusted in week 8 of the course; however that
adjustment is very small, and only adds one simple term multiplying 0 , to take care of the permeability
62

of other media than the classical vacuum.)


The closed loop in the first integral is then the Amperean circle, while the surface in the second (surface)
integral is the open surface attached to that closed loop. That is, the closed loop is the opening of the
bag, while the open surface is the bags surface.
The current Ipen is the real current that penetrates the surface. The second term, 0

d x ~ ~
E dA, is
dt
S

called the displacement current.


Note that whenever the electric flux is constant, the time derivative goes to zero, and the old Amperes
law pops out.
Lets try to re-calculate the magnetic field at P1, again using both the flat surface and the bag. First
out is the flat surface.
The left-hand side of the equation is unchanged, so 2rB, while the right-hand side turns out like this:


d ~ 2 
0r
= 0 (Ipen + 0)
2rB = 0 Ipen + 0
dt
... because there is no electric flux penetrating the surface - it is outside the capacitor, where the electric
field due to the capacitor is zero.
Thus, we find
B=

0 Ipen
2r

Same as before.
We re-start with:
I

~ = 0
~ d`
B

d x ~ ~
Ipen + 0
E dA
dt

The left-hand side is unchanged, as that is the closed Amperean loop, which we are not changing. Ipen
now changes, however: it is again zero - once again, no current penetrates the surface! What about the
second term? Lets fill it in:
2rB = 0 0


d
ER2
dt

The E-field is what is changing with time, so we put that in there (we found

dE
earlier):
dt

dE
I
R2 = 0 0 2 R2
dt
R 0
Note that the I here is not Ipen , the current penetrating the surface weve chosen, but the current in the
wire. It is not zero!
2rB = 0 0

If we simplify this, we get


0 I
2r
Excellent! Though we found I in a very different way, the answer is identical!
B=

63

Next up, lets try to calculate the B-field inside the capacitor (while it is charging; when it is not, we have
an electrostatic situation and it ought to be zero).
We set a new point P2 at distance r from the exact center of the capacitor (in all dimensions). Since the
choice of surface should not matter, we will make it simple an pick the flat surface again.
!
d x ~ ~
2rB = 0 Ipen + 0
E dA
dt
S

Ipen is zero, since there is no current inside - only air (or dielectric)! The flux, however, is EA with the area
being r2 (little r - the surface is smaller than the plates, so there is no contribution from the remaining
area).

d
Er2
dt
Again, we know the time derivative of the E-field from earlier, so we substitute that in there once again:
2rB = 0 0

2rB = 0 0

I
r2
R2 0

0 Ir
(for r < R)
2R2
Interesting, it is exactly zero in the middle (r = 0), then, and greatest at the edge.
B=

6.3.2

Displacement current

Why did Maxwell call the second term the displacement current?
If you have a dielectric between the capacitor plates, the changing electric fields will cause a current;
the induced charges will be rearranged due to the changing electric fields. In a vacuum, however, there
shouldnt be a current, so the name is perhaps somewhat poorly chosen.
We have a capacitor of area A filled with a dielectric slab with dielectric constant . If Q(t) is a function
of time, what is the polarization current in the dielectric? Express your answer in terms of the magnitude
of the electric field E(t) between the plates. Positive polarization current flows down in the picture.
The picture is then of a simple capacitor, with +Q on the top plate and Q on the bottom, with the
opposite induced charges Qind on the dielectric (minus up, plus down).
Hmm, OK. E(t) is given by
Q(t)
(t)
=
0
0 A
My first thought was to solve for Q and take the time derivative of both sides. That didnt work; I got
something close to the correct answer, yet far away: it had a factor instead of ( 1), where as well
see, only the latter means no current if = 1 (for a perfect vacuum).
E(t) =

Instead, we can solve it like this:


First, we know that the E-field in a capacitor in a vacuum is
Evacuum (t) =

(t)
Q(t)
=
0
0 A

With a dielectric, there will be an induced charge Qind (t) on the dielectric. This induced charge causes an
opposing electric field, Eind (t), which causes E(t) to equal the original E-field divided by :
64

Evacuum (t)
Q(t)
=

0 A
We can also write the net field as the vectorial sum of the original field, plus the induced field. The latter
is in the opposite direction, so we get a subtraction:
E(t) =

Q(t) Qind (t)

0 A
0 A
We can then set the two ways of writing E equal and finally solve for Qind (t):
E(t) = Evacuum (t) Eind (t) =

Qind (t) =

1
Q(t)

dQind
, so we need to calculate the derivative of the above. First, we
dt
solve the E-field equation for Q(t), so that we can substitute it in there:

Almost there. We are looking for

Q(t)
0 A
Q(t) = E(t)0 A
E(t) =

Make the substitution, and simplify:


1
E(t)0 A

Qind (t) = ( 1)0 AE(t)

Qind (t) =

And finally, take the time derivative:


Ipolarization =

dQind
dE
= ( 1)0 A
dt
dt

Thanks to H_Litzroth on the 8.02x forums for posting his solution, which the above is heavily based on.

6.3.3

Synchronous motors

Say we have a rotating conducting current loop in a magnetic field, like the first image here shows:

We then add two more loops, each 120 degrees rotated from the first, so that we get the result shown in
the second picture, seen from the side (from the side where the wires enter in the first picture). So we
have three loops, electrically isolated but rotating together.
As weve seen before, when they rotate, we will get an induced EMF in each loop. Due to their 120 degrees
(1/3 period) separation, we will get three sinusoidal waves, which differ only in their phase - assuming all
loops are identical in composition, area etc.
The EMF as a function of time would look something like this:
65

... with the current from loops 2 and 3 having a phase delay of 120 and 240 degrees from the first loop,
respectively.
The period of all three loops will be equal, as they are rotating together.
Now, lets put that aside for a while, to not cause confusion. We will use the current from that 3-phase
generator, but other than that, what follows is independent of the above setup!
We now take a look at a different setup, of three open solenoids.

To the left is the professors drawing, which I didnt understand at first. The second one, on the right,
is from the same angle - from straight above - and shows a snapshot at three points in time. The first
(leftmost) frame is when the magnetic field due to the red solenoid peaks, and the vectorial sum of the
three magnetic fields are in the direction of the arrow. (The red one alone is in that direction, and the
vector sum of the blue + yellow are also downwards.)
In the second frame, 1/6 of a period later (60 degrees phase later), the vector sum is now as the arrow
shows strongest due to the blue solenoid; again, the vector sum of the two others sum up and to strengthen
the field in this new direcion.
Finally, another sixth of a rotation later, the magnetic field now points as the arrow shows.
What we have, then, is a rotating magnetic field. The solenoids themselves are perfectly stationary, but the
net B-field they generate rotates around in a full circle, once per rotation of the original power generator.
(Once per Hz of the power we are feeding the solenoids.)
Of course, since the current is a set of smooth sinusoids, the magnetic field changes gradually throughout
this, and doesnt just skip between the 6 positions.
If we stick a magnet in the middle of all this, we have a synchronous motor, or a 3-phase motor. The
magnet will want to align with the magnetic field at all times, but the magnetic field is always rotating,
and so the magnet will rotate at the same frequency - the frequency of the supplied power.

66

If we stick a conductive object inside this field, it will want to rotate. Eddy currents are formed, and the
rotating magnetic field exerts a torque on the currents, causing the object to rotate. The above picture
shows a rotating, conducting egg as demonstrated in the lecture, using the solenoids drawn above.
If the object is roughly spherical (or egg-shaped), the rotational frequency will be very close to the frequency
of the current (and thus the magnetic field).

67

Chapter 7
Week 7
7.1

How do magicians levitate women?

This lecture is mostly about various concepts - the human heart, aurora borealis, and magnetic levitation.
There is little math, but much graphical content, and so it is hard to take very good notes without taking
screenshots once a minute!

7.1.1

The human heart

The human heart has four chambers: on top, the left and right atria (singular: atrium), and below that,
the left and right ventricles.

The aorta is on the left side.


The heart pumps about 4.5 liters of blood per minute. It beats about 70 times per minute (which varies
a lot between people, of course).
If the brain is without new oxygen for about 5-10 seconds, you pass out - after as little as five missed beats.
After a few minutes, permanent brain damage is very likely. After 5 minutes, it is virtually certain, and
death is likely.
Each heart cell is a small chemical battery. In the normal state, an individual heart cell, about 10 m in
diameter, has a potential of 80 mV on the inside, with respect to the outside.
A group of pacemaker cells, located in a small area of about 1 square millimeter on the right atrium,
start the process of a heart beat by changing their potential from -80 to +20 mV. Once they do so, the
neighboring cells do the same, and a wave propagates across the entire heart, from the atrium down to
the ventricles.
When the heart cells change their potential (by an exchange of ions), they contract, and so the heart
contracts, one cell after the other, in a wave.
68

About 0.2 seconds later, the cells return to their resting potential of -80 mV, in a similar wave, this time
going from the bottom up.
They then wait from a new incoming wave from the pacemaker cell, and the process starts over about a
second later.
In the resting state, the cell has repelled positive ions, which makes the inside negatively charged. The
electric field outside is zero, since the net charge around the cell is zero.

Here, we see a heart cell in the middle of the depolarization phase, when it changes from -80 mV to +20
millivolts (shown as 0 to simplify), by moving positive ions back inside.
The wave starts from above, and has passed through half the cell at this instant.
In the bottom half, we have a situation with negative charge on top of positive charge, which creates
a dipole-ish electric field. When the process is completed, the inside is at +20 mV with respect to the
outside, and there are positive ions on the inside, with the corresponding negative charge, making for a
zero electric field.
A while later, the repolarization wave comes in, from below, and the reverse process happens. The bottom
of the cell goes to the -80 mV, gradually changing into having the negative ions inside again, which will
then again create the dipole field for a time.
Only the cells that are part of the depolarization wave at a given time - a minority, and most of the time
indeed none of them are - contribute to the dipole field. The same is true for the repolarization wave as
well.
Since there is a net electric field from the heart, there will be a potential difference between different parts
of your body. Measuring this potential difference is how an EKG (or ECG - electrocardiogram) works.
Many electrodes - usually 12 - are attached to various body parts. The potential difference is only on the
order of a few (2-3) millivolts.
There is a condition known as ventricular fibrillation. In the United States, it kills on the order of 400 000
people every year. The basic cause is that the ventricular cells fire without a message from the pacemaker
cells, and so the fire randomly, and theress a non-coordinated depolarization, so that the heart stops
pumping any blood.
Clearly, then, this is a medical emergency, since as said earlier, loss of consciousness usually happens
within 10 seconds, and there is risk of permanent brain damage extremely quickly, even after a single
minute without oxygen. After a few minutes, it is almost certain.
One treatment for ventricular fibrillation is the well-known defibrillator - a direct-current electric shock
across the heart, to reset the cells and hopefully get back synchronization from the pacemaker cells. The
shock sends a total of roughly 200-350 joules of energy into the heart, over a short period of perhaps a
tenth of a second.
The hearts pacemaker can also be faulty, which may cause various problems. One treatment for such
a condition is to implant an artificial pacemaker, which can sense the hearts condition and take over if
necessary, triggering the depolarization wave.
There are also implantable defibrillators, which can apply small shocks automatically when fibrillation
may have caused death otherwise.
69

7.1.2

Aurora borealis

If we have a magnetic field, and move charged particle though it, lets say a positive one, we know that the
magnetic force on that particle is given by
~
F~B = q(~v B)
~ and one perpendicular to B.
~ We
We can do vector decomposition to get two vectors: one parallel to B,
get
~
F~B = q(v~ + v~k ) B
However, note that the parallel component v~k makes no difference for the force - the angle between that
and the magnetic field is zero (or 180 degrees), so the cross product is zero. We end up with
~
F~B = q(v~ B)
Consider, then, a situation with a magnetic field towards the right, and a velocity angled upwards (still
in the same plane). The perpendicular component will make the particles go in circles, but the parallel
velocity means it will still keep going forward at the same time, so the path is a helix, a bit like a spring.
The radius of the helix will then be given by, as usual (except that only the perpendicular component
matters know)
mv
qB
Meanwhile, it continues forward at the same parallel velocity, which remains unchanged.
R=

The Earths magnetic field is not made up by straight field lines, however. The field is similar to what
youd expect from a massive bar magnet buried within the Earth, with the magnets south pole near the
geographical north pole, and vice versa. (The magnetic poles drift and even shift completely with time,
over very long time periods.)
Thus, when a particle enters, it moves along a helical path around a field line, and enters near the one of
the magnetic poles, where the field lines terminate. Well, they dont ever terminate, but they would enter
the Earths solid part, where the particle is unlikely to get far (if it ever reaches the surface).
The sun sometimes emits massive amounts of plasma, mostly ionized gas so protons and electrons, called
the solar wind, in coronal mass ejections, or CMEs. The scale of these ejections are absolutely mindboggling; each arc of plasma that breaks off the sun is many, many times the size of the Earth. I feel an
image is absolutely necessary (image courtesy of NASA):

70

In a CME, such a huge lump of plasma can break off and leave the sun, moving out a a few hundred to
a few thousand kilometers per second, meaning it can reach the Earth in a few days or so. Thus, a few
days after a CME, massive amounts of charged particles can collide with the Earths magnetic field.
When that happens, it ionizes molecules in the Earths upper atmosphere, which releases light - the color
of which depends upon which particles are ionized. We call this aurora borealis, also northern lights in the
northern hemisphere; in the southern hemisphere, it is known as aurora australialis or southern lights.

Quite a stunning display. The second image is taken by the IMAGE satellite, a satellite designed to study
the response of the Earths magnetic field to changes in the solar wind. (The image of the Earth is added
digitally; that too is taken by a satellite, however, so that too is real.)
The third is from the International Space Station.
The colors can vary; green, white and red are the most common, but blue is another possibility. The color
depends on at what altitude the collisions happen, and also on which molecules are being excited.

7.1.3

Superconductivity and magnetic levitation

Superconductivity is a state of exactly zero electrical resistance, encountered in certain materials at very
low tempereatures. (High-temperature superconductors are under heavy research.)
The resistivity of a superconductor has a discontinuous drop to 0 below its critical temperature Tc .
Superconductivity was discovered by Dutch phycicist Heike Kamerlingh Onnes in 1911, and gave him the
Nobel Prize in Physics 1913.
He invented a method for producing liquid helium a few years earlier; helium remains a liquid down to
absolute zero (an unreachable temperature); indeed he got it as cold as 1 kelvin!

71

Unfortunately, quantum mechanics is required to fully understand superconductivity. Even then, hightemperature superconductors (materials that are superconductive above 30 K, previously thought to be
impossible) are not fully understood as of this writing.
High-temperature superconductors were discovered in 1986, and led to the Nobel Prize as soon as in 1987.
Because there are now materials known that are superconducting at up to around 133 K, liquid helium
or other extreme measures are no longer required, and the much more available liquid nitrogen, with a
boiling point of about 77 K (about 196 C) is cold enough.
No electric field can exist in a superconductor. If it did, then the electric field times a distance in the
conductor could be nonzero, meaning a potential difference would exist. If there was a potential difference
V
V
= , clearly things dont work out.
V , and I =
R
0
If we approach say a superconducting disk with a bar magnet, Faradays law tells us there is a change in
magnetic flux, and thus an induced EMF. But we just said there can be no EMF in a superconductor!
This has a very interesting consequence: we have said that Faradays law always holds, so the EMF in the
superconductor must be
dB
dt
... but since E must equal zero, so must the change in the magnetic flux!
The magnet will induce eddy currents in such a way that the magnetic flux through the superconductor is
held constant!
E=

The net magnetic field - the superposition of the fields due to the bar magnet and due to the eddy currents
- will look like so:

There will be a magnetic pressure between them, given by


P =

B2
N/m2
20

Because of the superconductivity, the eddy currents never dissipate - I 2 R = 0 because R = 0, and so we
can simply place a regular magnet above a superconductor, and it will levitate and stay in place.
There are other ways of magnetic levitation. In a train, we can have strong magnets and move them over a
conductive surface below the train. Since there will be a change in flux penetrating the conductive surface,
eddy currents will be induced, and we know via Lenzs law that they will oppose the external magnetic
field, thus creating a repelling force, possibly allowing the entire train to magnetically levitate above the
tracks - as long as it has a high enough speed. If the speed is low enough, the flux change through the
conductor will not be enough to produce enough force to lift the train, and it will crash (more or less) to
the ground. Clearly, such a train will need to be built to handle this either way, or it would have no way
to get moving from a standstill if it could only move while levitating!
Now then, weve seen two ways of magnetic levitation: one using superconductors, and another using
moving magnets.

72

A third form requires neither speed nor superconductors, but instead uses alternating current in a loop.
Imagine a current loop, through which we pass an alternating current, above a conducting plate.
At one moment in time, the magnetic field due to the coil will point downwards, i.e. have its north pole
towards the conducting plate. Lenzs law tells us the eddy currents will produce a magnetic field that
opposes this, and so the two plates will repel each other, and there will again be a magnetic pressure
between the two.
However, slightly later in time, the magnetic field will now be decreasing, and so the eddy currents flip
direction, such that the current loops field still has its north pole downwards (but the flux through the
plate is decreasing), but with the eddy currents field flipped, the two will now attract each other.
It therefore seems reasonable to conclude that they will repel and attract each other equally, on average;
this is not the case, and there will be a net repelling force, the source of which is discussed in next week,
when inductance is discussed.
(A preview: there is a lag in the eddy current versus the induced EMF, such that they repel each other
more than 50% of the time, so the net force on the current loop is upwards.)
The magnetic levitation of a woman
We then reach the section responsible for the lectures name. Magicians used to (seemingly) levitate people
- stereotypically, they were women - using various tricks. Can we do it using magnetic levitation?
We have the equation for the magnetic pressure,
B2
P =
20
Using a current loop of area 0.1 m2 , we find that using a B-field strength of about 0.15 tesla, the pressure
is
P 9000N/m2
Multiplying that by the area, we get a force of roughly 900 newtons, enough to lift a hair over 90 kilograms
in the Earths gravitational field - so that should be enough to lift a person.
However, generating such a strong B-field proves very difficult, as you need a very high current. Thus,
the demo is about levitating a blow-up doll - which tells us that magicians most likely dont use magnetic
levitation in their tricks.

7.2

Inductance and RL circuits

Self-inductance (as opposed to mutual inductance, discussed later in the course), often called inductance
for short, is a way to quantify the ability of a circuit to fight the change in magnetic flux produced by the
circuit itself.
We know that if you run a current through a circuit, a magnetic field is created. If the current is changing
with time, then the magnetic fields are also changing with time, which causes an induced EMF in the
circuit, fighting the change.
The magnetic flux is always proportional to the current, B I. The constant of proportionality is then
the self-inductance, for which we use the symbol L:
B = LI
If we combine this with Faradays law,
E=

dB
dt

73

We find that
dI
dt
... assuming that L is constant, i.e. it is not a function of time. That is usually a pretty good assumption,
as L is, just like capacitance C, only a function of the geometry. We will soon find many other similarities
between capacitors and self-inductors.
E = L

Self-inductance of a solenoid
Say we run a current I through a solenoid, with N windings, each of which has radius r, and the length
of the solenoid is `.
First, the B-field inside, given by a quick-and-dirty Amperes law calculation, is
Ba = 0 Ipen = 0 Ia

N
`

0 N I
`
... where a is the length of the Gaussian rectangle we choose. (See the section on Amperes law for a
proper derivation.)
B=

If we attach an open surface to the current loop in the solenoid - difficult to visualize, but it will have an
area where the flux penetrates of N r2 , with N again being the number of windings. The flux is then
that area, times the B-field inside, so
I
`
This is then equal to LI by definition, as we said earlier, so we can now calculate the self-inductance L
for a solenoid:
B = N 2 r2 0

LI = N 2 r2 0

I
`

N 2 r2 0
`
As promised, it is only a function of the geometry, plus two other constants and 0 .
L=

The SI unit of inductance is the henry, H.


Vs
A
All circuits have a nonzero self-inductance, just as how they have a nonzero capacitance, and (for nonsuperconductors) a nonzero resistance.
We can show this easily as follows: all current flows in loops. All loops, no matter their shape, produce a
magnetic field, which produces a magnetic flux. Therefore all circuits have a nonzero self-inductance, no
matter how small it might be.
1H = 1

From here on, I will mostly refer to self-inductors simply as inductors. The only potential confusion is with
mutual inductance, which will not be discussed at all in this chapter, only mentioned like now. Mutual
inductance however requires multiple coils with physical proximity, which makes it irrelevant here.

74

7.2.1

Direct-current RL circuits

Say we have a very simple series circuit: a battery, a switch, a self-inductor, and a resistor.
The switch is open to begin with, so the current at time t = 0, the current is zero.
The inductor will try to fight the change in current, and so the current will ramp up slowly. In the end,
V
the current in the circuit will be simply I = , with R being the resistance of the resistor.
R
(We often calculate as if inductors have zero resistance; that is of course not true, since all wire will have
some resistance. However, whenever that resistance is necessary to care about, we can often model it as
an ideal inductor in series with a resistor.)
V
So at t = 0, I = 0, and with t , I = . What about in between the two extremes? Well, lets try to
R
set up an equation that describes the circuit.
This is a critical point of this week, and perhaps of the entire course so far. Prof. Lewin states in no
uncertain terms that the way most books do this is incorrect - the end result is correct, but the thought
behind the result is misleading and/or incorrect.
Therefore I will list both the correct and the incorrect solutions here, and talk about the differences.
This is explained in greater detail in the supplemental lecture notes for lecture 20, and is probably better
explained there, by a physics professor, rather than by a student!
So, lets see. The circuit has an inductor. That means there will be time-changing magnetic fields. That
means that Kirchoffs loop rule does not apply! This is crucial. Instead, we will have to do a line integral
~ around the loop, and set it equal to dB , not set it equal to zero!
~ d`
E
dt
To clarify: Kirchhoffs rule is a special case of Faradays law. It only holds when the change in flux is zero,
because they state conflicting results for the same integral:
I
~ = dB
~ d`
(Faradays law, always true!)
E
dt
I
~ =0
~ d`
E
(Kirchhoffs rule, NOT always true!)
With Faradays law being the one that is always correct, clearly then Kirchhoffs rule is correct ONLY
when
dB
=0
dt
With that in mind, lets move on, and use Faradays law as necessary.

RL circuit analysis and a long note on Kirchoff s second rule


We start to the right of the battery. We treat the switch as ideal, so it doesnt contribute at all (we wont
even write a 0 for it in the equation, but treat it as if it doesnt exist). After that, we reach the inductor.
~ is ZERO for the inductor, as we treat it as having no resistance (inductors are made to have as low
~ d`
E
a resistance as possible). No resistance means no electric field, and so the dot product must be ZERO!
So far, our integral is still at zero.
After the inductor, we reach the resistor. The electric field inside the resistor is in the same direction as
~ is positive, and equals IR; Ohms law works here. We then reach
~ d`
the current, so the dot product of E
the voltage source/battery. Here, the electric field is in the opposite direction of the current - the E-field
always goes from positive to negative, but the current inside a battery does not. Therefore the voltage
source contributes with a V term.

75

Finally, the sum of all these terms must be equal not to zero, but via Faradays law, to the negative of the
dB
rate of change of the flux, that is,
. All in all, the equation is then
dt
dB
0 + IR V =
dt
We have said earlier that B = LI, so if we substitute that in there, we get
d
(LI)
dt
dI
0 + IR V = L
dt
Thus, we have used Faradays law in the proper way: the left side of the line integral is the sum of the
potential differences over the circuit: the potential difference across the inductor is zero, because we treat
it as having 0 resistance, and thus no E-field can exist in it.
The right side is equal to the negative of the rate of charge of the magnetic flux, as always in Faradays
law. THIS is where the inductors term is: it is due to the flux, and is not really a potential difference
per se, as electric potential is only applicable to conservative fields - and with changing magnetic flux, we
have non-conservative fields!
0 + IR V =

We therefore have a differential equation that we can solve, and thus find the value of the current in the
circuit. First, however, lets take a look at the incorrect approach used by most physics books.
The approach taken by them is to modify Kirchhoffs loop rule, such that the potential difference (or the
dI
voltage drop, different name for the same thing) across all inductors is L , if we move in the assumed
dt
direction of the current. Thus, when we do that, and move across the loop in the same direction, and set
it all equal to zero via Kirchhoffs here invalid rule. We now ignore the electric fields, and think in terms
of potential: when we go up in potential, we add (so going through a voltage source from - to + means
we add, otherwise subtract; going through a resistor where we enter from the + side means we subtract).
dI
This means we add L
for the inductor - remember, the minus sign is from this rule (see above), we
dt
are adding the term. We then subtract IR for the resistor, and finally add V for the voltage source. This
is then set equal to 0:
dI
IR + V = 0
dt
Note that this equation is identical to the one we found before, only that it has been rearranged. If we
dI
take our proper equation and move IR and V to the same side as L , we get the above. Thus,
dt
mathematically, both ways yield the correct answer.
L

So, if they give the same result, whats all the fuss about? The problem is in what the equations represent.
In the correct equation that we found, we follow Faradays law: the integral of the electric field around
~ - all added up, equals the negative of the rate of change of the magnetic flux.
~ d`
the loop - E
While the right-hand side also has the SI unit of volts, it is not a true potential difference, as the concept
of potential does not apply to non-conservative fields.
The incorrect equation,
on the other hand, is really implying that the sum of the potential
differences
H
H
~ is zero, which it is not! The sum of the potential differences, E
~ is equal
~ d`)
~ d`,
around the loop ( E
dB
to
and nothing else! Faradays law!
dt
The incorrect equation therefore also implies that the potential difference across the inductor is L dI
,
dt
which IS what you would measure with a voltmeter, but IS NOT correct. In the ideal inductor, the E-field
~ for the inductor is also zero!
~ d`
is ZERO, so E
76

The reason a voltmeter would measure the same value is not because there is a true potential difference,
but that when you connect a voltmeter, you get a RL-circuit with the voltmeters internal resistance
providing (most of) the resistance. Therefore, we must apply Faradays law to that circuit as well, and we
); however the physical cause is again a change in magnetic flux via Faradays
find the same result (L dI
dt
law, and not a true potential difference across the inductor!
Thus, the result (the number displayed by the voltmeter) is the same, but the physical reasons are vastly
different!
If this is still confusing to you (especially everything prior to the voltmeter part, which seems even more
confusing), read (or re-read) the lecture supplement, and perhaps especially the last sentence:
If you find all this confusing, you are in good company. This is one of the most difficult and subtle topics
in this course it trips up experts all the time. Not easy!
RL circuit analysis continued
So, getting back to where we were. We want to solve the differential equation
dI
dt
which governs this circuit. We add the additional constraint (boundary condition) that I(0) = 0, that
is, the current at t = 0 is 0. Without this condition, we cannot get a unique solution to the differential
equation.
Solving it, we get
IR V = L


Rt
V 
1 e L
R
x
Alternatively, a bit easier to read, using e = exp(x):


V
Rt
I(t) =
1 exp ( )
R
L
I(t) =

If we take the limit as t 0, we find that the current is indeed zero.


I
Also, in the limit as t the current is , as we expected.
R
In between the two, the current increases exponentially, asymptotically approaching the maximum current.
During this time, the inductor sucks up energy, and stores it in the magnetic field it creates. This energy is
later freed again when the field collapses, whenever the inductor fights a current change by itself providing
a current.
L
(the unit of henry divided by ohm is indeed seconds!), the current has
R
63.2% of its maximum value. After two time constants, the current is 86.47% of

After one time constant, =


increased to 1 e1
its maximum.

If we reverse our thinking: at what time is the current x times its maximum value, for 0 < x < 1?
R
The current multiplier, so to speak, is then both x and 1 exp( t), so we can set the two equal and
L
solve for t:
R
1 exp( t) = x
L
R
exp( t) = x 1
L
R
exp( t) = 1 x
L
77

R
t = ln (1 x)
L
L
t = ln (1 x)
R
We now know how the circuit behaves as the power is connected. What happens when it is disconnected?
That is, we remove the voltage source and replace it by a short circuit (not an open circuit!).
Since the inductance will fight the change, we expect the current to go down gradually. It has to stop
eventually; power is burned in the resistor, so without a power source, a current cannot run forever.
Until then, the magnetic field in the inductor collapses, and the magnetic energy will power the resistor.
To find the quantitative answer, we need to solve the same differential equation as before, but with V = 0.
We will find the answer to be
Rt
V
exp( )
R
L
V
with V being the now-removed batterys voltage, so that R was the stable current prior to the change we
just did to the voltage source.
So if we redefine t = 0 to be a time where the current in VR , we find that indeed, the equation holds; the
current is its maximum. Same goes for t , where the current will asymtotically approach zero, in the
same way it asymtotically reached its maximum before.
L
, then, we find the current to be about 36.8% of its maximum value; after
After one time constant = R
two time constants 13.6%, and so on.
I(t) =

A common rule of thumb in electrical engineering is to take the process as complete after 5 time constants.
Thus, for an increasing current, after 5 time constants, it will be at 99.33% of its maximum, while for a
decreasing current, it will be down to 0.67%, which we consider essentially 100% and 0%, respectively.
We know that the power dissipated in the resistor is I 2 R at all times, so we can use that fact, and the fact
that the circuit now only contains an ideal inductor, ideal resistor and ideal wires, to calculate the total
energy stored. We do that do integrating the power dissipated in the resistor over all time from t = 0,
when I = Imax , to t , when the current has died out:
2

Z
Z
R
R
2
2
R = Imax R
exp(2 t)dt
E=
I R dt = Imax exp( t)
L
L
0
0
Note the added two inside the exponential, since we raised it to the 2nd power. The result of the integral
is
Z
R
L
exp(2 t)dt =
L
2R
0
So the power is
L
L
1 2
2
= Imax
= LImax
2R
2
2
This relation holds for the current through the inductor at any instant in time, so we can remove the
max, and get an equation relating the current to the stored energy at any time:
2
Imax
R

1
E = LI 2
2

78

Dual circuit elements


Notice the striking similarity between this, and the energy stored in a capacitor:
1
E = CV 2
2
This is another of many similarities between the capacitor and the inductor, which are dual circuit elements.
Capacitors store energy in electric fields; inductors store energy in magnetic fields.
Capacitors fight a change in the voltage across them; inductors fight a change in the current through them.
The current through a capacitor is related to the rate of charge of the voltage across it:
dV
dt
The voltage across an inductor is related to the rate of change on the current through it:
I(t) = C

dI
dt
If we integrate both sides of these equations, we will also find that the voltage across a capacitor is related
to the integral of the current, and that the current through an inductor is related to the integral of the
voltage across it. This had the rather profound implication that to calculate the current through an
inductor, we must know its entire history, so to speak - we must know the voltage across it from t =
up to the time where we want to know the current. Either that, or just the voltage across it from t1 to t2 ,
plus i(t1 ).
The same, or perhaps rather the opposite, is true for capacitors.
This makes capacitors and inductors state devices, that essentially have a memory, of a sort.
V (t) = L

Magnetic field energy density


Going back to the magnetic field energy, since we know that stored energy, and we know the volume of
the solenoid, andt he fact that the magnetic field is essentially exclusively enclosed with it (the B-field
outside due to the solenoid is roughly zero), we can calculate the solenoids magnetic field energy density,
by dividing the stored energy by the solenoids volume.
First, we can substitute in values for L and I that we had found previously, into the equation. Namely,
these values:
B = 0 I

N
`

we solve that for I, and find


I=

B`
0 N

We also had a value for L:


L = r2

N2
0
`

Substituting them into the equation, we have




2

 2 2 
2
2
1 2 1
B`
1
B `
2N
2N
LI =
r
0
=
r
0
2
2
`
0 N
2
`
20 N 2
1 2 B 2 `2
r 0 2
2
`0
1 2 B2`
r
2
0
79

Note however, that because weve assumed that the magnetic field outside the solenoid is zero, and that
r2 ` is the volume where the field exists, we can divide out those terms, and find the magnetic field energy
density:
B2
J/m3
20
This, too has an electrical analogue. We calculated the electric field energy density for a capacitor, which
we found te be
U=

0 E 2
J/m3
2
In the electrical case, this represents the work required to assemble charges in a certain configuration.
In the case of the inductor, this represents the work required to get the current running through a pure
self-inductor (one with zero electrical resistance). It takes work because the inductor fights the change.

7.2.2

Alternating-current RL circuits

What if we have an RL circuit and power it by an alternating current? Say we have as simple series circuit
with a voltage source of voltage V0 cos t, a self-inductor L and a resistor R.
Say the current is clockwise at one moment in time.
H
~ d`, according to Faradays law, and
We walk around the circuit and add up the potential differences E
then set those equal to the negative of the magnetic flux change.
First, we reach the inductor. As before, the E-field is ZERO inside the ideal inductor, so it doesnt contribute at all to the potential difference sum.
We then reach the resistor, where the E-field is in the same direction as the current, an so we add a
potential difference of IR.
Finally, we reach the voltage source.
The E-field is a power source is always in the opposite direction of the current, so it contributes with a
V0 cos t term.
, via Faradays law, and the
Finally, on the right-hand side, we have the change in flux ddtB = L dI
dt
equivalence via the definition of inductance, LI = B .
All in all, we have
dI
dt
And again, we solve this differential equation. Unfortunately, the solution to this equation is a bit complex
- and this result is the simplified version; the unsimplified answer is quite a bit worse!
We find that
0 + IR V0 cos t = L

V0
cos(t )
I(t) = p
R2 + (L)2
where
tan =

L
R

We can note two imporant facts here. First, the current will be sinusoidal (as we would expect), with a
peak of
V0
Ipeak = p
R2 + (L)2
... which is just the term that multiplies the cosine; this comes simply from the fact that the cosine just
moves between 1 and +1.
80

The phase of means the current will appear later than the voltage; they will be out of phase. In an
RL circuit, this lag is between 0 degrees (purely resistive) and 90 degrees (purely inductive).
Another interesting - and vitally important - observation is that L plays the term of a resistance, in
two different places, no less. The phase lag is determined by the ratio of the inductive resistance L
(properly called the inductive reactance)
to the regular resistance. The maximum current, meanwhile, is
p
determined by the resistance of R2 + (L)2 .
The sum of a resistance (symbol R) and a reactance (symbol X) is called an impedance (symbol Z).
There are more things we can deduce from this relationship. When is high, the current will be low this makes intuitive sense, because the inductor fights the change. When the charge is more rapid, there
will be less time for the circuit to win the fight, and the current goes down. Likewise, when L is high,
the self-inductance has more strength to fight the change, and so the current goes down.
It is also intuitively pleasing that when is zero, i.e. we have DC, the entire equation simplifies down to
just
I=

V0
R

... which of course is just Ohms law.


Heres a table over values of the circuit, for different angular frequencies , for values V0 = 10 V, R = 10 ,
L = 102 H and V = V0 cos t:
f (Hz)
(rad/s)
L ()
Imax (A)

100
628
6.3
0.85
32

103
6.3 103
63
0.16
81

104
6.3 104
628
0.0016
89

At low frequencies R dominates over L, with the current coming relatively close to I = VR . At a 10 times
higher frequency, L dominates, and the current largely depends upon the inductance L. After another
tenfold increase in frequency, the inductor really dominates, and the resistance is almost negligible, which
causes a phase lag very close to the maximum 90 degrees.
This formes the basis of RL filters, which can be used as low-pass or high-pass filters, depending on where
you read off the output voltage.
Say that you input a piece of music, such that the voltage source is not a simple sinusoidal wave, but a
full piece of music, constructed by tons and tons of different sinusoids.
If we pass them through an RL circuit, different frequencies will affect the current differently. Low frequencies will pass through the inductor almost unharmed, which means the voltage drop across the resistor
will be about the same as it would be without the inductor.
High frequencies - high - will be blocked by the inductor, and so the current through the resistor due to
the high frequencies will be reduced drastically.
If we then read off the resistors voltage drop, and feed that into an amplifier and a speaker, we can hear
that all the high frequencies are gone from a piece of music, since the high frequencies of the current was
removed, thus causing no voltage drop across the resistor.

81

7.2.3

More on magnetic levitation

Lets return to the magnetic levitation we looked at last week, with a coil hovering about a conductive
plate. The setup was little more than that. We run 60 Hz AC through a loop (many loops, rather) of
conducting wire. This is positioned above a conductive metal sheet, and switched on. Given enough
current, the loop will levitate.
Lets begin by looking at the case where the current in the coul is in a clockwise direction. Via Lenzs
law, we know that there will be eddy currents induced in the metal plate, such that the induced B-field is
opposite to the original one.
Therefore, the loop will have its north pole downwards, and the current loop have its north pole upwards,
so they repel each other, and the loop levitates.
Slighly later in time, the current in the loop is still clockwise, but it is now decreasing. Faradays law and
Lenzs law doesnt care about the current direction, nor the magnetic field direction, but the sign of the
rate of charge of flux, which has now flipped - it is now decreasing!
Therefore, the current loops north pole is still downwards, but the eddy current will reverse, and have its
south pole upwards. Therefore, the two will attract each other! Say goodbye to magnetic levitation. Half
the time they will attract each other!
This is clearly not the case, as this was demonstrated and did work. The secret in inductive phase lag.
The EMF induced in the plate changes instantly1 , but the induced current does not - because the plate
has a nonzero inductance! Therefore there will be a phase lag as given by
L
R
If this lag is zero, because L = 0 (which cannot happen - but say L is extremely, extremely small), then
on average, the force repels and attracts each with 50%, so there is no net force.
If the lag is greater than zero, however, we can get a repelling force more than 50% of the time. If the
L
lag is the full 90 , which can only happen as the ratio
, the force would be repelling 100% of the
R
time.
tan =

The first image shows the situation with L 0, where the induced EMF (green) and eddy current (blue)
are in phase. Note that the induced EMF depends not on the coil current directly, but the derivative of
it, which is why the green and red curves appear out of phase.
The second image shows the case where L  R, or rather when the ratio goes to infinity, and the phase
lag goes to 90 .
1

Not faster than at the speed of light, of course!

82

The reality would be somewhere in between. Any phase lag greater than 0 degrees give a net repelling
force, thus allowing levitation; a greater lag would mean a greater net repelling force.
Of course, if the conductor is in fact a superconductor, then the phase lag will be exactly 90 degrees, and
the force will always be repelling.

7.3
7.3.1

Magnetic materials
A short note on motors

The lecture begins with some information about the MIT motor contest, which was held when the lectures
were recorded. Not a lot of physics is discussed, except one point about motors and current.
The ohmic resistance of a motors windings is often very low, so a very high current will flow when the
motor is just starting up. As it starts spinning, there is an induced EMF according to Faradays law,
which will counteract the external supply. Thus the current drops drastically, and later on settles at a
value, when the rotational speed is constant.
Therefore, if we hold a motor still while it is powered, the high current may damage it, so one should take
care to avoid this.

7.3.2

Magnetic dipole moment

As we have seen with some motor designs, a current loop in a magnetic field will experience a torque.
Imagine a rectangular current loop, with sides a and b, with a current that is counterclockwise as seen
from above. Say the left and right sides are a, and the other two b.
Say there is a uniform magnetic field B towards the right
~ (or I L
~ B,
~ since I is not really a vector).
The magnetic force on a straight wire will be I~ B
For the right a side, the force will be experience a force in the downwards direction; the left a side will
experience a force upwards. The b sides will experience no force, as the cross product is zero there - they
are parallel and antiparallel with the magnetic field, respectively.
Thus, there will be a net torque on the loop, given by |~ | = IabB. If we define A = ab to be the area of
the loop, we get |~ | = IAB.
If there is an angle between the loop and the B-field, the magnitude is given by |~ | = IAB sin . Thus, we
can write the torque as a cross product:
~B
~
~ = I A
~ has the magnitude of the loops area, and the direction of the upwards surface
where the vector area A
normal perpendicular to it.
~
The magnetic dipole moment
~ is defined as this product I A:
~

~ = IA

~ then has the unit of ampere-square meters, A m2 , or equivalently joule per tesla (J/T).
So the torque can also be written as
~
~ =
~ B
Because of its definition, the vector
~ always points from the dipoles south to its north pole, which makes
it a handy tool in several places. It is sometimes known as m
~ rather than
~.

83

7.3.3

The source of magnetism in matter

First off, lets state one thing clearly: what will follow here is mostly a classical interpretation of magnetism. For the truly correct answers, quantum physics is required. However, the results we find here are
often in fairly good agreement with the correct ones.
We know that current loops create a magnetic field. For a simple circular loop, going counterclockwise as
seen from above, the field (and thus the magnetic dipole moment
~ is upwards, via the right-hand rule.
This is the source of all magnetic fields, including those inside permanent magnets!
We also know that induced eddy currents create magnetic fields, but how does that apply to permanent
magnets, which do not rely on a change in external magnetic fields, or indeed do not rely on any external
magnetic fields?
The answer is that it doesnt apply: the mechanism is different.
Up until this moment in the course, I struggled to understand how a permanent magnet could possibly
work, given what wed learned prior... And then I read something similar to what follows in the next
paragraph, and it just clicked. There are simple current loops in all materials!
In the classical model of the atom, we have electrons in circular orbits, orbiting around the much more
massive nucleus. Because the definition of a current is a charge moving over time, the electron orbits
constitute tiny current loops - they are moving in a circle!
We can define the magnetic dipole moment of an electron orbital by first finding its current in amperes,
and then using the definion for = IA.
The current is the charge, divided by its period T . The period is in turn given by the distance travelled
(the circles circumference), divided by the velocity v:
2r
v
The electrons charge is e, so we divide that charge by T , looking only at the magnitudes for now:
T =

| e|v
2r
(The current is defined by the amount of charge passing by a point, per second. Dividing the charge by
the period will get us that number!)
The magnetic dipole moment is then the current times the loop area A = r2 :
|I| =

ev 2 evr
r =
2r
2
Because the electron has a negative charge, the current is then in the opposite direction of its orbit. If the
electron orbits clockwise as seen from above, we define the current I as being counterclockwise.
The magnetic field created follows the right-hand rule of the currents direction, so the magnetic dipole
moment is then pointing upwards, according to the right-hand rule.
=

So if all electron orbits produce a magnetic moment, and all materials contain orbiting electrons, why are
not all materials magnetic? The answer is simple: the magnetic moment due to one electron orbiting is
usually cancelled by one orbiting in the opposite direction, so that the net force in a material with many,
many electrons will be essentially zero.
We know that electric fields can induce electric dipoles in materials. In case the molecules or atoms themselves are permanent electric dipoles, an external electric field will attempt to align them. The degree of
success depends on how strong the external electric field is, and on the temperature: a lower temperature
means less random thermal motion, and so the dipoles are easier to align.

84

There is a similar situation with magnetic fields. If we have an external magnetic field, it can induce magnetic dipoles in a material, at the atomic scale. In case the molecules/atoms themselves have a permanent
magnetic dipole moment, then the external magnetic field will attempt to align them. Again, the lower
temperature will lead to a greater degree of success, due to less thermal motion.
When we bring a material into an external field, then, the field inside the material from the external field.
The external field will often be referred to as the vacuum field here.
When we dealt with dielectrics, we saw how an electric field could change inside a nonconductive material;
the effect was that the electric field inside was lowered by a factor , the dielectric constant of the material.
Since > 1 for all natural materials, at least (and = 1 for the vacuum), the electric field inside the
material was always lower than the external electric field that induced the change to begin with.
That is in stark contrast to what we will now explore with magnetism. There are three possibilites, rather
than the one for electric fields above, when we deal with magnetic fields. The magnetic field inside a
material can be
1) slightly lower than the external field (diamagnetic materials),
2) slightly higher than the external field (paramagnetic materials), or
3) a lot higher than the external field (ferromagnetic materials).
Ferromagnetism is the form that most people are familiar with, where e.g. iron is strongly attracted to
magnets.
The other two forms have effects that are much weaker, to the point that they often go unnoticed. For
example, water is diamagnetic, and thus repelled by magnetic fields - only that it is repelled very weakly.
Before we talk about the three forms of magnetism, we should introduce magnetization.

7.3.4

Magnetization

Now that we know where the permanent dipoles in matter come from, lets look at a larger scale than the
atomic one, where the electron orbits are, and switch our focus to what happens when we have lots and
lots of these tiny dipoles in a material.
Suppose we have a cylindrical piece of material, with end-cap area A and height L, consisting of N individual dipoles, each with a magnetic dipole moment
~ , spread uniformly throughout the cylinders volume.
All of the dipole moments are also aligned with the cylinders axis, i.e. they all point upwards.
In the absence of an external magnetic field, what is the field due to these dipoles alone?
~ to be the net magnetic dipole moment per unit volume:
We define the magnetization vector M
X
~ = 1

~
M
V i
In the case of the cylinder, since all moments are aligned, the net moment is simply N . To get the per
unit volume part, we just divide by the volume, AL:
M=

N
AL

~ is the same as all the individual moments


Clearly, the direction of M
~ , if they are all aligned.
2
Since
~ is in ampere-square meters (A m ), and the volume in cubic meters, the magnetization is then in
amperes per meter, A/m.
The average magnetic field due to this magnetization BM is then given by
85

~
B~M = 0 M
This is the large-scale average throughout the material; close to each of the dipoles, the field will be very
different.
This principle is similar to the E-field due to several nearby point charges; in the vicinity of the changes,
the field will be very complex, but from very far away, the field will look as one due to a single point
charge, with the same charge as the sum of the charges due to the collection.

7.3.5

Paramagnetism

Lets now start looking at the different forms of magnetism.


In paramagnetism, the atoms or molecules in the material themselves have a magnetic dipole moment,
such that they are in a way tiny magnets.
In the absence of an external field, these dipoles are randomly oriented, such that there is no net magnetic
field.
Once exposed to an external field (a vacuum field), that field will attempt to align them. The success
depends on the vacuum fields strength, and on the temperature, such that a lower temperature means
easier alignment, due to reduced thermal motion.
The alignment will be such that the induced magnetic field in the material adds to the external field, and
the field inside the material is slighly stronger than the external field. This is in contrast to diamagnetism,
where the field inside was weaker.
Because of this, paramagnetic materials are attracted to magnetic fields, while diamagnetic materials are
repelled.
More quantitatively, the alignment of the atomic-scale dipoles with the external field (which well call B~0 )
~ that is parallel to B~0 . Since the induced field B~M is parallel to M
~ , then
creates a net magnetization M
B~M too is parallel to B~0 . We see, then, that the field due to the paramagnetism indeed strengthens the
external field.
The total magnetic field in the material is then given by
~ = B~0 + B~M = B~0 + 0 M
~
B
~ is proportional to the external field B~0 . The proportionality constant
In most paramagnetic materials, M
is known as the relative permeability, and is written as m :
~ = m B~0
B
Because the effect of paramagnetism (and diamagnetism) is so small, m is usually very close to 1, and so
it gets cumbersome to talk about numbers such as the relative permeability of e.g. magnesium, which is
1.000012. Because of this, we can also define the relation in terms of the magnetic susceptibility m (
being the Greek letter chi) of a material. The relation between the two is such that
m = 1 + m
Thus, instead of the relative permeability of magnesium being 1.000012, we can talk about its magnetic
susceptibility of 1.2 105 .
Both relative permeability and magnetic susceptibility are dimensionless numbers.
We can then combine the relative permeability with the permeability of free space 0 to get the magnetic
permeability m (sometimes only , which can easily be confused with magnetic moment) of a material:
m = m 0 = (1 + m )0
86

For paramagnetic materials, m > 0 . However, m is often on the order of 106 to 103 , with many
materials having even lower values. In other words, the permeability of these materials is very, very close
to that of the vacuum, which also implies that the magnetic forces will be rather low.
Note that one of the meanings of this is that field lines will prefer to go through the paramagnetic
material, versus outside it, where the permeability is lower (assuming its in a vacuum or in air, with m
essentially 1).
Lets now look at why a paramagnetic material would be attracted to an external magnetic field.
If we imagine each molecule as a tiny bar magnet, with a dipole moment
~ pointing at an angle with the
vacuum field, then there will be a torque on the molecule, such that the torque causes its poles (or its
~
dipole moment) to align with the vacuum field. This torque would then by given by ~ =
~ B.
Paramagnetic materials will be pulled towards the strongest point in a non-uniform external field.
Lets look at an example.

Say we have a bar magnet as above, with the north pole downwards, creating a non-uniform magnetic
field. Below it, we have, say just an atom, with a dipole moment that is also downwards.
We can think of this in two ways. Either we simply realize that the downwards dipole moment of the atom
means that its north pole is downwards, and thus the south pole upwards, and so north and south attract
each other.
Another way to think of it is to consider the current loop as drawn. On the left side, the current goes
into the blackboard, and the magnetic force is given by the cross product of the currents direction and
the B-field due to the bar magnet, angled downwards, so the force will be at an angle upwards/to the left.
On the opposite side, we find the magnetic force to be upwards/to the right. The net force will then be
upwards, so the material will be attracted to the bar magnet.
What if the atom were at an angle, so that the dipole moment was towards the right in this picture? There
would be a torque on it that tries to align it with the field. This is yet again a quantum mechanical effect,
so we will unfortunately just take that for granted now.

7.3.6

Diamagnetism

Most of the things we have said about paramagnetism either applies to diamagnetism, or the opposite
does. The differences will be stated below.
Diamagnetic materials have no permanent magnetic dipoles; an external field B~0 can induce magnetic
~ and
dipole moments in atoms or molecules, however. These induced dipoles will be anti-parallel to B,
thus serve to weaken the field inside the material.
Diamagnetic materials have a magnetic permeability m < 0 , and therefore a relative permeability m < 1
(since m = m 0 ), meaning m < 0.
87

Here, the magnitude of m is even smaller than that of paramagnetic material, perhaps on the order of
105 to 109 .
Just as how field lines prefer to go through paramagnetic materials, because of their (slightly) higher magnetic permeability than the surrounding vacuum/air, field lines prefer to avoid /go around diamagnetic
materials, as the permeability is lower than that of the vacuum.
This effect is striking in superconductors, which are essentially perfect diamagnets, with m = 1 and
m = 0, such that they repel all fields entirely, except for a very thin layer near the surface (which is
usually less than 1 micrometer deep, given by the London penetration depth, which is likely not covered
by this course).
All materials are diamagnetic; however, for those that are also paramagnetic or ferromagnetic, as we can
see on the tiny susceptibility of diamagnetic materials, those other effects will usually overshadow the
diamagnetism.
As stated above, the induced dipoles weaken the internal magnetic field, and so they oppose the external
field. Diamagnetic materials will therefore be repelled by external magnetic fields, as their dipole moment
will always tend to oppose external fields.
Due to the low susceptibility, the effects of diamagnetism is usually very small, and often hard to even
notice, at least in everyday life. It does however result in water, a diamagnetic material, being repelled by
very strong magnetic fields, to the point that phycisists have levitated a living frog (and also mice) by the
repelling force on the water inside it. Doing so took a magnetic field with a strength of roughly 16 tesla!

7.3.7

Ferromagnetism

Ferromagnetism is perhaps the most interesting form of magnetism, because it forms the basis of permanent magnets.
Like paramagnetic materials, ferromagnetic materials also have permanent dipole moments.
However, (yet again!) for quantum mechanical reasons beyond this course, these dipoles are grouped into
domains, which are on the order of 106 to 104 m in size (very roughly), such that even large domains are
essentially microscopic. Inside each of these domains, all the individual dipole moments will be completely
aligned in one and the same direction. However, since there are many domains in a piece of material, there
may still not be a net magnetic moment. Each domain has a separate alignment, which may cancel out
with another nearby domain, to provide no net moment.
There may be up to on the order of 1017 to 1021 atoms in a single domain.
As weve seen before, when we expose the material to an external magnetic field, the dipoles (and thus the
domains) will try to align along the external magnetic field; the degree of success depends on the strength
of the external magnetic field, and on the temperature; the lower the temperature, the easier alignment
is, due to lower thermal motion.
The magnetic field inside a ferromagnetic material may be thousands of times stronger than that of the
external field, that is, m can be 1000 or more. Thats in fairly stark contrast to paramagnetic and diamagnetic materials, which had factors more on the order of 1.001 or less!
In addition, the domains in a ferromagnetic materials can stay aligned, to a certain degree, after the removal of the external field, creating permanent magnets.
This alignment can be removed either by sudden shock (such as hitting the material with a hammer), or
by heating the material; in either case, the domains will randomize again and the residual magnetism may
well go away entirely.

88

When we heat a ferromagnetic materials, at a certain temperature, the domain completely fall apart, and
disappear. The point where this happens is nown as the Curie temperature, or the Curie point. Irons
Curie temperature is 1043 degrees K, or 770 degrees Celsius.
At this point, the material becomes paramagnetic instead, in a very abrupt change.

89

Chapter 8
Week 8
8.1

Hysteresis and electromagnets

Last week, we talked about magnetic permeability, the relative permeability M of materials, and briefly
about ferromagnetism, the only form of magnetism where M has a significant value. We will now talk
about how strong a magnetic field we can theoretically get in a material, and about hysterisis in ferromagnetism - a phenomenon where the past state of the material changes its permeability, in a nonlinear fashion
(so the magnetization and permeability are no longer directly proportional to the external magnetic field).
If we were to align 100% of the atomic dipoles in a material, what would the strength of that field be?
Lets start with a slightly different derivation, that we will later use to answer the above question.
Say we have a hydrogen atom, consisting of a proton at the center, and an electron orbiting with a radius
R, in the classical model.
If the electron, as seen from above, orbits in a clockwise fashion, the current - we have a moving charge,
so there is a current - will by convention be in the counterclockwise direction.
The magnetic dipole moment
~ is then given by IA, where A = R2 is the area of the current loop. The
direction will be, via the right-hand rule, upwards.
Lets then try to find the value for I, which takes a bit more work.
If the electron is orbiting in a circle around a center point, there must, according to classical mechanics,
be a centripetal force on it, acting radially inwards. We can use Coulombs law to find the electric force,
2
:
and set that equal to the centripetal force mv
R
|e|| e|
me v 2
=
R
40 R2
e2
v =
40 Rme
e
v=
2 0 Rme
2

The time to go one loop around (the period T ) is then the distance around, 2R, divided by the velocity:

4R 0 Rme
T =
e
11
If we stick some numbers in there, using R = 510
for the orbit radius (the Bohr radius, about 5.291011
6
16
meters), we find v 2.3 10 m/s and T = 1.4 10
s - rather incredible numbers!

90

We can now calculate the current: current is defined as the amount of charge passing a point per second
1
- so if we choose a point along the orbit, the electron will pass it
times per second. We can
1.4 1016
therefore calculate
e
1.602 1019
=
1.144 103 A
T
1.4 1016
We find a current of about a milliampere - due to a SINGLE electron orbiting! In a copper wire, a current
of a milliampere would take roughly 6.24 1015 electrons passing by a point per second (a coulomb is
approximately 6.24 101 8 electrons, the reciprocal of the elementary charge 1.602 1019 C) and in this
case ONE electron is doing all that, because of its incredible velocity around the tiny orbit.
I=

The magnetic moment is then = IA, as we stated, so


= IA = IR2 8.98 1024 A m2
This value is known as the Bohr magneton. However, its value isnt really what we just found, but rather
the value of the Bohr magneton B is
B =

e~
9.274 1024 J/T
2me

... which is a result from quantum mechanics, which is where ~ comes from (the reduced Planck constant).
The value cannot be found accurately using classical physics.
The magnetic moment due to all electrons in orbit around an atom can only ever be a multiple of the
Bohr magneton - never anything it between. This is one example of the quantization (only a certain set
of discrete values are allowed) that gives quantum physics its name.
In another quantum weirdness effect, electrons themselves produce a magnetic moment due to a quantum
effect called spin, often thought classically as a literal spin of the electron, around its own axis.
The net magnetic dipole moment of an atom is then the vectorial sum of all the orbital dipole moments,
due to the electrons orbits, and of the spin dipole moments. The result of that is that most atoms and
molecules have dipole moments of either one or two Bohr magnetons. The limit on the maximum field
strength in a material (due to the material itself), then, will be on the order of 2 Bohr magnetons, times
the number of atoms we can manage to align all in the same direction.
When we expose a material to an external magnetic field, the field inside will be
~ = B~0 + B
~0
B
~ 0 is the field due to the aligned dipoles.
where B~0 is the external field (the vacuum field), and B
~ = M B~0 = B~0 + M B~0 . However, as we
For paramagnetic and diamagnetic materials, we then find that B
will soon see, that relationship does not hold so easily for ferromagnetic materials. The reason is that in
ferromagnetic materials, an external field can align dipoles so well that essentially all of them are aligned.
At that point, clearly, increasing the external field will have no effect on the materials dipole alignment ~ 0 is at a maximum.
they are already aligned, and so B
When all dipoles are aligned, and an external field cannot increase the materials induced field further, we
call that saturation.
Say we have a material where each atom contributes with 2 Bohr magnetons, with a density of N = 1029
atoms per cubic meter in our material.
Inside the material, a solid material, the atoms will be nicely packed together. We can imagine them as a
lot of small current loops in a grid; each column of the grid (which is really 3-dimensional, of course) then
looks a lot like a solenoid - many small windings, each with a current through them. Since weve said that
all dipole moments will be aligned, we can indeed approximate the B-field strength using the equation for
91

a solenoids magnetic field.


Each column will have an area A and a height `; lets ` = 1 meter, for simplicity. Each column is then
A` = A m3 in volume. The equation for the B-field inside a solenoid is
N
`
So somehow, we need to find a way to put B inside there. Note that N in the above equation is the
number of windings, while N is a different symbol (look closely), which is the number of atoms per cubic
meter.
If we take the area (not the volume!) A times N, we get the number of atoms per meter. Since each atom
is really a winding of this solenoid, we can substitute this term in for N` in the solenoid equation, since
that term means nothing but the number of windings per meter. Thus, we have
B = 0 I

B = 0 IAN
A-ha! But = IA, so we have
B = 0 N = 0 (2B )N
... using 2 times the Bohr magneton B as the value for the dipole moment. We can thus calculate the
B-field strength by putting numbers in the above equation:
B (4107 )(2 9.3 1024 )(102 9) 2.34 T
The units of Wb/Am times A m2 times m3 check out, and equal the unit of tesla (since 1 tesla is 1
Weber per square meter).
So for the values we have chosen, with 2 Bohr magneton dipole moment per atom, and 102 9 atoms per
cubic meter, about 2.3 tesla is the value we find for the maximum B-field strength due to the material
itself.

8.1.1

Ferromagnetism and hysteresis

Lets now look at the B-field strength inside a ferromagnetic material. Lets say that it has a relative
permeability M = 1000, to start with. That will only hold for a part of this section, however!
If we try to plot this, we will be unable to do so on a linear scale. Say we plot the vacuum field B0 on the
x axis, and the induced field B 0 on the y axis. Since M = 1000, for each unit we go towards the right, we
need to go 1000 units up! That is clearly not possible to draw in an easy way, on a to-scale linear plot.
Nevertheless, say we do this on a linear plot, just not to scale.
In the beginning, the slope will indeed be the m of 1000, but after a while, so many of the atomic dipoles
will have aligned, that the effect of further increasing the external B-field lessens. There then comes a
point where increasing it has no effect at all on B 0 , when all (or close to all) dipoles are aligned; this is
what we call saturation. If we do this on the material where we calculated this above, then B 0 will have a
magnitude of about 2.3 tesla when we reach the saturation point.
At this stage, then, the effect increasing the external field will have on B 0 is essentially zero.
The total field B = B0 +B 0 inside the material can however always be increased, by increasing the strength
of the external field; B 0 is the only field that is at its maximum strength.

92

Say we start with a nonmagnetic ferromagnetic material, and stick it inside a solenoid. Bvac , the vacuum
field, we can always calculate: we know how to find the B-field inside a solenoid, all we need to measure
is the current through it.
So we start at the origin, and increase the vacuum field, until we reach to point in the top right (in the
figure above), where B 0 saturates.

We then reduce the current in the solenoid to 0, and Bvac reaches zero (at point P , just above), as expected.
However, B is not zero, because the ferromagnetic material has now become a permanent magnet!
Note that we then see that B has two possible values for Bvac = 0!
But wait, theres more!
If we now change the direction of the current in the solenoid, the vacuum field will reverse direction, to
the left side of the y axis. If we increase the current (in the new direction) until we reach point Q, we
see that the net field in the material is now zero! The domains in the material are still pointing to the
direction they were in at point P , which is now in the opposite direction of the vacuum field, since we just
reversed that.
If we then move the current back to zero again, the vacuum field goes away, and we end up at point S
(also above). The material is now a permanent magnet again, but this time in the opposite direction as it
was before, since we coerced the domains to reverse direction.

93

Finally, if we increase the current in the original direction again, we go from S back up to the point it the
top right.
Note that if we were to decrease it again now, we would get back to point P ! We can not get back to the
origin again simply by reversing the current.
This also implies that given a material, if we expose it to an external field, we cannot calculate what the
field inside will be - since it depends upon the history of the material.
Looking at the relative permeability M , then, shows why we cannot use a fixed value for ferromagnetic
materials. At the two points where the hysteresis curve meets the x axis (outside the origin) , M must
be zero, because there is a vacuum field, but the B-field inside the material is zero! B = M Bvac , so M
must be 0 if Bvac is greater than zero, but B is exactly zero.
Consider the situation in the upper-left quadrant. The vacuum field is towards the left, but the B-field
inside the material is towards the right - so M must be negative! The same applies to the lower-right
quadrant.
Demagnetization
This raises the question: can we get the material back to being unmagnetized? The answer is yes, we can.
If we heat it above its Curie point, that will destroy the magnetic domains, and when it cools back down,
it should be unmagnetized. We can also possibly do it by shock - hitting it with a hammer, etc.
Another nice solution is to pass a sinusoidal current through it, with decreasing current, which is called
demagnetization.
Because Bvac is linearly proportional to the current I through the solenoid, if we pass a sinusoidal current
through it, the B-field inside the material will move along the hysteresis curve we just saw.
However, if we gradually decrease the currents magnitude, we gradually move inwards, so that the
points where we cut the x axis move closer and closer to the origin, and when the current reaches zero,
the material has been demagnetized.
Ferromagnetic materials and field changes
If we have a permanent magnet (top left in the illustration below), and we move a bit of nonmagnetized
ferromagnetic material near it, what will happen? I think we all know, in part even since childhood, that
the ferromagnetic material will be attracted to the magnet.

Using our new knowledge, we also know that the atomic dipoles inside the material will align to support
the external field, which in turn causes the net field to strenghthen in the left/right plane, and so there
will be an induced south pole near the north pole of the magnet. Due to the strengthened field inside
the material, the external field lines are sort of sucked in into the material (the pink lines).

94

Because of this, the external field will weaken towards the left of the picture, where two field lines are drawn.
This can be (and is, in the lecture) be shown experimentally, by tying a ferromagnetic nail to a thread,
with a magnet hanging down. The nail is tied such that it is held up, against gravity, by the attracting
magnetic force.
When we move another ferromagnetic object (a wrench, in the lecture) towards the other side, the field
lines are sucked in to the wrench, and the field weakens around the nail, and it falls down, without even
being touched.
It is also demonstrated that while this effect also happens with paramagnetic materials, such as aluminium,
it is so positively tiny that the nail stays up with no problems.

8.1.2

Maxwells equations

Now that we have learned about magnetic materials, we can finally write down the complete set of
Maxwells equations (in integral form). There is only one of the four that changes, and that one is
Amperes law.
Looking at the other three equations (below), there is no reason to suspect that magnetic permeability
would cause any change. However, Amperes law uses a 0 term, which is the permeability of the classical
vacuum. If the permeability of the medium in question is different, then we need to use the mediums
permeability, by adding in a M , so that we have M 0 instead. All in all, the four equations are, in
integral form (the only form taught in this course):
{

~ = Qf ree
~ dA
E
0
{
~ =0
~ dA
B
I
~ = dB
~ d`
E
dt


I
d
E
~ = M 0 Ipen + 0
~ d`
B
dt

(Gausss law)
(Gausss law for magnetism)
(Faradays law of induction)
(Amperes law)

95

8.2

Review for Exam 2

The list of topics covered on the exam, as stated on the transparency in lecture:

Magnetic fields & field energy


Lorentz force
Cyclotrons
BiotSavart Law
Solenoids
Amperes law displacement current
Lenzs law
Faradays law of induction
Motional EMF
Induction eddy currents
AC dynamos
3phase current
Induction motors
Aurora
Magnetic levitation bullet trains
Selfinductance
RL circuits

Quite a lot! Of course, knowing all the material from the first exam is most likely necessary, too, so the
full list ought to be everything learned in the course so far.
Everything covered in the review was, well, a review... So I didnt take any additional notes.

96

Chapter 9
Week 9
9.1
9.1.1

Transformers, Car Coils and RC circuits


RC circuits

We previously looked at series RL circuits - circuits with an inductor and a resistor, plus a power source
and a switch. We will now look at series RC circuits - so the only difference is that we replace the inductor
with a capacitor.
Thus, we have a circuit of a capacitor, resistor, battery with EMF V0 , and a switch, going around the
circuit on the blackboard clockwise (not that order matters!).
The two nodes the capacitor are connected to are labelled VA and VP , so the capacitor voltage is
VC = VA VP
Intuition goes a long way when it comes to solving this circuit, especially for the simple cases t = 0 and
t , just after we close the switch, and when it has been closed for a very, very long time.
Capacitors take time to charge up to a voltage, just as how inductors take time to charge with a current.
The two circuit elements are duals - as mentioned previously. That means that while inductors take time
to let a current through, with a EMF that goes down to 0 with time, capacitors are the opposite: the
current goes to 0 with time, while the voltage increases to its maximum.
V0
).
R
At t , the current must die down, at which point VC = V0 (otherwise part of the batterys EMF must
V
be across the resistor, which means I =
6= 0). So at this point, VC = V0 and I = 0.
R

So at t = 0, we expect the capacitor to be unchanged (VC = 0), but charging (I = Imax =

What about in between? Well, to find the correct answer, lets get serious and analyze this circuit properly.
The lecture questions asks us to use Faradays law, which is unnecessary since there is no changing magnetic
flux (Kirchhoffs rule works), but lets do so.
We start before the battery and go clockwise, adding whereever the electric field is in the direction we
are moving, subtracting otherwise. We find:
Q
dB
+ IR =
=0
C
dt
Alternatively, adding when we go up in potential, subtracting when we go down:
V0 +

+V0

Q
IR = 0
C

97

The equations are of course the same, with both sides having been multiplied by 1 in the second case.
dQ
We can write I as
, which it equals by definition. We then get, rearranged,
dt
Q
dQ
+R
V0 = 0
C
dt
... which is a differential equation in one variable. The solution to it, given the initial condition VC (0) = 0,
is



t
Q(t) = V0 C 1 exp
RC
Knowing the charge is generally not that useful, but knowing the current is; since current is the time
derivative of charge, we differentiate this result with respect to t:



1
t
dQ
= I(t) = V0 C
exp
dt
RC
RC


t
V0
exp
I(t) =
R
RC
Q
To find the voltage across the capacitor, we simply use the definition of V = , and the result for Q we
C
already found. We divide out the C, and find



Q(t)
t
VC (t) =
= V0 1 exp
C
RC
Much like RL circuits, RC circuits also have a time constant (tau). In the case of RC circuits, its given
by RC, the unit of which is seconds:
Volt
Ampere
Coulomb
Farad =
Volt
Ohm =

Volt Coulomb
Ampere Volt
Coulomb
=
Ampere
Ampere second
=
Ampere
= second

Ohm Farad =

1
As with RL circuits, after one time constant , the values have either gone down to of their initial value,
e
1
or up to (1 ) of their maximum, depending on in which direction they are going.
e
So the potential difference across the capacitor after one time constant = RC is 63.2% of its maximum,
while the current is down to 36.79% of its initial value.
If we now flip the switch in the circuit, such that the capacitor and resistor are in series with nothing else
in there, the capacitor will dischange and its stored energy will be burned as heat in the resistor.
The equation governing that circuit will be
Q
IR = 0
C
98

If we define I to be positive in the opposite direction, this turns into


Q
dQ
+R
=0
C
dt
The solution for that differential equation is


t
Q(t) = V0 C exp
RC
We use the same tricks (dividing by C and differentiating with respect to t, respectively) to find VC (t) and
I(t):


t
Q(t)
= V0 exp
VC (t) =
C
RC




dQ
1
t
= V0 C
exp
dt
RC
RC


V0
t
= exp
R
RC


V0
t
dQ
=
exp
I(t) =
dt
R
RC
So the current will now flow in the counterclockwise direction (remember that we redefined a positive I as
counterclockwise) as opposed to clockwise back when the battery was charging the capacitor.
So to summarize, when charging a capacitor in a simple RC circuit, the following equations apply:


t
V0
exp
I(t) =
R
RC



t
VC (t) = V0 1 exp
RC
When discharging, the following equations apply (assuming it is charged to voltage V0 ):


V0
t
I(t) =
exp
R
RC


t
VC (t) = V0 exp
RC
... where the current is in the opposite direction when discharging.

99

9.1.2

Transformers

Transformers are used to transform AC (and only AC!) currents and voltages.
To construct one, we take a coil, which we call the primary, with N1 turns, and self-inductance L1 .
We then add a second winding, electrically isolated, around the same core. This winding is called the
secondary, and has N2 turns and self-inductance L2 .

Since they are wound together (while electrically isolated), there is magnetic flux coupling between the
two. As the current changes in the primary, via Faradays law, a current is induced in the secondary.
So, we apply Faradays law to the primary side. We begin at the top, and first go through the primary
coil, where we know there is no electric field (since we treat the inductor as ideal, R = 0). Thus that
doesnt contribute to the integral at all.
We then reach the voltmeter, where the current is opposing the direction we are moving around the circuit,
so it contributes with a V1 term. According to Faradays law, that is then equal to the negative of the
rate of charge of magnetic flux, or
dI1
dt
... via the definition of inductance. This is a simplified result, where the proper result also has a term
related to the mutual inductance between the two coils, and the current in the secondary.
V1 = L1

We do the same on the secondary. The current is clockwise here as well, so we get a positive contribution
at the voltmeter:
dI2
dt
Again, we ignore the contribution of the mutual inductance.
V2 + 0 = L2

dI
The L are, as usual, the induced EMF due to Faradays law. Therefore, we can also write the equations
dt
as
dI1
dB
= N1
dt
dt
dI2
dB
|V2 | = E2 = L2
= N2
dt
dt
Due to the way the coils are constructed, assuming we have perfect magnetic coupling, the flux term is
the same for both equations! Therefore, we find that

V2 N2
=
V1 N1
|V1 | = E1 = L1

The result speaks for itself: we can choose the number of windings on each coil such that we create huge
voltages at the secondary - step-up transformation - or, we can do the opposite, and get a step-down
100

transformer instead.
Because ideal transformers are lossless, we can show that as power is conserved, a transformer that steps
down voltage will step up current, and vice versa. Thus, a transformer with N2 /N1 = 100 will draw a
much larger current on the primary than it delivers on the secondary, since the voltage at the secondary is
larger, but the power must be the same on both sides. If we flip the primary and secondary, the opposite
is true: with the flip, the output voltage is very low, but the current is much greater than the current
drawn at the primary.
The product V1 I2 is always equal to V2 I2 in an ideal transformer; more on this below.
Keep in mind that these voltages are peak values (or RMS values); transformers only work with AC, since
there would be no change in magnetic flux for DC.
Transformation is used a lot in power distribution. As discussed earlier in the course, power transfer is
done at very high voltages, which means we can power with less I 2 R losses, as I becomes smaller for a
given power transferred.
The power is generated, upconverted to a few 100 kilovolts or so, and fed into the power lines. When
it reaches a city, it is stepped down, but is still in the kilovolts range. Then, close to homes, it is again
stepped down, to the 110 120 volts or 230 240 volts used around the world.
Lets return to talking about currents in transformers. The currents are proportional/inversely propertional
to the windings ratio if a few things are true - that by no means are always true. They are:
R  L in both the primary and the secondary
No energy is lost to eddy currents, e.g. in the transformer core, which is often iron
The flux coupling is ideal, so that the flux in the primary equals the flux in the secondary
In these cases, power is conserved, so V1 I1 = V2 = I2 .
If all of these are true, then it is true that

I2 N1
=
I1 N2

9.1.3

Spark plugs / car coils

We can use our newfound knowledge on transformers to understand how the spark plug in a car works.
(A spark plug is used to ignite the fuel-air mixture in the engine.)

The car battery is a 12-volt DC battery. However, by placing a switch in series with the transformers
dI1
primary, we can still get a huge
value by letting the current built up, with the switch closed, and then
dt
opening the switch, thus instantaneously breaking the current.
L1 will then try to oppose this change, and generate an EMF in response. However, the magnetic flux
change responsible for the EMF, via Faradays law, is shared by the secondary.
101

The secondary is then wound such that N2  N1 , and the induced EMF in the secondary will be way
higher than the already high value in the primary.
Therefore, if we simply connect two wires to the secondary, and place them a short space apart, we will
get sparks between them if the electric field between them exceeds the roughly 3 million volts per meter
that causes electric breakdown in air.
For a distance of, say 3 mm, that voltage would be 9 kilovolts. We cound generate a hundred times that
with this method, but that would not be necessary for this usage.

9.2

Driven RLC circuits and resonance

We have discussed RL and RC circuits previously, and will now discuss RLC circuits (or LCR, or LCR,
etc.), i.e. series circuits with a power source, a resistor, a capacitor and an inductor.
We will only talk about the case of a sinusoidal voltage applied to the circuit.
Walking clockwise from the top-left corner in the circuit on the blackboard, using Faradays law, we get
Q
dI
+ 0 + IR V0 cos t = L
C
dt
The zero term comes from the self-inductor: we treat it as ideal, so the E-field inside is zero, and thus it
has no real voltage drop. Measuring across it will show a voltage, which is rather an induced EMF than
a true potential difference.
dI
with
We want to write everything in terms of Q, plus the constants (R, L, C and V0 , so we replace the
dt
dQ
its other form, via I =
, and move it to the other side at the same time:
dt
dQ
d2 Q
Q
+R
+ L 2 = V0 cos t
C
dt
dt
The way to solve this second-order differential equation wasnt shown in lecture, as that is the subject of
a course not yet taught at this stage at the MIT students studies.
The solution for the current in the circuit is given as
V0
I(t) = q
R2 + L


1 2
C

cos (t )

where
L
tan =
R

1
C

As before, the expression multiplying the cosine is the maximum current possible through the circuit, at
a particular V0 .
This solution is only valid for the steady state. When we power such a circuit up, the sine wave will be
multiplied by an exponential term, which turns into 1 once steady state is reached.
We can now define two (or four, perhaps) new terms. The term in the numerator of the tangent is called
the reactance, symbol X, and is effectively a frequency-dependent resistance:
1
C
Combined with the resistance, the reactance gives us the impedance, symbol Z:
X = L

102


Z = R2 + X 2 =


2
1
R2 + L
C

The two parts of the reactance are the inductive reactance XL = L and the capacitive reactance
1
XC =
, respectively.
C
Both reactance and impedence have the units of ohms, and can in many cases be treated as resistances,
as long as the input is sinusoidal.
The solution has a few quite interesting subtleties. One of them is resonance: there will be a value for
where the current is at a maximum. We can find this by doing some simple math (no calculus optimization
required). Looking at the expression for Imax :
V0
Imax = q
R2 + (L

1 2
)
C

... we see that no matter what, the R term will limit the current. Since the reactance is squared, we
cannot get it negative, only zero. So it will be at its lowest when
1
=0
C
2 CL 1 = 0
L

2 CL = 1
1
=
LC
This is known as the resonance frequency, often given the symbol 0 .
At resonance, the current is given simply by
V0
R
The current will also be in phase with the driving voltage, as becomes zero.
Do note, however, that when not at resonance, can be either greater than zero (which causes a phase
lag: current lagging the voltage), or smaller than zero, in which case the current leads the voltage. The
latter is due to the capacitive reactance, while the former is due to the inductive reactance.
Imax =

Lets now look at how the circuit behaves at different values of , the driving frequency, for fixed values
of R, L and C.
As the frequency is extremely low, 0, the current will go to zero, due to the capacitor. At that point,
our current really goes to DC, and so we charge the capacitor to V0 , and then the current stops. This can
1
of course be seen in the equation, from the
term.
C
As , the current again becomes zero. This time, the inductor is responsible. With the extremely
high dIdt, the inductor has a lot of power to say no to a getting a current through it.
We already saw that the maximum possible current is when the reactance becomes zero, at the point
where the inductive and capacitance reactances take each other out.
Between the two extremes, we got a plot that looks something like this (on a log scale):

103

As noted in the graph (in pink), below the resonance frequency, the capacitor plays the main role in deciding the current, and the phase in also in favor of the capacitor, such that the current leads the voltage.
Above the resonance frequency, the inductor dominates. The phase is in favor of the inductor here, such
that the voltage leads the current.
Exactly at the resonance frequency, there is no phase difference between the voltage and current, and the
current is at a maximum, as stated previously.
1
There are two points along the plot above where the current is 0.707 ( ) times its maximum value.
2
These two points are known as the half-power points: the power I 2 R will be exactly half of its maximum
1
when the current is of its maximum.
2
If we label the lower half-power point 1 and the upper 2 , the width 2 1 between these points is
known as the bandwidth, , which for a series RLC circuit is
=

R
L

(the derivation of this is not shown.).


The quality, or quality factor Q, not to be confused with charge, is given by
r
1
L
L
1 L
0
=
=
=

R C
LC R
R LC
... if we flip the fraction and multiply, instead of the unreadable 4-level fraction division.
Clearly, this is inversely proportional to the bandwidth, so the narrower the bandwidth, the higher the
quality factor, and vice versa. A high-quality resonator will have a very narrow and sharp peak.
Resonant circuits can be used in radios, TVs, etc, to select a station. The antenna will recieve a wide
variety of stations all at the same time, and a finely tuned, high-Q resonant circuit with adjustable 0
(often using an adjustable capacitor) is used to filter out one of them.
In reality, it is more complex, but the principle is absolutely used as part of the full solution.
Metal detectors also use resonant circuits. They are often built using two coils, finely tuned, and at
resonance. When a piece of metal comes near, eddy currents are induced in the metal, and so the magnetic
flux coupling between the loop changes, the mutual inductance between the two coils is disturbed, and the
system goes off resonance, and sounds the alarm.
The metal detectors at airport use this principle in a very simple matter: you walk in between the two
coils, having one of them on each side of you. If the circuit is sensitive enough, a piece of metal moved in
between them can disturb the resonance enough that it is detectable.
104

9.3
9.3.1

Traveling waves and standing waves


Traveling waves

Say we have a very simple equation, such as


1
y= x
3
and we want to make it move towards the right, at a certain velocity v = 6.
All we have to do to make this happen is to replace x with x vt, so we have
1
x
y = (x vt) = 2t
3
3
At t = 0, this simplifies down to the original equation, At t > 0, the equation will be shifted, and if
graphed, it will have moved towards the right.
Replace the minus sign in x vt with a plus sign, and it will instead move towards the left, again with
velocity v.
Lets look at a different function: y = 2 sin(3x). This time, we have a proper wave.

On this plot, we measure the wavelength (Greek lowercase lambda) as the length of a full cycle, so the
time between one peak and the next, or one trough and the next (or any two similar points).
2
In this example, the wavelength is =
.
3
2
We also introduce the symbol k for the wave number, defined as k =
, so k = 3 for this case. So k is

then the number multiplying x in the sine function.


Now, can we make this wave move, in the same manner as we did with the line? Indeed, we do as we did
before: we replace x with x vt. For v = 6 m/s again, we find
y = 2 sin(3(x 6t))
There is more information in this equation: the 2 multiplying the sine gives us the amplitude. The 3, the
2
2
wave number, tells us the wavelength (via k =
, so =
), and the 6 tells us the velocity. The minus

k
sign also tells us that it is travelling towards the right. Thus the equation is of the form
y = A sin(k(x vt))
If we attach a rope to a spinning wheel, we can create a traveling wave, propagating in the rope.
2
Say the wheel rotates with angular frequency . The period of one wave will as always be T =
. One

wavelength is as always the length between two peaks of the wave, or the distance traveled during one
105

v
1
period, which is simply = vT , which also equals = , where f is the frequency in hertz (f = , and
f
T
v
= 2f ). Its then also true that f = .

We can write the equation in a different form:


y = A sin(kx t)
Via the conversion formulas we just found, this too contains all the information. k gives the spatial information, such as . gives the angular frequency/frequency/period/velocity, either alone or combined

with k: v =
k
As an example, lets look at the equation
y = 4 sin(2(3x 9t))
If we multiply it out, we get
y = 4 sin(6x 18t)
... which tells us k = 6 and = 18, traveling towards the right.
2
2

=
18 = ,
Thus the amplitude is A = 4, the velocity v = = 3 m/s. The wavelength is = vT = v
k

3
3

9
and the frequency f =
= .
2

9.3.2

Standing waves

Say we have a wave


y1 = y0 sin(kx t)
which is the travelling towards the right. However, we also have a second wave, identical except for a plus
sign, which tells us it is travelling towards the left:
y2 = y0 sin(kx + t)
Via trigonometric identities, the sum of these is
y = y1 + y2 = 2y0 sin(kx) cos(t)
This is a standing wave. If we plot it at various values of t, we find that some points, called nodes, are

completely fixed: their y-value is fixed. These points occur at multiples of .


2
All points along the curve that are not nodes will move up and down, but not towards the left or towards
the right - note that there is no x t anywhere any more. So the sinusoid will essentially bob up and
down, while standing still.
We can make standing waves in a similar fashion to making traveling waves, but we need to get the frequency of them just right.
Say we have a length of string, and we hold it fixed at the ends, so the ends become nodes.
It is held by two people, and one of them is moving their node up and down slightly (which makes it not
a node, but with small movements, we can ignore that). When the wave reaches the end of the string, it
will reflect back towards the first person. When it comes back to the first person, it will again reflect, etc.
If the conditions are right, these reflections can support each other, and the amplitude will increase. This
happens at a set of frequencies, which we call the resonance frequencies, in a similar fashion to RLC
106

circuits. However, in this case, there are in theory an infinite amount of such frequencies.
For the lowest possible such frequency, which we call the fundamental (or first harmonic), there will be
nodes at the ends of the string, but nowhere else. Thus the length of the string will be half the wavelength
- or, the wavelength will be twice the strings length.
For the next possible frequency, the second harmonic, the frequency is exactly twice that of the first harmonic. Here, there will be a node at the exact center of the string, which will not move at all. So now we
have three nodes: one at each end, plus one at the exact center.
For the third harmonic, the frequency is exactly three times the first harmonics frequency, and there will
be another node, splitting the string into 3 parts that move, with a total of 4 nodes (one at each end, one
at 1/3 of the strings length, and one at 2/3 the strings length).
There are always n + 1 nodes for the nth harmonic, assuming we count nodes at the string ends.
The fundamental, denoted by 1 , is then 2L, where L is the length of the string, as we stated above.
v
v
.
The frequency of the fundamental, f1 , is given by f1 = in general, so in this case, f1 =

2L
For the second harmonic, 2 = L, which follows from f2 = 2f1 (or from drawing it out, in which case it is
immediately obvious).
In general, then,
n =
fn =

9.3.3

2L
n

v
nv
= nf1 =
n
2L

Musical instruments

When we excite a string in an instruments whether it is a string inside a piano being hit by a hammer,
a guitar string hit by a guitar pick, or a violin string hit by a bow it will be exposed to a whole range
of frequencies. However, it will tend to oscillate only in the resonant frequencies, which is why we can get
well-tuned sound out of such instruments.
So a ring that has a resonant frequency of 400 Hz, when struck, will tend to oscillate at 400 Hz, 800 Hz,
1200 Hz etc., all at the same time.
So for designing a stringed musical instrument, a key equation is
f1 =
v is decided by the strings material and tension:
s
v=

v
2L

Tension
Mass/length

(proved in the next MIT physics course, 8.03 Vibrations and Waves, not here).
A regular steel-stringed guitar usually has roughly the same tension for all strings, something on the order
of 7-10 kg-force of tension per string for a 6-string guitar in standard tuning, with standard string gauges.
The difference between the strings is then mostly in their material - indeed, they usually all differ in thickness, from 0.010" for the thinnest E string, to 0.046" for the thickest. (Different guitarists have different
tastes; 0.009"-0.0042" is about the thinnest anyone uses, and they can go up to about 0.013"-0.056", still

107

in standard tuning, which then causes much more tension).


The tension is what we adjust when we tune a stringed instruments - we wind the string either more (more
tension) or less, until it resonates at the (fundamental) frequency we want.
When we play the instrument, we change the strings length, by shortening it with our fingers.
On fretted instruments, like guitars and electric basses, the string is shortened by a metal bar (the fret)
going across the neck; there are usually 17-24 such frets on a guitar.
The instruments sound is then generated as the string vibrates, which pushes and pulls on the surrounding
air, creating pressure waves in the air. Those pressure waves eventually reach our ear drums, and causes
the ear drums to vibrate at the same frequencies. That is then interpreted as sound by the ear and our
brains.
Lets now discuss wind instruments.
Say we have a completely closed box, of length L. At the left end of the box, we put a loudspeaker, which
generates a certain sound frequency.
We can now get resonance, and a standing wave, in the air inside the box. The frequencies are goverened
by the same equation as for the stringed instruments, except that v is now fixed: it is the speed of sound
in the air inside the box.
We cant do much about the temperature - but if it does change, the fundamental of the instrument will
also change. This can be compensated for by adjusting L to cancel out the error, if applicable to the
instrument.
Because of the equation that governs them (above), the smaller L is, the higher the fundamental frequency.
Therefore a trombone, which is large, has a lower frequency than a trumpet, which is smaller. A tuba has
an even lower frequency sound.
For a flute, if you cover all the drilled holes on it, the resonant chamber will be long, and the frequency
will be low. If you take your fingers off the holes, so that they are open, the frequency will go up.

108

Chapter 10
Week 10
10.1

Resonance, electromagnetic waves

The lecture begins with talk about mechanical resonance, without much math or things to take notes of.
It is demonstrated how playing a loud tone at a wine glasss resonance frequency can shatter the glass.
After that, a movie is shown about the Tacoma Narrows Bridge, that collapsed only 4 months after its
construction in 1940.
The bridge resonated with the wind, such that huge waves went along it. On the day of its collapse, it
went into a twisting motion that proved too much for it to handle, and the bridge collapsed. I would
recommend looking up footage of this on the internet, as the collapse was actually filmed.
Lets now turn to electromagnetic waves. Two of Maxwells equations are extra important here:
I

~ = dB
~ d`
E
dt


I
d
E
~ = 0 m I + 0
~ d`
B
dt

(Faradays law)
(Ampere-Maxwell law)

At least to begin with, we will introduce EM waves without the rigorous differential equation treatment.
As the above equations show, we can get an electric field due to a changing magnetic field (Faradays law).
Not only that, but we can also get a magnetic field due to a changing electric field (the Ampere-Maxwell
law).
One possible EM wave that meets all four of Maxwells equations is
~ = E0 x cos(kz t)
E
~ = B0 y cos(kz t)
B
Thus, the electric field is solely in the x direction, the magnetic field solely in the y direction, while the
wave propagates in the (positive) z direction, as seen by the term inside the cosine. Furthermore, we see
that they have the same frequency/wavelength, and that they are in phase with each other. That is, when
E = 0, B = 0, and when E = E0 (the maximum value), B = B0 , etc.
The wave propagates with v = k = c, with c being the symbol for the speed of light (in a vacuum), about
3 108 m/s (to within 0.07%!).
The equations above are for a plane wave. If we take a plane perpendicular to z, at a single moment in
time, the E and B vectors are the same everywhere in that plane. They satisfy Maxwells equations only
if
B0 =

E0
c

109

and
c=

1
0 0

Now, lets move into some math.

At t = 0, we draw a loop in the y-z plane. Due to the nature of a plane wave, the field is constant along
one value of y, so the marked strip with width dz has a constant field along one side. If we assume dz is
very small, we can consider the field constant through its width.
We mark the width (in the z dimension) as /4 as it will simplify the result we get.
We attach a flat surface to the loop, bounded by /4, ` and the y and z axes, and apply Amperes law to
it:
I
~ = 0 0 dE
~ d`
B
dt
Because we are in a vacuum, m = = 1, and I = 0 as there can be no conduction current through a
vacuum.
First, what is the flux through one such slice `dz? Flux is the dot product of E-field strength dot area, so
the flux is
dE = `dzE0 cos(kz t)
~ = x, i.e. dA
~ points upwards.
if we choose dir dA
We need to find the flux through the entire surface, though, so we integrate the above:
Z

/4

Z
`dzE0 cos(kz t) = `E0

E =
0

/4

cos(kz t)dz
0

We could simply integrate this, take the time derivative, and have our answer - but doing that in math
software yields a relatively complex answer, as we need to substitute a few values to make the answer
(very) simple.
dE
d
= `E0
dt
dt

/4

cos(kz t)dz
0

We can differentiate this inside the integral, instead of first calculating the integral and then taking the
derivative of what we find; this will make the calculation involve fewer terms.
When we take the time derivative of cos(kz t), a term comes outside. Since is a constant, we
move it outside the integral, too. The cosine turns into sin inside the integral; we then substitute for
R /4
t = 0, so that the integral turns into 0 sin(kz). Combining these steps, we get
110

dE
= `E0
dt
Z
= `E0

/4

sin(kz)dz
0
/4

sin(kz)dz (cancelling minus signs)

= `E0

We get this result after calculating the integral, and then making the substitution k =
the opposite substitution as well). We can simplify it further:

2
(we could make

dE

= `E0
dt
2

= `E0
2
= `E0 f
= `E0 v
= `E0 c
2
. In the fourth, we

use v = f , and in the last, the velocity v of EM radiation is c, the speed of light (or perhaps rather the
speed of EM radiation!) in a vacuum.

The second line is simply a rewrite with no actual change. In the third, we use f =

So we finally have the change in electric flux, and we now need to do the closed loop integral of the
Ampere-Maxwell law. With the result we just found, we now have
I
~ = 0 0 `E0 c
~ d`
B

~ is strictly in the +
Keep in mind that B
y direction, i.e. out of the blackboard. If we do the closed loop
integral, counterclockwise as seen from above, starting at the outer edge, we find that for the outer edge,
~ are perpendicular; one is in +
~ and d`
B
y and one in +
z , so that contributes nothing to the dot product.
For the rightmost side, the B-field is zero everywhere along the line, by the definition of the plane wave.
(It is zero for all y along that line.)
For the short inner portion, they are again perpendicular, so that part doesnt contribute.
The only part that does contribute is the part along the y axis, with length `. The field is constant along
111

that line - again, definition of a plane wave, so that part simply contributes B0 `. So with the integral
evaluating to that simple value, we find
B0 ` = 0 0 `E0 c
B0 = 0 0 E0 c
The ` cancels, and we find a relation between the two field strengths.
The following properties are always true of traveling EM waves:
~ ~v
E
~ ~v
B
~ B
~
E
~ and B
~ are in phase
E
= v
E B
~ = E0
B
c
1
c=
0 0
The second-last one does not contradict our finding above, because of the last one. We can solve the two
as a system of equations and will find the value for c shown, which means that the two equations are really
equivalent.
We know that electromagnetic waves travel at (propagate at) c 3 108 m/s, we can then calculate some
rough times required to move certain distances.
One foot takes about a nanosecond. 30 meters, the length of the lecture hall where the lecture is recorded,
about 0.1 microsecond. In one second, it can move all the way to the moon, and about eight minutes to
the sun. That is, it takes light 8 minutes to move between the surface of the sun and the Earth.
The closest star (except for the sun, of course!), Proxima Centauri, is about 4.2 lightyears away, so light
would (by the definition of a lightyear) take light 4.2 years to travel to us from there.
The nearest large galaxy, the Andromeda galaxy, is about 2.5 million lightyears away. When we look at
it in the sky, we see it as it were 2.5 million years ago!
Since one year is (in the definition used) 365.25 days (on average), one year is 365.25246060 = 31557600
seconds, so multiplying that by the speed of light, we find that one light year is about 9.46 1015 meters.

10.1.1

Radar and measuring distance

We can measure distances by measuring how long it takes for light to reflect off a target.
We sent a pulse, and begin measuring the time taken. When the reflection returns, we stop measuring,
and calculate the distance as
2d = ct
ct
d=
2
where t is the measured time, and c is as usual the speed of light. We have to divide by two since the light
has travelled both there and back, and we only want the one-way distance.
We can measure the distance to the moon in this way, by sending focused laser pulses. Astronauts from
both the US Apollo missions and the Soviets have left corner reflectors / retroreflectors on the moon.
112

These devices reflect light back in the exact direction it came from - as opposed to a mirror/mirror-like
surface, where the angle would be of the same magnitude, but the reflection would only come back if you
looked at it exactly straight on.

10.1.2

Radio

Now to something slightly different: radio, as in both radio waves and radio stations. An AM radio station
might transmit somewhere in the 1000-1000 kHz range (as an order of magnitude). How is audible sound,
<20 kHz, transmitted that way?
Well, we use AM: amplitude modulation. First, we generate a carrier wave, a high-frequency sinusoid.
The frequency of the carrier wave is the frequency a listener will have to tune in to.
We then modulate that signal, by changing its amplitude, by essentially simply multiplying the two
frequencies. Thus we will end up with a signal that is of the (simplified, not truly accurate) form
y = sin(2106 t) cos(2103 t)
To listen to such a station, we tune an RLC circuit in our radio to the carrier frequency, such that it is at
resonance only at the frequency we want to listen to. (We need a high-Q / low-bandwidth resonator for
this, or we might pick out nearby stations as well!)
After that, we need to demodulate the signal, to get only the envelope, which is what was the signal to
begin with (at the radio station).

10.2

Index of refraction and Poynting vector

10.2.1

Poynting vector

Lets now look at the energy content of electromagnetic waves. We have seen earlier that electric fields
and magnetic fields both have an associated energy density, given by
1
UE = 0 E 2 J/m3
2
1
1 E2
UB = 0 B 2 =
J/m3
2
20 c2
E0
The latter follows from the equivalence B0 =
that we found earlier.
c
However,
c=
c2 =

1
0 0

1
0 0

Therefore,
1 2
1
E 0 0 = 0 E 2 J/m3
20
2
So the energy denssity of the B-field is exactly the same as the energy density of the E-field in EM waves!
The total energy density is then the sum, so we just get rid of the one-half, ane we find
UB =

Utotal = 0 E 2 = 0 EBc
where we use that E = Bc (via B =

E
).
c
113

Now that we know that EM waves carry some energy, we can ask the question: if an EM wave passes
through a surface, how much energy passes through one square meter per second? That is, what is the
energy flux, in watts per square meter (joules per second per square meter)?
Imagine a square surface, of side one meter, perpendicular in space to the waves propagation. Thus, the
dot product works out to not have the cosine term, as that would equal one (cos = cos 0 = 1).
Since light travels a distance of c meters in one second, we can calculate the energy stored in a box that
goes back such a distance from the square we are interested in. We know that all energy inside this box
will pass through the square, so we calculate the total energy by multiplying the energy density by c 1 1
cubic meters, and find
Utotal c = 0 EBc2
Because c2 =

1
, that is equivalent to
0 0
Utotal c =

EB
0

The unit of this is then (as noted above) watts per square meter, or

J
m2

What we have found is called the Poynting vector, named after its inventor (though it was simultanesously
created by others, as well). The pronunciation is very close to pointing vector (many likely say it exactly
like that), perhaps with a bit more of the y sound. It is generally written as
~
~
~ = EB
S
0
~ and B
~ are always perpendicular in a traveling EM wave, so the cross product can be seen as being there
E
~ = EB .
only to provide the direction; the magnitude is always |S|
0
~ and B
~ are not constant in value, however! They are sinusoidal in nature, and so the Poynting vector
E
must be described by a sin2 or cos2 function.
We can think about its time averaged value, which may be more useful, since it varies constantly (and
usually with extremely high frequency - visible light is in the 1014 Hz range, for example!).
The time average of a sin2 or cos2 is one half the maximum, so we find
hSi =

10.2.2

1 E0 2
1 E0 B0
=
2 0
2 0 c

Waves due to accelerating charges

Now that we have learned a bit about plane waves, lets look at some other types. Plane waves are useful
for modelling some situations, but far from all.
Given the equation for a plane wave, we can calculate its value for any time, past or present, at any point
in space, no matter how distant. Thats not too realistic.
Lets look at the electric field lines of an accelerating point charge.

114

To the left, we have a few point lines of a point charge (postive or negative; we cant tell from the lines
alone, and it doesnt matter.)
To the right, inside the circle, we have one point charge, at two points it time: t = 0 and t = t.
The circle - really a sphere in three dimensions - has a radius of ct, which is the distance light can travel
in the time that has passed.
Just outside the circle/sphere, we draw the field lines as they are at time t = 0 (in white), radially outwards
from the point charge in the center.
In pink, we have the new field lines of the charge, at its new location, at t = t.
Because these are the same field lines, only at a different location, they must be connected to the old field
lines. Information about the change in location cannot have propagated to outside the sphere, because
light travels at a finite velocity. Therefore, the old and new field lines must be connected, which we
draw as the green kinks.
Note that the field line on the direction of movement is straight and without any kinks.
At this point in the lecture, there is a video demonstration of this, with moving/accelerating charges,
which really must be seen in video.
So similarly to what we have in the picture above, and the video, we can look at an antenna, with an AC
current going through it. Clearly, if the charges move back and forth, they need to start and stop, too,
which means they must accelerate (both positive and negative acceleration, i.e. deceleration).
Along the direction of the current, there will be no EM waves produced, for the reason shown above and
in the video. In the plane perpendicular to the current, the field strength will be at a maximum, and for
angles in between, the value will be somewhere between zero and the maximum.

10.2.3

Spherical waves and the Poynting vector

Lets return to the Poynting vector.


If assume that the Sun has a symmetric, radial power output of 3 1026 watts, then we can calculate a
rough number for the amount of received power per square meter, at the surface of the Earth, 150 million
km away:
p = 3 1026 W

1
1061 W/m2 1 kW/m2
9
2
4(150 10 m)

This is modelled as a spherical wave - a plane wave has a constant value, as the energy is not spreading
out, but moving together as a plane.
1
We can think of spherical waves as emanating from a point source, such that the intensity falls off as 2 .
r
The greater the distance (the 150 109 m here) is, the greater the surface area of the sphere where the
energy flux density is calculated, and the lower the energy flux density is. (Of course, if we could surround
115

the entire sun, then we would capture the entire output regardless of the radius of our sphere, just that
the energy flux per square meter of the sphere would be lower for greater radii.)
If we instead think of this as a plane wave, we can calculate the equivalent electric field:
1000 =

E0 2
0 c

E0 613 V/m
(the lecture says 870 V/m, but Im not sure how that number is found!)
Regardless of the exact number, what does this represent? Well, its some equivalent E-field value, that
would produce this energy flux if it were a plane wave. Is that useful? Perhaps, perhaps not, depending
on what we are trying to do.
We can often choose a model to use - plane waves, spherical waves, individual photons - to explain some
phenomenon.

10.2.4

Photons and radiation pressure

Next, lets have a look at photons. Photons are individual light particles - quanta of light. Each photon
has a certain frequency, and a certain energy (the energy is proportional to the frequency).
Because photons have energy, they have momentum - despite having no mass!
In classical mechanics, the magnitude of momentum is given by p = mv, which clearly must be 0 for a
massless particle.
The answer comes from Einsteins relativity, and the worlds most famous equation... except the famous
form is simplified:
E = mc2
This is only a special case of the full version:
E 2 = p2 c2 + m2 c4
Thus, for a photon, with mass m = 0, we find
E 2 = p 2 c2
E = pc
E
p=
c
(E = mc2 is the special case where p = 0, for a body it its rest frame.)
So we see that the momentum of a photon is given by the energy it carries, divided by the speed of light.
d~p
But now, something else happens: the transfer of momentum is what we call force (F~ =
- Newtons
dt
second law)!
So say we have a square meter, perpendicular to some incoming EM radiation (at a wavelength that would
be absorbed by the surface). Using the Poynting vector, we have
S=

energy
m2 s

If we divide both sides by c, we find


S
energy
=
c
c m2 s
We find that E/c term again, which is the momentum. So this can be rewritten as
116

S
momentum
=
c
m2 s
The units of this is then momentum per unit time, which is force, per square meter, which is pressure.
We call this pressure radiation pressure. So what this is saying is that for a given area, with a certain
amount of incoming EM radiation perpendicular to it, there will be a force proportional to the area, and
proportional to the energy contained in each photon, pushing this surface backwards (in the same direction as the radiation wave is propagating).
Keeping in mind that the time-average value of the Poynting vector is what we are really interested in, we
can write the pressure as
p=

hSi

where

0, if the material is entirely transparent to the radiation,


= 1, if 100% of the radiation is absorbed,

2, if 100% of the radiation is reflected


Of course, values in between are also possible.
The reason for these values can be seen quite easily from the conservation of momentum.
The professor used the examples of throwing tomatoes earlier in the lecture, so we will use that mechanical
analogue.
Since classical momentum is the product of mass and velocity, a (very large!) tomato of mass 1 kg, moving
at 5 m/s, will have a momentum of 5 newton-seconds. If one such tomato hits a target per second, and
falls straight down after hitting, a force of 5 newtons will be applied to the target.
If the tomato were instead somehow bouncing back, in a perfectly elastic collision, then the force would
be twice that: first the momentum is absorbed by the target, and then it must add the same magnitude
of momentum back in the opposite direction, so the transfer of momentum is twice as large.
In between the lecture segments, we have the following question:
What is the radiation pressure on a spherical iron asteroid of approximate radius 100 m? The asteroid is
at a distance of about 400 million km from the sun. Assume that all radiation is absorbed. Again assume
that the power output of the Sun is 3 1026 W as shown in the lecture (note that the actual measured
value is closer to 3.8 1026 W).
We take the Suns power output to be spherically symmetric, so the energy flux density at the distance is
hSi = 3 1026

1
149.2 W/m2
4(400 109 m)2

We divide that by c, and find the radiation pressure:


149.2
5 107 N/m2
8
3 10
The pressure will only be on half the asteroid, as only one side is facing the Sun.
While we will find a half-decent approximation by multiplying the area facing the sun (2r2 ) with the
pressure, we really need to consider that parts of that area are almost entirely parallel to the pressure,
while the area near the asteroids center will be mostly perpendicular.
p

F = r2 p
We can then calculate the force due to the radiation pressure:

117

F = (5 107 )1002 0.0157 0.02 N


(Im not a fan of the rounding, but apparently this is the way to calculate it; I did it differently to begin
with, which found exactly 0.02, but that method was incorrect!)
Thats a very small force indeed! We can compare it with the gravitational force between the asteroid and
the sun. The Suns mass is 2 1030 kg, and the density of iron 7.87 g/cm3 (= 7870 kg/m3 ) (as given in the
question).
Gravitational force is given by
m1 m2
r2
with the m variables being the masses, and G the gravitational constant, G 6.67 1011 , and r the
distance between the objects. We find
Fg = G

Fg 6.67 1011

(2 1030 )(7870 4/3 1003 )


2.8 107 N
(400 109 )2

Well, that force is about about 1.4 billion times stronger than the force due to radiation pressure! Radiation
pressure thus has a very small effect, but one that still cannot be neglected! Interplanetary spacecraft can
veer off course by hundreds or thousands of kilometers if radiation pressure is neglected.

10.2.5

Polarization

Lets look at the electric field due to an oscillating charge, moving back and forth with frequency , at
the center of the coordinate system above.
Say we are at point P, with the position vector ~r to the origin, and angle from the z axis.
Knowing some rules will allow us to find the E-field at point P.
~ is always perpendicular to ~r.
First off, E
~ are always located in one plane.
Second, ~a, ~r and E
If we double the charge, or double the acceleration, the E-field strength will double. That is
E qa
As stated earlier, however, no EM radiation will go out in the direction of accerelation ( = 0), and the

maximum will go out perpendicular to that direction ( = ). We can show this by adding a sin term
2
to the proportionaly equation.
The E-field strength is also inversely proportional to the distance r, so we find
118

qa sin
r

~ is proportional to the product of


Because E is also proportional to B, and because the Poynting vector S
~ and B,
~
E
q 2 a2 sin2
S
r2
1
For the same reasons as the suns energy flux density falls of as 2 , this must do so, too, or conservation
r
of energy would be violated.
Here is a transparency sheet showing linear polarization:

At the center, we have our oscillating point charge, constantly accelerating up and down.
We see that there is no EM waves going out upwards, since that is in the same direction that the charge
is oscillating.
To the middle-left, bottom-left, middle-right and bottom-right of the image, we are in the plane perpendicular to the oscillation, and so we see a maximum of the EM radiation here.

In the top right of the image, we see that for , the magnitude of the E-vector is smaller, but still
4
not zero as it is at the top ( = 0).
~ are all in one plane (to see this, first look at the leftmost or rightmost set of
We also see that ~a, ~r and E
~ and then the angled one).
~r and E,
We call this linearly polarized radiation. The electric field and the magnetic are in phase, as this is a plane
wave.
If instead we had two perpendicular plane waves of equal amplitude, but with a 90 degree difference in
phase, we would call that circular polarization.
The ending of the lecture (lecture 28) should really be seen, as it clearly demonstrates polarization, and
explains and shows it much, much better than I could hope to do using only text and a few images here
119

and there.
(Of course, these notes are not intended to ever be a replacement for the lectures! They are incomplete, may
contain some errors, and are ridiculously unlikely to explain things as well as a world-famous professor!)

10.3

Snells law, refraction and total reflection

Say we have a water surface, surrounded by air, and a light beam hits that surface, as seen here:

Some light will be reflected, and turn back up into the air. Some light will be refracted, and enter the
water.
The incident light (the incoming light), the reflected light and the refracted light all lie in one plane.
For the reflections, 1 = 3 , where the angles are measured with respect to the normal to the surface, as
shown. That is, the reflection will exit at the same angle as it entered, something we all see intuitively
from using mirrors.
Willebrord Snellius introducted the concept of index of refraction, or n.
A materials index of refraction is the ratio between the speed of light in a vacuum, to the speed of light
in the material. Thus, if light travels at v = 0.5c in a material, that materials index of refraction is 2.
n=

c
v

For this reason, n 1 for all regular materials.


The index of refraction of vacuum is, by definition, one. For air, it is extremely close to 1, and is therefore
usually treated as 1. Waters index of refraction is about 1.3, and glasss about 1.5.
Snells law states that
sin 1
n2
=
sin 2
n1
Or, equivalently, and perhaps easier to remember,
n1 sin 1 = n2 sin 2
120

Thus, the ratio of the materials indices of refractions is related to the ratio of the sines of the angles.
Alternatively, we can think of it as a ratio of the speed of light in the material, in which case we find
sin 1
v1
=
sin 2
v2
Snells law appears to have been first discovered by Muslim scholar Ibn Sahl, in 974 AD, 637 years prior
to Snellius! The western name remains Snells law, however.
Using Snells law - which remains its name - we can calculate the angle 2 given the materials refraction
indices and the angle of incidence 1 .
As an very quick example, lets look at the case above, going from air to water. Say the angle of incidence,
1 , is 60 degrees. n1 = 1 and n2 = 1.3, so
sin /3
= 1.3
sin 2
sin 2 =
sin

 
3

3
, so
2

2 = arcsin

10.3.1

sin /3
1.3

3
41.77
2 1.3

Total internal reflection

Lets then look at the reverse situation:

Here, the light ray is going from water, to air.


The lecture question asks, for n1 = 1.331 (n2 = 1, as it is air), how large must 1 be for 2 to equal 90
degrees?
We set up Snells law again:
sin 1
1
=
sin /2
1.331
1
sin 1 =
1.331


1
1 = arcsin
48.70
1.331

121

If the angle is greater than that, nature cannot deal with that refraction, and we get total reflection: all
the light is reflected, back into the water. Total reflection happens when the angle is greater than a critical
angle, that is
n2
, if n1 > n2
n1
So in this case, the critical angle is 48.7 . If 1 is greater than that, there would be no refraction, only
reflection.
sin c r =

This principle (total internal reflection) has some interesting uses. Perhaps most significantly, it is the
principle behind optical fiber, used (among other things) to create high-speed data links between computers, cities, and even countries and continents.
Such a fiber is made from a continuous piece of a form of glass or plastic. Light enters one end, runs in
a straight line until it hits the outside, which is manufactured to have a lower refractive index than the
inside. Therefore, as long as the angle is greater than the critical angle, total internal reflection causes the
light to keep bouncing away inside the fiber.
This process is repeated over and over and over, until it reaches the end of the fiber.
Materials research have made modern optical fiber extremely low-loss: a single unbroken piece of fiber can
be used to communicate over a distance of several hundred kilometers without any elecrtrical equipment
to strengthen the signal.
When a surface is smooth, parallel incoming light beams will reflect with equal angles (3 = 1 ), and thus
exit in parallel; we call this a specular reflection. Shining a laser pointer on a mirror is a great example of
a specular reflection.
If the surface is rough, incoming light may be reflected in essentially random directions, and the light
will not exit with the same angle, but exit in all sorts of directions. We call this a diffuse reflection; one
example might be a laser pointer on a wooden table - or on most non-shiny surfaces, really.
We will use the term reflection to mean specular reflection, as those are the kinds that can be easily
calculated, without knowing near-atomic-scale details of surfaces.
If a light ray enters a material, and then exits again, the new twice-refracted light ray will always be
parallel with the incident ray. It will not be on the straight line, however, unless n1 = n2 .

10.3.2

Frequency and wavelength in refraction

As light enters a medium, its frequency stays constant, but its wavelength changes. The relationship
v = f
must always hold, but because v changes upon entering a different material, one of the other two must
also decrease. The frequency cannot change, imagine a wave passing two observers inside two different
material; point A is just before the boundary between the two materials, and point B just after. If
the waves frequency differs between the two points, that would imply that energy were piling up or
disappearing around the boundary.
Thus, the wavelength has to change. The wavelengths follow the relationship
v1 = 1 f
v2 = 2 f
v1
n2
1
=
=
2
v2
n1
We saw previously that the speed of light is given by

122

c=

1
0 0

As this result uses only two results, the (electric) permittivity of the vaccuum 0 , and the (magnetic)
permeability of the vacuum 0 , it would make sense that if we instead used a materials permittivity and
permeability values, we would get the answer for the speed of light inside that medium. And, indeed, we
do. That is:
v=

1
0 0 M

We can also write this as


v=
... which tells us that because

c
M

c
v=
n

n = M

Thus, we can calculate the index of refraction for any material for which we know its dielectric constant
and relative permeability.
Both of these two values are functions of frequency. Of course, M 1 for most materials (paramagnetic
and diamagnetic materials), so that doesnt have a huge effect on the refractive index.
That they depend on frequency makes intuitive sense, when we think about what they represent.
The dielectric constant tells us how strongly dipoles in a material align to cancel out the external field.
However, they only align due to the external field, and so when the external field alternates back and
forth extremely quickly, the dipoles dont quite have time to align. Therefore, decreases as frequency
increases.
The same argument can be applied to the relative permeability.
As an example, for water, M 1 at all frequencies. However, varies: at near-zero frequencies, it is
about 79; at 108 Hz (100 MHz) it is almost the same, at about 78. At the frequencies of visible light,
about 5 108 however, it has dropped drastically to around 1.33 or so (depending on the exact wavelength,
as well see next).

10.3.3

Dispersion, prisms and white light

Due to the effect discussed above, the index of refraction depends on the wavelength of the light.
In water, the index of refraction for red light is about nred = 1.331, while that for blue light is about
nblue = 1.343; so we see that red light is about 0.9% faster than blue light in water.
We call this effect dispersion.
What we perceive as white is a combination of others colors. Therefore, we can use a prism to split white
light into its constituent colors, using dispersion. Because the index of refraction is different for each color
(or each wavelength/frequency), the colors will split up as soon as they enter the prism. Because the
side they exit the prism on will not be parallel with the side they enter it on, the refraction will not be
cancelled, but rather enhanced further, i.e. the different colors are separated further.
So if we shine white light (combining all wavelengths) into a prism, we will see the visible spectrum of
colors on the other side. In order of decreasing wavelength: red, orange, yellow, green, blue and violet.
These are the colors of the rainbow, and are also often given as red, orange, yellow, green, blue, indigo
and violet (Roy G. Biv as a mnemonic - think of it as a name).
123

Say that we are in a room lit by a regular, incandescent light bulb. The light will be comprised on various
frequencies, and will be roughly white (more yellow-reddish, but still comprising many colors).
Without this wide-spectrum light, we could not see colors properly. In a room lit exclusively by a
monochromic (single-color) red, we could not perceive green or blue, only shades of red.
Therefore, we always design general-purpose light bulbs to produce many colors of light.
(You may have seen the yellow-orange-ish light produced by sodium-vapor street bulbs, and how everything looks orange-y in such light.)

10.3.4

Primary colors

The human eye has three types of cone cells, that respond to different wavelengths of light (they each have
a wavelength that is the most likely to cause a response). One type responds best to short wavelengths
(blue), one to medium wavelengths (green) and one to long wavelengths (green-yellow); the latter two have
significant overlap.
Because of these three cone cell types, we can create almost any color of light (to a human observer) by
mixing red, green and blue light in the correct proportions.
Those three colors are then known as primary colors.
Computer graphics (and televisions, etc.) are based around this concept. A modern LCD display works
by mixing these three colors, as in this example:

124

(Photo of LCD screen by Mattia Luigi Nappi, licensed under CC-BY-SA 3.0)
The top picture shows a zoomed-out version of a picture of an LCD screen (displaying a picture of a man
in a tie). The bottom picture is the full-sized version, displaying the same pixels ase the area marked in
white.
Its clear here that each pixel (each dot on the screen) is made up by three subpixels: one red, one green
and one blue.
Other display technologies exist, but all combine these colors in one way or another, to create an image
with millions (16.7 million for 8 bits per color channel) to billions (1.07 billion colors for 10 bits per color
channel) of colors, from having just varying intensities of red, green and blue arranged in some pattern.

125

Chapter 11
Week 11
11.1

Polarizers and and Maluss law

This lecture will focus entirely on the polarization of visible light.


The light from the sun or light from a light bulb is not polarized.
If we think about individual photons as plane waves, each photon will have a well-defined linear polarization.

A collection of multiple photons may then look like the above, where each photon is coming straight out
of the blackboard, with the lines representing the direction of oscillation of the E-field.
Some are horizontal, some vertical, some at angles... and there is no preferred direction if we average over
time.
In 1938, Edwin Land invented a material that can polarize such light. Consider the case of a diagonally
polarized photon entering the polarizer, when the polarizer is held such that it polarizes vertically:

The resulting polarized photon will have a magnitude that is equal to the vertical component of the incoming photon, in other words, E0 cos , where is the angle between the polarizer direction and the photon,
126

as shown.
We can find light intensity (in watts per square meter) using the Poynting vector, which is proportional
to E 2 - or E B, where B is proportional to E.
Because of this square dependence, the intensity is reduced by cos2 .
In order to find the intensity change over the average of all incoming photons, we take an average of this,
over all possible angles. The average of a cosine squared function is one half the maximum, as weve seen
before. Thus, in a best-case theoretical scenario, we halve the intensity, but in return get 100% linearly
polarized radiation, when inputting entirely unpolarized radiation.
If we had such a polarizer, it would be an HN50 polarizer - meaning 50% of the light gets through and
becomes polarized.
In reality, we can only get smaller numbers than that, such as HN25, HN40, etc, for polarizers where 25%
or 40% of the incoming light gets through, respectively.
Lets try to apply that:
Linearly polarized light of intensity I0 passes through an HN25 linear polarizer. An HN25 polarizer lets
through 1/2 as much light as an ideal HN50 polarizer which has no attenuation at all. The angle between
the E-vector of the incoming light and the E-vector of the outgoing light that emerges from the other side
of the HN25 polarizer (i.e. the polarization direction of the HN25) is 30 . What is the intensity of the
light that emerges?
Lets see. We have
I = I0 cos2
... but thats for the ideal case of the HN50 polarizer. We need to further halve the result, so the answer
is
I0 3
1
= 0.375I0
I = I0 cos2 =
2
24
This result, that the intensity is reduced by cos2 , is known as Malus law.
Two things should be noted here, however: one is that not 100% of the light exiting a polarizer will be
polarized. The ratio of the transmission of the unwanted components to the wanted is called the extinction
ratio, which is about 1:500 for Polaroid film, but can be 1 : 106 for certain polarizers.
The second is that the way we talk about individual photons essentially losing energy through a polarizer
is not true.
Blue light has a higher frequency (shorter wavelength) than red light. Violet is the shortest wavelength
we can see; ultraviolet is shorter. On the other side of the scale, red is the longest wavelength we can see,
and longer wavelengths are infrared.
A photon either comes through a polarizer, or it doesnt. If it did come through with less energy, its
wavelength would have shifted to become longer (as wavelength and photon energy are related via c), and
the color of the photon would have become redder. This is not the case.
As usual with small things, quantum mechanics has the correct answer. Malus law does hold, however,
even through our derivation was a bit of a cheat.

11.1.1

Polarization by reflection

We can produce polarized light without a polarizer per se, by relecting it off a dielectric, like glass or
water.
Unfortunately, the full derivation (using Maxwells equations) isnt shown, and instead its mentioned that
127

the derivation will be part of MITs 8.03 course (Vibrations and Waves).
Lets have a look:

In order to do this, we need to decompose the vectors into two components: one perpendicular to the
blackboard (in to/out of the blackboard), and one parallel to the blackboard. Both are perpendicular to
the rays propagation, however, which is a must for traveling EM waves.
Apart from the decomposition, there is nothing new going on here: we have a reflection, where the reflection angle equals the angle of incidence, and a refraction, for which we can use Snells law.
The incident light, reflected light and refracted light all lie in one plane (the plane of the blackboard, in
this case): we call this the incident plane, or plane of incidence.
The incident light is not polarized in our example. That implies that that the perpendicular and parallel
components of the incident light must be equal, or else the light would have a preferred direction, and
thus not be entirely unpolarized.
Using Maxwells equations, in the derivation not shown, we can relate the parallel components of these
beams. We get two relations, and two equations: one that relates the parallel components of the incident
light and the reflected light, and another that does that same for the parallell components of the incident
light and the refracted light.
We can do the same with the perpendicular components, which gives another two equations, for a total of
four.
Looking at those four (unmentioned) equations, it can be seen that while the parallel and perpendicular
components of the incident light must be equal, that will no longer be the case for the reflected light, nor
for the refracted light. In other words, they will be partially polarized.
One of the four equations, for the parallel components of the incident and reflected light, is:
n1 cos 2 n2 cos 1
n1 cos 2 + n2 cos 1
We can simplify this using Snells law (again, derivation not shown):
E0kref l = E0kinc

E0kref l = E0kinc

tan(1 2 )
tan(1 + 2 )

Remember, however, that the light intensity is related to the Poynting vector, which in turn is proportional
to E0 2 , while the above gives the value for E0 ; we need to square the above to get a relation useful for
128

light intensity.
In the equation above, we can see that as 1 + 2 goes to 90 degrees, we get the tangent of 90 degrees,
which in infinite in magnitude. That means the parallel component goes to zero - and so if any light exits,
then clearly it must be 100% polarized in the penpendicular direction!
If 1 + 2 is 90 degrees, via trigonometry,
sin 2 = cos 1
So in this special case, we can replace all sin 2 s with cos 1 s. That means we can write down Snells law:
n2
sin 1
tan 1
sin 1
=
=
=
sin 2
n1
cos 1
So when the condition is met that
n2
n1
we get 100% polarized reflections (or possibly no reflections at all). The angle for which this happens is
then
 
n2
1 = arctan
n1
This is known as the Brewster angle (or Brewsters angle), after its discoverer.
This has another interesting meaning: when 100% polarized light hits this surface - polarized in the correct
direction, of course - there will be no reflection whatsoever, and the light is perfectly transmitted through
the surface.
The above relation only holds for dielectrics only, not for conductors. Light that reflects off metals will
act differently.
tan 1 =

11.1.2

Polarization by scattering

We have now seen two ways of polarizing light: using polaroid film - devices created to be polarizers - and
by reflection.
There are many ways light can be come polarized, and by scattering is another.
When light shines through very fine particles (much smaller than 1/10th of a micron), the light exiting in
one plane will become 100% polarized; namely in the plane where all photons exit at 90 degrees compared
with the angle at which they enter.
Say a single photon comes in from the left, towards the right, polarized in the up/down direction, all in
the plane of the blackboard.
(We talk about a single photon to begin with, however the light as a whole is unpolarized, with individual
photons polarized in all possible directions.)
The photon enters some fine dust, and so charged particles in the dust (electrons, protons) will experience
an acceleration in the same direction as the photons polarization, given by
~
qE
F~
=
m
m
~ is the electric force on a charged particle.
... where q E
Electrons, having a much, much lower mass than protons, will experience a much greater acceleration.
~a =

129

The charges accelerating and oscillilating up and down to the left (in the picture below), will produce an
oscillating electric field at a point P , that is perpendicular to the position vector between them, and in
the same plane as ~r and ~a:

There is only one possible direction for those two constraints combined, and it is drawn above.
The photon enters the dust, is absorbed by the dust, and another photon is then re-emitted at exactly the
same frequency. Why?
Well, the incident photon, with angular frequency , causes an acceleration in charged particles, again
with that same frequency . Accelerating charges cause EM radiation, and the frequency of the radiation
will be the frequency of acceleration, and so the re-emitted photon will have the same frequency as the
incident one.
The direction of the new photon will have changed, however!
There will be no radiation in the direction of acceleration (i.e. in the direction of polarization for the
incident photon), as no EM waves are ever emitted in the direction of acceleration, as we have seen earlier.
There is however a high probability that it leaves in the plane perpendicular to the acceleration (that plane
has the highest probability to contain the re-emitted photon). For angles between the acceleration and
the emission direction other than 90 degrees, the probability is lower, but still nonzero. It is only zero at
= 0, i.e. in the direction of acceleration.
Light that is scattered at exactly 90 degrees will be 100% polarized.

Consider a light beam (the cylinder), consisting of many photons, each individually polarized in random
directions, so that the light beam as a whole can be considered unpolarized, drawn here as the lines in all
directions inside the cylinder.

130

Each photon individually moves into the dust, and is scattered according to its own polarization.
The ones that exit at a 90 degree angle with respect to the incident beam will become 100% polarized, i.e.
the ones going to the left or right as shown, plus out of or in to the blackboard.
So in that plane, all light will become 100% polarized. We can look at that plane from above, look at the
beam head-on, so that the photons would be coming straight out of the blackboard.

Now, we see three individual photons (drawn in white, pink and blue), each with a different polarization,
coming out of the blackboard (we are looking into the beam).
If we then draw a position vector ~r as shown, how will the light be polarized at the end of that vector,
after having been scattered?
Well, it must still follow the same two rules: perpendicular to ~r, and in the plane of ~r and ~a. There is only
one solution per photon - and only one solution altogether, and so, as shown, they are all polarized in the
same direction, and so the light is 100% polarized when scattered at a 90 degree angle.
At a 45 degree angle (or angle between 0 and 90 degrees), the light exiting will only be partially polarized.
At 0 degrees, it will remain unpolarized.

11.1.3

Scattering demonstration and Rayleigh scattering

In the demonstration for this 90-degree-polarization, by shining light on cigarette smoke (made up of very
fine particles), another phenomenon is obvious: most of the light that exits is blue. That is because the
probability of light scattering is way higher for blue light (short wavelengths) than it is for red light, so
the smoke appears blue.
This phenomenon is known as Rayleigh scattering. The equation involved for the scattering light intensity
is fairly complex, but its notable that it is proportional to 4 . In other words, smaller values for lambda
(bluer light) will yield much higher values; the scattering probability is about seven times higher for blue
light than it is for red.
When the particles we scatter the light off are much larger than the above less than 0.1 microns, the effect
the wavelength has is reduced, and for much larger particles, it is negligible.
Rayleigh scattering is the reason for why the sky is blue (and red, at dawn and sunset). Mid-day, the
sun is high in the sky, and light scatters off dust in the atmosphere, and even off the air particles, due to
thermal fluctuations that cause density differences in the atmosphere.

131

The light that reaches us on the ground, that is not in the direction of the sun itself, is then in big part
scattered light. Due to the dependence on wavelength, more of that scattered light will be blue, and so
the sky appears blue.
At a 90 degree angle to the sun, because of what we discussed above, the light will be linearly polarized.
At other angles, it will be partially polarized.
Clouds are white because the particle sizes involved are large enough that scattering is about equally likely
for all colors.
Rayleigh scattering also tells us why the sky is red at dawn or sunset (as mentioned above).
It those cases, the sun is very low in the sky, and the sunlight has to go through a lot of more atmosphere
in order to reach someone on Earths surface. Because blue light is scattered more than red light (and
green light, being in between blue and red in wavelength, also being scattered more than red), most of
that light will have scattered away from the rays that finally reach you.
And so, only the red light remains.
Clouds are reddish at dawn/sunset for about the same reason: the light that reaches the cloud, and
eventually reaches you, is mostly devoid of shorter wavelengths, since those have been scattered away
already.
Having only red light shine on an otherwise white object will make it appear red, and that is what we see.

11.2

Rainbows

The lecture begins with the professor asking 15 questions about rainbows: if the red is on the inside or the
outside, the radius and width of the bow in degrees, whether it is brighter on the inside or the outside,
where they can be seen in the sky (in which direction), and many more, to illustrate that while everyone
has looked at a rainbow, not many have truly seen them.

132

Above is a single raindrop, with the sun at the horizon for simplicity.
So sunlight hits the raindrop. We draw a single ray of light, which hits the raindrow with angle of incidence
i (1 in Snells law is then i).
Some light is reflected at point A, and some is refracted, at an angle r, which we can find by using Snells
law.
The refracted light will hit point B, where some is again refracted, back into air, and some is reflected to
point C.
The light at point C again hits a water-to-air boundary, where some light is refracted out and some reflected.
Via Snells law, the exit angle for the refraction at point C is i, same as the incidence angle, and via
geometry and the law of reflection, the internal angles are all r.
I should point out the angles and as I personally found them hard to see.
is the angle between the line through the drops center (the i = 0 line) and the ray leaving at point C.
is the angle all the way around from the same line, so they are related as shown in the second equation
below:
= 180 + 2i 4r (by adding up the deflections)
= 180 = 4r 2i
... where r is given by Snells law:
n2
sin i
=
sin r
n1
sin i
= sin r
n


sin i
r = arcsin
n
... where n is the refractive index of the water, for the given wavelength. So the relation between i and
is


sin i
2i
= 4r 2i = 4 arcsin
n
has a minimum value of about 138 degrees, which means has a maximum of about 42 degrees (180 138 degrees). This can be seen if the above equation is plotted, for 0 i 90 degrees; 90 degrees being
the maximum possible incidence angle, when light hits the raindrop at the very top.
So far, weve only really talked about light entering the top half, and leaving the bottom half of the
raindrop. However, it is after all spherically symmetric, so the opposite can happen. Light can enter the
bottom half, and refract/reflect its way to exiting at the top.
Because of this, and because is in the range 0 to 42 degrees, we get a cone of light that exits the raindrop
on the left side:

133

The angle between the center line and the top is 42 degrees, as is the other angle.
There is axial symmetry around the center line.
None of this light can exit behind, since the angle between the center line an behind is much greater than
the maximum of 42 degrees.
However, these values are for red light, with n 1.331. For blue light, n 1.343, for which we find
another result! For nblue , we instead find a maximum for of 40.7 degrees, instead of 42.4 for red light!
In other words, the angle of maximum intensity varies with the wavelength, which is one of the crucial
effects that produces a rainbow. As mentioned earlier, this effect is known as dispersion. What we get
then, is this:

For red light max is 42.4 degrees. Since that is the maximum, it can vary between 0 and 42.4, depending
on the angle of incidence. If light enters with i = 0, then it will just reflect back along the center line;
thats no problem. The only problem is for the angle to be greater than 42.4 degrees (again, for red light).
For blue light, the maximum angle possible is smaller at about 40.7 degrees, and so that will create a
smaller cone, as shown above. Green is between the two, and will create a cone with an angle in between
40.7 and 42.4 degrees.
However, as emphasized above, those are maximum angles only. For angles of 0 40.7, all colors of
light are allowed (assuming the blue at 40.7 is the shortest visible wavelength; violet is shorter, but harder
to see), and so we will see white light in the entire center, since white is the mixture of all other colors!
That is, the inside of the rainbow is bright, then we have the bow in all the colors, and the outside is dark
- the light cannot exit that way, since the angle would be greater than the 42.4 degrees maximum for any
visible wavelength of light!
This next part should probably be seen in lecture - video is much better than still pictures for this explanation. I have two pictures, where there should be 4-5 (so that we can see them being drawn one part at
a time).

Here, we have the sun coming in from our left, and rain to our right. If it were also raining to our left, the
light wouldnt reach the rain on our right side, and we wouldnt see a rainbow.
134

Light hits the leftmost marked raindrop (and all drops along the line between us and it), and we get the
cone (half-angle of about 42.4 degrees, for the outer ring, the red light) as shown above. However, we are
looking outside that cone, and so we see no light there.
It also hits the bottom marked drop, and we are then looking into the cone, and we see white light at that
location in the sky (and from all other drops on that line).
It also hits the middle marked drop, and we see red light from it - as we are looking right at the edge of
the cone of light.
Of course, all raindrops will be hit by light, and all will act in the same way - but only some will be
visible to us, creating a rainbow in that direction - the direction opposite the sun. We could never see
a rainbow towards the sun, since in order to create a rainbow, light needs to be reflected (after some
refraction inside the drop) back towards the sun, and so if we are looking at rain between the sun and us,
the rainbow-colored light would be away from us, and never reach our eyes.

So say the sunlight comes in from an angle, and the sun is higher in the sky that previously.
The rain is still to our right.
We envision a line from our head to our shadow, which then is the line to the sun behind us. At at angle
of 42 degrees above that line, we should see the red in the raindow, with the other colors being slightly
lower, and having white light on the inside.
Because of this relationship, when the sun is low, the bow will be relatively high in the sky. The higher the
sun is, the lower it will become (as the 42.4 degrees from the line will be closer and closer to the ground).
When the sun is at 42 degrees or more in the sky, we cant see rainbows any more - unless the water is
right next to you (you can create rainbows with a garden hose, for example!).
If we return to the first picture in this section on rainbows, with the reflection and refraction angles for a
single raindrop, note that at point C, where the light is drawn as exiting by refraction, there will also be
some reflection.
If we follow the line of this reflection, but backwards - so that sunlight comes from above, shines down
towards point C, refracts in, and reflects twice inside the drop before leaving, we will find that the light
creates a secondary bow. All we said above applies to the primary.
The secondary bow is fainter than the primary (even faint enough to not be visible at times), but wider
and reversed in the color order.
If we do the math for this secondary bow, we will find that the maximum angle for the red will be about
50.4 degrees, while for the blue, it will be about 53.5 degrees, giving a difference of about 3.1 degrees,
compared to about 1.7 degrees for the primary.
However, because the sun is not a point source - it is about half a degree in the sky - the bows will be
slightly larger than these numbers of 1.7 and 3.1 degrees.

135

Note also that the blue maximum angle is now greater than that of the red, so the colors in the secondary
will be reversed, with violet/blue on the outside and red on the inside.
We can also see, looking at these angles, that the secondary bow will be about 10 degrees above the
primary, so we only need to look slightly above the primary bow to find the secondary.
Now, on to answering 12 of the 15 questions asked prior to the lecture.
Red outside or inside? The red will be on the outside of the primary bow, inside the secondary.
Radius of the bow in degrees? About 42 degrees, for the primary.
Length? Depends on the suns location, where there is rain, etc. If the sun is high, the bow goes
downwards, and not much is left to see. The bow is part of a circle with a radius of 42 degrees, so
we can find the length by knowing how much of it is visible.
Width (in degrees)? As mentioned above, a bit above the 1.7 degrees we find from just looking at a
single raindrops angles.
Light intensity, inside vs outside? The light intensity will be greater inside, as all colors will be
present there (and appear as white light). None of this light can end up outside, so the outside
should be darker.
At what time of day can we see them? Well, the sun should preferably be fairly low in the sky, to
make a bow visible, so morning or afternoon is perhaps better than mid-day.
In what direction should we look? In the direction opposite to the sun. (In fact, there is a tertiary
bow in the direction of the sun, but it was only first photographed in 2011, after people had tried
for over 100 years, so it is extremely difficult to capture. With your naked eye you would only see
the sun.)
Is there one or two bows? Two; there is a secondary. (Not counting the ones we essentially cannot
see.)
If two, where is the second? The secondary is about 10 degrees higher than the primary.
If two, what is the color sequence of the second? The secondary has its color order, when compared
to the primary.
What is the radius of the secondary? About 52 degrees.
Width of the secondary? About 3.5 degrees or so, based on the difference in angles plus the suns
apparent size.
Those are the first 12 questions; the remaining three have to do with polarization of the rainbows light,
which we will look at shortly. First, a picture from Newton himself:

136

Here, we can see how the sunlight (S) is reflected and refracted to reach an observers (O) eyes, creating
the primary (lower) bow, and the secondary (higher).
Note how the light in the secondary goes the other way around the raindrop, and is subject to one more
reflection.
Below, we see a supernumerary bow.

Note that the colors arent all in order, and that there are multiple green and blue-purple-ish areas.
This cannot be explained by the reflection and refractions weve talked about previously. Instead, this is
137

because of the wave nature of light.


Interference between rays of light along slightly different paths, with different phase, cause constructive
interference (for in-phase waves) and destructive interference (for waves out of phase by about half a
wavelength). In the case of destructive interference, the waves cancel each other out.
This can be seen by maxing a graph of two waves, e.g. of sin + sin( + ) where varies; at = the
wave is completely cancelled out, and at = 0, the waves amplitude is doubled.
Diffraction (the interference mentioned above) can also cause entirely white rainbows to appear. In this
case, the water droplets are extremely small, which (for complex reasons) causes the light to essentially
smear out, and so the bows colors are all mixed together, creating a box that appears partially or entirely
white.
This can also be combined with the destructive interference effects, causing a white bow with smaller dark
bows inside!

11.2.1

Other atmospherical optical phenomena

Next, we have some other atmospherical optical phenomena.


Around the sun, and around the moon, we can sometimes see a 22 halo.
They are the result of ice crystals in the atmosphere, and form complete circles around their source, with
an angular radius of 22 degrees, giving them their names.
22 degree halos are fairly common. There are also 46 degree halos, that are then of course larger in radius,
but much fainter and thus rare to see.
Then there are sun dogs, or more properly, parhelia (singular parhelion):

Sun dogs are always 22 degrees distant from the sun (or moon, in which case they are then called moon
dogs), i.e. at the same distance the 22 halo is. They are always at the same height at as the sun.
They can be seen anywhere in the world, year-round, though thats not to imply that they are always
visible.
Like the above mentioned halos, sun dogs are due to light refracted through ice crystals in the atmosphere.

11.2.2

Polarization of rainbows

We now return to the last three questions, all about the polarization of the light in rainbows.
Is light from rainbows polarized, and if yes, how strongly and in which direction?

138

The answer is yes, and very strongly; as for the direction, the light will be polarized along the bow: at
the leftmost part, the polarization will be up/down, curving along the bow, being horizontal at the bows
top, and then again turn vertical at the very end.
Looking again at the diagram of the single raindrop, we know that the actual bow shows up when 42
degrees, which happens when the angle of incidence is about 60 degrees.
The angle of refraction r is related to i via Snells law, as the light is refracted:


sin i
r = arcsin
n2
So for n2 1.333 in water, and i of 60 degrees, r 40.5 degrees.
At that point, B, where light is reflected back towards the front (left side) of the raindrop, the Brewster
angle is given by
Br = arctan

n2
n1

... where n2 is the refractive index of the medium the light would be going to (the air, n2 = 1), and n1
the refractive index of the water, n1 1.333. With those values, we find Br 37.88 degrees, which is of
course very close to the 40.5 or so degree angle of the light coming there.
Because light would be 100% polarized at the Brewster angle, the light will be not 100%, but very strongly
polarized.

11.3

Review for Exam 3

There was just about nothing reviewed that had not been covered previously, as expected, so I did not
take any notes watching this lecture.

139

Chapter 12
Week 12
12.1

Double slit interference and interferometers

Light can be seen as both waves and particles, under different circumstances. When light waves interfere
with each other, which this and the next lecture are all about, clearly we use the wave model. It doesnt
make much sense to think of two photons, each with the same energy, cancelling each other out to become
darkness. With waves, on the other hand, it does - at least from a mathematical viewpoint, where e.g.
sin(x) + sin(x + ) is zero everywhere.
A simple physical example of this is when we have two sources, producing waves with exactly the same
frequency.

In this example, we can think of the waves as two-dimensional waves, such as waves on the surface of a
body of water. However, the same principle applies to EM waves in three dimensions.
Say we are at point P, at t distance r1 and r2 away from the two sources, respectively.
If the waves from both sources arrive at the same time, in phase, they will add: the amplitude will become
twice as high, as both the mountains and the valleys of the waves will add. This is called constructive interference: the two waves interfere in such a way that the net result is greater than either wave alone.
If they were to appear exactly 180 degrees out of phase, and are of the same amplitude, then there will
be points where the net amplitude is zero - the waves cancel each other out. We call this destructive
interference. Therefore, we need to have the waves differ by half a wavelength - which is the same as
saying 180 degrees out of phase. So the difference between the distances to the two sources, at the point
we are, must be
(2n + 1)
(for destructive interference)
2
... where n is an integer. This will then yield 0.5, 1.5, 2.5 etc.
|r1 r2 | =

140

To get constructive interference, the waves need to arrive in phase, so the distance must be a multiple of
the full wavelength. Thats easier:
|r1 r2 | = n (for constructive interference)
We should perhaps call the above total constructive/destructive interference, or something to that effect,
since e.g. two waves, each of amplitude A might add to a wave with an amplitude of say 1.1A, which is
then also constructive interference. The same but in reverse applies to destructive interference: if the sum
of the two waves is smaller than each wave alone, then there is destructive interference.
When the sum of distances like this is a constant, so that r1 + r2 =constant, that gives us the equation
for an ellipse, e.g.
x2 y 2
+
=1
22 32
... which graphs as an ellipse with height 3 and width 2 (semi-minor axis 2, semi-major axis 3). If we
switch to a difference, so that we only change from a plus sign to a minus sign, we get a hyperbola instead.
The points (-2, 0) and (2, 0) are solutions to both equations, however.
In three dimensions, if we rotate the entire thing, we get a hyperboloid instead, which (as expected) has a
similar shape, but in three dimensions.
Lets have a look at how this might look.

(Note that the lines can also be mirrored on the other side! They were only drawn on one side, but they
are equally valid mirrored.)
We have the separation d between the two sources, which are in phase and at the same frequency/wavelength.
At the line where r2 r1 = 0, there will be a maxima in all cases. We can see this from the maxima
equation with n = 0: there will be a maxima there regardless of the wavelength.
In the case of 3 dimensions, the line will instead become a plane, perpendicular to the blackboard.
The yellow lines, where the destructive interference completely cancels out the waves, are called nodal lines,
or nodal surfaces in the three-dimensional case. The maxima (white lines) are then sometimes known as
anti-nodes, or simply maxima.
If we look at the distance between the nodal lines, right in between the two sources (so to the very left
of the long lines), the distance between two adjacent nodal (yellow) lines will be /2, and likewise for the
spacing between two adjacent anti-nodal lines.
Therefore, the approximate number of maxima or minima is given by
# maxima, # minima
... where d is the separation between the two sources.

141

d
2d
=
/2

Because the curves will be hyperbolas, which are asymptotic to a line (for example, y 2 x2 = 1 is asymptotic to y = x in the second and fourth quadrants (x < 0, y > 0 and x > 0, y < 0), and to y = x in the
first and third (x > 0, y > 0 and x < 0, y < 0).
Therefore, we can define an angle between the origin (the center of the two sources), and the hyperbolic
line in the first quadrant, at least assuming that we are looking from far away, where we can treat the
hyperbola as equal to its asymptote, without any significant loss of precision.

In the approximation that r1  d and r2  d, i.e. the separation between the sources is negligible
compared tot he distance to them, we can make some simplifications. First, we can think of the lines
between our point P to the sources as parallel, despite that they obviously cannot be exactly parallel,
since they meet up at P.
This means we can measure the angle as shown, which will then prove to be equal at many places, again
as shown.
r2 r1 , shown in pink, can be found via simple trigonometry:
r2 r1
d
r2 r1 = d sin
sin =

We can now easily find the directions where we have constructive and destructive interference.
As we saw earlier, for constructive interference, we want r2 r1 = n With our newfound equality, that
means
n
(for constructive interference)
d
As for destructive interference, we do the same in replacing r2 r1 with d sin and find
sin n =

(2n + 1)
(for destructive interference)
2d
... adding an index n to the angles, as there will be multiple maxima angles and multiple minima angles.
sin n =

Now that we know the angles, let us look at what happens if we project this onto a screen a distance L
away, where L is a very large distance (L  d).
If we use x to denote distance on the screen, perpendicular the length L (thus a vertical distance), then
we will find that
tan =

142

x
L

When is small, tan sin (as tan =

sin
, and cos 1 for small angles).
cos

We can then calculate where the maxima will be on the projection, i.e. where there is constructive interference on the screen, still at the distance L, as given by the x value. There will be multiple, as there are
multiple angles of maxima being projected.
We know that for constructive interference to occur, the waves must arrive in phase. That is, the difference
is distance r2 r1 = d sin must be a multiple of the wavelength. What implication does that have for
the vertical distance? When x = 0, we will have a maxima - as we saw earlier (as = 0 where x = 0). As
x grows, as does . The relation is simple, when we are using radians for the angle:
x = L
For small angles, sin , which means that, using the above equivalence
sin n =

n
d

we have
n
d
(I am not sure about the above reasoning, but since tan sin for small angles, it should be true.
The answers are the same as given in lecture, so those are certainly correct.)
n

We then multiply that by L as we found above, and find:


nL
(for constructive interference)
d
that tells us at what linear separations from the center line there will be maxima, at a distance L from
the sources.
xn

Using the similar equivalence for the destructive interference angle, the x values where there are minima
are given by
(2n + 1)L
(for destructive interference)
2d
However, it is important to remember that these are approximations! They become less and less accurate
as grows, and since tan for certain angles, the approximation is truly awful for larger angles. In
such cases, we need to use the actual tangent instead of just the angle or the sine.
xn

Anyhow, lets look at a different way of creating these two sources:

143

We shine bright light - laser light is excellent, as it is essentially monochromatic (only one wavelength) onto two very very small slits. According to the Huygens principle, these slits will then act as sources for
secondary waves, and will emit in-phase spherical waves, at the same frequency as the incoming light.
So above, we have the incoming light as plane waves, and on the other side of the two slits, we have two
sources of spherical waves, which will interfere at a screen a certain distance away.
We shine the laser beam, which has a diameter of about 3 mm, onto these slits, which are cut-out of a
black material. The separation between the two lines is d = 0.088 mm, or 8.8 105 m.
The wavelength of the laser light is 6328 ngstrm, which is 632.8 nm in units I personally prefer. With
the distance d = 8.8 105 m between the slits, and L 10 meters, we can calculate the x value for the
first maximum (except for the one at x = 0, which is certainly always there if the sources are in phase).
x1

L
7.2 cm
d

The result looks like this:

We indeed see that there are places where the two waves cancel each other out, and there is darkness.
The light intensity of the maxima is non-uniform, which will be talked about more next lecture. If the
slots were much thinner (much thinner than the separation between the two), the intensity would be more
uniform, but then less light would pass through the now-thinner slots, and so it is a tradeoff between the
two. Clearly, visibility is important for a demonstration such as this one.
If we do this experiment with white light instead, there will probably not be locations of darkness. The
reason is that since white light is made up by tons of different wavelengths, and the location of maxima/minima is dependent upon the wavelength, each color of light would have its maxima and minima at
different locations in space (except for x = 0, as usual), and so the colors would smear out a bit, and there
would not be any well-defined locations where all light cancels out.
These equations are demonstrated in several different ways, showing the extreme similarities between different types of waves: sound waves ( 0.113 m), water waves (f 7 Hz), the red laser light ( 630
144

nm) and radar waves ( 3 cm).


Lets look at the radar example. We have
= 3 cm
d = 23 cm
L = 120 cm
What is x1 , the first maximum (except x = 0)?
First, we can calculate the angle, using

sin 1 =
d
 

(for small angles)


1 = arcsin
d
d
Using this small angle approximation, we can calculate x1 as follows:
L
15.65 cm
d
If we ignore the small angle approximation, and use the more correct equation:
x1

x
L

 

x = L tan = L tan arcsin


d
tan =

x = 15.787 cm

12.2

Gratings and resolving power

Last lecture, we talked about interference between waves, due to two coherent light sources. This lecture
will talk about this when we have dozens to thousands of sources, rather than just two.
In the case where have many such thin slits (or sources of in-phase spherical waves of the same frequency),
say N such sources/slits, where the distance between two adjacent ones is d (d was previously the distance
between the two sources, where we only had those two), we will in fact find the same angles and linear
locations of maxima:
n
n (small angle approximation)
d
n
So in other words, we will find minima in the direction of n
, and we can turn that into a linear
d
distance by multiplying it by the distance L to the screen. All this is assuming the angles are measured
in radians, and are fairly small.
sin n =

An intuitive way of thinking about this result that if the top two sources costructively interfere, then
the second and third will also constructively interfere, as will the third and fourth, etc., as the distance
between two adjacent sources is always d.
As for the points where there is destructive interference, that is tricker, and for a full derivation, we are
told to have a look at MITs 8.03 course (Vibrations and Waves).
As for the result, with N sources, we will find that in between each pair of maxima, we will have N 1
minima - i.e. places where there is complete destructive interference.
145

Note that with N = 2 as in the previous lecture, we indeed find the result of having one minima in between
the maxima.

Here, we see how the interference pattern may look for N = 5, giving 4 minima in between each maxima.
Assuming the secondary peaks (the small bumps) are identical, we can calculate the angle between 0 = 0
radians and the first minima, simply by dividing the distance to the next primary maxima by N :

(measured from the middle of a primary peak)


Nd
is then the width of a peak, in radians (from peak to the next minima).
=

The larger N is, the higher (but also narrower) the primary peaks will be. If N increases by a factor of
e.g. 3, the electric field vector also increases by a factor of three. However, light intensity is given by the
Poynting vector, which is proportional to E 2 - and therefore, the intensity of the peaks is proportional to
N 2.
The peaks will also become narrower by a factor of three, which restores balance and ensures that there
is no violation of the conservation of energy going on here.
As a demonstration, the same red laser as before is used, together with a grating (a transparent sheet with
lots of thin lines blocking light) of 2500 lines per inch, and with L = 10 meters. With the laser beams
diameter of 2 mm (hmm, it was given as 3 mm earlier!), we can calculate some values. The given values
are then
d = 10.16 m
= 6.3 107 m
L = 10 m
lines
984.25
cm
lines
cm
2 mm 0.1
200 lines
cm
mm
We can then find the first-order maximum using the good old formula,
N 984.25

0.062 rad 3.55


d
The width of the peak, from the center to the next minima (so that the peak is perhaps really twice as
wide, in a way) is then given by above:
1

0.018
Nd
146

We can then multiply this angle (in radians!) by the distance to the screen, to find the rough width in
meters, which turns out to be about 3 mm. The real with will probably be greater, as the laser beam used
is spreading out more than the 0.018 degrees we found above, and becomes the limiting factor.
As we saw with the double-slit interference, if we use white light instead, the peaks will spread out, since
their positions are proportional to the wavelength of the light. Blue light will have the peaks closer together than red light, and looking at the equation for , should also have thinner peaks.
If we do this with white light, we will see white at the center maximum - all wavelength have a maximum
at = 0, but at other places, the light will tend to spread out according to its wavelength.
Heres what we see when shining white light through (or on, as this is a reflecting grating) a second grating,
with d 2.5 m:

As we would expect, there is white light at the center, where there is a maxima regardless of wavelength.
Further out, we see other colors, with blue always being the closest to the center (the angles for blue
maxima are always smaller than other colors of greater wavelength), and red farther away.
If we add in the red laser again, which is aimed exactly the same, we expect that to fall in the same places
where the red light from the white light ends up, and that is indeed what happens:

We can apply the same principles to light of other colors, of course. As an example, a demonstration
is given in the lecture using a neon light source, which looks red to the naked eye. Looking through a
grating, we can see discrete lines at particular wavelengths - many of which are indeed in the red, a few
in the orange, and a few in the green as well.

12.3

Single-slit diffraction

Lets look at another different case, whereby we have a single slit. Even then, there will be interference,
and there will be places of total destructive interference (at least assuming coherent light, such as the red
laser weve been using).
For a single slit of width a, we will find a maximum at = 0, as always. We will find minima at
n
n
a
... where the derivation is once again shown in the follow-up course, 8.03 Vibrations and Waves.
This looks suspiciously similar to the one we found for the two- and multiple-slit interference. As always,
in the small-angle approximation, we can just multiply the angle in radians by a distance L, to find the
linear separation on a screen a distance L away:
sin n =

Ln
a
I refer to the lecture and the book for more details about this.
Heres what the intensity will look like, as a function of x on some screen a distance L away:
xn

147

So we find a very broad maxima at the center, which ends at the first minima, where the intensity is zero
After that, we find the first-order maxima, which is very weak compared to the center; the second order
is fainter yet, etc.
L
- or twice that, if you prefer, since
In other words, the width of the center maxima is then really just
a
it is symmetric around the other side of the zero.
Theres an extremely interesting phenomenon (not very) hidden in this simple equation. Since a is the
width of the slit, and the width of the peak is inversely proportional to a, making the slit narrower will
make the peak on the screen wider! Very non-intuitive.
We can make the peak extremely wide (but also faint, since less light can come through) by making the
slit very, very narrow.
What about if we dont have a slit, but a circular pinhole instead?
We get something similar to the above graph, except we rotate it around its axis, as we now have axial
symmetry. We would see a circular maximum at the center of a certain radius, followed by a ring of
darkness of greater radius, followed by a ring of the first order maxima, etc.
For reasons not explained in the lecture, the radius (in radians) of 1 , the angular radius of the first
minima, will be
1

1.22
a

We use the small-angle approximation here as well.


(The source of the 1.22 appears to be from the first zero of a first-order Bessel function of the first kind something I have no experience at all with.)
If we have a pinhole, and we look at the image of two light sources through the pinhole - where the sources
are far apart. Two examples given are the headlight of a car, and two stars - far apart is relative, of
course, and angular separation is what really matters.
Each source would produce a circular diffraction pattern. As we move the two sources closer and closer
together, the diffraction patterns will also move closer together. The angular resolution is the smallest
angular separation where we could still say that we see two separate patterns, rather than having them
merge into one. For the definition of angular resolution, we take the light sources as equal in intensity,
and for simplicity we now assume that they are monochromatic.
Heres a drawing:

148

The commenly accepted definition is then that the center of the second (rightmost) pattern must be no
closer to the first than that it is centered on the first minima, i.e. at

a
If they are any closer together, we well tend to see it as a single source. We call this the Rayleigh criterion
of resolution. In other words, the Rayleigh criterion says that the separation between the two light beams
has to be larger than the above angle, or we will see it a as a single source. Note that it is a function of
a, the pinhole diameter, such that we can increase the resolution by making a larger.
1 1.22

This is also known as the diffraction limit on angular resolution. Whether we have a pinhole, or a lens or
a concave mirror, the latter two which we might find in a telescope or a regular camera, we can never beat
this limit.
Using this formula, we find that the diffraction limit for a 20 cm lens is about half an arc second for 500
nm light, while that for a 2.4 m lens is about 1/25 of an arc second (calculated by ignoring the factor of
1.22).
(A degree, or arc degree can be subdivided, like time, into 60 arc minutes, each of which is 60 arc seconds.
Thus 1 degree = 1/360 of a circle = 60 arc minutes = 3600 arc seconds.)
In other words, the larger the telescope we have, the better the angular resolution.
However, for Earth-based telescopes (on the ground), there is another, much more problematic limit: air
turbulence in the atmosphere. Air turbulence limits the resolution to about half an arc second, in a process
called astronomical seeing. Seeing is one reason that we have space telescopes: there is essentially no air
whatsover at the height of e.g. the Hubble space telescopes altitude of about 150 km, and so Hubbles
lenses are indeed diffraction limited.
The angular resolution of a human eye is diffraction limited to about half an arc minute (about 1/120
of a degree), calculated for a pupil size of about 4 mm and with 500 nm light (green). According to the
professor, however, most students will have vision worse than that, such that it is not diffraction limited.
Nobody could possibly have better optical resolution, however, unless their pupil size was greater than the
4 mm used in the calculation.

12.4

Doppler effect and the Big Bang

The Doppler effect is the reason for the familiar effect where the pitch of an ambulance/police car/fire
trucks siren changes as it travels past you.
When the sound transmitter is moving away from you, the pitch you hear is lower than the pitch that is
actually transmitted at the source. If the transmitter is coming towards you, the pitch instead increases.
This is given by the equation
f0 = f

vs vrec
vs vtr

149

... where f is the original frequency, f 0 is the frequency received/percieved/heard, vs the speed of sound
in air, vrec the velocity of the receiver and vtr the velocity of the transmitter.
In this equation and in what follows, we take the velocities as positive when moving towards the right of
the blackboard, and negative when moving towards the left.
Note that the case of a stationary transmitter and a receiver moving away is not equal to the case of aa
stationary receiver where the transmitter is moving away!
If the transmitter is stationary, and is at say 440 Hz (the standard tuning in modern music is to the A440
note), and the receiver moves away at the speed of sound, then there will be no sound heard at all - the
receiver (just barely) outruns the sound waves entirely. The equation above gives 0 Hz for that case granted, it sounds weird talk about a 0 Hz sound, but since 0 Hz means no wave motion, I suppose that
is technically true.
On the other hand, if the transmitter is moving away at the speed of sound, while the transmitter is
stationary, what do we find?
340 0
= 220 Hz
340 (340)
The frequency is halved, but we still hear the sound. If you find that unintuitive, think about it a bit
more - its very clear that while the transmitter moves, the waves will still reach you as they always travel
at 340 m/s in your direction.
f 0 = 440

The equation also has a very curious effect, which can be hard to show in practice: if the transmitter is
moving away at twice the speed of sound, the sound (a piece of music, perhaps) will reach a stationary
observer in time and in tune, but backwards!
How do we intuitively explain the Doppler effect? It is actually quite simple, but is best shown using
animated graphics, which I cant really use here. I suggest looking it up online.
In short, when moving towards a transmitter (or the transmitter is moving towards you), the wave fronts
become compressed, and the wavelength becomes shorter (meaning that for sound, the pitch increases;
for light, it becomes blueshifted ; more on that soon). When the distance between is increasing, the wave
fronts become separated from each other, and the wavelength becomes longer (sound: lower pitch; light:
redder color, or redshift).

12.4.1

The Doppler effect and light

Lets look more at the Doppler effect in light (and other EM radiation).
The derivation requires special relativity, but the result is given by
0

f =f

1
1+

1/2

where

v
c
where v is the relative velocity between the transmitter and receiver, and c is the speed of light.
If > 0, then the two are moving apart; if < 0 the two are approaching each other.
=

Much of the idea behind special relativity, as the name implies, is that motion is relative. There is no such
thing as an absolute reference frame, and therefore, there is only one term of velocity in the equation. It
does however, of course, matter whether the two are approaching or receding from each other.
The reason this doesnt apply to the sound waves above is that sound waves require a medium to travel
through, whereas electromagnetic do not.
150

c
In terms of wavelength, which is physically identical but perhaps more useful, we find (as = and thus
f
c
0
= 0 ):
f
1/2

1+
0
=
1
Note that the denominator and numerator have swapped places, as compared to the frequency-based equation.
The velocity v in the above equations is the radial velocity. If the velocity between the source and the
receiver is not is the straight line connecting the two, then there is an angle between the velocity vector
and the position vector between the tow. The velocity we need is then given by v cos , which is a result
weve seen many times before when using vectors. Only the component of the velocity in the direction of
the object matters.
Police often use radar guns, which can measure the speed of cars by reflecting radar waves of a known
frequency/wavelength off cars, and measure the wavelength of the returning waves. The radial velocity of
the car can then be calculated.

12.4.2

Big Bang cosmology

Doppler shift is also used by astronomers to calculate the radial velocity of distant stellar objects. Most
stellar spectra consist of lines, spectral lines, of a discrete wavelength, from the atoms and molecules that
they consist of.
If we happen to know a certain spectral line, and we see the same line, but shifted in wavelength, we have
two names for that, as mentioned earlier:
0 > : redshift
0 < : blueshift
So if the actual wavelength, from the stellar object, is shorter than we would have expected, that means
the object is approaching us (the light is blueshifted), and vice versa if it is longer than expected (the light
is redshifted).
Strictly speaking, if a spectral line is at e.g. 1000 nm when stationary, and say 1100 nm when measured,
we still call that redshift, despite that it is technically moving away from the red (red being at 620-700
nm or so) now that the wavelength has surpassed that of red.
The same applies to blueshift: any decrease in wavelength is known as blueshift, and any increase is known
as redshift, regardless of the actual wavelengths involved.
If we know a certain spectral line, as above, and we can generate that same spectral line in our observatory,
we can compare the two, and via the equation above, find and then find v = c, and so we know the
relative radial velocity between us and the object (perhaps a star, or an entire galaxy).
Edwin Hubble, the astronomer after which the Hubble Space Telescope is named, discovered a curious
relation between the radial velocity of a galaxy and its distance.
The further away a galaxy is, the higher its redshift is. In other words, the further away it is, the faster
it is moving away from us! This fairly linear relation it known as Hubbles law, and the constant of proportionality H or H0 (the latter is commonly used for the current value, as it actually changes over large
time periods) is known as the Hubble constant. The value of this constant was most recently measured
(on March 21 2013) to be 67.80 0.77 km/s/Mpc (kilometers per second per megaparsec).
One parsec is about 3.26 light years, so a megaparsec is 3.26 106 light years, or about 3.1 1019 km.

151

The value has fluctuated over time, however, as measurements have gotten more precise. (Hubbles original
measurement was on the order of 500 km/s/Mpc!)
Hubbles law, in mathematical form, states that
v = H0 d
where v is then measured in km/s, H0 in km/s/Mpc and d in Mpc.
As an example, if we see an absorption line in the spectrum of a galaxy that is is 7% shifted towards the
red, we can calculate by solving the wavelength equation for :
=

(0 )2 2
(0 )2 + 2

If we plug in 0 = 1.07 (0 > , or else its not redshifted but blueshifted), we find
0.0675556
If we then use v = c, we find
v 20266 km/s
We can then calculate the approximate distance using Hubbles law:
v = H0 d
20266
v

298.9 Mpc
d=
H0
67.80
One way to describe why this happens is to assume we are at the center of the universe, which was
formed from a single point, if we only turn back the clock. The galaxies that obtained the largest speed
at that initial explosion (the Big Bang) are not the furthest away from us.
This isnt strictly true, as the universe doesnt even have a center according to current theories on the
shape of the universe. (For a two-dimensional analogue, think about the surface of a balloon/sphere:
where on the surface is the center? The universe can be thought of in a similar way, except we have three
dimensions rather than two.)
Thats also ignoring that there were no galaxies right after the Big Bang. Still, lets assume this is true
for now.
We can then ask the question: if the universe had a beginning, where everything was together, as we would
see it if we simply ran time backwards (everything would get closer and closer to everything else), how
long ago did this happen?
According to data from two probes that monitor the Cosmic Microwave Background Radiation (CMBR),
the WMAP (W-map, the Wilkinson Microwave Anisotropy Probe) and the Planck satellite, essentially
the successor to WMAP, the universe is 13.7980.037 billion years old. Numbers in this order of magnitude
are widely accepted in the scientific community, and indeed there is plenty of proof, such as very, very old
known galaxies that are only a few hundred million years younger than the universe itself.
As a side note, the solar system and Earth are about 4.5 billion years old, while life on Earth (simple cells)
are thought to have existed for about 3.6 billion years.

12.5

Farewell special

I didnt take any notes for this lecture. While the content is certainly interesting, Im not sure exactly
what to write down for it; it must certainly be seen either way.
152

Chapter 13
Homework problems
13.1

Week 2

13.1.1

Problem 1: Motion of charged particle in electric field

An electron of mass me and charge q = e is injected horizontally midway between two very large
oppositely charged plates. The upper plate has a uniform positive charge per unit area + and the lower
plate has a uniform negative charge per unit area . You may ignore all edge effects. The particle has
an initial velocity ~v0 = v0 x.00
a) What is the magnitude [and direction] of the electric field between the plates?
I didnt bother solving this one manually, since I remembered the answer. Each infinite plate has a field

magnitude of
. The positive, top plate has a field pointing outwards, while the bottom, negative plate
20
has a field pointing inwards.
Since they are acting in the same direction between the plates, the answer is twice the above. Thus, the
~ = (
electric field between the two plates is E
y)
0
b) What is the magnitude of the acceleration of the electron when it is between the plates?
e
F~
F~ = m~a, so ~a = . F~ is given by the electric field strength times the magnitude charge e: F~ =
m
0
Therefore,
|~a| =

e
0 me

c) What is the y-component of the position of the particle when it reaches the plane defined by x = L?
This one is a bit harder to calculate - unless my solution is a bad one.
We were given that the initial velocity is purely in the x direction, and the electric force (the only force
we are to calculate) acts strictly in the y direction. Therefore, the velocity in the x direction will in fact
be constant, so we can easily calculate the time t taken for the electron to reach x = L:
L x0
L
=
v0
v0
We can now use a fairly complex equation from mechanical physics - it can a least be complex with all
values filled with expressions:
t=

yinitial

1
yf inal = yinitial + vinitialy t + ay t2
2
is given as 0, as is vinitialy , so that simplifies our situation a lot:
153

1
yf inal = ay t2
2
L
We found the acceleration previously; we just need to multiply it by t2 , where we found t = :
v0
 2
e
L
yf inal =
20 me v0
That concludes problem 1!

13.1.2

Problem 2: Three plates capacitor

Three infinite uniformly charged thin sheets are shown in the figure below. The sheet on the left at
x = d is charged with charge per unit area of 3 , The sheet in the middle at x = 0 is charged with
charge per unit area of +, and the sheet on the right at x = d is charged with charge per unit area of
2 . Find the x component of the electric field Ex in each of the regions 1, 2, 3, and 4.
The figure is simply three vertical lines, the sheets, and four regions. Region 4 is to the left of all of
the sheets, region 3 between the rightmost and the middle sheet, region 2 between the middle and the
rightmost sheet, and region 1 to the right of all the sheets.
This problem is very straightforward, as long as you know the electric field of an infinite plate to be

.
E=
20
We then calculate the electric field due to each plate, and use superposition to add them together, keeping
the directions in mind for each region.
3
0

E2 =
0
2
E3 =
0
E1 =

(2 + 1 3)
20

Region 2:
(3 + 1 + 2) = 0
20

(3 1 + 2)
Region 3:
20

Region 4:
(3 1 + 2)
20
Region 1:

Thats really all there is to it!

13.1.3

Problem 3: Electric field, potential, and electrostatic potential energy

Point charges Q1 , Q2 , and Q3 reside on three corners of a square with sides of 1 m; the distance from Q2
to P3 is 2m (see diagram).
(a) What is the electric potential, V , at P1 ? (Normalize the potential to be zero at and give your
answers in Volts). (Note: P1 is located at the unoccupied corner of the square.)
Well, we begin by calculating the electric potetial V for each of the charges alone, using V =

154

Q
:
40 r

VQ1 =

11 106
40 r

VQ2 =

3 106
40 r

7 106
40 r
The question then asks for the potentials at P1 , P2 and P3 . All we have to do is to add the three potentials
above, while substituting the distance to that point from each charge for r. That is a mess to write, but
easy to do, so Ill leave that to the reader. Its either as simple as 1 m or 3 m, or youll have to use the
Pythagorean theorem.
VQ3 =

(b) Are there points or surfaces in space (other than infinity) where V is zero?
Yes. We have both positive and negative charges, which means we will have both positive and negative
potentials at different locations in space. Potentials dont charge abruptly, so in an area where the potential transitions from positive to negative or vice versa, there will be a point where it exactly equals zero.
(c) What is the electrostatic potential energy of the system? Express your answer in Joules.
To solve this, we add up the electrostatic potential energies of each pair of charges:
Utotal = U12 + U13 + U23
Since U = qV , this is the potential at each point times the charge at that point, e.g.
U12 = V1 Q2
U13 = V1 Q3
U23 = V2 Q3
For r, part of the potentials, we use the distance between the two charges in question.
The answer to the question is then Utotal .
(d) Suppose we release the three charges so that they can move freely in empty space. How much energy
is released in the form of kinetic energy?
This is the only question of the homework so far that I didnt solve on my own, even with the book and
Google. The correct answer is The question is not well defined, if Ive understood it correctly because
the answer depends on the order you release the charges in, and (I think?) the times you do so.
I still dont see how there can be anything but one correct answer, if you release them all exactly simultaneously, though, even if we havent learned how to calculate it.

13.1.4

Problem 4: Electric field of a charged ring

A point-like negatively charged particle of mass m and charge q is initially released from rest from the
point P along the positive z-axis where the magnitude of the electric field is largest.
The diagram shows a charged ring, with radius R and total charge Q, centered on the z axis and located
o the x-y plane. The point P has coordinates (0, 0, z), where we are to find z:
(a) What is the distance from the point P to the origin? Express your answers in terms of the following
variables, if relevant: q, m, R, Q, and 0 .

155

Well, all we know about P is that the magnitude of the electric field is the maximum there. Clearly, our
first task is to find the electric field due to the ring, to begin with!
By symmetry arguments, along the z axis, the x and y components of the electric field will cancel out, so
we only care about the magnitude of the z component, Ez .
~ each of which carries a charge dq =
First, we divide the ring into tiny segment d`,
distribution is uniform, though the problem didnt actually state that).

Q
(the charge
2R

~ to the total electric field at the point we are interested in.


Each of these contribute a small amount dE

The distance between a point on the ring and P is given by r = R2 + z 2 , via the Pythagorean theorem.
So far, we have (from Coulombs law)
dq
Q
= 2 2
2
40 r
8 0 r R

dE =

We are only interested in the z component, dEz = dE cos (by vector decomposition), with being the
angle between the point P and d` (if we draw a line between the two, the angle is what that line makes
with the z axis).
z
Via geometry, we can find see cos = . If we make this substitution, and then substitute the value for
r

r = R2 + z 2 :
dEz =

Q
8 2 0 r2 R

cos =

Q
8 2 0 r2 R

z
r

Qz
8 2 0 r3 R
Qz

dEz =
2
8 0 R( R2 + z 2 )3
dEz =

dEz =

Qz
8 2 0 R(R2

+ z 2 )3/2

Finally, we want to integrate this over the entire circle. That turns out to be an easy integral, because
everything above is a constant!
I
Ez = dEz d`
The infinitesimal segment d` = Rd (arc length):
Z
Z 2
Ez =
dEz Rd =
0

Qz
8 2 0 R(R2

Qz
Ez = 2
8 0 (R2 + z 2 )3/2
Ez =

+ z 2 )3/2

Rd

d
0

Qz
40 (R2 + z 2 )3/2

We want to know where the field is at its maximum, so we start off by calculating the derivative of the
electric field:


d
1
Qz
Q(R2 2z 2 )
=
dz 40 (R2 + z 2 )3/2
40 (R2 + z 2 )5/2
For that, I thank Mathematica. We then set the derivative equal to zero, to find extreme points; we know
that Pz > 0, so we can discard the negative solution we find. One solution remains:
156

R
Pz =
2
... which answers part (a).
(b) What is the speed of the particle when it reaches the origin? Express your answers in terms of the
following variables, if relevant: q, m, R, Q, and 0 .
For this part, we want to know the potential difference between P and the origin. One way of doing that
is to integrate the electric field dot the direction vector. However, we know that they point in the same
direction (
z ), so we can skip the dot product and do a regular integration with dz:



Z 0
Z 0
z
1
1
Q
Q
~ =
~ dz

E
dz =
VP Vorigin =
40 Pz (R2 + z 2 )3/2
40
R
R2 + z 2
Pz
I admit, more Mathematica for that integral.
At least now the worst part is over. Only algebra remains.
We now know the potential difference, and can very easily calculate the potential energy difference by
multiplying the voltage difference by q, the charge of the particle.
Via conservation of energy, the gain/loss in potential energy plus the gain/loss in kinetic energy must
equal zero:
U + Ke = 0
The particle is released from rest, so Ke starts at zero, and Ke = Ke . U is
U = V (q)
So what is Ke ? Its the kinetic energy the particle has when it reaches the origin. We use classical
mechanics for this, so
1
Ke = mv 2
2
We set the sum of the two equal to zero, and solve for the velocity v, which will be our answer:
U + Ke = 0
1
V (q) + mv 2 = 0
2
2V (q) + mv 2 = 0
mv 2 = 2V (q)
2V q
v2 =
m
r
2V q
v=
m
Almost done! The only thing left is to put our ugly expression for V in there... Hold on:
v 


u
Q
u2
1
1

q
t
40
R
R2 +z 2
v=
m
R
... actually, I lied. z isnt allowed in the answer, but we solved that the point in question has z = .
2
157

v=

v 
u 

u
Q
1
u2

40
R
t

1
q
R2 +( R )2


q

m
... and that can be our final answer, if we choose to not clean it up.

13.1.5

Problem 5: Two spherical conductors

Two spherical conductors, A and B, are placed in vacuum. A has a radius rA = 25 cm and B of rB = 35
cm. The distance between the centers of the two spheres is d = 225 cm.
A has a potential of VA = 110 volts and B has a potential of VB = 40 volts.
(a) An electron is released with zero speed from B. What will its speed be as it reaches A? Express your
answer in m/sec.
The distance betwee is in fact superfluous information! We can calculate this using only the potential
difference:
VB VA = 150 V
VA VB = 150 V
(Well need both, as part (b) asks about a proton.)
The kinetic energy (in classical mechanics) is:
1
Ke = mv 2
2
We can solve that for v and use the equation to convert from potential energy to velocity:
1
Ke = mv 2
2
2Ke
= v2
m
r
2Ke
v=
m
So with that done, we need to know Ke and m. m is the mass, which for an electron is about 9.109 1031
kg. Ke can be found easily by the definition of the electron volt (eV):
1 eV = 1 e 1 V = 1.602 1019 J
where e is the elementary charge. We have a potetial difference of 150 volts, so
Ke = 150 V e = 150 1.602 1019 J
If we substitute that in the velocity equation, we get
r
2 (150 1.602 1019 )
v=
7.264 106 m/s
9.109 1031
... which is our answer for (a).
(b) A proton is released with zero speed from A. What will its speed be as it reaches B? Express your
answer in m/sec.

158

The charge of a proton is exactly the opposite of the electron, i.e. the magnitude is the same. Since the
question is now reversed (A to B instead of B to A), the only thing we need to charge from the answer
above is the mass. A protons mass is about 1.673 1027 , so:
r
2 (150 1.602 1019 )
v=
1.69 105 m/s
1.673 1027
Due to the much greater mass, the proton arrives with a much lower velocity than the electron.
(c) We now change the potential of B to VB = +25 volts. For this new configuration, what is the ratio of
the speed of the electron (as it arrives at A) and the speed of the proton (as it arrives at B)?
To solve this one, we recalculate the potential difference to be of magnitude 85 volts, and substitute that
in our equations:
r
2 (85 1.602 1019 )
velectron =
5.464 106 m/s
31
9.109 10
r
2 (85 1.602 1019 )
vproton =
1.275 105 m/s
1.673 1027
The ratio, and answer, is then
5.464 106
velectron
42.85

vproton
1.275 105
... which answers (c), and we are done!

13.1.6

Problem 6: Two conducting hollow cylinders

Two conducting thin hollow cylinders are co-aligned. The inner cylinder has a radius R1 , the outer has a
radius R2 . Calculate the electric potential difference V(R2 ) - V(R1 ) between the two cylinders. The inner
cylinder has a surface charge density of a = , where > 0, and the outer surface has a surface charge
density of b = 3.
The cylinders are much much longer than R1 . Thus, you may ignore end effects and neglect the thickness
of the cylinders.
(a) What is the electric potential difference between the outer cylinder and the inner cylinder V(R2 ) V(R1 )?
Okay, lets get to work! We choose a co-axial Gaussian cylinder and place it in between the two cylinders,
such that it encloses the inner, but not the outer cylinder.
Because they are co-axial, we dont need the surface integral from Gausss law, but can use EA instead:
Qins
0
A is the surface area of our Gaussian surface 2r`, where r is the radius and ` the length.
What is Qins ? Well, its the enclosed surface area of the inner cylinder, 2R1 `, times the charge density
a = : 2R1 `. Thus we have
EA =

2r`E =
We can cancel out a few terms, and solve for E:

159

2R1 `
0

R1
0
R1
E=
0 r
This is the the electric field for R1 < r < R2 , i.e. between the cylinders. Keep in mind that this is not
valid for the later sub-questions!
rE =

~
Now then. The question was about the potential difference, so we integrate the field dot the direction dr,
from R2 to R1 :
Z R1
Z
R1 ~
R1 R1 dr

V(R2 ) V(R1 ) =
dr =
0 r
0 R2 r
R2
~ and E
~ to both be radially outwards, we reduce the integral to a very, very simple
Because we know dr
one.
 
Z R1
R1
R1
dr

= ln r = ln R1 ln R2 = ln
r
R2
R2
R2
Lets add in the rest, the constant stuff we had before the integral:
 
R1
R1
ln
V(R2 ) V(R1 ) =
0
R2
That answers question (a)!
(b) What is the magnitude of the electric field outside the cylinders, r > R2 ?
Well, lets do what we did for (a), since that worked a treat. Th difference here is that Qins has changed,
but the calculation is the same other than that. Where we had 2R1 `, we now have a density that
depends on R1 and R2 as well as two sigmas. Lets start over with Gausss law:
Qins
0
A is still the area of the Gaussian surface, so A = 2r`. Now, lets find Qins :
EA =

QA = R1 2`
QB = 3R2 2`
Qins = QA + QB = (3R2 R1 ) 2`
Put it all together:
2r`E =

(3R2 R1 ) 2`
0

rE =

(3R2 R1 )
0

E=

(3R2 R1 )
0 r

... which answers (b).

160

(c) What is the electric potential difference between a point at a distance r = 2R2 from the symmetry axis
and the outer cylinder V(2R2 ) - V(R2 )?
Well need to integrate the electric field again.
Z R2
Z
(3R2 R1 ) R2 dr
(3R2 R1 ) ~
V(2R2 ) V(R2 ) =
dr =
0 r
0
2R2
2R2 r


R2
We have already solved that integral: it is ln
. Thus, we have
2R2


R2
(3R2 R1 )
ln
V(2R2 ) V(R2 ) =
0
2R2
... which answers (c), and we are done!

13.1.7

Problem 7: Speed of an electron

Finally, the last problem of the week. Writing these down in this LATEXdocument has taken a lot of time
- Ive essentially re-solved them from scratch, having only a few scratch equations left from when I solved
them the first time. That is surely an excellent way to learn and truly understand, though!
An electron is projected, with an initial speed of vi = 1.74e + 05 m/sec, directly towards a proton that is
essentially at rest. If the electron is initially a great distance from the proton, at what distance from the
proton is its speed instantaneously equal to twice its initial value? Express your answer in meters.
My first instinct was that more information must be required, but as expected (they wouldnt ask unanswerable questions!), we can solve this. Fairly easily, too, once we know how.
We know the initial velocity, and so we can calculate the initial kinetic energy. We also know the final
velocity - twice the original one - and can calculate the final kinetic energy. Via conservation of energy,
the difference is the two is the change in electric potential energy U , which we can relate the the electric
potential V of the proton, to find the distance between them.
So, lets begin. First, the initial kinetic energy of the electron is

Kef inal

1
Keinitial = me vinitial 2
2
1
1
= me vf inal 2 = me (2vinitial )2
2
2


1
1
1
U = me (2vinitial )2 me vinitial 2 = me (2vinitial )2 vinitial 2
2
2
2
Lets now find the electric potential due to the proton. That one is easy - we know the formula:
Qp
40 r
The distance between the two is initially great, so we treat it as if the electric potential energy is 0 to
begin with. Thus, U = U . We also know that U = V q, where V is the electric potential due to the
proton, and q is the charge of the electron.
Therefore, we can now set the sum of our two expressions for U equal to zero, and solve for r. The charge
of the electron is q = e, so we get:
V =

 e Qp
1
me (2vinitial )2 vinitial 2 +
=0
2
40 r
Messy, but if we solve for r and substitute Qp = +e, we get:
161

r=

e2
60 me vinitial 2

If we stick our values in there, we get


r=

(1.602 1019 )2
5.556 109 m
12
31
5
2
6 (8.854187 10 )(9.109 10 )(1.74 10 )

That is, about 5.56 nanometers! After travelling for a very long distance, it only reaches twice its initial
speed less than a picosecond before they collide. Wow.
Finally, last question of the week:
In your opinion do we need to worry about the special theory of relativity for the speeds given in this
problem?
The answer is no. The speed is only about 0.1% of the speed of light, so relativistic effect are negligible.
Even at 10% of the speed of light, the relativistic effects are fairly small.

13.2
13.2.1

Week 3
Problem 1: Spherical capacitor

A capacitor consists of two concentric spherical shells. The outer radius of the inner shell is a = 0.6 mm
and the inner radius of the outer shell is b = 2.96 mm.
(a) What is the capacitance C of this capacitor? Express your answer in Farads.
Q
Lets begin by stating that capacitance C = , where V is the potential difference between the two
V
spheres. Q is a given, so if we find V , we should be able to solve this problem easily. To find V , we can
integrate the electric field between the two radii, so lets begin by finding the electric field!
We know since before that a spherical shell has the same electric field as a point charge would, as long
as youre outside the shell; and we know that the electric field inside the larger sphere, due to the larger
sphere, is zero.
Therefore the electric field for a < r < b is given by the charge on the inner sphere a only:
~ =
E

Q
r
40 r2

Z b
Q
dr
~
~
Va Vb =
E dr =
r
40 a r2
a
We know that the electric field from a sphere is radially outwards and thus always parallel to dr, so we
1
can ignore the vectors and the dot product, and simply integrate 2 and evaluate at the limits:
r
 


Z b
Q
Q
Q
dr
1 b
1 1
Va Vb =
=
=

40 a r2
40
r a
40 a b
Z

The latter part of the result can be simplified:


1 1
ba
=
a b
ab
And therefore, the reciprocal, which well need very soon, is
1
a

1
b

=
162

ab
ba

Now then. Back to our potential difference result. Now that we have V (the potential difference Va Vb )
we can divide Q by that, which will get rid of the Q and flip everything else upside down, so we get:
Q
ab
= 40
V
ba
We were asked to answer numerically, for a = 0.6 mm and b = 2.96 mm:
C=

(0.6 103 )(2.96 103 )


8.37299 1014 F
3
3
2.96 10 0.6 10
(b) Suppose the Maximum possible electric field at the outer surface of the inner shell before the air starts
to ionize is Emax (a) = 3.0106 Vm1 . What is the maximum possible charge on the inner sphere? Express
your answer in Coulombs.
C = 40

Well, the magnitude of the electric field is (again) given by:


E=

Q
40 r2

If we solve the equation for Q and substitute E = 3 106 V/m and r = a = 0.6 103 m we should get the
answer:
Q = 40 r2 E = 40 (0.6 103 )2 (3 106 ) 1.2059 1010 C
(c) What is the maximum amount of energy stored in the capacitor? Express your answer in Joules.
The energy stored can be calculated in these ways:
1
1
1 Q2
Energy stored = CV 2 = QV =
2
2
2C
Since we know Q and C, we prefer the latter form, and so our answer is

(d) When E(a) = 3.0 106 V m1


Express your answer in Volts.

1 Q2
8.6838 108 J
2C
what is the absolute value of the potential difference between the shells?

We have the equation from the potential difference from question (a), we have the charge of a at this
E-field strength from question (b), and we have the value of all the constants required to find the answer:




Q b a 1.2059 1010

(2.96 0.6) 103



1435.1 V
|V | =
=
40 ab
40
(2.96 103 ) (0.6 103 )

13.2.2

Problem 2: Coaxial cylinders

A very long cylindrical capacitor consists of two thin hollow conducting cylinders with the same axis of
symmetry. The inner cylinder has a radius a, the outer one has a radius b. You may ignore end effects.
There is a figure that shows the inner cylinder with charge +Q and the outer with charge Q.
(a) What is the capacitance per unit length?
Well, we need to do roughly the same thing as we did with the sphere. First, we want to find the electric
field.
Lets set up a Gaussian cylinder with a < r < b. The total charge on the cylinder is Q, so the charge per
Q
unit length is = . Via Gausss law, we have that
`
163

EA =

Qenc
0

A is the surface area, so:


E2r` =

Qenc
0

Qenc
2r`0
The enclosed charge is the charge per unit length times the length, so:
E=

E=

`
20 r`

20 r
The ` cancels - anything else would be bizarre; surely the length we choose for the Gaussian cylinder cant
change the electric field strength.
E=

We carry out the good old integration to find the potential difference:
 
Z b
Z b

b
dr

~ dr =
=
ln
Va Vb =
E
20 a r
20
a
a
We know that C =

Q
, and Q = `:
V
C=

`
20 `

=
|Va Vb |
ln ab

The question asked for the capacitance per unit length, so the ` disappears.
C=

20

ln ab

(b) Now consider the limit where b is very close to a. Express b as a + ; where
 1. What is the
a
capacitance per unit length in that limit? Hint: in that limit you can use the following approximation:
ln (1 + x) x
Express your answer in terms of a, and 0 .
OK, well, we can rewrite with b = and see how that looks:
C=

20
20
=

a+
ln a
ln 1 + a

Following the hint, we then ignore the 1 and the whole logarithm:
C

20 a

... which is our final answer for (b).

164

13.2.3

Problem 3: The effect of a dielectric medium on capacitance

Consider a capacitor made of two square plates of side `. The distance between the two plates is d.
(a) We insert a dielectric of dielectric constant K > 1 and width ` a distance x (as in the diagram). What
is the total capacitance of this arrangement?
We should be able to treat this like two separate capacitors. From the diagram shown, it seems to me as
if we then need to add the capacitance of the two, as they would be connected in parallel. Lets try that
and see what happens.
The plates have area ` `, but are then split up into two regions. Sketch it up on paper and it will make a
lot of sense. The capacitor with the dielectric is then made of sides x and `, so Adielec = lx while the one
without the dielectric is ` (` x) = `2 lx, which certainly looks sensible.
We have the good old parallel plate capacitor equation:
A0
K
d
Lets call the capacitor without the dileectric C1 , and the capacitor with the dielectric C2 :
C=

(`2 `x)0
d
`x0
C2 =
K
d

C1 =

Ctotal

(`2 `x)0 `x0


+
K = 0
= C1 + C2 =
d
d

(`2 `x) `x
+ K
d
d

(b) The capacitor is now connected to battery which provides a difference of potential V0 across the capacitor. What is the energy stored in the capacitor?
The energy stored is

1
CV 2 , so we can mostly copy and paste from the previous answer:
2
 2

(` `x) `x
1
0
+ K V0 2
2
d
d

(c) While the battery is still connected to the capacitor, we now move the dielectric slab a bit further in
between the plates, increasing x by an amount . What is the change in the energy stored in the capacitor?
Well, lets see. Lets first calculate the increase in capacitance. First, the new total will be:
 2

(` `(x + )) `(x + )
Cnew = 0
+
K
d
d
If we subtract the two and simplify, we get the difference in capacitance:
0 (K 1)`
d
(The notation becomes slightly awkward as C is not times C, but rather the change in C. Same for
E below.)
We can then use the half-C V squared formula to find the extra energy stored in C:
C =

1
1 0 (K 1)` 2
E = CV 2 =
V0
2
2
d

165

That answers part (c), or (c1). Then come the highly related follow-ups, which are unlabeled, so lets call
them (c2) and (c3). First, (c2), with my label:
(c2) What is the work done by the battery while we push the dielectric slab in from its original position
x to x + ? (Make sure you have the correct sign!)
Uh oh! Well, intuitively, I entered the same as for (c1), since it seemed like it would make sense for the
batterys work to be the same as the added energy. Nope!
The correct answer is twice of (c1), so just remove the half factor in the front. The answer to the next
question adds some insight...
(c3) What is the work done by us while we push the dielectric slab in? (make sure you have the correct sign!)
The answer here is the negative of (c1). That is, we do negative work, equal in magnitude to the added
charge on the capacitor. The battery, meanwhile, must provide both the added energy stored in the capacitor and cover up the negative work we do!
The reason this happens is that due to electrostatic forces, the dielectric is pulled into the capacitor.

13.2.4

Problem 4: Coaxial cable with dielectric

A certain coaxial cable consists of a copper wire, radius a, surrounded by a concentric copper tube of
inner radius c. The space between is partially filled (from b out to c) with material of dielectric constant
K. The goal of this problem is to find the capacitance per unit length of this cable. You may neglect edge
effects.
Note that there is space a < r < b which is vacuum! Thus, we can not simply calculate a single
electric field and use that everywhere!
Note that for technical reasons, we use the symbol ` for charge per unit length, rather than the more
typical . Do not get confused, ` is not a length!
Argh, okay then. Well see how many times I screw that one up before I remember.
a1) Assume that the copper wire has uniform positive charge per unit length ` and the copper tube has
uniform negative charge per unit length on its inner surface `. Calculate the radial component of the
electric field in the region 0 < r < a.
Well, since there is no mention of a applied potential difference, and this is conductor, this is the likely
the easiest sub-question of this week: the answer is zero.
a2) Calculate the radial component of the electric field in the region a < r < b.
Frankly, Ive already done that in earlier examples, so Ill just nick the answer from there. The equation
looks like the one for electric potential due to a point charge, except Q is replaced by the charge per unit
length, and its multiplied by two. So:
`
20 r
Calculate the radial component of the electric field in the region b < r < c.
Er =

In other words, inside the dielectric. Well, we just divide the above by the dielectric constant. No more,
no less.

166

`
20 Kr
Calculate the radial component of the electric field in the region r > c.
Er =

Ah, finally some work to do. Well, not really. We apply Gausss law and find no net enclosed charge (+`
and ` per unit length, respectively, for the two cylinders), so the electric field outside is also zero. Too easy.
(b1) What is the potential difference V(b) - V(a) (be careful about sign)?
Well, is there any work this time around, then? Yes! There is an electric field in the region, that we can
integrate. It is, as per above:
Er =

`
20 r

~ vector we would use too, so this is a simple integral:


As usual, the field is radial and the dr
Z a a
Z a
a
`
`
dr
`
dr =
ln
Vb Va =
=

20 b r b
20
b
b 20 r
(b2) What is the potential difference V(c) - V(b) (be careful about sign)?
Clearly, we need to integrate just as above, but with the field that is also divided by K (see above), and
from c to b instead of b to a. We can thus add K to the solution and rename some variables:
 
Z b
Z b b
`
`
dr
`
b
Vc Vb =
dr =
ln
=
20 c r c 20 K
c
c 20 Kr
What is the magnitude of the potential difference |V(c) - V(a)|?
Uh oh! We have two different electric fields between those points, so we need to integrate twice. Or
integrate zero times, perhaps, because we already have. Lets see then, we want
Z a
~
~ dr
E
Vc Va =
c

~ between those points, so we integrate in two parts:


... but there is no one E
Z b

Z a


~ +
~
|Vc Va | =
E~cb dr
E~ba dr

c

We just solved those integrals:



 
 a 
`
b
`

|Vc Va | =
ln
+
ln
20 K
c
20
b
Lets factor out that term:


 
 a 
`
1
b

|Vc Va | =
+ ln
ln
20 K
c
b
Now we just need to take the absolute value of that expression, since the absolute value bars wont be
accepted as part of the answer.
` is the magnitude of the charge per unit length, so that must be positive; all the other constants are
positive. Only the natural logs could cause this to become negative... so when are they zero? They are
when the argument is less than one. Since a < b < c, what can we say about the ratios inside? It seems
they will both become less than one!

167

Therefore, we need to negate the whole thing (or otherwise turn it positive, by mucking around with
signs/order inside):

 
 a 
`
1
b
|Vc Va | =
ln
+ ln
20 K
c
b
(c) What is the capacitance per unit length?
If we temporarily ignore this problems variable names, and use sensible ones:
Q
V
Q=`

C=

where is the charge per unit length, and ` is the length. V is the big chunk we found above, the potential
difference between the inside and the outside, so to speak.
In this problem, ` is charge per unit length. Due to this confusion, I will temporarily use x to mean length,
such that Q = `x. We then find the answer to our problem by dividing this Q first by V above, and then
by x to get per unit length. If it seems nonsensical to introduce a variable only to divide it out, I do so to
Q
as-is. Well then, lets go:
us C =
V
`x


C=
1
`
20 K ln cb + ln ab
First, lets divide out x. Easy enough, it just disappears from the top there. After that, note that `
cancels out - the capacitance doesnt depend on the charge per unit length. So remove the x, turn ` into
1, and factor out that fraction within a fraction. If we factor out the a fraction from the bottom, it turns
upside-down, so we now have:
C = 20

1
K

ln

b
c

1

+ ln

a
b



Ill leave it at that. Still a bit messy, but better than the original expression.

13.2.5

Problem 5: Capacitor network

Three parallel plate capacitors (C1 , C2 and C3 ) are in series with a battery of 100 V. C1 = 2500F;
C2 = 2C1 , C3 = 3C1 .
(a) What is the potential difference over each capacitor and how much charge is on each capacitor?
To solve this problem, we begin by stating an important rule: the charge on capacitors in series is the
same for each capacitor. The magnitude of charge must always be equal on to adjacent plates, whether in
the same capacitor or not, so Q1 = Q2 = Q3 . With that in mind, we can do some equation juggling.
Q
Vi
Q = Ci Vi
Ci =

Q
Ci
We also know that the potential difference of the three must add up to the 100 volts of the power supply,
so we set up that equation:
Vi =

Q
Q
Q
+
+
= 100
C1 C 2 C3
168

Q
Q
Q
+
+
= 100
C1 2C1 3C1
If we solve that equation for Q, we get
Q=

600
C1
11

So given our values for this problem,


3
0.1363636 C
22
Since the charge is equal for all three capacitors, this answers 3 out of 6 questions.
Next up is finding the potential difference across each capacitor. Now that we know Q (and the given Ci ),
this is easy.
Q=

3
Q
=
C1
22
3
Q
=

V2 =
C1
22
Q
3
V3 =
=

C1
22
V1 =

1
2500 106
1
2 2500 106
1
3 2500 106

(b) We now connect a 4th parallel plate capacitor (C4 = 4C1 ) to the battery. One side of the capacitor
is connected to the negative side of the battery, the other side is connected to the positive side of the battery.
Now C4 is in parallel with C1 + C2 + C3 .
Solving for C4 should be easy, since its essentially independent on the others. We know C = 4C1 =
4 2500F and V = 100V , so we know Q and V for that one.
Using the same logic, the other capacitors are unaffected by the addition of a fourth capacitor parallel to
the battery, so we simply copy and paste our answers for those three capacitors!
(c) We make one more change: We still have our 4 capacitors as above, but we now place a dielectric
(K = 3) between the plates of the 4th capacitor. What is the potential difference across C4 and what is
the charge on it?
Well, a voltage source is conneted, so V cannot change and will remain at 100 volts.
Because V cannot change, and C must change (Cnew = KCold ), the charge will also increase by a factor
of K via Q = CV :
Q4 = KCold V = 3 4 2500F

13.2.6

Problem 6: Resistances of conducting wires

Three conducting straight wires (though insulated from each other) are each 2 m long. They are electrically
connected to each other only at their ends points A and B. AB = 2 meter.
The wires are cylindrical. One is made of copper, one of aluminum and one of iron. Their radii are 2, 3
and 4 mm, respectively.
a) What is the ohmic resistance between point A and B? Express your answer in Ohms. (you might need
to look up the resistivity of these elements).
Lets start out by finding the symbolic answer; after that, we can look up the resistivity and calculate the
numerical answer.
First off, when connecting multiple resistors in parallel, the formula to use for the equivalent resistance is:
169

Requiv =

1
R1

1
R2

1
+

1
R3

Looking about for a useful equation to use, I found Pouillets law:


`
A
where is the resistivity in ohm-meters, ` the conductors length and A its cross-sectional area. We can
then begin to find the resistance of each wire alone, using the above formula.
R=

lCu = lAl = lF e = 2 m
ACu = (2 103 )2 m2
AAl = (3 103 )2 m2
AF e = (4 103 )2 m2
Substistuting in some resistivity values, we find that
RCu = 1.68 108

2
(2 103 )2

RAl = 2.82 108

2
(3 103 )2

RF e = 1.0 107

2
(4 103 )2

The total resistance (which is lower than the resistance of each individual conductor, since there are
multiple ways for the current to flow) is then
Rtotal =

1
1
RCu

1
RAl

1
RF e

0.000887591 0.888 m

We now attach at B a straight copper wire which is in electrical contact with the 3 wires at B. This 2nd
copper wire is identical to the one between A and B. It runs from B to C. The distance AC is 4 m.
(b) What is the ohmic resistance between A and C? Express your answer in Ohms.
The resistance of the wire is of course the same as RCu above, so we take Rtotal and add another RCu ,
since the current must pass through the sum of the resistances this time (they are in series).
RAC = Rtotal + RCu 0.00356139
We now turn AC into a near perfect circle. A is very close to C but it is not touching C.
(c) What is the ohmic resistance between A and C now? Express your answer in Ohms.
Well, if its not touching, then nothing has changed. Whether the conductor is a straight line or a near-circle
does no difference, so the answer is same as in part (b).

13.2.7

Problem 7: Resistor network

Ugh, this was more than a pain... I wont actually list the full solution. For finding the currents, I used 1
KCL equation at node B and 2 KVL equations, around each of the two smaller loops (the ABMGA and
BDNMPB).
For node B, I noted that
I1 = I4 + I2
170

For the KVL equations, I went clockwise, adding resistor voltage drops if the current was in the same
direction I was going, otherwise subtracting. For voltage sources, I added if I hit the + side first, and
otherwise subtracted.
I1 R1 I4 R4 V2 I1 R5 + V1 = 0
V3 I2 R2 I2 R3 + I4 R4 + V2 = 0
After solving this system of three equations, you get a RIDICULOUSLY complex expression for each
current, with roughly 14-17 terms per equation. If we substitute the values from the problem into those,
we get the currents:
2442
29431
723
I2 =
1549
11295
I3 =
29431
Note that I3 is negative, indicating the real direction is the opposite of what the problem assumed, so that
current goes downwards.
I1 =

After that, youre asked to find three potential differences, which was even more of a pain, at least using
my approach. Id probably solve this using the node method and superposition, if I did this again.

13.3

Week 4

Theres no howework this week due to the upcoming midterm exam!

13.4
13.4.1

Week 5
Problem 1: Lorentz Force

An electron has velocity components vx = +100 m/s, vy = 80 m/s, and vz = +75 m/s. It enters a
region of uniform B-field with components Bx = 103 T, By = 7 104 T and Bz = 5 104 T.
What are the components of the force acting on the electron? Express your answer in Newtons.
Since no electric field is mentioned, well assume that Lorentz force here means the magnetic force alone,
so
~
FB = q(~v B)
This problem could probably be solved in a single line of Mathematica, which is what Ill do, for the most
part. Some quick analysis first, though. Since the force is given by the cross product of the velocity and
the magnetic field, only components which are perpendicular to each other will be part of the answer.
Indeed, the cross product, in components, is defined as
hFx , Fy , Fz i = q hvx , vy , vz i hBx , By , Bz i = q hBz vy By vz , Bx vz Bz vx , By vx Bx vy i
Very, very ugly... but note how all multiplied quantities are perpendicular to each other. If we use this,
and stick our numbers in, including for q, we get

171

Fx = 2.003 1021
Fy = 4.005 1021
Fz = 1.602 1021
... which is marked as being correct.

13.4.2

Problem 2: Motion of a charged particle in magnetic field

An ion of charge q and mass m, is accelerated from rest by a potential difference of V = 25 kV. The
particle then enters a magnetic field region (with strength B = 0.01 T) where the B-field is uniform and
perpendicular to the ions velocity. The ion then travels on a circular path with radius R = 2 m.
(a) Write an expression for the mass, m, of the ion in terms of q, B, R and V . Note that the answer
should be a formula, not a number.
We have our good old radius equation, which is
s
R=

2mV
qB 2

We get what we need if we solve it for m:


R2 =

2mV
qB 2

R2 qB 2 = 2mV
qB 2 R2
2V
(b) Now suppose we have another particle: a positively ionized deuteron (the deuteron mass is 3.341027
kg) accelerated through the same voltage and then entering the same magnetic field. What is the radius
of its trajectory? Express your answer in meters.
This ought to be even easier, we simply put the numbers into the formula we had above, before we solved
it for m:
s
s
2mV
2(3.34 1027 )(25 103 )
R=
=
3.22869 m
qB 2
(1.602 1019 )(0.01)2
m=

(c) How long does it take the deuteron to move around a full circle once. Express your answer in seconds.
Looking back at the lecture notes, a useful formula is
T =

2m
qB

... which is independent of the velocity, very cool. We need to multiply by to take care of relativistic
correction, but that should not be an issue as an electron (with a much lower mass) with 25 keV is nonrelativistic, so this particle must be, too.
Plugging in the values, we get
T

2 3.34 1027
1.30998 105 s
19
1.602 10
0.01
172

13.4.3

Problem 3: Cyclotron

Consider a deuteron in a cyclotron with field strength 0.5 T. The deuteron is accelerated twice per rotation by a potential of V = 25 kV.
(a) If the radius of the cyclotron is 2 meter, what is the maximum energy of the deuteron? Express your
answer in Joules (the deuteron mass is 3.34 1027 kg)
I will again assume that relativistic corrections are not necessary. We can use the radius equation again,
but solve for the velocity, and use that to figure out the energy.
R=
v=

mv
qB

(1.602 1019 )(2)(0.5)


qRB

4.7964 107 m/s


m
3.34 1027

The energy is then given by


1
1
E = mv 2 3.34 1027 (4.7964 107 )2 3.8419 1012 J
2
2
(b) Starting from a negligibly small velocity, how many full rotations does the deuteron need before it
reaches this maximum energy?
We can figure out the potential difference required to accelerate it; E = qV , so V =

E
:
q

3.8419 1012
23.982 106 V
19
1.602 10
Divide that by the 50 kV per rotation, and we get 479.64 480 rotations.
V =

(c) What is the time it takes for the deuteron to make one complete rotation when its energy is about
500 keV and when it is about 5 MeV? Ignore possible relativistic effects. Express your answer in seconds.
We have the time formula since before, so
T =

2m
qB

m and q are the same as in the previous question, but B is now 0.5, so:
T

2(3.34 1027 )
2.61995 107 s
19
(1.602 10 )(0.5)

Since the time is independent of velocity, and thus kinetic energy, this is the answer for both questions!

13.4.4

Problem 4: Rectangular current loop

A current I travels counterclockwise through a closed copper wire loop which has the shape of a rectangle
with sides a and b.
What is the magnitude of the magnetic field at the center, C, of the rectangle? Express your answer in
terms of a, b, I and 0 .
Well, the illustration is hardly required; it only labels the sides, essentially. a is the left/right sides, while
b are the top/bottom sides. Not that it really matters.
Anyway. Due to symmetry, we only need to calculate the contribution due to one a and one b side, multiply
each by two to account for the other, duplicate side, and add the results.
Using the right-hand curl rule, its clear that the field is additive: all four sides will contribute to a

173

magnetic field that comes out of the page at the center of the rectangle.
Well, thats about as far as we can go qualitatively. What about some numbers? Or, at least, some
equations? Well have to use Biot-Savarts law for this one.
Remember that
Z
I ~
0
~
B=
(d` r)
4
r2
If we draw a diagram, for which I recommend you watch the week 5 problem solving videos, we get
something like this (screen capture from the video):

As shown there, if we call the distance to the point we measure at d, and the distance along the axis x,

d
we get r = d2 + x2 via the Pythagorean theorem. We also get that sin = , which comes in useful.
r
~ r = |d`|sin,
~
The reason we have a sine in there is because of the cross product d`
since the magnitude
of r is 1 by definition. If we make that substitution of the sine in the shown |dB| equation, while also
substituting the square root-value for r, we get the final equation shown:
Z 0.5`
Z 0.5`
0
dx
0
dx
~
|B| = 2
Id 2
=
Id
2
3/2
2
4 (x + d )
2
(x + d2 )3/2
0
0
The integral is
Z
dx
x
=
(+ C)
2
2
3/2
(x + d )
d 2 x2 + d 2
... where ` is the length of the wire segment. We now use this equation to find the magnetic field due to
a and b, multiply each by 2 (the 2 in the equation has a different origin: the result without in is only for
one-half of the length) and add them up, and we should have our final answer.
Lets now look at the rectanglular loop we had again. d is the distance to the center, measured from the
a
b
middle of the wire. For the b part, that distance is then , while it would be for the a part. What is
2
2
then x (and dx)? Well, thats the coordinate along the axis of the point we are at on the wire.
Lets try to calculate the magnitude of the magnetic field due to one a (short) side alone. We use the
integral result, and make sure not to forget about the constants we moved in front, and evaluate the
integral at 0.5` and 0. We also multiply the whole thing by 2, to take care of both wire segments:
174

x=0.5`

|Ba | = 2

0 b
x

q
I
2 2 ( b )2 x2 + ( b )2
2

x=0

After some painful algebra and simplification, the above is


2aI
0
b a2 + b2
If we do the same for the b parts, it would be fairly shocking if the result was anything else than the same
thing with a and b swapped, so
|Ba | =

2bI
0
a a2 + b2
And, finally, the net answer is the sum of the two. Lets factor out the common parts first:


20 I
a b
+
B=
a2 + b 2 b a
|Bb | =

Finally, we can simplify that a bit further; if we first combine the fractions to the right, we get
a b
a2 + b 2
+ =
b a
ab
20 I a2 + b2
20 I(a2 + b2 )

B=
=
a2 + b2 ab
ab a2 + b2
Because

a2 + b 2

= a2 + b 2
a2 + b 2

c2
(think about right triangles, with c2 = a2 + b2 and c = a2 + b2 ; it then essentially says that
= c), this
c
becomes

20 I a2 + b2
B=
ab
... and we are finally done. Prior to this problem, I thought this weeks homework was uncharacteristically
easy. That changed a bit!

13.4.5

Problem 5: Resistor network

Consider this circuit with the following arrangement:


[Diagram of a cube, with a battery connected to opposite corners of the cube.]
Each edge of the cube is a resistor with resistance R = 10 (there are a total of 12 resistors).
What is the equivalent resistance Req from one corner of the cube to the diagonally opposite corner? (Hint:
think about a steady current I flowing from the battery, how does the current split as it reaches and then
moves through the wires making up the cube?)
I had a look at the recommended reading material (helpful content from the book), and... the second
listed item had the answer. Uh, I feel like I just cheated! I looked it through, and sure enough, the problem
is indeed identical, and the books answer is correct for this problem.
Heck, there shouldve been a spoiler warning there!

175

Because of the graphical layout of the problem, this is explained simply in the book, while an explanation
here would be harder to follow. For that reason, I refer to the book (or a search engine, which will no
doubt find many solutions) for this problem.
For what its worth, the answer is
5
Req = R
6

13.4.6

Problem 6: Coaxial current loops

Two concentric circular loops have radii R1 = 9 cm and R2 = 26 cm. They lie in the same horizontal
plane. The direction of the two currents are opposite. Seen from above the current through the outer loop
is I2 = 15 A clockwise, the one through the inner loop is I1 = 10 A counterclockwise.
What is the magnitude [and direction] of the magnetic field at the center of the loops? Express your
answer in Teslas.
The problem diagram helpfully also shows two unit vectors; z upwards, and r radially outwards, with the
current loops centered in the coordinate system.
Via Biot-Savart:
0
B=
4

I ~
(d` r)
r2

~ r| sin = d`, since the two are always perpendicular, and the magnitude of the
The cross product is |d`||
unit vector is 1:
Z
0 I
d`
B=
4r2
The integral of d` will come out to be 2r, so
0 I 2r
0 I
=
2
4 r
2r
We add the magnetic fields due to each current loop alone, keeping direction in mind (the fiels oppose
each other in the middle):


0 I1
I2
B=

3.3564024 105 T
2 R1 R2
B=

13.4.7

Problem 7: Parallel plate capacitor

A parallel plate capacitor has plates of area A and separation d. We connect the capacitor to a battery
of V volts. We then disconnect [the battery] when the capacitor is fully charged.
(a) How much energy is stored in the capacitor? Express your answer in terms of the following variables
if needed, A, V , d and 0 .
Capacitance is given by
A0
d
(times , but theres no mention of a dielectric, and we cant use it in our answer either). Stored energy is
C=

1
AV 2 0
U = CV 2 =
2
2d
176

Alternatively, we can use the field energy formula:


Z
1
0 E 2 dV
U=
2
R
dV = Ad, because the electric field is zero everywhere else. Thus the integral, which is over all space,
is just over the volume enclosed by the capacitor plates. Thus we have
1
U = 0 E 2 Ad
2
V = Ed, so E =
1
U = 0
2
U=

V
d

 2
V
Ad
d
AV 2 0
2d

Unsurprisingly, we get the same answer either way!


We now increase the plate separation by a small amount dz.
(b) What now is the Electric field in the capacitor? Express you answer in terms of the following variables
if needed, A, V , d, dz and 0 .
Remember that the battery has been disconnected ! V can and will vary as we separate the plates! The
charge Q on the plates is fixed, however. Q = CV ; C will go down as d increases, and so V must go up
to compensate.
What about the electric field? E =
must stay the same, so the answer is

V
, but also E = . The latter makes it clear that the electric field
d
0

E=

V
d

(since we arent allowed to use or Q in the answer).


(c) What now is the energy stored in the capacitor? Express you answer in terms of the following variables
if needed, A, V , d, dz and 0 .
Okay; so energy stored is given by many different equations:
1
1 Q2
1
AV 2 0
U = CV 2 =
= QV =
2
2C
2
2d
Since all must obviously be true, we can pick the one(s) that suits us. From the first, it looks as if the
stored energy has gone up, as V has increased by the same factor C has decreased, but V is squared.
From the second, it also looks like its gone up. Same for the third. And fourth. So clearly, we did work
in separating the plates, and the energy has increased.
To what?
1
Looking at U = QV , we know that Q in constant and V increases linearly with the distance. Therefore
2
U must also increase linearly, so
Unew = Uold
177

d + dz
d

AV 2 0 d + dz
AV 2 0 (d + dz)

=
2d
d
2d2
(d) What force do I have to apply on one plate (the other plate is fixed), to separate the plates by the
amount dz? Express you answer in terms of the following variables if needed, A, V , d, dz and 0 .
Unew =

At first glance, this confuses me a bit. I guess the force required is an infinitesimal amount higher than
the force pulling the plates together, but how do we specify that tiny extra bit? Lets analyze it, though.
Force times distance is work, so if we figure out the work required, that should help us out. Okay.
Well, the plates attract each other via their electric fields. However we must be careful to not use the sum
of their two electric fields; objects cant exert a force on themselves, so only the force from one plate on
the other will matter.
I will imagine that we move the bottom plate downwards, and that the bottom plate has negative charge
on it.
The electric field that the bottom plate is in, due to the top plate, is

20

V
Since the plates contribute equally to the total field of
= , the field due to the top plate should be
0
d
V
Etop =
2d
The magnitude of the force pulling the bottom plate upwards is then
E=

F =

V
Q
2d

We can find Q by knowing C and U :


U=

1 Q2
2C

2U C = Q2

Q = 2U C
Now, clearly, if we try to find the work by multiplying this by the distance moved, and then divide away
the distance to get force, we get back what we have. So lets substitute Q in there:

F =
=
=
=

r
V
V
AV 2 0 A0
2U C =
2(
)(
)
2d s
2d
2d
d
 2 2 2
V
A V 0
2
2d
2d2


V AV 0
2d
d
2
AV 0
2d2

... and that is indeed accepted as the correct answer!

178

13.5
13.5.1

Week 6
Problem 1: Amperes law in action

A long coaxial cable consists of two concentric conductors. The inner conductor is a cylinder with radius
R1 , and it carries a current I0 uniformly distributed over its cross section. The outer conductor is a
cylindrical shell with inner radius R2 and outer radius R3 . It carries a current I0 that is also uniformly
distributed over its cross section, and that is opposite in direction to the current of the inner conductor.
~ as a function of the distance R from the axis.
Calculate the magnetic field B
Well, the problem name says what we should do.
First, lets think about what will happen with the magnetic field inside the outer shell, due to the outer
shell.
We choose an Amperean circle with radius r = R2 , and an open surface that is just the circles area, and
get
!
d x ~ ~
E dA
2rBinside = 0 Ipen + 0
dt
S

Since we are only considering the field due to the outer shell, Ipen is zero, since the surface is smaller than
the shell. What about the electric flux? That is also zero - the electric field is zero inside! Even if it
werent, there would be zero net flux through the Amperean surface, since the electric field is radially
outwards (+
r) from the cylinder, and in the same plane as the circle. And even if that were not the case,
the flux should be constant, so its derivative would be zero. In short: the magnetic field inside due to the
shell must be zero, so we can ignore it for the calculations where r < R2 .
Lets start looking at the questions.
What is the direction and magnitude of the magnetic field for 0 < r < R1 ? Express your answer in terms
of I0 , R1 , R2 , R3 , r and 0 .
Inside the conductor, in other words. Well, we use Amperes law again, as usual.
The electric flux will, again, be zero in our circle, so we can use the old, simplified version of the law, as
the second term is known to be zero either way. The left-hand side is just the integral around the circle,
as usual, so we start out with
2rB = 0 Ipen
What is Ipen , though? Since r < R1 , it will be less than the full current. Namely, it will be the ratio of
the areas times the full current:
2rB = 0
B=

r2
I0
R12

0 I0 r
2 R12

The direction is given by the right-hand rule, so it will be in the + direction.


What is the direction and magnitude of the magnetic field for R1 < r < R2 ?
We barely need to re-solve it; instead, consider that Ipen = I0 , and so we only need to solve the first
equation we had with that substitution:

179

2rB = 0 I0
B=

0 I0
2r

The direction is unchanged.


What is the direction and magnitude of the magnetic field for R2 < r < R3 ?
Ah, now the outer shell begins to matter. The magnetic field due to the inner wire will be exactly the
same as the previous answer, however!
Once again, the second term in the law will be zero - there will be no magnetic flux penetrating the surface
we choose, so the change in magnetic flux will certainly also be zero. We set up the simple one instead:
2rBshell = 0 Ipen
Ah, that Ipen again... Well, this time, it will be the ratio of the area of the part of the shell inside r, to
the full shells area. Also, its negative, since its in the opposite direction as before. That is
Ipen = I0

r2 R22
R32 R22

For the top term, we have the area of the circle with radius r, minus the area of R2 which is the void
inside. Same for the outer part. Simplified, and substituted into Amperes law, we have
2rBshell = 0

r2 R22
I0
R22 R32

... keeping the minus sign in mind. So, the magnetic field due to the shell, inside the shell, is
Bshell = 0

r2 R22
I0
2r(R22 R32 )

The direction of this is then in the + direction as well, since we used the minus sign for the current.
The net magnetic field inside the shell is the above, plus the old one:
B = 0

r2 R22
0 I0
I +
(for R2 < r < R3 )
2
2 0
2r(R2 R3 )
2r

Simplified, we get
B=

I0 0 (r2 R32 )
(for R2 < r < R3 )
2r (R22 R32 )

And finally:
What is the direction and magnitude of the magnetic field for r > R3 ?
This one is the easiest, actually!
To see why, imagine an Amperean circle, with its usual simple flat surface attached to it. The circle is
larger than the cable (center + shell), and so Ipen = +I0 I0 = 0! The Ipen term is zero, and the electric
flux is zero (and thus the flux change is zero), for the same reasons weve seen before.
The net field outside is zero!

180

13.5.2

Problem 2: Intuition breaks down

(a) In the diagram above the small semi-circle in the wire above R1 indicates that it is not in contact
with the horizontal wire which it appears to intersect due to the 2-dimensional projection. Additionally,
the horizontal wire is continuous, not broken.
Calculate |V1 | in terms of the emf E, R1 , R2 .
Oh boy, non-conservative fields and weirdness. Lets see how this goes.
We can NOT use Kirchoffs rule here, which is the entire point. We can however find the current, and use
Ohms law to find voltage drops, so lets see... I think were to assume that I1 = 0 and I2 = 0 since there
is no mention about the volt meter resistance, etc. Lets see if that assumption gives us correct answers.
E
R1 + R2
The magnitude of the voltage on V1 would then be that current times R1 ... except that the voltmeter
is also wound around the changing flux, so we get a +E, as there will be an induced EMF inside the
voltmeter as well, separately from the current loop I.
I=

E
R1 + E
R1 + R2
They then ask for |V2 |; that is trivial: it is the current times the resistance, nothing more, nothing less.
Why? Because the voltmeter isnt part of a loop where there is changing magnetic flux, so we get nothing
but basic circuit analysis rules.
|V1 | =

E
R2
R1 + R2
Lastly, they want the ratio of the two, so we divide them and get
|V2 | =

|V1 |
2R1
=1+
|V2 |
R2
Now wrap that same wire not once around the circuit but 100 times before you connect it again to the
D-side. Without realizing it, in doing so you have been building some kind of a transformer (we will
discuss transformers later in the course).
(b) Calculate |V1 | [and |V2 |, and their ratio] in terms of the 1 loop emf E ,R1 , R2 .
Cool. Its immediately obvious that |V2 | doesnt change, as that is still connected directly, and there will
be no extra EMF there. There is also no extra EMF that increases the current through R1 , so only the
voltmeters extra EMF will change, from E to 100E. That indeed gives the correct answers, yay! The
three answers are then
|V1 | =

E
R1 + 100E
R1 + R2
181

E
R2
R1 + R2

|V2 | =

|V1 |
101R1
= 100 +
|V2 |
R2

13.5.3

Problem 3: Helmholtz coils

A current I = 2 Amperes flows in a circular single loop coil of radius R = 10 cm.


(a) Evaluate the magnitude of the magnetic field (in Tesla) at a point P on the axis of the coil, and a
distance z = 5 cm away from the plane of the coil itself.
Magnetic field due to a circular current loop
Superposition is valid for magnetic fields, so we can start out by making the calculation for a single ring.
Unfortunately, that is not easy by itself - and the rest problem gets a lot worse from there. I suggest using
some form of computer math software for the rest of the problem... I use Mathematica, but there are
plenty of other alternatives, many (cost-)free and/or free+open source.
This part isnt too hairy mathematically, though.
For a better explanation than mine below, watch this video: http://www.youtube.com/watch?v=lN296gUXkl4
Since the solution (both the process and the final equation) are in the book, watching that is hardly any
more cheating than reading the textbook is. In fact, I could simply use the books equation, but that
wouldnt make me learn, so I derive it myself as far as possible.
Now, then. Well have to use Biot-Savart, to begin with.
~ ~r)
~ = 0 I (d`
dB
4 r3
This would all be easy for z = 0, which we have done for a previous homework. Since we cant make that
simplification, r is no longer just the radius of the circle, but the distance from a point on the circle to
P , which is at height z above the center of the loop. This probably needs to be drawn on paper to make
complete sense.
What else? Well, the direction of ~r also changes; its no longer simply the vector pointing inwards towards
the middle of the circle. However, as long as we stay on the z-axis, via symmetry arguments, this should
all cancel out.
That is, the fields direction, if we stay along the z-axis, should be simply +
z.
Drawing this out and using the Pythagorean theorem, we find that the distance r to the point is given by
r=

R2 + z 2

This makes an angle with the xy-plane.


One more thing. If we look at the symmetry of this problem, its clear that only the z component isnt
cancelled out (by the symmetry above). Therefore, are only interested in the z component of the field
We then have
dBz =

0
I
(d`|~r|) cos
2
4 (R + z 2 )3/2

where the cosine comes from vector decomposition, to get the z-component.
Via geometry, we can see that
cos =

R
R
=
r
R2 + z 2
182

Therefore we have
0
I
R

d`|~
r
|
4 (R2 + z 2 )3/2
R2 + z 2

I
R
0
R2 + z 2
dBz =
d`
2
2
3/2
2
4 (R + z )
R + z2
0
IR
dBz =
d`
2
4 (R + z 2 )3/2
dBz =

We can finally integrate that:


Z
Bz =

0
IR
IR
0
d` =
2
2
3/2
2
4 (R + z )
4 (R + z 2 )3/2
Bz =

Z
d`

IR2
0
2 (R2 + z 2 )3/2

Thus, we can answer part (a) by plugging in the values given.

Two circular loops


Now consider two N -turn circular coils of radius R, each perpendicular to the axis of symmetry, with
`
their centers located at z = . There is a steady current I flowing in the same direction around each
2
coil, as shown in the figure below.
The image shows a similar situation to what we had, then, but with two coils, above and below z = 0,
where the old one was.
(b) Assuming the N = 100, I = 2 Amperes, R = 10 cm, ` = 2 cm, calculate the magnitude of the
magnetic field (in Tesla) at a distance z = 0.5 cm from the midpoint between the centers of the two coils.
The field due to N loops is simply N times stronger, as each loop simply adds to the rest. (I suppose we
can think of it as integrating over the circle N times, so we integrate from 0 to N 2, which then just
spits out a factor N .)
With that in mind, and superposition, this part should be easy enough. When using the equation we just
derived, the z value is really the distance from the center of the ring, which isnt at z = 0 in the coordinate
system any longer. We need to adjust it slighly; same goes for the circle below. Ill call the top one 1 and
the bottom one 2:
Bz1 =
Bz2 =

N IR2
0
2 (R2 + ( 2` z)2 )3/2

0
N IR2
2 (R2 + ( 2` z)2 )3/2

The sum is then


0 N IR2
Bz =
2

1
1
+ 2
`
2
2
3/2
(R + ( 2 z) )
(R + ( 2` z)2 )3/2

If we just plug in the numbers given now, and sum the two answers, we get the answer they want.
(c) Evaluate the first derivative with respect to z of the magnetic field calculated in part (b) using the
same values for the parameters. Express your answer in Tesla per meter.

183

Goodness, this is why I recommend using math software. Im not differentiating that - the z is hidden
inside two exponents, in the denominator of a fraction - and they want the second derivative later on, too!
The first derivative is given by
dBz
= 40 N IR2
dz

6(l 2z)
((l 2z)2 + 4R2 )5/2

6(l + 2z)

((l + 2z)2 + 4R2 )5/2

The second derivative is given by


d2 Bz
= 40 N IR2
dz 2

60(l + 2z)2
((l + 2z)2 + 4R2 )7/2

12
((l + 2z)2 + 4R2 )5/2

12
((l 2z)2 + 4R2 )5/2

60(l 2z)2
((l 2z)2 + 4R2 )7/2

What a friggin nightmare! My heart goes out to the students that solved this on their own. (I just noticed
that these equations are in the book. Im sure some worked it out manually, though.)
Once again, though, plugging in the values gives the correct answer.
The rest of the questions are for the same problem, with R = l = 10 cm and z = 0, so we just plug in the
numbers to solve those, the point being that the first and second derivatives are both exactly zero in that
case.

13.5.4

Problem 4: Spinning loop in a magnetic field

A square loop (side L) spins with angular frequency in field of strength B. It is hooked to a load R.
(a) Write an expression for current I(t) in terms of B, L, R, and t.
Well, we can find the EMF using Faradays law. Its the negative of the change in magnetic flux through
the loop. So lets first find the flux through the loop; we have a planar surface, and a uniform, constant
magnetic field of strength B (since there is no other information). Magnetic flux is B-field strength times
area, so
B = L2 B cos
... where is the angle between B and the normal to the surface.
Since we are rotating the loop, changes with time according to the angular frequency :
= 0 + t
We can choose 0 = 0 to simplify, so we do that, and then differentiate and negate it to find the EMF:
B = L2 B cos t
dB
= L2 B sin t
dt
dB
E(t) =
= L2 B sin t
dt
The current is then this, divided by the resistance R:
L2 B sin t
R
(b) How much work is done by the generator per revolution? Express your answer in terms of B,L,R and
(enter omega for ).
I(t) =

184

The work done will be equal to the heat dissipated in the resistor, so we could integrate that power over
the time of one rotation.
=

2
2
, so T =
T

The power is given by I 2 R:



P (t) =

L2 B sin t
R

2
R=

L4 B 2 2 sin2 t
R

The work is the integral, then, from t = 0 to t = T :


Z T 4 2 2 2
Z
L B sin t
L4 B 2 2 T
W =
=
sin2 t
R
R
0
0
2

When we integrate the sine, and substitute T =


, the result of the integral is simply , so we get:

4 2
L B
W =
R
(c) To make it twice as hard to turn (twice as much work), what factor would you have to multiply the
resistance R?
Its clear from the equation just above that, since no other variable is dependent or R, we simply need to
1
halve R, so the answer is .
2

13.5.5

Problem 5: Loop in a magnetic field

A rectangular wire loop of dimensions a b, with a = 3.5 cm and b = 4.5 cm, is pulled parallel to side a
~ = 0.125 T, is uniform and
at a constant speed v = 4.3 m/sec into a region where the magnetic field, |B|
perpendicular to the loop. The loop enters the magnetic field at time t = 0.
(a) Calculate the rate at which the magnetic flux through the loop is changing. Express your answer in
m2
.
Tesla
sec
I wouldve preferred an illustration, but all right. I assume that pulled parallel to side a means there is
some external force in the same direction as the line a is in. That makes the area of the loop exposed to
the magnetic field ba(t). The flux is
B = b a(t)B
The rate of change is then
dB
da
= b B = bvB
dt
dt
(b) What is the magnitude of the induced EMF at time t?
This confuses me, since they are expecting a numerical answer... The magnitude of the EMF is the same as
the first answer, though, which is accepted. I suppose the point they want to make is that it is independent
upon t? However, as soon as the loop has entered the magnetic field fully, surely the flux would become
constant.

185

13.5.6

Problem 6: Electrodynamic tether

A L = 3 mile long conducting cable tethers a satellite to the International Space Station. The Cable
moves through the Earth magnetic field of about B = 0.02 Gauss at a speed of v = 8 km/sec. The cable
points radially from the Space Station to the satellite. The ISS moves Eastwards and the magnetic field
is Northward.
What is the potential difference between the two ends of the tether?
This is really nothing more than a simple motional EMF problem. Unless Im missing something (despite
having the answer right), most of the work is in the unit conversion, really. Most of this solution is restating facts about motional EMF.
Say we have a conductive bar moving rightwards, in a magnetic field going into the page. The free electrons
in the bar experience a magnetic force upwards:
~
F~B = q(~v B)
Positive charge gathers closer to the top, and negative charge towards the bottom.
This produces an electric field inside the bar, from positive to negative as usual, which forces positive
charges down, and negative charges up, with force
~
F~E = q E
In an static situation of constant magnetic field and velocity, well find an equilibrium where the forces
are equal:
~ = qE
~
q(~v B)
~ =E
~
~v B
We evaluate the cross product, which is simply vB, since the angle between them is 90 degrees, so the
sin term in the cross prodcut is one.

E = vB
Given that the electric field should be constant (since v and B are constant), the potential difference is
then given by V = Ed, where d is the distance, here `
V = vB`
Going back to our problem, we can now solve for the potential difference:
1609 m
m
)(0.02 104 tesla)(3 miles
) = 77.232 V
s
1 mile
Which end has the higher potential? If the velocity is eastwards, and the B-field northward, then the
magnetic force is upwards (outwards from the surface), so the positive charge gathers at the far end, so
the potential is highest at the end the farthest away from the Earth.
V = vB` = (8000

13.5.7

Problem 7: Motor

A simple motor has N = 250 turns of wire on a coil measuring 12 cm 4 cm. Its resistance is R = 10
ohms. The magnetic field is B = 1 tesla.
(a) How much current (in Ampere) in the coil is needed to produce a maximum torque of 40 N m?

186

Motors arent my strong side - they havent been talked about a whole lot in the course, and I didnt take
8.01 prior to this course, so Im not super-familiar with torque... so I had a look in the suggested reading.
The torque on the loop is given by
| | = IAB
... where A is the area of the loop.
In this case, the current will be given by N I, as the multiple windings simply multiply the torque. So, we
have
| | = N IAB
| |
N AB
100
= 33.3333 amperes.
Now we just need to stick the numbers in, and find I =
3
I=

(b) What maximum EMF (in Volts) is needed to drive the motor at 50 Hz if the current is constant?
Lets see. There will be an induced EMF due to the changing magnetic flux through the loop, as it rotates.
The current through the loop (due to the external source) in the books drawing is counterclockwise, so
lets assume that it will be at one instant, just for visualization.
What about the induced EMF, in which direction will that one be? In the one that opposes the change
in the external flux, which turns out to be in the opposite direction, so the induced EMF will counteract
the external one the power supply provides.
What will the induced EMF be?
The flux through the surface is
B = N AB cos
Since it rotates, as weve done before, we set = t, and then differentiate the flux, and negate it, to find
the EMF:
dB
= N AB sin t
dt
dB
E=
= N AB sin t
dt
The EMF maximum will then occur when sin t = 1, so the maximum induced EMF will be N AB.
= 2f = 2 (50 Hz) = 100, so the maximum induced EMF is
E = (250)(4 cm)(12 cm)(1 T)(100 rad/s) 377 V
Heres the interesting point. The above is not only the maximum induced EMF, but actually also the
answer to the question.
Motors and generators are strongly related. If we think of this motor as a generator, the maximum EMF
generated at angular frequency is the same as the maximum EMF required to turn the motor at the
same angular frequency .
Im still unsure of exactly why this is the case, though. Im hoping next weeks talk about inductance and
phase lag help a bit.

187

13.5.8

Problem 8: Auroral zone

This problem has a very long introduction, with a derivation of the equation used, etc, and several figures.
I suggest reading that first!
... The solar wind ram pressure causes the magnetic field of the earth to terminate at about 10RE on the
sunward side of the earth (see figure).
The auroral zone is defined by the last field line from the Earth that returns to the Earth. If the field
of the earth extends no further than 10RE in the sunward direction, at what angle (in degrees) in the
picture above does the last field line that returns to the earth on the sunward side leave the polar regions
of the earth at 1RE , assuming that the fields are always described by r = R0 sin2 ? In the drawing, this
is equivalent to asking what is the angle with respect to the vertical of a line from the center of the earth
to the point that the red curve intersects the light blue circle.
Admittedly, I first solved this one by graphing it, and will solve it more mathematically below.
In polar coordinates, r = 1 gives a circle; if we choose our coordinate system such that RE = 1, this then
represents the Earth.
The field lines are then plotted by the equation given, r = R0 sin2 , with various values for R0 . When
R0 = 10, we get that last field line the questions asks about.
When weve graphed both on the same plot, we can simply zoom in and read off the angle. Well, almost polar coordinates define angles as 0 on the +x axis, and so to find the answer - the angle from the vertical
axis, we need to shift it. Additionally, this angle will be in radians, while they requested degrees.

conversion, and then


Anyhow, I read it off as 0.3975 from the horizontal axis; with the 90 degree =
2
from radians to degrees, the answer is approximately
 180
0.3975
18.45
2
rad
It turns out the mathematical solution isnt very much harder. We just set up


RE = 10RE sin2
... since those are the r coordinates where the lines meet, and solve the equation. The solution pops out:
10 sin2 = 1
1
sin2 =
10
1
sin =
10
1
= arcsin 18.43495
10
Keep in mind that
sin2 = (sin )(sin )
if that part was unclear.

13.6
13.6.1

Week 7
Problem 1: Magnetic energy of a solenoid

Compare the total magnetic energy in two solenoids, each with N turns, area A, and current I, but length
x and 2x. Denote Ux the total magnetic energy in the solenoid with length x, and U2x the one in the
188

solenoid with length 2x. What is U2x /Ux ?


We calculated the magnetic field energy density back in this weeks lecture notes, and since it featured a
fairly ugly integral, a circuit to solve etc., I wont redo it all here, as I would if it were simpler.
Instead, we found the inductance L of a solenoid and the field energy density U to be
N2
0
`
B2
U=
20

L = r2

The B-field inside the solenoid can be derived relatively easily using Amperes law, which we have done
previously (week 5), so I will do the quick-and-dirty version here. There is no displacement current through
the Amperean surfacae we choose, so we can use the simplified Amperes law:
I
~ = 0 Ipen
~ d`
B
N
`
N
B = 0 I
`
We then want to square that, and then divide by 20 , to find the energy density:

2
1
0 IN
0 I 2 N 2
U=
=
20
`
2`2
Ba = 0 Ia

The ` we have used is the length of the solenoid, which then is to be x or 2x.
Since the B-field is assumed to be constant inside a solenoid, and 0 outside, the total magnetic energy is
the above times the volume, A`, where we substitute in the value for `:
0 I 2 N 2
Ax
2x2
0 I 2 N 2
=
A2x
2(2x)2

Ux =
U2x

Finding the ratio U2x /Ux , we first get rid of all the terms that are the same in both expressions, and get
A2x . Ax
A2x x2
2Ax3
U2x
=
=

=
Ux
(2x)2 x2
4x2 Ax
4Ax3
With all that cancels, we find
U2x
1
=
Ux
2
1
Since the total energy is proportional to (the Ux = ... equation above), this does appear to make sense.
`
1
The energy density is proportional to 2 , but we then multiply that by a ` term to find the total energy.
`

189

13.6.2

Problem 2: Displacement current

Consider the process of charging a parallel plate capacitor with circular plates of radius R = 5 cm separated by a distance d = 0.2 cm. At some time t1 , the capacitor is being charged with a current I = 0.035 A.
Consider a point P on the plane which is equally distant from the two plates and is a distance r = 0.068
m away from the axis of the capacitor.
(a) Calculate the magnitude of the magnetic field (in Tesla) at a point P at time t1 during the charge of
the capacitor.
We will want to use the (almost) full version of Amperes law for this one. (Next week we will finally have
the fully complete version.)


I
d x ~ ~
~
~
E dA
B d` = 0 Ipen + 0
dt
We choose our Amperean loop as a circle with radius r, concentric with the capacitor plates, and centered
in the distance between the plates as mentioned. We must then attatched an open surface to our closed
loop (the circle), and we choose a flat surface in the plate of the circle (think of it as the circles area) for
simplicity.
The left-side integral becomes B2r; Ipen = 0 since the surface is in in mid-air and no real current can pass
through it. The rate of change of the electric flux through the surface will however be nonzero! Replacing
Q
~ with its magnitude
E
and the area the flux goes through by R2 (capital R! r > R, so the flux
2
0 R
only goes through part of r2 !) we get, with all our changes:


d
Q
2
B2r = 0 0
R
dt 0 R2
The dot product inside the integral turns into a regular multiplication, and the integral is trivial since the
~ are the same.
~ dA
E-field is uniform, so all infinitesimal products E
In taking the time derivative, we find that the only non-constant is Q, which will change over time, so


1 dQ
I
Q
R2 d
d
2
R
(Q) =
=
=
2
2
dt 0 R
0 R dt
0 dt
0
where I is the current in the wire to (or from) the capacitor, not Ipen which we established was zero.
Putting it all together and solving for B:
B2r = 0 0
B=

I
0

0 I
2r

For the values given,


B=

13.6.3

(4)(107 )(0.035)
2(107 )(0.035)
=
1.0294 107 T
20.068
0.068

Problem 3: RL circuit

Consider a circuit with an RL series with L = 0.09 H and R = 0.05 Ohm. At t = 0 the circuit is connected
to a battery which provides V0 = 12 V.
(a) How long does it take to the current to equal a fraction 0.95 of the steady state current?
We could set up and solve a differential equation, but theres little in point in doing that, since we found
the solution already in this weeks lectures. The current is governed by the equation
190

V
I(t) =
R




Rt
1 exp
L

The steady state current is then just VR , as the inductance will have no effect for the steady state.
Thus, we want to find when the first equation equals 0.95 times the second:



Rt
V
V
1 exp
= 0.95
R
L
R


Rt
1 exp
= 0.95
L


Rt
= 0.05
exp
L


Rt
exp
= 0.05
L
Rt
= ln 0.05
L
L
t = ln 0.05
R
For the values given, t 5.392 seconds.
(b) What is the energy stored (in Joules) in the magnetic field when the current equals a fraction 0.95 of
the steady state current?
The energy stored is given by
1
U = LI 2
2
Since I at that time equals 0.95 VR ,

2
V
1
2339.28 J
U = L 0.95
2
R
(c) What is the total energy delivered (in Joules) by the battery up to the time t found in part (a)?
How much energy (in Joules) has been dissipated in the resistor?
The total energy delivered is the energy stored (in the ideal inductor) plus the energy burned in the resistor
up to that time, the integral of I 2 R from t = 0 to t = (answer a).
Z

5.392318
2

I R dt =
0

5.392318

V2
R2



2


2
Z
Rt
V 2 5.392318
Rt
1 exp
R dt =
1 exp
dt
L
R 0
L

I didnt see an easy way to get that integral down to something simple, so I solved it with Mathematica.
2
Simplified, and keeping the VR in front in mind, I found
  2RT


RT
V 2 L e L 4e L + 3 2RT

2R2
with T being the upper integration limit; T = 5.392318 for this problem. Putting the values in there, we
find that the energy burned is 8265.796 joules.
That is then the answer for question (c2), while the answer to (c1) is (b) plus (c2).
(The total energy delivered is the energy stored plus the energy burned in the resistor.)
191

13.6.4

Problem 4: RL circuit

(Yep, the problem name is the same as the previous one!)


Consider the following circuit. We have R1 = 11 Ohms, R2 = 15 Ohms, R3 = 13 Ohms, V = 7 Volts,
L = 0.09 H.

At t = 0 the switch is closed.


(a) Immediately after the switch is closed, what are currents I1 , I2 , I3 ?
Well, we dont need differential equations nor regular equations for this one. I1 = I3 =
because of the inductor.

V
.
R1 +R3

I2 = 0

(b) What are the three currents after the switch has been closed for a long time?
Again simple: we now treat the inductor as a short-circuit with R = 0, and again we just have Ohms law
with a few parallel resistors.

I1 = I2 + I3
is clear, since the current through I2 and I3 must go through R1 first.
We can set up two loop equations for the outer and lower loops, keeping in mind that we need Faradays
law for the lower one. For the outer loop, we start in the bottom-right corner and go clockwise, simply
subtracting drops in potential, and adding climbs (the battery); the sum is then 0, Kirchhoffs rule applies.
I1 R1 I3 R3 + V = 0
~ potential
~ d`
For the lower loop, we use Faradays law! The left-hand side will be the sum of the E
differences, while the right-hand side will be the negative of the time derivative of the flux. We start in
the same corner. In this method, we instead think about the E-fields direction and currents direction
when deciding the same. For the same direction we add, opposite directions we subtract. We find:
dB
dt
Remember that E = 0 inside the inductor, so it doesnt contribute to the left side! Instead, we use the
definition of inductance LI = B to get
I1 R1 + I2 R2 + 0 V =

dI
dt
We then have three equations and three unknowns. Because the current has stabilized,
term disappears, and our differential equation simplifies:
I1 R1 + I2 R2 V = L

192

dI
dt

= 0, so that

I1 = I2 + I3
I1 R1 I3 R3 + V = 0
I1 R1 + I2 R2 V = 0
Solving the system, we get (I used Mathematica):
I1 = I2 + I3
R3 V
I2 =
R1 R2 + R1 R3 + R2 R3
R2 V
I3 =
R1 R2 + R1 R3 + R2 R3
The switch is now opened again.
(c) What are the three currents after the switch is reopened?
The question is slightly unclear, but the answers work if we calculate the answer for t = 0+ where it was
opened at t = 0. That is, an infinitesimal amount of time later, so that the inductor current is unchanged.
Note that the circuit changes. S, V and R1 are now disconnected/left open, so the circuit is now L, R2
and R3 in series. Thus I3 = I2 , with I2 unchanged from our answer above.
I1 = 0, since it is disconnected.

13.6.5

Problem 5: Opening a switch on an RL circuit

Even more RL circuits!


The LR circuit shown in the figure contains a resistor R1 and an inductance L in series with a battery of
emf E0 = V0 . The switch S is initially closed. At t = 0, the switch S is opened, so that an additional very
large resistance R2 (with R2  R1 ) is now in series with the other elements.
(a) If the switch has been closed for a long time before t = 0, what is the steady current I0 in the circuit?
Express your answer in terms of, if appropriate, V0 , R1 , R2 and L.
Well, that parts crazy easy. I =

V0
, since everything else is a short circuit.
R1

(b) While this current I0 is flowing, at time t = 0, the switch S is opened. Write the differential equation
for I(t) that describes the behavior of the circuit at times t > 0. Solve this equation (by integration) for
I(t) under the approximation that V0 = 0. (Assume that the battery emf is negligible compared to the
total emf around the circuit for times just after the switch is opened.)
Express your answer in terms of the initial current I0 , R1 , R2 , t and L.
Well, thats clearly a lot harder! What the problem means is then that V0 is replaced by a short circuit,
while the switch of R = 0 is replaced by the resistance R2 . So the circuit is R1 , R2 and L all in series,
with nothing else to worry about.
I would normally just apply the solution to the differential equation directly, but since they insist...
Starting at the bottom-right again (why not?), applying Faradays law; leftside closed line integral, right
side change in flux:
dI
dt
Not having taken a differential equations class, hmm... Solve by integration, they say. Im not sure how
to do that, and itd feel a bit like cheating to just skip to the known solution, so Ill attempt the method
IR2 + IR1 = L

193

taught in 6.002x (circuits and electronics). First, we try write it in a different form. We set R = R1 + R2 ,
to simplify things.
L dI
I =0
R dt
First, we find the particular solution, which is any solution that satisfies the equation. If we choose I = 0,
that works, so Ip = 0.

L
times
For the homogeneous solution, we need a function such that the derivative of Ih is a constant R
itself. The exponential function has this property, so we guess (yes, really) that might be a solution, only
that we have two unknown coefficients. We choose Ih = Aebt :

L dIh
+ Ih = 0
R dt

L d
Aebt + Aebt = 0
R dt
L
Abebt + Aebt = 0
R
We can divide out Aebt at this point:
L
b+1=0
R
We can now find b:
L
b = 1
R
R
b=
L
Well, that looks familiar.
The answer to the differential equation is the sum of the particular and homogeneous solutions, we we add
the two. However, our particular solution was 0, so the answer is just the homogeneous solution:


R
I(t) = A exp t
L
(using exp(x) = ex notation, to make it readable)
We can find the value of A by substituting an initial condition, I(0) =
we substitute in that value for I and t = 0:

V0
R

(by analysis of the circuit), so

V0
R
The exponential turns into 1, so we find the answer at once. The full solution, with A and b known, is
then


V0
R
I(t) =
exp t
R
L
A=

... which we really knew already, having seen the answer in lecture. At least now we have derived it
ourselves, too.
The task was to express this answer, using I0 for the initial current, I0 =
set R = R1 + R2 earlier):
 
 
R1 + R2
I0 exp
t
L
194

V0
R

V0
R1 +R2

(remember that we

(c) Using your results from part b), find the value of the total emf around the circuit (which from Faradays law is LdI/dt) just after the switch is opened. Express your answer in terms of, if required, V0 ,
R1 , R2 and L.
Well, I suppose we should differentiate the above (since we want to know dI
and we know I), and multiply
dt
it by L, then.
 
 
 
 
d
R1 + R2
I0 (R1 + R2 )
R1 + R2
I0 exp
t =
exp
t
dt
L
L
L
The EMF is then the above times L, so
X

 
 
R1 + R2
E = I0 (R1 + R2 ) exp
t
L

The answer does make intuitive sense - the current running times the total resistance ought to equal the
total EMF!
At t = 0, the above simplifies down to
X
E = I0 (R1 + R2 )
P
0
Time to be careful! I originally substituted in I0 = R1V+R
and got
E = V0 , several times over. However,
2
R2 was just added - the circuit hasnt had time to react to that, yet. So instead,
I0 =
X

V0
R1



V0
V0 R2
R2
E=
(R1 + R2 ) = V0 +
= V0 1 +
R1
R1
R1

How reasonable is your assumption in part b) that V0 could be ignored for times just after the switch is
opened?
Well, considering that we got the answer while assuming that, it appears it was a fairly great assumption.
d) What is the magnitude of the potential drop across the resistor R2 at times t > 0, just after the switch
is opened? Express your answers in units of V0 assuming R2 = 100R1 .
It would have to be the current, answer (b), times R2 :
 
 
 
 
R1 + R2
V0
R1 + R2
exp
VR2 = I0 exp
t =
t R2
L
R1
L
With R2 = 100R1 :
VR2

 
 
R1 + R2
= 100V0 exp
t
L

The times t > 0 part is a bit confusing; they are really asking for the value JUST after the switch is
opened, i.e. at t = 0+ . Thus the exponential term disappears (with t = 0, the whole term becomes 1),
and VR2 = 100V0 , or, in units of V0 as they ask for, just 100.

195

13.6.6

Problem 6: Self-inductance of a toroid

A coil consists of N = 105 turns of wire wrapped uniformly around a plastic torus. The inside radius of
the torus is r0 = 0.21 m and the outer radius is r1 = 0.24 m. Thus, each winding has a diameter d = 0.03
m. What is the self-inductance (in H) of this coil? Work in the approximation d  r0 .
To find the self-inductance, we first apply Amperes law to find the B-field inside. Once we have that,
LI = B
can be used to calculate the inductance.
Lets see. We begin by choosing the Amperean circle they have drawn for us (the dashed line), with the
radius r, to which we attach the flat open surface that it bounds, r2 . The B-field inside is approximately
constant at all points along the line, so we start out with
!
d x ~ ~
E dA
B2r = 0 Ipen + 0
dt
S

If I is constant, I see no reason for there to be any change in electric flux. (If I varies, Im not sure.)
We get rid of the displacement current term to get
B2r = 0 Ipen
0
Ipen
B=
2r
What is Ipen , however? Thats a bit trickier than with the straight solenoid... Or is it? The length of the
toroid must be the circumference of our Amperean circle, centered in the toroid, so ` = 2r. Other than
that, the number of times it penetrates should be the same - the number of loops per unit length, times
the length, so
Ipen = I

N
` = NI
`

In that case,
196

N I0
2r
This turns out to be the correct B-field for a toroid! All right, lets try to find the flux, so we can find L.
B=

As can be seen above, the B-field is not constant inside, so we really need to integrate this over the area,
to find the correct flux.
However, we are told to [w]ork in the approximation d  r0 .
We can write the B-field as
N I0
2(r0 + )
Factoring out r0 from the sub-expression in the denominator gives us
B=

B=

N I0
2r0 (1 + /r0 )

Now, since  r0 , the /r0 term becomes far less than the 1, so we neglect it, and find
N I0
2r0
If we use that B-field approximation, we see that in the approximation, it is now constant over the crosssectional area. We can now find the flux using B = BA, with A being the cross-sectional area, and then
multiply by the number of windings to find the total flux:
 2
d
N I0

B
2r0
2
B

N Id2 0
8r0

The inductance is then given by


N B
N 2 d2 0

6.73 H
I
8r0
This result is consistent with that of a a solenoid, which shouldnt come as a huge shock, seeing that a
toroid is essentially a wrapped-up solenoid.
We also see that the toroid has a fairly huge inductance, which also shouldnt be entirely surprising given
its size and 105 windings!
L=

13.6.7

Problem 7: RL circuit

(Yes, really, RL circuit!)


A circuit consists of a self inductor of L = 0.003 H in series with a resistor R1 = 5 Ohm. Parallel to these
is a resistor R2 = 10 Ohm. A battery of V0 = 9 volt is driving the circuit.
Current has been running for 10 minutes.
(a) How much energy (in Joules) is now stored in the self-inductor?
The time constant is less than a millisecond, so the 10 minutes means steady-state has been reached long
ago.
V0
. Power stored is
We treat the inductor as having zero resistance, so the current through it is I1 =
R1
1
given by U = LI 2 , so we can easily find the answer using that combination.
2
197

1
U= L
2

V0
R1

2
= 0.00486 J

(b) How much power (in Watt) is then generated by the battery into the circuit? (ignore internal resistance
of the battery).
The inductor no longer draws any power, so the answer is
V2 V2
+
24.3 W
R1 R2
The connection to the battery is now broken (so that the battery is not connected to the circuit anymore).
P =

(c) How long will it take (in seconds) for the current through R1 to be reduced by 50%?
We derived the equation for this earlier on in this weeks homework.
First, the circuit will change to be R1 , R2 and L in series.
Second, the current will die out following an exponential decay, given by


R
I(t) = I0 exp t
L
If we set that equal to 0.5I0 , and solve:
R
t = ln 0.5
L
L
ln 0.5 1.386 104 s
R
keeping in mind that R = R1 + R2 . This also answers (e), since the circuit is now a series circuit.
t=

(d) How long does it take (in seconds) till the energy stored in the self-inductor has been reduced by
50%?
Energy stored is


2
1 2 1
R
U = LI = L I0 exp t
2
2
L


1
R
= LI02 exp 2 t
2
L
1
We set that equal to 0.5 times the current stored energy, LI02 :
2


1 2
R
1
LI0 exp 2 t = LI02
2
L
4


R
1
exp 2 t =
L
2
R
2 t = ln(2)
L
L ln(2)
t=
6.93 105 s
R 2
Since we solved part (e) above (same answer as for part c), so we are now done!

13.7

Week 8

Theres no homework this week, as the second midterm is next weekend.


198

13.8
13.8.1

Week 9
Problem 1: RC circuit

In the following circuit V = 9 V, R = 25 Ohm, C = 0.0025 Farad.


At t=0 we start charging the capacitor with no charge initially on the capacitor.
(a) Calculate the time t (in seconds) when the potential across the capacitor is V/2.
The circuit is a simple series RC circuit, so I hardly see a need to draw it. The capacitor voltage and
current (the latter is the same for the entire circuit, of course) is given by



t
VC (t) = V 1 exp
RC


t
V
I(t) = exp
R
RC
To find the answer, we set the potential VC equal to V /2, and solve the exponential for t:



t
V
V 1 exp
=
RC
2


t
1
1 exp
=
RC
2


t
1
exp
=
RC
2
t
1
= ln
RC
2
t
= ln 2
RC
t
= ln 2
RC
t = RC ln 2

We plug our values in, and get t 0.04332 seconds.


(b) How much energy (in Joules) has the power supply generated between t = 0 and the time when the
potential across the capacitor reaches V/2?
1
There are several ways to solve this. We could find the energy stored in the capacitor ( CVC 2 ) and the
2
Z t

energy dissipated in the resistor ( I 2 R dt), or we could integrate the power sources power over time,
0
Z t
V I dt. Ill choose the later approach, since it should be slightly easier, and it is also the approach that
0

immediately answers the question, without invoking conservation of energy etc.


We have the integral
Z
E=
0





Z
t
V
t
V2 t
exp
V I(t)dt = V
exp
dt =
dt
RC
R 0
RC
0 R



t
2
E = V C 1 exp
RC
Z

E 0.101247 J
199

13.8.2

Problem 2: RC circuit

In the following circuit R1 = 3 Ohm, R2 = 10 Ohm, C = 0.003 Farad, V = 5 Volts.


At time t = 0 we connect the power supply to the circuit. At t = 0 the capacitor is uncharged.
(a) What is the time constant (in seconds) to charge up the capacitor?
The time constant is given by = RC, where R is the equivalent resistance of the circuit, as seen by
the capacitor. We can find that by shorting out the voltage source (replacing it with ideal wire), and
calculating the resistance by doing series/parallel analysis.
Doing so, we find that all current to the capacitor must go through R1 . It can go then either go through
R2 or R1 at the bottom; they are in parallel. So we have
Req = R1 + (R2 ||R1 ) = R1 +

R2 R1
5.308
R2 + R1

therefore
= Req C 5.308 0.003 0.015924 s
(b) What is the electric potential over the capacitor when it is fully charged?
When it is fully charged, the current through the top resistor is zero, and the voltage drop there is zero.
Thus the capacitors voltage is in parallel with V + VR1 , and the two must be equal.
The current through the lower loop is I =
volts. Thus

5
15
V
=
A, and the voltage drop across R1 therefore
R1 + R2
13
13

VC = V VR1 3.846 V
(c) Calculate the current delivered by the power supply as a function of time and evaluate it (in Ampere)
for t = 0.6 , where is the value obtained in part (a).
The function needs to have two parts: one that decays with time, and one that doesnt.
5
The latter must either be a constant
, the current when the capacitor is fully charged (see (d) below),
13
or move up to that value as time passes.
5
and the capacitor current (the difference between the current
13
when its completely uncharged and acts like a short, and when its charged and acts like an open) times
the exponential, that is


t
I(t) = I() + (I(0) I()) exp

It turns out that the two parts are the

Therefore


5
V
5
t
I(t) =
+(
) exp
13
Req 13

Using the questions values for all variables and constants:


I(t) =

5
5
5
+(
) exp (0.6) 0.6905 A
13
5.308 13

(d) What is the current delivered by the power supply when the capacitor is fully charged?
200

Well, the capacitor is an open circuit at that point, so its simply the current in the lower loop,
I=

V
5
=
A
R1 + R2
13

(e) How much energy (in Joules) is in the capacitor when it is fully charged?
1
U = CVC 2 0.0222 J
2
... using the result for its voltage from (b).

13.8.3

Problem 3: RLC circuit

In the following circuit R = 5 ohm, L = 0.02 H, C = 0.0065 Farad and V = 13 Volts. The capacitor is
initially uncharged.
(a) What is the current delivered by the battery immediately after the switch is closed?
The inductor will act like an open, while the capacitor will act as a short circuit. The current will be at a
maximum through the capacitor, so I = V /R = 13/5 A.
(b) What is the the current delivered by the battery a long time after the switch is closed?
Now the inductor is a short, so the same answer applies.
What is the potential difference across the capacitor a long time after the switch is closed?
0, because the inductor is a short circuit in parallel with it.

13.8.4

Problem 4: An LRC circuit

A circuit contains a self-inductance L in series with a capacitor C and a resistor R. This circuit is driven
by an alternating voltage V = V0 sin t. We have L = 0.015 H, R = 80 , C = 5106 F, and V0 = 40 volts.
(a) What is the value (in radians/seconds) of the resonance frequency, 0 ?
The resonance frequency for a series RLC circuit is given by
0 =

1
LC

so 0 3651.48 rad/sec.
(b) Consider three separate cases for which = 0.250 , = 0 , and = 40 respectively. For each case
calculate the the peak current I0 in Amperes.
To do this, we need to find the equation that governs the current in the circuit. Finding that is not a
trivial task, so I will refer to the known result from lecture/the book instead:
V0
I(t) = q
R2 + (L
where
L
tan =
R

1
C

201

sin(t )
1 2
)
C

Pay attention to that we use the sine function, rather than the usual cosine, as the problem description
states the driving voltage is a sine. The only difference is in the phase, so the old result should certainly
still be valid, however.
I didnt realize this until I was essentially 100% sure I calculated the energy stored in the inductor correctly.
Turns out I did, if the driving voltage had been a cosine!
So, with that in mind, the peak current in the term that multiplies the sine, so we just need to stick our
values in there, and find
I0 0.181467 A (for = 0.250 )
I0 0.5 A (for = 0 )
I0 0.181467 A (for = 40 )
Note how at = 0 , the resonance frequency, the reactance is cancelled out and the peak current is

V0
.
R

(c) Find the energy UC(t) and the energy UL(t) stored in the capacitor and in the inductor, respectively,
at time t1 = 0.0003 seconds for = 0 . Express your answers in Joules.
OK. How should we approach this? We have an expression for the current in the circuit, and the current
must be the same through all elements at all times. Therefore, we can use the current expression and
1
U = LI 2 to answer the second question:
2
1
1
UL = LI 2 L0.4445662 0.0014823 J
2
2
We can find UC trivially by knowing a fact about RLC circuits: the energy stored in the capacitor plus
the energy stored in the inductor is always the maximum energy stored in either. So there is a time where
1
the total stored energy in the circuit is CVC2 , and at that time UL = 0. At another time, the inductors
2
stored energy is at its maximum, which is when the capacitors energy is 0.
Between the extremes, the sum of their stored energies is always the maximum stored energy, which we
1
find easily with LI0 2 (since, as stated, UC = 0 at that time). Therefore,
2
1
UC = LI0 2 UL = 0.001875 0.00148229 = 0.00039271 J
2
We can also find this by integrating the capacitors voltage:

Z t1
1
VC (t) =
I(t)dt + VC (0)
C
0
Z
Z t1
1 t1 V0 sin t
V0
VC (t) =
dt + VC (0) =
sin t dt + VC (0)
C 0
R
RC 0
since we are at resonance, and the inductive and capacitive reactances cancel, so that only the resistor
contributes to limiting the current, which is in phase with the driving voltage.
Solving it, we find
V0 1 cos(t1 )
+ VC (0) 14.8528 + VC (0)
RC

Well, what is VC (0)? Its not zero; keep in mind that the equations we are working with are only true
at steady state, so the capacitor doesnt necessarily have to be uncharged at t = 0; the circuit must have
been running prior to t = 0.
One way to find it is to use the knowledge that the current is 0 (because the driver is a sine, and it is in
VC (t) =

202

phase with the current), which means the capacitors voltage is at a maximum (or else the current would
not be zero). Therefore, the capacitors voltage is
1
0.5

27.386 V
C
3651.48 5 106
(via essentially Ohms law, using the impedence).
We can then apply the potential energy formula:
VC (0) = Imax

1
UC = (5 106 )(14.8528 27.386)2 0.0003927 J
2
... though this was more complex than the solution that uses the knowledge that U = UC + UL is constant.

13.8.5

Problem 5: Design a flute

A flute can be regarded as a tube open at both ends. It will emit a musical note if the flutist excites a
standing wave in the air column inside the tube.
The lowest musical note that can be played on a flute is C (261.7 Hz). What must be the length of the
tube? Assume that the air column is vibrating in its fundamental mode.
Interesting! But very simple. We use the equation from lecture,
v
2L
v 340 m/s, the speed of sound in room-temperature air. L is the length of the flute, which we want to
find. So we rearrange it, and get
f1 =

L=

v
2f1

With v = 340 m/s and f1 = 261.7 Hz, we find L = 0.65 meters.

13.8.6

Problem 6: Width of resonance peak

Consider the following RLC circuit: (standard series RLC circuit)


with V = 3 Volts, R = 25 Ohms, C = 0.006 Farad, L = 0.085 Henry. Define the frequencies as the
1
frequencies such that the absolute value of the current of the circuit I0() equals I0max .
2
What is the difference = + ? Express your answer in radians/sec.
Well, the problem doesnt actually specify the driving waveform, except that theres a sine wave on the
voltage source in the diagram, so Ill take it to be V cos t. The phase shouldnt matter for this question
anyhow.
Given that, we have this:
V0
I(t) = q
R2 + (L

cos(t )
1 2
)
C

where
tan =

L
R

1
C

If we set
X = L
203

1
C

Z=

R2 + X 2

to reduce the clutter, we have


V0
X
cos(t arctan )
Z
R
If Z = R, we have the maximum possible current (at resonance). Therefore we should have half the
maximum current at twice that impedance, Z = 2R (where half the impedance comes from the L and C).
I(t) =

1 2
) = 2R
C
The equation has four solutions, but two are negative (with the same absolute value as the other two), so
we find
R2 + (L

+ = 513.247 rad/s
= 3.82035 rad/s
= 509.4267 rad/s
(Im sure we could do this with less math, but
I used Mathematica for the equation solving here; I couldnt

find a really simple way to solve this; solving R2 + X 2 = 2R for X, and then the definition of X = 3R
which we find from the first equation gives us two frequencies, the absolute values of which are correct,
but one is negative.)
The resonance frequency is
1
44.281 rad/s
LC
We can find the resonance frequency as the geometric mean of the half-current frequencies:
0 =

0 =

13.8.7

+ 44.281 rad/s

Problem 7: Standing wave

Consider the following standing wave, (all units are SI)


z = 0.2 sin(0.4y) cos(350t)
(a) what is the wavelength?
The equation is of the form
z = A sin(ky) cos(t)
so we find the wavelength as =

2
2
=
.
k
0.4

(b) what is the angular frequency in radians/sec?


= 350, see above.
(c) what is the frequency (f) in Hz?
f=

350
=
2
2
204

(d) The values of y where the displacement is always zero, are of the form y = a + bn, where n=0, 1,
2, etc.
What are a and b?
2
2
=
, and this happens at multiples of half that. At y = 0,
k
0.4
sin(0.4 0) = 0, so there is a node there. Therefore a = 0, so that when n = 0, a + bn = 0.

Lets see. The wavelength is =

b must then be half the wavelength (above).


(e) The values of t where the displacement at all values of y equals to zero are of the form t = c + dn,
where n=0, 1, 2, etc.
What are c and d?
So what this is saying that if we plot the wave, it will be a straight line through the y axis (x axis on a
x-y plot) twice per cycle, at times t = c + dn.
This should happen when cos t = 0. Solving that, we find
t=

+ n
2

which nicely translates into c and d.

13.8.8

Problem 8: Traveling wave

Consider the following traveling wave (all units are SI).


x = 0.05 cos(3.5y + 126t)
(a) What is the wavelength in m?
Here we have the form
x = A cos(ky + t)
The wavelength is again =

2
.
k

To save time writing: the other questions are for , f , the waves direction, the speed of propagation v
and the amplitude.

; the direction is y; minus because there is a plus between ky and t (the


2
direction is opposite that sign) and y because we have ky.
The speed of propagation is found as
is given in in the t; f =

= vT
so

= f
T
Finally, the amplitude is the number multiplying the cosine, so A = 0.05.
v=

205

13.8.9

Problem 9: Lightly damped undriven circuit

On to what looks like the weeks hardest problem.


Consider the RLC circuit shown in the figure below in which R2 < 4L/C (underdamped).
Assume that at t = 0 , the charge on the capacitor has its maximum value.
(a) The differential equation obeyed by the potential across the capacitor can be written in the following
form:
d2 V
dV
+ bV = 0
+a
2
dt
dt
Express a [and b] in term of, if necessary, R, L, C.
OK. We start by writing down the circuit in terms of voltage, which means using a few equations used for
capacitors and inductors:
dV
IC (t) = C
dt
Z t
1
IL (t) =
VL (t)dt + VL (0)
L 0
Using Kirchhoffs bastardized loop rule (the one that holds, but confuses the physics for many),
VR (t) + VC (t) + VL (t) = 0

 Z t
dI
1
I(t)dt + I(0) + L = 0
RI(t) +
C 0
dt
The current through the circuit can be writtes as I = C dV
, where V is the potential across the capacicitor
dt
(V = VC ):
 Z t

1
dV
d2 V
dV
+
C
dt + I(0) + LC 2 = 0
RC
dt
C 0
dt
dt
I(0) = 0, because the capacitor is fully charged at that time, so the I(0) term disappears, and the integral
of the derivative simplifies:
RC

d2 V
dV
+ V + LC 2 = 0
dt
dt

Re-order:

d2 V
dV
+ RC
+V =0
2
dt
dt
Remove coefficient from the second derivative:
LC

d2 V
R dV
1
+
+
V =0
2
dt
L dt
LC
So we see then, that
R
L
1
b=
LC
These values have other often used names, a = 2 and b =02 . Here we see that 2 = (which we also
1
havent discussed) is the bandwidth, and 0 =
is the resonance frequency of the circuit.
LC
a=

206

In the circuit above, R = 251 Ohm, L = 0.07 H, C = 2 106 F.


(b) What is the decay constant (in seconds) according to which the charge on the capacitor is decaying?
It seems to me this would be easier if we have an equation for Q, so lets solve a differential equation for
Q instead, and see what happens.
Going clockwise with the current, starting at the top-left corner, using Faradays law:
Q
dI
= L
C
dt
Q
dI
I(t)R + L = 0
C
dt

I(t)R + 0

Writing I(t) in terms of

dQ
:
dt

dQ Q
d2 Q
L 2 =0
dt
C
dt
dQ
< 0. We can multiply by -1 throughout,
... keeping the sign in mind, as the current is clockwise, but
dt
and sort:
R

Q
dQ
d2 Q
+R
+L 2 =0
C
dt
dt
We solve this for the initial conditions Q(0) = Q0 and Q0 (0) = 0 (as the current will be zero if the
capacitors fully charged at the moment). We find a complex solution, however, the beginning of it is


Rt  
Q(t) = Q0 exp
...
2L
Using the standard form of


t
exp

we find
2L
R
which gives the correct answer. We could also have watched the problem solving videos where this decay
constant is given.
=

(c) The times at which the total energy stored in the RLC circuit is exclusively of electric nature can be
written as
t = a + bn n = 0, 1, 2, 3 ...
What are a and b?
This happens when all the energy is in the capacitor (stored in an electric field), which happens when
I = 0.
Finding the equation governing the current may not be easy, but we dont have to. We know that I(0) = 0,
2
T
.
and by definition, the zero crossing occur with a period , with T given by T =
2

(There are two zero crossings per period of a sinusoid.)

207

isnt easy to find either (Mathematicas solution of the differential equation doesnt contain it in an
easy-to-find manner, at least), so I will use the equation shown in the problem solving video:
r
1
R2
=
2
LC 4L
So we have
2
0.003170045

a=0
0.003170045
b
2
a = 0 because we know a zero crossing is at t = 0, so when n = 0, bn = 0, and a + bn = 0.
T =

13.9
13.9.1

Week 10
Problem 1: Traveling electromagnetic waves

Consider two examples of a plane, monochromatic, electromagnetic wave traveling in a homogeneous


medium. The electric field vector is given in the two cases by
case (1) Ex = 0; Ey = 0; Ez = 50 sin(4.71x + 9.12 108 t)
case (2) Ex = 0; Ez = 0; Ey = 55 sin(4.71x 1.05 109 t)
~ is measured in V/m, t in sec, and x in m. For each case, answer the following questions:
where |E|
(a) What is the propagation direction of the wave?
The direction of propagation is given by the term inside the sine; in the first case, we have the form
sin(kx + t) which implies movement in the negative x direction. In the second case, the opposite is true.
2
, so
They then ask for the wavelength and wave number. The wave number we use is defined as k =

2
=
, with k being 4.71 in this case, so that answers the four sub-questions here.
k

After that, they ask for the frequency. That is given by f = , but v 6= c here, and we do not know it, so
v
we have to find some other way to calculate it. We can use , the term multiplying t in the sine.

= 2f , so f =
, which answers these two questions.
2
Next up, the propagation velocity in meters per second (which we couldve used above). We can relate
that to the wavelength and frequency: f = v.
c
Next question: index of refraction. n = , so we can pretty much take 3 108 m/s and divide it by two
v
previous answers. Easy.
where we are to give them
~ = B0 sin(kx t )d,
After that, they want the corresponding equations for B
the direction, B0 , k and , with the correct sign.
They are in phase with the electric field, so we just copy the contents of the sine above.
= v, so we have z B
=
= y via the
As for the direction, E B
x for the first case. That means that B
right-hand rule.
= +
= +
For the second case, we have y B
x, which means B
z.
What about magnitudes? In a vacuum,
208

E0
c
...but this is not in a vacuum. I assume we can replace c by v and be done with it, and that indeed gives
us green checkmarks, using v from above instead of c.
B0 =

Finally, the last sub-question: what is the time-averaged Poynting vector (magnitude and direction) for
x = y = z = 3 for case 2 ?
~
~
~ = E B , and its time-averaged magnitude by hSi = E0 B0 .
The Poynting vector is given by S
0
20
~

Thus the direction is given by dir S = E B = y z = x.


The magnitude is simply found using the formula above - we have E0 and B0 already, and know 0 =
4107 .
The coordinates do not enter into this, because this is a plane wave.

13.9.2

Problem 2: A standing electromagnetic wave

~ = E0 x cos(9.5z) cos(1.84 1011 t) where z is meaA wave solution to Maxwells Equations is given by E
sured in centimeters and t in seconds.
(a) What is the wavelength (in meter) of the wave?
Ugh, centimeters. Well, lets look at this the right way, and not divide by 100 where we should really
multiply, etc.
2
, where k is given in radians per meter.
The wavelength is =
k
We are given k in rad/cm, so we need to multiply it by 100 to get rad/m:
=

2
9.5 100

and index of refraction n = of the medium.


2
v
11
1.84 10

=
Hz
f=
2
2

1.84 1011
v= =
1.936 108 m/s
k
9.5 100
c
3 108
n= =
v
1.936 108
~ = B0 f1 (kz) f2 (t) d
(c) The associated magnetic field can be written as B
What is the direction, the value of B0 , k, and what are f1 and f2 ?
They then ask for the frequency f =

The direction must be


y , because the wave is propagating in the z direction (though this is a standing
wave, so the movement is cancelled out), and the E-field is in the x direction.
= z, so B
= y for that to be true.
We should find that x B
E0
c
, where v = , so we have the values we need to answer that.
v
n
k and have the same values as for the E-field, we can just copy and paste them (but watch out for the
units of k being in cm, so we must multiply it by 100 here as well).
f1 and f2 are both the sine function. The E-field uses cosines, and the B-field is 90 degrees out of phase
in a standing wave (see the problem solving videos).
The magnitude can be found via B0 =

209

Use the following trigonometric identity


2 cos cos = cos( + ) + cos( )
to express the standing wave as a superposition of two traveling waves. Then, obtain the magnetic fields
associated to those traveling electric field, and combine them together to get the magnetic field associated
with the standing electric field.
Well, we have
~ = E0 x cos(9.5z) cos(1.84 1011 t)
E
that we need to match up with the identity, and split up. Clearly, = 9.5z and = 1.84 1011 t. The
identity has a factor of two prior, and our equation has the factor E0 ... so it looks like we can write it as
~ = E0 x cos(9.5z) cos(1.84 1011 t)
E

~ = E0 x cos(9.5z + 1.84 1011 t) + cos(9.5z 1.84 1011 t)
E
2
So the standing E-field can be written as the above sum. Writing the two traveling waves as two equations,
we find
E0
x cos(9.5z + 1.84 1011 t)
2
E0
x cos(9.5z 1.84 1011 t)
Etr2 =
2
... with tr1 signifying traveling wave 1, etc. Next, they want us to find the magnetic fields of those waves.
We now have two traveling waves, so we use B0 = E0 /v, with v 1.936 108 m/s as we found earlier.
The B-field should now be in phase, so we find
Etr1 =

E0
y cos(9.5z + 1.84 1011 t)
2v
E0
=
y cos(9.5z 1.84 1011 t)
2v

Btr1 =
Btr2

= v, but
Note the minus sign in Btr1 , which is absolutely crucial! Without it, we dont find E B
v
instead, which is of course incorrect. That screws up our entire result, too.
We can combine the result using the identity
cos( ) = cos cos sin sin

E0
y cos(9.5z) cos(1.84 1011 t) sin(9.5z) sin(1.84 1011 t)
2v
With less confusing minus signs (by distributing the one in front):
Btr1 =

Btr1 =


E0
y cos(9.5z) cos(1.84 1011 t) + sin(9.5z) sin(1.84 1011 t)
2v


E0
y cos(9.5z) cos(1.84 1011 t) + sin(9.5z) sin(1.84 1011 t)
2v
We now see that when we add these two, the cosine terms cancel, and the sine terms add:
Btr2 =

~ = E0 y 2 sin(9.5z) sin(1.84 101 1 t)


B
2v
~ = E0 y sin(9.5z) sin(1.84 101 1 t)
B
v
210

This is indeed what we need to get part (c) correct (above).


(d) What is the time-averaged Poynting vector for x = y = 3, z =
square meters per second.

3? Express you answer in joules per

We barely need to do math to solve this one - the answer is 0 for all components, because this is a standing
wave! The time-averaged Poynting vector will be zero in such a case, since there is no net movement.
However, lets have a quick look at the math that proves this.
E0
11
11
~
~

~ = E B = (E0 cos(9.5z) cos(1.84 10 t))( v sin(9.5z) sin(1.84 10 t)) (E B)


S
0
0

Thats way too messy, so I will calculate the result with and instead:
E02
cos cos sin sin
v
What is the time average of the trigonometric mess on the right? Looking at the functions of time (), we
have one cosine and one sine, essentially sin t cos t. Because the argument is the same for both functions,
we get the shape of a sinusoid, with lower amplitude than plain sin t. Indeed, its equivalent to 12 sin(2t),
and the time-average of that is zero! Since we are multiplying all the terms together, and one of them has
a time-average of 0, the time average of the Poynting vector is also zero.
S0 =

13.9.3

Problem 3: E-M waves - Maxwells equations, and the speed of light

We discussed in lectures that traveling Electromagnetic waves in vacuum of the form


~ = E0 x cos(kz t)
E
~ = B0 y cos(kz t)
B
satisfy all 4 Maxwells equations. In lectures, I showed that an application of the generalized Amperes Law
(closed loop surrounding area A2, see below), leads to: B0 = E0 0 0 c, and I mentioned that independently
it follows from an application of Faradays Law that B0 = E0 /c. Combining these two results then leads
1
to the fantastic result that the speed of light in vacuum c =
. I want you to show that Faradays
0 0
Law indeed leads to the result B0 = E0 /c. You can show this by choosing a similar special area as we did
in lectures:

We define the normal of the surface A1 in the figure above to point in the +
y direction. In the following,
12
assume that `=1 m, E0 = 1 V/m, f = 610 10 Hz, where f is the frequency of oscillation.

211

H
H
~ where E
~ is the closed loop integral taken along the contour of the area A1
~ d`,
~ d`
(a) We define f1 (t) = E
in the figure above. Evaluate the function f1 (t) in Volts for the following value of t: t = 2.21016 seconds.
Wow, thats a lot of text! Well, most of the intro text is irrelevent for solving the problem (its a nice
background, but we dont need to memorize it!).
The E-field is in the x direction, so we can think of it as a sinusoid drawn upon the figure above, in the
z x plane, going left to right.
We go counterclockwise for the line integral. The reason for this is that the problem defines the surface
normal as +
y , so according to the right-hand rule, with the thumb pointing out of our screens, we curl
the fingers counterclockwise.
The top and bottom segments are both perpendicular to the E-field, and so their dot products are both
zero.
~ up, and the E-field up, so that contributes a positive term +`E(/4, t).
The rightmost segment has d`
~ down, but the E-field up, so that contributes a negative term `E(0, t). Thus
The leftmost segment has d`
we have:
I
~ = `E(/4, t) `E(0, t)
~ d`
E


k
t) cos(0 t)
= E0 ` cos(
4



= E0 ` cos( t) cos(t)
2
= E0 ` (sin(t) cos(t)) 0.0817035 V
(b) Consider the function f2 (t) = B (t). Following the method used in Lecture 27, calculate the function
f2 (t), and evaluate it (in Volts seconds) for the following time t: t = 2.2 1016 seconds.
The method used in lecture was to integrate over the area shown in the figure above, so lets try that from
memory (it should be obvious how to do this anyway).
Since the B-field is in the +
y direction, we need to do this on the area perpendicular to that, A1, or we
will surely find B = 0.
This being a plane wave, the B-field is constant in the xy plane, but not in the z plane. We can thus
integrate it, with ` being the length of the side where its constant, and dz the width, integrating from
z = 0 to z = /4.
By the way, the area is perpendicular to the B-field, so we need not worry about any cos here.
Z

/4

B =
We plug in the values, with B0 =
B 3.68366 1016 T/m2 .

/4

cos(kz t)dz

B`dz = B0 `

B =

B0 `
(cos(t) + sin(t))
k

E0
2f
(which we are trying to prove!), = 2f and k =
, and find
c
c

Next they want the negative of the derivative, i.e. the change in flux, that is, Faradays law.
dB
B0 `
d
=
(cos(t) + sin(t))
dt
k
dt
dB
B0 `

=
( sin(t) + cos(t))
dt
k

212

dB
0.0817035 V
dt
We find the same result as in part (a), which shouldnt be surprising, as Faradays law states
I
~ = dB
~ d`
E
dt
... and all weve done is to calculate the left and right sides of the equation separately!
E0
However, we calculated B0 using
, so at least for these numbers, we have proven the two to be equal.
c
Can we prove it generally? Lets see, if we equate the symbolic answers for the line integral and the
(negative) flux derivative:

E0 ` (sin(t) cos(t)) =

B0 `
( sin(t) + cos(t))
k

Factor out and cancel `:


E0 (sin(t) cos(t)) =

B0
( sin(t) + cos(t))
k

Factor out 1 on the right-hand side:


E0 (sin(t) cos(t)) =

B0
(sin(t) cos(t))
k

We can now cancel the trig terms completely:


E0 =

B0

= B0 = B0 c
k
k

Success!
E0 = B0 c
E0
B0 =
c

13.9.4

Problem 4: Polarized radiation

Write down the electric field and associated magnetic field in vacuum for a traveling plane wave with the
following properties. The amplitude of the electric vector is E0 and the frequency is . The radiation is
linearly polarized in the y-z plane at an angle of /4 with respect to the y-axis, and it is traveling in
the +x direction. The electric and magnetic field can be written in the following way:
~ = E0 cos(t + (1)bE kx)(cE x + dE y + eE z)
E
~ = cos(t + (1)bB kx)(cB x + dB y + eB z)
B
There are two options for the direction of . What are the parameters for both electric and magnetic
field? Give answers for the two possible options:
They then want values for bE , cE , dE and eE , for = /4 from the +y direction (rotated towards +z) and
for = /4 from the +y direction (rotated towards -z).
Lets look at the first case first. But even before that, what do they even mean?
~ = E0 y
If the radiation were linearly polarized in y, that would mean that the E-vector is described by E
times some sine or cosine. That is, the x and z components are exactly zero.
Since this wave is polarized in the yz plane, with an angle with respect to the x axis, we will have a zero
x component, but nonzero y and z components. (However considering the angle of 45 degrees, I assume
the y and z components will be equal in magnitude!)

213

The direction of the polarization must always be perpendicular to the direction of propagation, which it
is: the wave propagates in the +x direction (in phase with the B wave), while the polarization is in the
yz plane.
(The B-field is perpendicular to the E-field, as usual, and is in phase, as usual. However, we define linear
polarization via the E-vector only, and we will ignore the B-field for now, and find the E-field first.)
Okay, so lets look at this thing. There should be a minus sign in the cosine, since the wave is propagating
in the plus x direction. So
bE = 1
cE = 0
... as cE is the x component, which is zero as the polarization is in the yz plane. What remains is
~ = E0 cos(t kx)(dE y + eE z)
E
Two unknowns remain. We know that the direction we want is part y, and part z. Equal parts, since the
direction is exactly 45 degrees from the y axis (with 90 degrees, it would be in the z plane only).
So that essentially leaves us with one unknown, since the values should be equal (at least in magnitude).
When we combine unit vectors, we get this result: a vector (of non-unit length! the magnitude will be
larger than 1) pointing in a direction that is neither y nor z, but something in between, is y + z. This
principle applies here, as well.
In order to get the unit magnitude of such a vector, we need to divide it by its magnitude. So what is the
magnitude?
Lets ignore that third dimension, and think of this as a 2D problem. We draw a coordinate system, and
set up a vector from the origin
to (1, 1)- 1 away from the y axis, 1 away from the z axis. What is the
distance to the origin? r = 12 + 12 = 2. So that unit vector, lets call it r (just picking something), is
y + z
r =
2
I hope this doesnt get too confusing. The above was just the explanation of the principle. Because the
magnitude of the E-field must be E0 (at its maximum), the magnitude of the vector components we find
must be 1, or we will scale it upwards (or downwards)! That is,
=1
|dE y + ee E|
p
must hold for our chosen values. We need dE 2 + eE 2 = 1 and dE = eE . It looks fairly apparent that the
two values that makes this relation hold are (plus the bE and cE values we already found):
bE = 1
cE = 0
1
dE =
2
1
eE =
2
So that answers the first part. Next up: the B-field. It will be perpendicular to the E-field, so we can
work from that. bB = 1, so that it the travel direction and phase match up with the E-field.
Here, there is no magnitude specified in the equation given, so rather than having our vector components
E0
add to a magnitude 1, they must add to B0 =
.
c
214

What will the direction be? If the E-vector is in the direction of y + z, and the B-field is plus or minus 90
= x must hold... What do we find?
degrees from that, and E B


y + z
= x

B
2
Honestly, Im not sure how to solve this mathematically, but its easy to find more intuitively, at least as
far as the direction is concerted. The B-field should be perpendicular to the E-field weve found, so there
are two possible choices that may work out with also being perpendicular with the propagation direction.

3
One is that we rotate it in the same direction as we did with the E-field, from
to + =
. The
4
2
4
4
other is that we go in the other direction. What happens in the first case? Drawing a coordinate system
(which partly helps with 3D visualization, but not entirely), it seems as that this one is the one that will
work (keep in mind that the right-hand rule should apply!).
So now, we want one part +
z and one part
y , and then we correct for the magnitude by dividing by

E0
2 (so that the net vector has magnitude 1), and finally multiply by the
that we want as the net
c
magnitude of the wave:
bB = 1
cB = 0
E0
dB =
c 2
E0
eB =
c 2
E0
, as we want. the negative in dB is to get it in
When we multiply all this out, the magnitude becomes
c
the
y direction, i.e. downwards (the way Ive drawn this, +x is to the right of the paper, +y upwards in
the plane of the paper, and +z out of the paper).
Next up: case 2, where the rotation is in the other direction. The E-field is first out.
Clearly, the x component must still be zero, as we are in the yz plane still. The direction of propagation
has not changed, so bE = bB = 1.
Looking at the old intuitive directions, we now want part of
z and part of +
y for the E-field vector, so
that it points upwards, inwards using the coordinate system sketch mentioned above. The magnitudes
are the same as before, though, so we just throw a negative on the eE value, but other than that, the
E-field vector is unchanged from before.
bE = 1
cE = 0
1
dE =
2
1
eE =
2
And finally, the B-field. That is now located where the old E-field was; rotated 90 degrees from the /4
we had, it is now at +/4 degrees rotation, just as the first E-field. So we just copy and paste the values
E0
from there... except that we need to multiply them by
for the magnitudes!
c
bB = 1
215

cB = 0
E0

dB =
c 2
E0

eB =
c 2

13.9.5

Problem 5: Polarization of electromagnetic radiation

(a) Describe the polarization state of the plane E-M waves represented by the following equations for the
~
electric field E(x,
t)(Ex = 0 in all three cases):
case(1):
Ey = E0 sin(kx t), Ez = 4E0 sin(kx t)
The two components are in phase, so this is linear polarization.
Linear polarization is characterized by wave components that are in phase, such that the resulting wave
is confined to a plane.
Circular polarization is has wave components 90 degrees out of phase, which causes the resulting vector
to rotate and trace out circles.
Elliptical polarization is like circular, except the phase or magnitudes are such that an ellipse is traced,
rather than a circle.
If you have not already seen such videos, I highly recommend searching for videos that show linear and
circular polarization via 3D graphics software. Its very helpful to understand the concepts.
case(2):
Ey = E0 cos(kx + t), Ez = E0 sin(kx + t)
Here we have a 90 degree phase difference (sine versus cosine), but with equal magnitudes. That gives us
circular polarization.
case(3)
Ey = 2E0 cos(kx t + /2), Ez = 2E0 sin(kx t)
We can rewrite the cosine as a minus sine:
Ey = 2E0 sin(kx t), Ez = 2E0 sin(kx t)
In phase, equal magnitude: linear polarization.
(b) For case (1), where Ey = E0 sin(kx t), Ez = 4E0 sin(kx t), the magnetic field can be written in
the following way:
~ = sin(kx + (1)bB t)(cB x + dB y + eB z)
B
In other words: Bx = cB sin(kx + (1)bB t), By = dB sin(kx + (1)bB t), Bz = eB sin(kx + (1)bB t)
~
What are the values of the different parameters in the above equation for B?
Heh. Not that easy to read, but its no harder than the previous homework problem.
bB = 1, because we want that minus sign, to keep it in phase with and in the same direction as the E-field.
The wave propagates in the x direction, with the E-field having only y and z components. The B-field also
only has y and z components, though its still perpendicular to the E-field, of course.
As in the previous problem we did with this, the x component must be zero.
E0
in magnitude, because E0 is
Unlike the previous problem, we cant simply try to get the B-field to be
c
not the magnitude of the E-field. We need to find that magnitude first!
216

~ = Ex x + Ey y + Ez z
E
~ = sin(kx t)(E0 y + 4E0 z)
E
So the magnitude will be given by
~ = |E0 y + 4E0 z| =
|E|
So B0 =

E0 2 + (4E0 )2 =

17E0 2 =

17E0

E0
. However the E-field has different magnitudes for the different components, unlike last
17 c

time.
The Ey components should have a B-field component perpendicular to it of the same magnitude, I would
guess. So Bz = Ecy = Ec0 .
The same thing goes for the Ez component, which turns into By as we rotate it 90 degrees. Thus we find
bB = 1
cB = 0
4E0
dB =
c
E0
eB =
c
...which is marked as correct! Intuition proved useful once again!

13.9.6

Problem 6: Poynting vector

The Poynting vector associated with a plane electromagnetic wave is described as follows:
~ = e0 E0 2 c cos2 (ky t)
S
y
with k = 433 rad/m and = 1.31 1011 rad/sec. The direction of the electric field oscillates along the x
axis.
(a) One possible vector description of the electric and magnetic field in the plane wave associated with
this Poynting vector is
~ t) = E0 x cos(ky + (1)aE t)
E(y,
~ t) = (bB x + cB y + dB z) cos(ky + (1)aB t)
B(y,
~ and B
~ that correspond to the given Poynting vector.
Determine the parameters in these expressions for E
Even more of this... Im getting a bit tired of it, to be honest.
which in this case should equal y. With E = x,
Well, the direction of the Poynting vector is S = E B,
the B-field should then be in the
z direction for the cross product to work out:
x (
z ) = y
So now we at least know the B-fields direction,
z.
What about the cosine signs, i.e. aE and aB ?
The Poynting vector should be propagating in the same direction as the wave, which means the wave
should propagate in the +
y direction, as the Poynting vector does. Therefore, we want the minus signs
inside the cosines, and aE = aB = 1 causes that to happen.

217

As for the component values, that is pretty easy. We have found the direction as
z , so there must be a
negative z components, and 0 x and y components. The one nonzero component is really simple, as the
E0
magnitude of that will be the magnitude of of net vector 0 =
, so we find
c
aE = 1
aB = 1
cB = 0
dB = 0
E0
eB =
c
... with a negative to get it in the negative z direction, as mentioned above.
Could a different pair of electric and magnetic fields have this same Poynting vector?
= +
Hmm, well, lets see. The direction must be E B
y ; can we get that using other values that E = x
=
and B
z ? Well, yes: negate both the E-field and the B-field, and it works out... But does that really
get us a different pair of fields? They oscillate back and forth, and we just changedt he phase...
(The answer is that it is possible, but Im not sure the above is enough to confirm that.)
(b) What is the wavelength in meters [and period T , in seconds] of the wave?
This should be second nature by now.
2
2
=
m
k
433
1
1
2
2
T = =
=
=
s
f
/2

1.30 1011
=

13.9.7

Problem 7: Intensity of the sun

At the upper surface of the earths atmosphere,D the Etime-averaged magnitude of the Poynting vector is
~
2
referred to as the solar constant and is given by S
=1.35 103 W/m .
(a) If you assume that the suns electromagnetic radiation is a plane sinusoidal wave, what is the magnitude
of the electric field in V/m?
Digging up the definion of the time-averaged Poynting vector:
1 E0 B0
1 E0 2
hSi =
=
2 0
2 0 c
So we can set that latter thing equal to the value we have, and solve:
1.35 103 =

1 E0 2
2 0 c

2.7 103 0 c = E0 2

E0 =

2.7 103 4107 3 108 1008.89 V/m

What is the magnitude of the magnetic field in T?


We can find this as B0 = E0 /c, knowing that equivalence. Or, we could use the other equivalence (which
is really found by using B0 = E0 /c anyway!):

218

1 (1008.89)(B0 )
2
4107
2.7 103 4107 = (1008.89)(B0 )
1.35 103 =

2.7 103 4107


3.36 106 T
B0 =
1008.89
(b) What is the time-averaged power (in Watt) radiated by the sun? The mean sun-earth distance is
res = 1.5 1011 m.
We usually do these calculations the other way around, but this shouldnt be a problem.
The intensity at the Earth is inversely proportional to the distance, as the sun radiates (roughly) equally
in all directions:
Isun

1
= 1.35 103
2
4res

Isun = 1.35 103 4res 2 3.81 1026 W

13.9.8

Problem 8: Snells law in action: fiber optics!

An optical fiber is a flexible, transparent fiber devised to transmit light between the two ends of the fiber.
Complete transmission of light is achieved through total internal reflection. This problem aims to calculate
the minimum index of refraction n of the optical fiber necessary to obtain total internal reflection for every
possible incidence angle.
(a) Express sin , where the angle is defined in the figure above, in terms of the incidence angle and
the index of refraction n of the optical fiber. Evaluate this function for n = 1.55 and = 84 . Take the
index of refraction of air to be 1.
Lets begin by writing down Snells law, in two forms.
n1 sin 1 = n2 sin 2
n2
sin 1
=
sin 2
n1
Lets think this through a bit before we go further.
The beam enters and leaves (if it does leave at the top) on non-parallel sides, so it will not leave parallel
to the incident light, unless n = 1.
Looking at the triangle drawn,

++ =
2

=
2
... since the sum of angles in a triangle must be 180 degrees, i.e. radians.
We can then attempt to find as a function of , using Snells law:
sin 1
n2
=
sin 2
n1
sin
=n
sin
sin = n sin
219

sin
n


sin
= arcsin
n
sin =

So that means that

= arcsin
2

sin
n

We want to find the relationship between and , however, so we cant stop here!
We have another refraction here, with as the incident angle:
sin 1
n2
=
sin 2
n1
1
sin
=
sin
n


sin
sin 2 arcsin n
1
=
sin
n

We flip the whole thing, to find sin , and apply sin( x) = cos x:
2
sin
 = n
sin arcsin sinn



sin
sin = n cos arcsin
n

... which answers the first part!


(b) The condition on n for total internal reflection of all beams entering the fiber is achieved when = 90
or unphysical (i.e. sin 1) for all values of . Determine the smallest value of n that satisfies that condition.
Well, they essentially set up an equality and told us to solve it, so lets go.



sin
1
n cos arcsin
n
Ugh, the cosine and arcsine are a bit annoying... This would be so easy without the n inside. Ah well.
I didnt know the exact identity, but I did know that cos(arcsin(x)) could be written in a different way,
using some square root. Indeed:
cos(arcsin(x)) =

1 x2

So we can rewrite our inequality:


s
n 1

sin
n

2
1

sin2
n 1
1
n2


sin2
2
n 1
12
n2
n2 sin2 1
220

n2 1 + sin2
p
n 1 + sin2
We need
to find the smallest
n
for which this
holds true for all . The right-hand side can only move

between 1 + 0 and 1 + 1 = 2, so n = 2 is the smallest value of n for which this is always true,
which solves the problem!

13.10

Week 11

Theres no homework this week, due to the third midterm.

13.11

Week 12

13.11.1

Problem 1: Primary rainbow

Combine = 4r 2i with Snells Law to express the angle as a function of i and n only.
(a) Evaluate (i, n) for the following values (give your answers in degrees): i = 5 , n = 1.55
We use Snells law to accomplish this, as mentioned in the question. The incident light has angle i, and
the refracted light (at point A) has angle r, which is a function of i according to Snells law.
n1 sin i = n2 sin r
Using n1 = 1 for air, and n2 = n from the problem statement:
sin i = n sin r
sin i
sin r =
n


sin i
r = arcsin
n
Therefore, (i, n) is

(i, n) = 4 arcsin

sin i
n


2i

Evaluating the three angles is now easy (just keep track of using radians and degrees correctly!).
(b) For a given index of refraction n, determine the value of the incidence angle i0 that maximizes , and
the correspondent value of max (give your answers in degrees). All answers should be positive angles.
The following identity to differentiate the inverse function f 1 may be useful:
221

d f 1 (x)
=
dx

1
d f (z)
dz
z=f 1 (x)

Okay, so they want us to differentiate (i, n) with respect to i and find its maxima. The formula is clearly
intended for finding the derivative of the arcsine function. Using f 1 (x) = arcsin(x), f (x) = sin(x) (since
d
sin(z) = cos(z):
the arcsine is the inverse of the sine) and
dz
d
1
arcsin(x) =
dx
cos(arcsin(x))
That doesnt answer the entire thing, however. We still need to apply the chain rule, not to mention we
cant forget about the constant multiplier and the 2i. All in all, we have


(i, n)

sin i
= 4 arcsin
2
i
i
n
The (partial) derivative of the arcsine is then just the chain rule: the derivative of the outside with respect
to the inside, times the derivative of the inside with respect to i (in this case) is how I remember it best.
We know the first part already, and the second part is just the the derivative of sin i times the constant
one over n inside. Keeping all this in mind:
(i, n)
1
=4
i
cos arcsin

sin i
n



cos i
4 cos i
2=
n
n cos arcsin

sin i
n

 2

Frankly, this is a bit painful, and as it is allowed, I will use Mathematica to substitute in n and set the
whole thing equal to zero. Doing so (the second part, leaving n as is), I find

v
s
u
u n2 + 2
2

n 1
2 arctan t
2 3
2
4n
(n2 4)2
... there has got to be a better way to solve this... But the above does give correct answers.
For n = 1.55, we find i0 = 46.8634 , while for n = 1.3 we find i0 = 61.3418 .
We then plug those values into (i, n) and find
(46.8634 , 1.55) = 18.6158
(61.3418 , 1.3) = 47.1321
(c) What is the angle of incidence i of light on a spherical raindrop that will lead to the red in the primary rainbow (give your answers in degrees)? The index of refraction of red light in water is nred = 1.331.
The red in the rainbow is where max is for red light, so we just plug n = 1.331 into the above formula, to
and then plug that into (i, n) as above.
i0 = 59.5267
(59.5267 , 1.331) = 42.3698

222

13.11.2

Problem 2: Polarization of primary rainbow

The average index of refraction of water is n=1.336. (a) What is the Brewster angle Br when light reflects
in water off air (i.e. at point B) (give your answer in degrees)?
Easy, assuming we know the definition of the Brewster angle. It is
Br = arctan

n2
n1

... with n1 being the index of refraction of the current medium (the water), and n2 of the medium we
reflect off (the air, n2 = 1). Thus
1
36.815
1.336
(b) Assume that i is the incident angle which gives the largest value of as found in Problem 1. Then, r
is both the refracted angle at Point A and the angle of incidence of the light ray at point B in the figure
above. What is r Br (give your answer in degrees)?
Br = arctan

Well, we return to our formulae for i0 and r(i, n) with n = 1.336:


i0 = 59.2362
arcsin(sin 59.2362 )
r(59.2362 , 1.336) =
= 40.0291
1.336
r Br = 40.0291 36.815 = 3.2141
(c) Which of the following sentences do you think is most accurate about the light from the primary
rainbow?
(The choices are completely unpolarized, weakly linearly polarized and strongly linearly polarized.)
Strongly linearly polarized, as the angle of incidence for the reflection is very close to the Brewster angle. If
the angles were equal, the light would be 100% polarized; instead, it will be fairly close to 100% polarized.

13.11.3

Problem 3: Glassbow

What would be the radius (max , in degrees) of a glass bow? The glass beads have an index of refraction
n = 1.5. We spread them out on the ground and we observe a glass bow as the sun is high in the sky.
The maximum for the angle of incidence, i0 , would be i0 (n) = 49.797 , and (49.797 , 1.5) = 22.8415 , and
so the answer is 22.8415 . Piece of cake, now that we have the previous problems figured out!

223

13.11.4

Problem 4: Secondary rainbow

All answers should be in degrees.


(a) Using the result of Lecture 31 Question 3, and Snells law, derive and evaluate = (i, n) for the secondary rainbow for the following values of the incidence angle i and index of refraction n: i = 6 , n = 1.35.
The result from question 3 is that
(i, r) = 2 + 2i 6r
is then given as = from the figure.
(i, r) = + 2i 6r
We use Snells law to get rid of the r:
n1 sin i = n2 sin r
sin i = n sin r
sin i
sin r =
n


sin i
r = arcsin
n
Nothing new there. Combining the results:

(i, n) = + 2i 6 arcsin

sin i
n

We can plug in the values and find the first three answers. All three answers are in the range 90 > >
180 .
(b) For a given index of refraction n, determine the value of the incidence angle i00 that minimized , and
the correspondent value of min .

224

This is in a way the same thing as we did for the primary rainbow earlier on. Because the secondary bow
is flipped inside-out, the minimum angles are where we will find the colors; all colors are allowed outside
(there will be while light outside the secondary).
We could differentiate it again, but I will simply use Mathematicas FindMinimum function here, for the
various values.
For n = 1.35:
i00 = 71.2981
min = 55.249
For n = 1.5:
i00 = 66.7163
min = 86.8651
(c) What is the angle of incidence i of light on a spherical raindrop that will lead to the red (n = 1.331)
in the secondary rainbow?
This question is again exactly like the two previous, so we do the same thing for n = 1.331:
i00 = 71.9073
= 50.3651

13.11.5

Problem 5: Diffraction pattern

Light of a red laser (wavelength = 650 nm) goes through a narrow slit which is only a = 2 microns
wide. After the light emerges from the slit, it is visible on a screen that is L = 5 meters away from the
slit.
What is the approximate width on the screen (in cm) of the bright central spot? Here width is defined as
the distance between the center and the first minimum.
If we have the formula we found in lecture, this is very easy.
Ln
m
a
m, and the answer should be in cm, so
xn

n = 1, L = 5m, = 650 109

5 650 109 100 cm

162.5 cm
2 106
1m
Note: this solution uses the small angle approximation, which was good enough to be marked as correct,
but it is actually more than 5% off the correct answer, due to the relatively large angle (I didnt pay
attention to that, since my first try was marked as correct). For a more correct answer, we use the formula
we found this week, that is

 

x = L tan = L tan arcsin


d
xn

225

13.11.6

Problem 6: Optical resolution of the human eye

A car with its 2 headlights on is approaching you at night. Approximately how close (in meters) does the
car have to be to you so that you can distinguish the two headlights? Assume that the diameter of your
pupils is 6.5 mm, the distance between the two lights is 1.5 m, and the wavelength of the light is 550 nm.
Again, this should be fairly easy. There will be diffraction associated with each source on its own, and
they need to be further apart, in angular distance, than the Rayleigh criterion limit of
sin = 1.220

where d = 6.5 mm, the diameter of the lens.

sin = 0.10323 103


(For such small angles, the sine of the angle and the angle in radians are essentially equivalent.)
OK, so we have that but we need the actual distance in meters.
We draw the situation on paper: an isosceles triangle (from our position (right in front of the car) to the
two headlights.
Because it is symmetric, we can split it in two, and have /2 radians between one headlight and the cars
center. One side (the long, non-hypotenuse side, adjacent to ) of that triangle is then the distance d
between us and the car, and we find
1.5/2

=
2
d

d tan = 1.5/2
2
tan

d=

1.5/2
= 0.75 cot(/2) 14531 m
tan(/2)

Wow, thats insanely far! This only counts the diffraction limiting, though... Id be surprised if this was
actually true in a real situation, given non-ideal eyesight and all that. Hmm.

226

Chapter 14
Exam problems
14.1
14.1.1

Midterm 1
Problem 1: Electric field on the surface of a conductor

The electric field at point A on the surface of a conductor is 49 103 V/m. What is the surface charge
density (C/m2 ) at that point?
Lets call the surface charge density . The total charge for that tiny area at the surface is Q = A.
We choose a Gaussian pillbox that is located such that the end cap is located just above the surface.
Via Gausss law
A
0
A on the left side is the area of the pillbox, and A on the right side the area of the surface. Of course, we
choose them to be equal, so they cancel:
EA =

E=

0

Thus
= E0
We know E at that point, so
= 49 103 8.854187 1012 4.3385 107 C/m2
We can sanity check the units of this. The electric field is in V/m, 0 is in F/m = (C/V)/m and in
C/m2 :
V F
C
?
=

m2
m m
C
V
C
?
=

m2
m Vm
C
C
=
m2
m2

227

14.1.2

Problem 2: Non-conducting charged planes

Two parallel non-conducting sheets of area A = 4m2 are separated by a distance d = 0.007 m. They carry
equal but opposite surface charge densities of = 7 106 C/m2 .
(a) What is the electric field (V/m) between the plates? (ignore end effects)
We could use a Gaussian pillbox to solve this for an infinite plane (since we are to ignore end effects), but
the result is so easy to remember that I wont feel guilty for using it. The electric field between two such
plates is
E=
... half of which (

0

) is from the top plate, and half from the bottom.


20

So the answer for (a) is


7 106
790586.41974 V/m
8.854187 1012
(b) What is the total energy (Joules) contained in the electric field between the sheets?
E=

We can use U =

1 Q2
here.
2C

Q is given by A = 4 = 28 106 C.
40
A0
=
5.059535 109 F. Thus,
C is given by the geometry, C =
d
0.007
1 (28 106 )2
U=
0.077477 J
2 5.059535 109
(c) A conducting sheet with the same area as the two nonconducting sheets and with thickness h = 1 mm
(0.001 m) is inserted between the two sheets. What is the potential difference (Volts) between the sheets
with this conducting sheet in place?
Uh oh. Well, the potential difference without the sheet in place, since the electric field is uniform, is simply
V = Ed 5534.105 V.
If we insert a dielectric, an induced electric field will appear, that cancels out part of the original field; the
net field is reduced by a factor of , the dielectric constant for the material. However, thats for dielectrics
- and only when they completely fill the void between the two plates, too.
It turns out the answer is that since the sheet is a conductor, and the electric field inside it is zero, we can
simply integrate over the rest of the distance. That is, we add up the potential difference from the top
plate to the conductor, add the potential difference across the conductor (zero), and then the potential
difference from the bottom of the conductor to the bottom plate.
Since the field is uniform, V = Ed. The distance d to use is then the distance between the plate minus
the thickness of the sheet (where the field is zero):
V = E(d h)
7 106

(0.006 0.001) 4743.5 V


V = (d h) =
0
8.854187 1012

228

14.1.3

Problem 3: Incandescent bulbs

An incandescent light bulb A consumes PA = 21 W when it is connected to a V = 126 V battery. Another


incandescent light bulb B consumes PB = 253 W when it is connected to a V = 126 V battery. We place
the two light bulbs in series. You may ignore the internal resistor in the power supply and you may also
assume (for simplicity) that the resistance of the light bulbs are independent of the temperature.
We connect these two light bulbs in series to the V=126 V battery.
(a, b) How much power (in W) is used by bulb A (and B)?
Ah, finally something thats not intimidating at all to me. Exams always freak me out a bit...
V2
. We know P and V, so well use the last equation and solve
Well, power is given by P = V I = I 2 R =
R
it for R.
V2
R
V2
R=
P

P =

We can now calculate the bulb resistances:


1262
= 756
21
1262
62.750988
RB =
253
We put them in series with the battery of 126 volts, and get a current:
RA =

I=

V
0.15389296 A
RA + RB

We can then use I 2 R to find the power dissipated in each bulb.


PA = (0.15389296)2 756 17.90438 W
1262
1.486135 W
253
All of a sudden the previously weak bulb is the brightest! Lets sanity check the answers, since this is an
exam.
The total power must equal the power put out by the battery, V I 126 0.15389296 = 19.93 watts.
Indeed it does.
PB = (0.15389296)2

(c, d) What will be the potential difference across bulb A (and B)?
Well, V = IR and we know I and R. Easy.
VA = 0.15389296 756 = 116.343 V
1262
9.6569 V
253
Again, we can sanity check: the sum must equal the batterys voltage, and it does.
VB = 0.15389296

229

14.1.4

Problem 4: Circuit

(a) Find all the currents in the circuit above, where R1 = 16 Ohm, R2 = 1 Ohm,R3 = 3 Ohm, V1 = 10 V,
V2 = 1 V.
Doh. I had to insert a picture for this one. Excuse the size/image quality, if it looks bad. It looks good
here, though.
We can use loop analysis to solve for the currents in this problem. After that, we can calculate voltage
drops across the resistors.
Lets begin by working symbolically.
First, we set up an equation for the left loop, starting at point A, going clockwise around (as an arbitrary
choice). We add when we move over an increase in potential, and subtract when we move over a decrease:
I1 R1 V 1 I2 R2 + V 2 = 0
For the right loop, starting at the middle node (where V2, R2 and R3 meet), going clockwise:
R2 I2 + I3 R3 = 0
We now have two equations, but three unknowns (I1 , I2 and I3 ). We can relate the three with a KCL
equation, at the middle node mentioned above:
I1 + I2 = I3
I1 enters from the left, and I2 from above, while I3 leaves to the right. Alternatively, we can sum currents
entering a node and enter the leaving current as a minus, but we would get the same equation, only with
everything on one side and set equal to zero.
This gives us the nasty answers:
I1 =

(R2 + R3 )(V1 V2 )
R1 R2 + R1 R3 + R2 R3

I2 =
I3 =

R3 (V1 V2 )
R1 R2 + R1 R3 + R2 R3

R2 (V1 V2 )
R1 R2 + R1 R3 + R2 R3
36
A
67
27
I2 =
A
67
I1 =

230

9
A
67
We can now calculate the voltage drops over each resistor:
I3 =

36
576
16 =
V
67
67
27
27
VR2 = I2 R2 = 1 =
V
67
67
27
9
3=
V
VR3 = I3 R3 =
67
67
Adding up the voltage drops along the left loop gives us
VR1 = I1 R1 =

1 +

27
576
+ 10
67
67

which equals zero.


For the right loop, the sum is also zero, as VR2 = VR3 .
Our answers make sense!
(b) The potential at point G is V(G)=0. What is the potential at point A (in Volts)?
Well have to add up the voltage drops we encounter when moving from G to A.
94
27
+1=
V
67
67
If we add VR1 and subtract V1 from that, we get zero, for the outer loop, so all of our answers satisfy KVL,
an excellent sign that they are correct. At the very least, its not as sign that theyre wrong!
VR3 + V2 =

14.1.5

Problem 5: Point charge in a hollow conducting sphere

A very thin hollow conducting sphere has a radius of R = 0.14 m. The center of the sphere is at S. A
charge of q = 8 106 C is present at point P which is located d = 0.06 m from S. Assume that this charge
stays at this position (thus it cannot move). We draw a line from S to P and extend it beyond the radius
of the sphere.
What is the Electric field strength (V/m) at point D which is located on the line from SP at a distance of
` = 0.76 m from S (thus 0.7 m from P)?
Aha! While this looks like it could be very scary due to the non-centered charge, the fact that the shell is
conducting and a sphere makes it almost trivial!
The positive charge inside will cause the inside surface of the sphere to hold a charge of q to cancel
the E-field inside the conductor out. This charge will be non-uniformly distributed, because the charge is
off-center.
However, the sphere was neutral to begin with, so there will be a charge of +q on the outside.
This charge will, amazingly, be uniform. We know from earlier on - or could re-calculate it using Gausss
law - that a charged sphere produces an electric field exactly as a point charge at the center would. Thus,
we can use the good old point charge formula:
E=

Q
40 r2

where Q = +q is the same as the charge inside the sphere, and r is the distance between the center (not
the surface!) and the point D.

231

E=

8 106
124481.34 V/m
40 (0.76)2

Thats our final answer. Since the conductor acts as a Faraday cage, the position of the charge inside is
completely irrelevant! As long as the charge is inside the cavity, the external electric field will be the same
- after it has settled down (which happens extremely quickly after a change in position).

14.1.6

Problem 6: Charges on an equilateral triangle

A charge of q1 = 2 106 C is located at corner A of an equilateral triangle. A charge q2 = 4 106 C


is located at corner B and a charge of q3 = 8 106 C is located at corner C. The distance AB = 0.26 m
(the other two sides of the triangle are the same).
The point P is located exactly in the middle of q1/point A and q3/point B, along the x axis, and exactly
below q2/point B.
(a) What is the x-component of the Electric field (in V/m) at point P which is located half way between
A and C?
Before we start crunching numbers, lets think about this (which is always a good strategy)! The electric
field from each point charge alone is always radially outwards (or inwards, for negative charges). Thus, the
x-component of the electric field from a charge directly above you (xcharge = xpoint ) will be zero! Therefore,
we can treat this as a one-dimentional problem, and completely ignore q2 .
Therefore, the x component is given by
Ex =

q1
q3
+
2
40 (AP )
40 (P C)2

0.26
AC
=
= 0.13 m.
2
2
So, accounting for directions (q1 is negative, and its field is towards the left at point P; q2 is positive, its
force is also towards the left at point P):


8 106
2 106
Ex =
+
5.318085 106 V/m
40 (0.13)2
40 (0.13)2
AP = PC =

(b) What is the y-component of the Electric field (in V/m) at point P?
Similarly as before, due to the location of the point and the charges, we can now completely ignore charges
q1 and q2 , so this should be a piece of cake. All we need to do is to find the distance BP, and plug it into
Coulombs law.
BP is equal to the height of the equilateral triangle. As I dont remember the formula for that, I drew it
up on paper. Via Pythagoras theorem, we get
 2
d
2
h +
= d2
2
4d2 d2

4
4

3
h=
d
2

h2 =

Thus

BP =

3
0.26 m
2

232

Ey =

4 106

709077.12957
40 (0.5 3 0.26)2

Note that since the field is radially outwards from the charge, and P is located below the charge, the field
will be downwards, and so the y-component must be negative. The distance from the charge is greater
than that of the charges in the previous problem, so we expect the magnitude of the field to be quite a bit
lower, which it is.
(c) What is the Electric Potential (in Volts) at point P?
Work, work! Well, at least potential is not a vector, so we dont have directions and such to think about.
The potential at a point is the sum (superposition) of the potential due to each charge alone, as with
Q
):
electric field strength. We just add the potential due to each charge alone (V =
40 r


q1
q2
q3
1
VP =
+
+
40 AP
BP
CP


106 2
4
8

VP =
+
+
574470.61 V
40 0.13 0.5 3 0.26 0.13
(d) What is the Electric Potential Energy (configuration energy) in Joules of this system of 3 charges?
Last one. Well, for this problem anyway. Say the potential at infinity is zero. We start out with nothing,
and so the potential energy is zero. We add charge q1 , and the potential energy is still zero - but the
potential is not. We add a second charge, and the potential energy is now V1 q2 . We add the third charge,
and get a potential energy of V1 q2 from before, plus V1 q3 + V2 q3 from adding the new charge. That is:
U=
Looking at the diagram, r12 = r13 = r23
U=

q1 q 2
q1 q 3
q2 q3
+
+
40 r12 40 r13 40 r23
= 0.26 m, so we can factor out that, too:
1
(q1 q2 + q1 q3 + q2 q3 )
40 0.26

106 106
U=
((2)(4) + (2)(8) + (4)(8)) 0.27661247 J
40 0.26

14.1.7

Problem 7: Capacitor network

Two ideal plate capacitors in air, with capacitance C1 = 4 106 Farad and C2 = 4 106 Farad, are
connected to a battery of voltage V = 10 Volts as shown schematically below. The plates are separated
by a distance d = 0.003 m.
(The diagram is too simple to reproduce: it has two capacitors, C1 to the left and C2 in the middle,
connected in parallel with a voltage source. The upper plates are connected to the positive side of the
voltage source.
(a) What is the total charge residing on the upper plate of capacitor C1 (in Coulombs)? Make sure you
have the correct sign.
Q = CV , so this one is simply
Q = 4 106 10 = 4 105 C

233

(b) What is the total charge residing on the lower plate of capacitor C2 (in Coulombs)? Make sure you
have the correct sign.
Note the lower plate, and of C2 . The magnitude is the same as above (C1 = C2 , as given), but the lower
plate will have the negative charge, so we negate the above answer.
(c) What is the electric field (direction and magnitude) between the plates of capacitor C1 (direction and
magnitude)?
V
. V and d are given, so
d
10
= 3333.33 V/m
E=
0.003
The direction is downwards (from positive to negative).
V = Ed for a parallel plate capacitor, so E =

We now disconnect the battery but leave the two capacitors connected together as shown. After disconnecting the battery, we fill the the entire air gap of the capacitor C2 with a dielectric ( = 4).
(d) Now, what is the total charge residing on the upper plate of capacitor C1 (in Coulombs)? Make sure
you have the correct sign.
OK, so lets think a bit. The battery is disconnected, so the potential difference between the plates is no
longer fixed. It must however be the same for both capacitors.
The charge is trapped on the capacitors, but we have two, and they are still connected, so while Q1 + Q2
must clearly equal the initial total charge on them, it should be able to move around between the two.
After a very long struggle I solved this one. I first wrote down a few things that simply can not change,
period. V1 = V2 due to the connection, so I simply call that V :
Q1
V
Q2
C2 =
V
Q1 + Q2 = Qinitial = V (C1 + C2 )
C1 =

C2new = C2 K
C1new = C1
A0
C1 simply can not change; it is given by
, and none of those change, so it must be constant.
d
With the same argument, C2 must increase by a factor , as the above is true for C2 as well, except wi
insert a dielectric which always increasese capacitance by .
The voltage can and will change, but the total charge cannot. With all this in mind, we set up and solve
this system:
Q1 + Q2 = Qinit
Q1
V
Q2
4 C1 =
V
... with C2 = 4C1 to avoid adding a fourth line. We also have the constraint that V > 0.
This gives us a solution of
C1 =

234

Qinit
5
Q2 = 4Q1

Q1 =

V =

Q1
C1

After we plug in Qinit = V (C1 + C2 ) = 8 105 C, we get


8
1
105 =
C
5
62500
1
32
105 =
C
Q2 =
5
15625
8
1
V = 105
=4V
5
4 106
Q1 =

V
They now ask us for Q1 , Q2 , and the electric field between each set of plates. The electric field is E =
d
for both capacitors, so we plug in V = 4 and get our answer.

14.2
14.2.1

Midterm 2
Problem 1: RL circuit

The switch in the circuit below has been open for a long, long time, R1 = 4.0 Ohm, R2 = 5.0 Ohm,
L = 0.045 H, V = 5 V. The internal resistance of the battery is negligibly small.
Determine the currents I1 , I2 , I3 (in Ampere) in the resistors and in the self-inductor at the moment
(a) just after the switch is closed:
OK, so looking at the circuit (which is not reproduced here), if the switch has been open for a long time,
then the current everywhere is zero.
Just after closing the switch, the inductor current will still be zero, and so I3 = 0.
Because that part of the circuit is essentially an open circuit, we have a series circuit with the battery, R1
and R2 :
I1 = I2 =

5
V
= A
R1 + R2
9
I3 = 0

(b) a long time after the switch is closed.


A long time after, the current in the inductor will be at a maximum, limited only by the battery and R1 .
R2 will be completely bypassed by the inductor, which now looks like a short circuit, and so I2 = 0, and
I1 = I3 .
V
5
= A
R1
4
I2 = 0

I1 = I3 =

235

14.2.2

Problem 2: Non-conservative fields

Two voltmeters, Vright and Vleft , each with an internal resistance of 106 are connected through wires of
negligible resistance (see the circuit below). The + side of both voltmeters is up as shown. A changing
magnetic field is present in the shaded area.
At a particular moment in time Vright reads 0.7 Volt (notice the - sign).
(a) What, at that moment, is the magnitude of the induced EMF (in Volts) in the circuit?
OK. The circuit diagram is very simple: two voltmeters, side by side, connected to each other. So both
have the plus side upwards, and then the + sides are connected at the top, and the - sides connected at
the bottom. Theres a changing magnetic flux in the middle of the loop.
Lets not fool ourselves here; we will use Faradays law, and map out the circuit based on the electric fields
in it, not based on Kirchhoffs rules, which dont apply here!
If we start at the bottom node, going clockwise, we first encounter Vright , which measures 0.7 volts,
meaning the current though it goes into the bottom and out of the top, or else it would display a positive
value. Thus, the current is going counterclockwise in the loop.
We now know the voltage across the voltmeter (which is not really a voltage drop, though, but an induced
EMF), and the resistance, so
I=

V
R

0.7
= 7 107 A
6
10
6
Since the total resistance of the loop is 2 10 ohms, and Ohms law holds, the total EMF is
I=

V = IR = 7 107 2 106 = 1.4 V


Question (a) wants the magnitude of this, so 1.4 volts.
(b) At that moment in time, what is the reading of Vleft ? (make sure you have the correct sign!)
That voltmeter must have a a current of the same magnitude through it, but it is positive (as it goes into
the positive terminal), and so
Vlef t = IR = 7 107 106 = 0.7 V

14.2.3

Problem 3: Bainbridge mass spectrometer

A Bainbridge mass spectrometer is shown in the figure. A charged particle with mass m, charge
|q| = 4.8 1019 C and speed v = 3 106 m/s enters from the bottom of the figure and traces out
the trajectory shown in the fields shown. The only electric field E = 10 103 V/m is in the region where
the trajectory of the charge is a straight line.
(a) When the particle is moving through the first (straight-line) segment of its trajectory, what is the
magnitude of the magnetic field B in Tesla?
The particle moves towards the left in a magnetic field pointing into the page, so it must be positively
charged. The E-field is towards the right, and so for the particle to move in a straight line, the magnitude
of the electric force and the magnetic force must be equal.

236

The E-field is uniform between the plates, at 10 103 V/m (given above); alternative usits would be N/C,
which will be more useful here. The electric force on the particle is thus
FE = qE = q 10 103 = 4.8 1019 10 103 = 4.8 1015 N
... and the direction is, then, to the right. The strength of the magnetic force on the particle must equal
this, only that the magnetic force will be towards the left. The magnetic force is given by
FB = q(v B) = qvB
... since the B-field is perpendicular to v, sin = 1, and we get a nice, simple equation. Solving for the
B-field strength, we find
FB = qvB
B=

FB
qv

Since FB must be equal to the electric force (in magnitude), we find


B=

FE
4.8 1015
=
= 0.0033 T
qv
4.8 1019 3 106

as the answer to (a).


(b) The charge hits the left wall of the spectrometer at a vertical distance h = 0.157 m above where it
entered the upper region and a horizontal distance L = 0.374 m to the left of where it entered the upper
region (see sketch). What is the radius r of the trajectory in m?
Well, first, lets have a guess at the rough size of the answer. Via the sketch, r is slightly larger than L/2
(L is a bit smaller than the diameter of the circle the particle would trace). So if we find r 0.2 then
that would look about right.
We know that
R=

mv
qB

... but we dont know R and we dont know B (the B-field above is for another region of the device!), so
we cant use that as-is.
Looking at the diagram, we can simply use the Pythagorean theorem to find r:
h2 + (L r)2 = r2
h2 + L2 2Lr = 0
h2 + L2 = 2Lr
h2 + L2
r=
2L
We stick our values in and find r 0.219953 m.
(c) The mass of the particle can be determined using the radius r, the charge q, the speed v, and the
magnetic field B0 . Using a value of B0 = 0.2 T, evaluate the mass of the particle in kg. (Note that the
magnitude of the field in the curved section, B0 , is NOT the same as the magnitude in the straight section,
B, found in part a).

237

Now we can use the radius equation! We just need to solve it for m.
R=
m=

mv
qB0
qRB0
v

With our values, m 7.0385 1027 kg.

14.2.4

Problem 4: RL circuit

In the circuit shown in the figure below, the switch closes at t = 0, R = 2.0 Ohm, E = 7 V, L = 0.085 H.
(a) What are the currents (in A) through the two bottom branches at t = 0+ (just after the switch is
closed)?
As in the previous (very similar) circuit, I1 = 0 at this point, because the inductor wont allow any current
to pass through without at least some time passing.
I2 is then depends only upon the EMF and the resistances 2R + R = 3R, so
7
E
= A
3R
6
(b) What are the currents (in A) through the two bottom branches at a much later time t ?
I2 =

In this case, the current through the inductor has reached its maximum, and it is a short circuit. Therefore
I2 = 0, and
I1 =

14.2.5

E
7
= A
2R
4

Problem 5: Magnetic field of a loop

A current I = 4.9 A flows around a continuous path that consists of portions of two concentric circles of
radii a and a/2, respectively, where a = 1 cm, and two straight radial segments. The point P is at the
common center of the two circle segments.
Calculate the components of the magnetic field (in T) at point P.
Okay. So neither of the straight wire segments contributes to the B-field at point P, so we can neglect
them completely, and focus on calculating the B-field due to the two arcs.
The field ought to be in the +
z direction alone, so I would expect the x- and y-components to be zero.
More on that later.
Well, lets see then. We use Biot-Savart, which says that
~ r)
~ = 0 I (d`
dB
4 r2
~ is a segment along the current-carrying wire, always aligned with the currents direction.
... where d`
Since we need to calculate the B-field due to two arcs, and then vectorially add the two, we will do this in
a general way, and solve it for an arc of radius r, and then substitute in r = a and r = a/2 later on.
~ r| = d`, since they are always
Since an arc is part of a circle, we have the wonderful properly that |d`
exactly perpendicular, and the magnitude of r is 1 by definition. The direction is given by the cross
product, and will be z (out of the page) for all d`.
238

~ r = |d`||
~ r|sin = d` 1 1 = d`
d`
z
So we now have
~ = 0 I d`
dB
z
4 r2
r is the distance between all points on the wire, and point P, so we dont need to make any substitutions
there. (We will later, when we are done with the integration.)
Via arc length formulas we find that d` = rd, so
~ = 0 I rd
dB
z
4 r2
Thus, we can integrate:
~ =
B

~ =
dB

arc

Z
0

0 I
0 I
rd
z
=
4 r2
4 r

d
z
0

0 I
~ = 0 I
B
z=
z
4 r
4r
This resembles the B-field in the center of a full circle - in fact, its exactly half of that! So that makes
sense.
Lets now make the substitutions, then. We have one arc with radius r = a/2 where the current goes
upwards (in the plane of the page, so +
y at the center), which makes the B-field go out of the page.
Another arc of radius r = a has the current going down at the center, which contributes to a B-field going
into the page.
Since the current is the same, and the one with radius a/2 is closer to the point, the net B-field will point
out of the page (the closest one will win). So we have then, that
0 I
0 I
~ = 0 I z + 0 I (
z) =
z
z
B
2a
4a
2a
4a
~ = 0 I z
B
4a
So the components, and our answers, are
Bx = 0 T
By = 0 T
Bz =

14.2.6

4107 4.9
0.000153938 T
4 0.01

Problem 6: Magnetic field of a current-carrying ribbon

Consider a thin, infinitely long conducting ribbon that carries a uniform current density j (current per
unit area). The width of the ribbon is w and its thickness s is extremely small (s  w). P is a point in
the plane of the ribbon, at a large distance (x  s) from the ribbon edge. (See the figure below)
What is the magnitude of the magnetic field B (in T) at point P for the following values of w , j, s and
x? w = 6 cm; s = 0.1 cm; j = 1 A/m2 and x = 16 cm.
Oh my. Well... Lets see. If we divide the cross-sectional area s w into pieces of s dr (dr to avoid
ambiguity with dw and w - r is the distance between the piece and the point P) and integrate the B-field
due to each current j s dr , will that be enough? Or do we need to do a double integral where we consider
239

ds and dr?
Since s  x, and s is described as extremely small, I will in fact assume that we ignore the thickness,
and treat it as essentially 0. That means that we wont bother calculating the B-field due to the top layer,
the bottom layer, and an infinite amount of layers in between, but indeed treat it as a 1-dimensional set
of currents, each at a different distance from the point P , with the closest being r = x and the furthest
being r = x + w.
Since the question only asks for the magnitude, as a single number, we wont have to worry about directions and such.
We can find each current i (I wont call it di since we wont integrate it to find the full current) as
i = j dA = j s dr
At this point, I started working with Biot-Savart... and realized that it wont work. I got stuck when
I thought I could treat the system as an infinitely short (depth-wise) wire, and realized that I need to
integrate d` in the direction in which is is 0... Doh!
Well, Amperes law can work instead, actually. To use it, we treat the system as a set of infinite wires,
stacked width-wise. So each wire has area s dr, but since s is very small, we can neglect it (via the problem
description), and we get a set of infinite wires of zero radius, where Amperes law applies just fine for
the really rectangular shape.
Lets then look at the B-field due to one such wire. Clearly,
it will be equal at all equidistant points
H
~ d`
~ = dB2r. Since there is no changing
from the wire, so that we can use Amperes law and get dB
electric flux (and point P is in empty space), we use the simple version of Amperes law:
dB2r = 0 Ipen
What is Ipen here? Well, we choose the Amperean circle with the radius that equals the distance to the
point P , and attach a flat surface to that, so it is the full current j s dr though the tiny wire segment.
That means that we have
dB2r = 0 j s dr
0 j s dr
2r
Well, the dimensions work out to be tesla, so this looks promising.
We can now integrate the above, with r (the distance between the wire and P) x to x + w.
Z
Z x+w
0 j s dr
0 j s x+w dr
B=
=
2r
2 x
r
x


0 j s
x+w
B=
ln
2
x
dB =

This does indeed give a green checkmark for the answer. With the given values we find B 6.36907 1011
tesla.

14.2.7

Problem 7: Magnetic field of a rotating charged sphere

A spherical shell of radius R carries a uniform surface charge density (charge per unit area) . The center
of the sphere is at the origin and the shell rotates with angular velocity (in rad/sec) around the z-axis
(z = 0 at the origin). Seen from below, the sphere rotates clockwise. (See the figure below)

240

(a) Calculate the magnitude of the total current (in A) carried by the rotating sphere for the following
values of , and R:
= 5 104 C/m2 , = 8 rad/s and R = 1 m.
I managed to get a correct answer on this on the test, despite using incorrect methods to solve it. Now
that Ive realized two things I did wrong, Ill try again and see whether I can find the correct answers (not
just something close, but the actual answer!) or not.
First, we will model this as a set of circular (with an infinitesimal height) current loops. That is, we cut
the sphere up in the x-y plane, such that only extremely thin slices remain. If the radius of the sphere is
R, and we call the radius of each circle r, we will find via trigonometry that
r = R sin
where is the polar angle (the angle from the z axis). Clearly, each circle will have a different radius,
depending on where it is on the sphere. Only the one centered at the origin (the great circle) will have
r = R, while ones closer to the poles will approach 0 radius.
Thats one dimension; to find an area element, we need to multiply that by a d or something or another.
Well, we know that arc length is given by Rd, so that the circumferance of the sphere (or of the spheres
great circle, rather) is 2R; for half the sphere, its simply R, etc. So the angle in radians times the
radius.
It really isnt any harder than that, so we have
dA = 2r(Rd)
= 2R2 sin d
As a sanity check, lets integrate that over the entire sphere, and see if we find A = 4R2 . If we dont, it
is clearly incorrect.
Z

2R2 sin d
0
Z
2
= 2R
sin d

A=

= 4R

Excellent! Let us continue with the physics.


First off, part (a): the total current. This can be found easily: current is charge moving past a point
per unit time, and the period of each ring of current will be the same, as they are rotating together.
2
Therefore, the total current is the total charge, 4R2 , divided by the period T =
, or

Q = 4R2
2
T =

2
Q
4R
I=
=
= 2R2 = 0.008 A
T
2
We can also do this the proper way to prove that the above is correct. In this case, we use the area
element dA we found, and integrate it over the surface area, while multiplying by , and dividing by the
period.
An infinitesimal current di is given by
241

dA
dA
(2R2 sin d)
=
=
= (R2 sin d)
T
2
2
Integrating that over the sphere, the total current is
di =

R2 sin d
0
Z
2
sin d
= R

I=

= 2R2
... which is exactly the equation we found above, so our answer of 8 mA looks correct!
(b) Calculate the magnitude of the magnetic field B(z) (in T) that is generated by the circular current of
the rotating shell at a point P on the z-axis for the following values of ,, z and R:
(same values, plus z = 2.3 m)
Okay. Since we are considering the currents as circular loops with an infinitesimal height, we can use
Biot-Savart to find the B-field along the axis of one such loop. We have done that before, so I will simply
refer to the result. It says that
Bz =

0
IR2
2 (R2 + z 2 )3/2

Since we are integrating a whole set of such loops, I will refer to that as dB instead. In the above, R refers
to the radius of the circular loop, which is given by little r = R sin in our case.
z is the distance from the center of the current loop to the point P where we find the B-field (lets call
that Pz , as in the z-coordinate of point P), so we need to substitute in an expression for that distance for
z. We get
dB =

di r2
0
2 (r2 + (Pz z)2 )3/2

Substituting in the value for di we found, we have


dB =

0 (R2 sin d) r2
2 (r2 + (Pz z)2 )3/2

Ugh, so now we have both z and as variables, plus r (which is R sin ). Still, two is one too much. We
need to rewrite in terms of z or vice versa; I chose to use z = R cos (conversion from cylindrical to
spherical coordinates) here. That gives us, if we also replace r = R sin at the same time,
dB =

0
(R2 sin d) (R sin )2
2 ((R sin )2 + (Pz R cos )2 )3/2

What a mess! We can do some of the squares, to simplify:


dB =

0
R4 sin3 d
2 (R2 sin2 + (Pz R cos )2 )3/2

Hardly pretty, but we can actually integrate this now; we only have constants and one variable, . Of
course, doing this integration manually would be madness. Doing it symbolically, using software, would
probably also be madness. We can do a numerical integration, however, by giving the software the values
for all the constants, and telling it to integrate numerically. With Mathematica, this is done by setting
the constants, and then using the NIntegrate function.

242

Using NIntegrate, or something similar, we evaluate the following integral:


Z
0
R4 sin3 d
B=
2 (R2 sin2 + (Pz R cos )2 )3/2
0
With constants moved outside the integral:
Z
sin3 d
0 R4
B=
2
2
2 3/2
2
0 (R sin + (Pz R cos ) )
Solving the integral gives us B 2.75419755 1010 T, which is correct! (Not only accepted as correct i.e.
within 5%, but as far as the answer is given, looks to be exactly correct.)

14.3
14.3.1

Midterm 3
Problem 1: RC Circuit

Here we go.
The circuit below consists of three identical resistors each with resistance R = 21 Ohm, two identical
batteries with emfs E = 17 V, and a capacitor with capacitance C = 551F. The capacitor is initially
uncharged at t = 0.
The first sub-questions ask for i1 , i2 , i3 and the capacitors voltage after a very very long time t  RC.
Well, the capacitor will act as an open circuit after such a long time. Thus, the entire left loop will at as
if it is disconnected, so i1 = 0. We can redraw the circuit with only the right loop, and find an extremely
simple circuit:
i1 = 0
17 V
E
=
0.40476 A
i2 = i3 =
2R
2 21
For the rest, we can do turn the circuit into its Thevenin equivalent.
First, we find the open-circuit voltage for the port where the capacitor is located. That is, we remove the
capacitor, and then analyze the circuit that remains.
The left-hand loop becomes disconnected, and the voltage is decided by the right-hand loop alone.
We can either use KVL around the loop, or simply see that the circuit is a voltage divider, with R1 =
R2 = R, so the voltage across the middle resistor is half of E. In addition to that, we have the leftmost
voltage source, which is upside down (the way I think of it), that is, its voltage is positive downwards.
We go around the left loop (never mind that theres a hole where the capacitor was, we include that),
clockwise, starting in the bottom middle (below the center resistor):
i3 R E + VT H = 0
VT H = E + i3 R
VT H = 17 + 8.5 = 25.5 V
VT H = 25.5 V
Next, we need to find the equivalent resistance into that port. We turn off the voltage sources by turning
them into shorts (current sources, if any, are turned into open circuits), and redraw the circuit. It becomes
obvious that the equivalent resistance is
Req = R + (R||R) = R + (
243

R2
) = 1.5R
2R

So we can now analyze a much simpler circuit: a series circuit of a voltage source (25.5 volts), a resistor
(1.5 21 = 31.5 ) and the capacitor.
After a long time, clearly, the capacitors voltage will equal the batterys, or there would still be a current
running. Therefore,
VC (t ) = 25.5 V
(d) Assuming that the capacitor starts uncharged, how long will it take (in seconds) for the voltage across
the capacitor to reach 3/4 of its maximum value?
We can use the same circuit, and the equation


t
)
VC (t) = VT H 1 exp(
RT H C
Set it equal to three-quarters VT H and solve for t...

VT H 1 exp(

t
RT H C


)

3
= VT H
4


3
1 exp(
) =
RT H C
4
1
t
)=
exp(
RT H C
4
t
1

= ln
RT H C
4
1
t = RT H C ln
4
t = RT H C ln 4 0.02406 s

14.3.2

Problem 2: RLC circuit

A solenoid has N = 470.0 turns, length d = 10 cm , and radius b = 0.4 cm, (b  d). The solenoid is
connected via a switch, S1 , to an ideal voltage source with electromotive force E = 5 V and a resistor with
resistance R = 49 Ohm . Assume all the self-inductance in the circuit is due to the solenoid. At time
t = 0, S1 is closed while S2 remains open.
(a) When a current I = 1.02 102 A is flowing through the outer loop of the circuit (i.e. S1 is still closed
and S2 is still open), what is the magnitude of the magnetic field inside the solenoid (in Tesla)?
All right. So we dont need to do any circuit analysis for this step, but we do need to calculate the B-field
inside the solenoid (and then the inductance). We could use a known result, but I will derive it using
Amperes law.
We choose an Amperean loop, in the form of a rectangle, placed such that one side (length c) is inside
the solenoid; the other sides dimension is irrelevant, if it is long enough that the second length c side is
outside the solenoid.
The basic version of Amperes law (we wont need the displacement current term, as the change in electric
flux will be 0) tells us
I
~ = 0 Ipen
~ d`
B
In a solenoid, the B-field is approximately constant inside, and approximately 0 outside. It will be completely parallel to the length c side, but completely perpendicular to the other side. Therefore, only one
244

side of the loop contributes to the left side: the one inside.
We attach a flat surface to this loop, to find the current that penetrates it. The current may/will penetrate
multiple times, since N is large. It will penetrate N times, multiplied by the ratio of the loops length to
the solenoids:
c
d
The left-hand side gave us c for the line integrals length:
Ipen = IN

cB = 0 IN
B = 0 I

c
d

N
d

This result looks familiar, except d is usually called `.


We can now answer the first sub-question, of the B-field:
470
N
= 4107 1.02 102
6.024 105 T
d
0.1
(b) What is the self-inductance L of the solenoid (in H)?
The definition of self-inductance is the flux, divided by the current:
B = 0 I

L=

B
I

Which also means that


B = LI
dB
dI
=L
dt
dt
(keeping in mind to multiply by N to find the total flux, as needed).
In other words, we need to find the flux inside the inductor. To do so, we attach a surface to the inductors
loop - a very weird surface (a helical surface), but we can find the flux easily:
B = N AB
... where A is the cross-sectional area of the solenoid, and N of course the number of windings.
B = N b2 0 I

N
d

B
N2
= b2 0
I
d
L 0.000139533 H

L=

(c) What is the current (in A) in the circuit a very long time (t  L/R) after S1 is closed?
At that point, I = ER (since the inductor has no resistance in this model).
5
E
=
0.102 A
R
49
(d) How much energy (in J) is stored in the magnetic field of the coil a very long time (t  L/R) after
S1 is closed?
 2
1 2 1
5
U = LI 0.000139533
7.26432 107 J
2
2
49
I=

245

For the next part, assume that a very long time (t  L/R) after the switch S1 was closed, the voltage
source is disconnected from the circuit by opening switch S1 . Simultaneously, the solenoid is connected to
a capacitor of capacitance C = 851F by closing switch S2 . Assume there is negligible resistance in this
new circuit.
(e) What is the maximum amount of charge (in Coulombs) that will appear on the capacitor?
Hmm. We will have an LC circuit, since the rest is disconnected.
Since there is stored energy to begin with (in the inductor), and no energy is lost to resistance, the circuit
should oscillate forever.
1 Q2
, but we
I dont see an easier way to solve this (actually, for this question, we could probably use U =
2C
need some other solution for the next quetion), so lets look at setting up the differential equation using
Faradays law:
I
~ d` = dB
E
dt
First, we go around the circuit starting at the top (arbitrarily), going clockwise - in the direction of the
initial current.
The inductor doesnt contribute to the line integral, as there can be no electric field inside it, having no
resistance.
For the capacitor, we find VC = Q/C (C = Q/V , V C = Q, V = Q/C); charge will build up on the bottom
plate for this current direction, and so the E-field is is the same direction we are moving, and
dI
Q
= L
C
dt
We can use I =

dQ
:
dt

d2 Q
Q
= L 2
C
dt
We use the initial conditions Q(t = 0) = 0 (no charge on the capacitor) and Q0 (t = 0) = I0 (initial current)
to solve the differential equation, and find (I used Mathematica)



t
Q(t) = I0 LC sin
LC
That can easily answer the question, since the sine will move between -1 and +1. The maximum charge is

E
5
Qmax = I0 LC =
LC
0.000139533 851 106 3.516 105 C
R
49
(f) How long does it take (in s) after S1 is opened and S2 is closed before the capacitor first reaches its
maximum charge?
t
t
That would be when sin
= 1, or when
= /2.
LC
LC
t

=
2
LC

LC
t=
2
t 0.000541281 s
BTW (they didnt ask for it), the current is given by the derivative of the charge, of course, so


t
I(t) = I0 cos
LC
246

14.3.3

Problem 3: Energy flow of a capacitor

Consider a charging capacitor made out of two identical circular conducting plates of radius a = 9 cm.
The plates are separated by a distance d = 10 mm (see figure below, note that d  a). The bottom plate
carries a positive charge


t
Q(t) = +Q0 1 +
T
with Q0 = 2 106 C and T = 0.006 sec, and the top plate carries a negative charge Q(t). The current

through the wire is in the positive k-direction.


You may neglect all edge effects.
(a) Calculate the components of the electric field (in V/m) inside the capacitor for t = 2T .
so they clearly want us
Three questions (this being one) want the answers as components in r, and k,
to use cylindrical coordinates for this one.
As for the electric field, that is given by
E=

0

... in the direction from the positive to the negative charge, so +k in this case. can be found by dividing
the total charge by the plate area:
=

Q
Q
= 2
A
a

So
Q

=
0
0 a2
For the change at t = 2T , using the equation, we find


2T
Q(t = 2T ) = Q0 1 +
= 3Q0
T
E=

We find then,
Er = 0 V/m
E = 0 V/m
Ek =

Q
3Q0
=
2.66298 107 V/m
2
0 a
0 (0.09)2

... that seems insane, as it is several (not far from 9) times higher than the maximum possible E-field in air,
before breakdown occurs... I keep getting the same answer though, after having double- and triple-checked
my calculations! Lets see...
A0
(0.092 )(8.854187 1012 )

2.25 1011 F
d
0.01
Q
6 106
V =
=
266666.67 V
C
2.25 1011
Well, the results are in line with everything else. I still find them bizarre, though!
I actually posted about this on the wiki, to ask whether unphysical values could be valid, and the answer
was yes, they didnt choose the values to avoid the breakdown limit. So the above is most likely correct
C=

247

after all.
(b) Calculate the components of the magnetic field B(r, t) (in Teslas) at time t = 2T inside the capacitor
at a distance r = 0.9 cm from the central axis of the capacitor.
First, as a sanity check, lets calculate the B-field outside the wire. Using Amperes law (without the
displacement current term),
B2r = 0 I
0 I
2r
B 2.222e 5 I
B=

The current is given by the derivative of Q at t = 2T , which is constant at Q0 /T (where T = 0.006, given
early in the problem), so I 3.333 104 A.
B 7.405 109 T
... for the B-field outside the wire. I do believe this is equal to the answer (it turned out it wasnt!), but I
will calculate it using the displacement current term inside the plates, to do things right.
The equation we need is then
I



d
E
~ = 0 Ipen + 0
~ d`
B
dt

Ipen = 0 since we will be mid-air (or mid-vacuum). We again to around a circle with radius r = 0.9 cm, so
dE
dt
0 0 dE
B=
2r dt
OK, time to calculate the change in electric flux. First, the flux itself. Since r is much smaller than the
plate radius, we are inside the plates. Only the flux through our Amperean loop matters, not the flux
through the entire plate area.
B2r = 0 0

We attach a flat surface to our Amperean loop, and find


E = EA =

2
Q
r =
r2
0
0 a2

Qr2
0 a2
So the bigger our loop, the stronger the flux. Makes sense. Its also inversely proportional to the plate
area, as the E-field depends on the charge density - lower density (larger plate area) equals less flux per
area. That also makes sense.
We then need to find the derivative of this. Well, thats extremely easy - we have a bunch of constants
multiplying Q(t), so we get the same constants times dQ/dt = I:
E =

dE
Ir2
=
dt
0 a2
And so the B-field is
B=

0 0 Ir2
0 Ir
=
2r 0 a2
2a2
248

... which matches a result we found in lecture many weeks ago.


With our values, we find
B 7.40667 1011 T
... which is certainly not equal to the value we calculated previously. The values are equal at the very
edge of the plates (r = a), but not inside. (This equation cant describe anything outside the plates, so
we cant check thete without recalculating.)
Interesting. Still, lets move on, as this certainly should be correct, despite not being what I expected.
With the current going
The direction of the magnetic field will be azimuthal, i.e. in the direction .
upwards, we use the right-hand rule to find the B-field curling counterclockwise, as seen from above. Im
not sure about the sign, but it appears to be positive if curling counterclockwise.
(The unit vector is positive inwards, to the right of the drawing, but to the left of the k and r vectors.
Slightly confusing, but it appears common to have azimuth positive when counterclockwise seen from
above.)
~ t) (in W/m2 ) at time t = 2T between the plates
(c) Calculate the components of the Poynting vector S(r,
at a distance r = 0.9 cm from the central axis of the capacitor.
~
~
~ = EB
S
0
Looks like we need to do a cross product in cylindrical coordinates. The Poynting vectors direction will
be given by
S = k =
r
because
k = r
The magnitude will be the product of the E- and B-fields, divided by 0 , as weve seen:
~ =
|S|

EB
QIr
= 2 4
0
2 a 0

For our values, at r = 0.9 cm:


(3 2 106 )(3.333 104 )(0.9/100)
~
|S| =
1569.57 W/m2
2
4
2 0.09 0
The direction is mentioned above, and the other components will be zero. Keep the direction in mind
however, as the value must be entered as negative!
(d) What is the flow of energy (in W) into the capacitor at time t = 2T ?
This is the Poynting vector, times the surface area that the Poynting vector works over, 2ad (r = a at
the outer edge, so we need to take the different B-field in mind and cannot use the previous numerical
values!):

dU
QIr
QId

= SA = 2 4 2ad
= 2 88.766 W
dt
2 a 0
a 0
r=a
(e) How fast is the energy stored in the electric field changing (i.e. what is the rate of change in W)
within the capacitor at time t = 2T ?
Hmm. The stored energy per unit volume is given by

249

1
1 2
UE = 0 E 2 = 0 2
2
2 0
UE =

1 2
2 0

UE =

1 (Q/a2 )2
2
0

UE =

1 Q2 / 2 a4
2
0

UE =

1 Q2
2 0 2 a4

We multiply by the volume of the capacitor, a2 d, to find the total stored energy:
U=

1 Q2
a2 d
2 0 2 a4

1 Q2 d
2 0 a2
The rate of change in energy stored is then the derivative of that:
U=

QdI
dU
=
dt
0 a2
For our values, we find
(3 2 106 )(0.01)(3.333 104 )
dU
=
88.766 W
dt
(8.854187 1012 )(0.09)2
This answers both (d) and (e), though we calculated it as the answer to (e). Thus, the energy flow into
the capacitor is equal to the growth of the stored energy. That makes quite a bit of sense!

14.3.4

Problem 4: Electromagnet with small air gap

An electromagnet has a steel core (M 2500) with an approximately circular cross sectional area of
6.0 cm2 . The radius of the magnet is 12.0 cm; there is a small air gap of only 2.4 mm (see sketch). The
current through the magnets N = 140-turn coil is 20 A.
What will the magnetic field strength be inside the air gap? Express your answer in Tesla. Assume that
the magnetic field is azimuthal (i.e. points around the circle formed by the steel) everywhere and d  the
radius of the electromagnet.
The magnetic field lines will approximately follow the circle (which is also stated in the description), even
in the gap, since magnetic field lines must be continuous and connect back. They dont have a starting
or ending point.
We use Amperes law around the magnet:
I
~ = M 0 Ipen
~ d`
B
Choosing our Amperean loop to be a circle, concentric with radius of the magnet, and attaching a flat
surface to it, we find Ipen = N I.
As for the line integral, we need to calculate it in two parts: in the steel, and in the air part. Due to the
massive difference in permeability, ignoring the air part is not an option.

250

~ inside the steel is roughly the B-field times the length, so that contributes with a term B(2R d)
~ d`
B

(since there is no steel in that last part).


For
Z the air, the same deal applies, except the permeability is waaay lower, and we need to adjust for that:
~ = BM d
~ d`
B
All in all, we have
B ((2R d) + dM ) = M 0 N I
B=

M 0 N I
(2R d) + dM

B 1.30287 T
An alternate way to look at Amperes law is to rearrange it:
I
1 ~ ~
B d` = 0 Ipen
M
This is of course the same thing, but now its easy to see that we need to use the different permeability
values in the separate line integral parts, one with a high M and one with M 1.

14.3.5

Problem 5: RLC Circuit

A circuit consists of a resistor of R = 3 , a capacitor of C = 6F, and an ideal self-inductor of L = 0.06


H. All three are in series with a power supply that generates an EMF of 16 sin(t) Volt. The internal
resistance of both the power supply and the inductor are negligibly small. The system is at resonance.
(a) What is the time averaged power (in Watts) generated by the power supply?
At resonance, the circuit acts exactly as though the inductor and capacitor werent there (in steady state,
that is). The time-averaged power is thus the same as it would be if we had a voltage source-resistor circuit.
V (t)
(since we are at resonance), so
R



16 sin t
hP i = hV Ii = (16 sin t)
3

The current will be given by I(t) =

162
2
sin t
3
The time-average of a sine-square function is one-half, which means that
hP i =

hP i =

162
42.667 W
6

V2
162
=
, which it indeed is!
R
3
1
We decrease the frequency of the power supply to a value for which the reactance (
L) becomes
C
equal to R; the maximum EMF remains 16 V.
This should be half of what we would find using the DC method of P =

(b) What now will be the time averaged power (in Watts) generated by the power supply?
We will now have to use the full equation for the RLC circuit:
I(t) =

V0
sin(t )
R2 + X 2
251

where


= arctan

X
R

1
, the negative of what we were given), we find = /4, and
With X = R (because X = L
C
therefore
16
I(t) =
sin(t + /4)
2
3 + 32
16
I(t) = sin(t + /4)
3 2
16
P (t) = (16 sin t) sin(t + /4)
3 2
162
P (t) = sin(t + /4) sin t
3 2
Now we just need to find the time average. We can find it by integrating the power over one period, and
then dividing by the period:
Z
1 T
hP i =
V (t)I(t) dt
T 0
2
, so
T =

Z T
162

sin(t + /4) sin t dt


hP i =
2 0 3 2
Z
162 T
hP i =
sin(t + /4) sin t dt
6 2 0


162

hP i =
6 2 2
162
21.333 W
12
So the power is exactly half of what it were earlier. Quite a lot of work to find a factor of one half!
hP i =

14.3.6

Problem 6: Electromagnetic wave

The magnetic vector of a plane electromagnetic wave is described as follows:


~ = Bm x sin(12 rad/m z 3.60e9 rad/sec t)
B
where Bm > 0. This represents the full magnetic field, so that By = Bz = 0.
(a) What is the wavelength of the wave, in meters?
(b) What is the frequency f of the wave, in cycles per second?
(c) In which direction does this wave propagate?
OK. We can use some simple formulas for these few.
2

2
=
= m
k
12
6

3.6 109
f=
=
Hz
2
2
=

252

As a sanity check, f = c (in a vacuum), which is true here.


The direction of propagation is +
z , because we have a kz in the sine, and a minus t; the direction is
opposite the sign.
~ x, t) can be written as
(d) The associated electric field E(~
~ = A0 sin(kx x + ky y + kz z t)m
E

where kx , ky , kz and are all positive (or zero).


Determine m, A0 (in V/m), kx , ky , kz (in rad/m) and (in rad/s), assuming Bm = 9.73 107 T.
The EM wave propagates in the +
z direction, and so the E-field must do the same. Therefore kx = ky = 0.
It needs to be in phase with the B-field, so kz = 12 rad/m and = 3.6 109 rad/s.
E0 = B0 c
, only that here we call them by other names, so
A0 = Bm c = 9.73 107 3 108 291.9 V/m
= v must be true, so in this case, E x = z; therefore the E-field must
And lastly, the direction. E B
be in the
y direction, so we need to set A0 to a negative value (since there are only x, y and z choices
for the direction).
(e) What is the time-averaged Poynting flux associated with this wave, assuming Bm = 9.73 107 T?
~
~
~ = E B ). As for the
The Poynting vector is in the direction of propagation (since it is given by S
0
magnitude, we will have to take the time average of the above, and find
D E AB
~ = 0 m z
S
20

14.3.7

Problem 7: Radiation pressure on the Earth

The average energy flux in the sunlight on the Earth is


hSi = 1.4 103 W/m2
You might need to use some of the following constants:
Distance from earth to the sun = 150 109 m
REarth = 6.4 106 m
RSun = 7.0 108 m
G = 6.67 1011 m3 /(kg s2 )
MEarth = 5.97219 1024 kg
MSun = 1.9891 1030 kg
(a) What force (in N) does the pressure of light exert on the Earth? Assume that all the light striking
the Earth is absorbed.
We can find the radiation pressure by dividing the time-averaged Poynting vector by c:

253

2
hSi
1.4 103
6 W/m
=
= 4.6667 106 Pa
= 4.6667 10
8
c
3 10
m/s

We can then multiply this by a number , which depends upon whether the radiation goes through ( = 0
for a fully transparent material), is absorbed ( = 1 for 100% absorption) or reflected ( = 2 for 100%
reflection). In this case, = 1, so the number above is the one we need.
We then multiply this by the Earths cross-sectional area (as if it were a circle, facing the Sun) - I previously (during the lecture sequence) solved this by integrating it as a cross product, which gave me exactly
the 0.02 N that was marked as correct in that case, but it turned out the correct answer was found using
R2 , and the answer was then rounded up to 0.02.
Therefore, Ill use the R2 method here, of course.

F = pA = 4.666 106 (6.4 106 )2 6.004 108 N


Wow, thats much more than I would have guessed. Well, the Earths radius is huge compared to the
other example of an asteroid with a 100 meter radius, so it makes sense. Not to mention that it goes with
the radius squared.
(b) What is the gravitational force (in N) that the Sun exerts on the Earth? (Think about how that
compares to the force due to the pressure of light. Does your answer make sense?)
Time for Newton to shine.
Fg = G

24
30
m1 m2
11 (5.97219 10 )(1.9891 10 )
=
6.67

10
3.52155 1022 N
r2
(150 109 )2

Whoa, that was also larger than I would have expected. Still, it should be way higher than the radiation
pressure, and indeed it is.

254

14.4
14.4.1

Final Exam
Problem 1: Loop in a Magnetic Field

An infinite straight wire carrying a current I = 10 A flowing to the right is placed above a rectangular
loop of wire with width w = 6 cm and length L = 15 cm, as shown in the figure below. The distance from
the infinite wire to the closest side of the rectangle is h = 2.3 cm. The loop of wire has resistance R = 0.36
Ohm.

(a) What is the magnitude (in Tesla) of the magnetic field due to the infinite wire at the point P in the
rectangular loop, a distance r=4.7 cm from the wire (see figure).
Okay. Well, first, we can completely ignore the rectangular loop for this first question. All we need to
know is the current and the distance.
Using Amperes law, we can find the B-field at point P:
I
~ = 0 I
~ d`
B
B2r = 0 I
0 I
B=
0.0000425532 T
2r
The B-field is circular. It points into the screen at point P (and at all other points below the wire), and
out of the screen above the wire.
(b) Calculate the magnitude of the magnetic flux (in Tesla m2 ) through the rectangular loop due to the
magnetic field created by the infinite wire.
Okay, we need to be careful here. The field will be constant along the L dimension, as all points on such
a line is equidistant from the wire. However, the distance from the wire will matter greatly, of course. We
need to integrate the B-field over the rectangle. Calling the flux , an infinitetesimal amount of flux d
is given by d = BL dr, where dr is an infinitesimal distance in the direction of r. The total flux is given
by the integral over the area,
Z
Z h+w
0 I
=
d =
L dr
2r
S
h
We can move most things out of the integral, since the distance r is the only thing that isnt constant:


Z
0 IL h+w 1
0 IL
h+w
=
dr =
ln
2 h
r
2
h
255

3.85004 107 T m2
As a quick sanity check, what happens if we just multiply the B-field at P by the area of the rectangular
loop (i.e. treat the B-field as constant throughout the loop)? The result should be somewhere in the same
order of magnitude as the answer we just found. Doing that, we find 3.83 107 Tesla m2 , which is quite
reassuring! It looks like our integral is correct.
(c) Suppose the current in the infinite wire starts increasing in time according to I = bt, with b = 60
Amps/sec. What is the magnitude (in Amps) of the induced current in the loop? Neglect any contribution
to the magnetic flux through the loop due to the magnetic field created by the induced current.
The magnitude of the current will be given by
1 dB
R dt
which is really just Ohms law, using Faradays law to find the magnitude of the induced EMF (which
is equal to the rate of change of the magnetic flux through the loops surface). The magnetic field is
linearly proportional to the current in the wire, and the magnetic flux through the loop is likewise linearly
proportional to the B-field (and thus the current).
|I| =

Weve already found (or B ; I chose the shorter name earlier, but they are the same here):


h+w
0 IL
ln
B =
2
h
Which variables here will change with regards to time? Assuming the loop doesnt move (h is constant),
only I can change. Therefore, when we take the time derivative, we find


dB
dI 0 L
h+w
=
ln
dt
dt 2
h
dI
Weve given I = bt; b is constant, so the time derivative is
= b = 60 A/s. Therefore, the above
dt
dI
= b A/s is the magnitude of the induced EMF. We only need to divide that by the
expression with
dt
resistance of the loop to find the current:


0 L
h+w
Iind = b
ln
2R
h
Iind 6.4167 106 A
The direction of this current will be such that it opposes the increasing flux from the wires B-field. The
flux from the wires B-field increases into the page, so the our current must be induced to create flux out
of the page, which means counterclockwise according to the right-hand rule.

14.4.2

Problem 2: Non-conducting infinite sheet and infinite parallel slab

We have an infinite, non-conducting sheet of negligible thickness carrying a uniform surface charge density
+ = 2.00106 C/m2 and, next to it, an infinite parallel slab of thickness D = 10 cm with uniform volume
charge density = 1.60 105 C/m3 (see sketch). Note that the slab has a negative charge. All charges
are fixed in place and cannot move.

256

Assuming the infinite sheet is in the x-y plane, and z points as shown in the figure, calculate the components of the electric field at the locations listed below. Since we do not ask for the direction separately,
these components could be positive or negative.
Give all of your answers in Volts/m.
(a) a distance h = 3 cm above (i.e. in the +
z direction) the positively charged sheet.
Because the sheet is infinite in both x and y, those components must be zero for all questions, by symmetry.
Only the z components wont be cancelled out, so only those can be nonzero.
The first step, after figuring that part out, is to find the E-field due to the sheet, and due to the slab (and
also due to the slab, inside the slab).
We start with the sheet; I will do the full (but quick) derivation using Gausss law as practice.
We use a cylinder (pillbox) of height 2r, centered on the charged sheet, so that r of the cylinder is above
the sheet, and r below. The area of the cylinders end caps is A.
(Pretend that the slab isnt there, since we are calculating the E-field due to the sheet alone!)
Using the same symmetry argument as above, there can be no flux through the rest of the cylinder; only
through the end caps. We have Gausss law:
I
~ = Qencl
~ dA
E
0
The area in question is the entire surface area of the cylinder. However, as mentioned, there will be no
flux through the side, so only the end cap area is relevant. Because the E-field is constant over the area,
we get
Qencl
0
as the flux through the two end caps. The charge enclosed is given by Q = A, and is a given, so
E(2A) =

A
0

2E =
0

E(2A) =

~ = (
E
z ) (due to the sheet alone)
20
The direction, since the sheet is positively charged, will be outwards. That is, upwards (+
z ) above the
sheet, and downwards (-
z ) below the sheet.
Next up, we calculate the E-field due to and outside the slab.
Again, we ignore that the sheet is there, since we want to calculate the E-field due to the slab alone.

257

We use a Gaussian pillbox again, again with end-cap area A (per cap). We again center the pillbox,
and make it 2r high, such that 2r > D (the thickness). That is, the cylinder is higher than the slab and
encloses the entire thickness of the slab, so to speak.
Using the same arguments as before, we will end up here again:
E(2A) =

Qencl
0

The enclosed charge is now given by the volume V = AD, times the charge density (charge per unit
volume) :
AD
0
D
E=
20

E(2A) =

~ = D (
z ) (due the slab alone, outside the slab)
E
20
As the thickness decreases to where it no longer matters, this starts to look like the equation we have for
the thin sheet, which we would expect.
Finally, we do the calculation for the E-field inside the slab, due to the slab alone.
Once again we use the pillbox, but this time we choose it such that r is much smaller than D, so that
it is completely inside the slab. We center it in the slab; the E-field should be exactly zero at the exact
center, then grow linearly until the edge (since the charge density is constant) and then stay the same
until r , since the E-field due to an infinite plane/slab doesnt decay with distance.
Using Gausss law, we arrive at this step again:
E(2A) =

Qencl
0

The charge enclosed is now given only by the size (volume) of our pillbox, V = 2rA (2r being the height
of the pillbox):
E(2A) =

2rA
0

~ = r (
E
z)
0
So to summarize:
~ = (
E
z ) (due to the sheet alone)
20
~ = D (
E
z ) (due the slab alone, outside the slab)
20
~ = r (
E
z ) (due to the slab alone, inside the slab)
0
Lets not forget that the slab is negatively charged !
Next, lets gather together expressions for the E-fields above both, inside the slab, and below both.
Above both, the E-field due to the sheet will point upwards (it is positively charged), and the E-field due
to the slab point downwards (it is negatively charged).
The net E-field above is the superposition of the both, that is, the difference between the two, in this case:

258

~ = ( z) + ( D (
z ))
E
20
20
~ = ( z) ( D z)
E
20
20
Does this make intuitive sense? If wins, the field points upwards; if D wins, it points downwards.
Sounds good to me, so lets simplify it down:
~ = D z (above the sheet and slab)
E
20
Next up, we calculate the E-field below everything.
Below both, the sheets E-field will point downward, and the slabs upward.
D
~ = ( (
z )) + (
z)
E
20
20
~ = ( D z) ( z)
E
20
20
~ = D z (below the sheet and slab)
E
20
Again, we check to make sure that it makes sense. If the sheet wins, the net field is now negative (downwards), and it is positive (upwards) if the slab wins. This indeed makes sense, given their charge polarities.
We note also that this is the opposite/negative of the E-field above both, which also makes sense.
Almost there... We need to find the combined E-field inside the slab, as well. Well, lets see.
(This is really a bit of overengineering: we dont need this equation to solve it, we can just calculate it for
a particular location, since the answer should be a number, not an expression.)
There should be no sharp transitions when moving through the slab. At the very top, the E-field should
be the sum of that of the sheet and that outside (above) the slab. At the center, the slabs E-field is zeroed
out, and only the sheets should remain.
At the very bottom, we should have the same value as below both. And this with a linear transisition.
Lets have a look at this equation:
~ = r (
z ) (due to the slab alone, inside the slab)
E
0
r will go from +0.5D to 0.5D as we move from top to bottom. We need to express this as d, a distance
below the sheet (above the slab), which can then go from 0 at the top, to D at the bottom (and 0.5D in
the middle).
r = 0.5D d seems to fit nicely. Thus, we have
~ = (0.5D d) (
E
z ) (due to the slab alone, inside the slab)
0
We can now also take care of the direction properly. Since the slab is negatively charged, the direction
will be downwards above the center, and upwards below the center.
~ = (0.5D d) (
E
z ) (due to the slab alone, inside the slab)
0
We can finally calculate the net E-field inside the slab, by simply adding to this the E-field due to the
sheet, which is downwards here (below the sheet):

~ = (0.5D d) (
E
z) +
(
z)
0
20
259


~ = (0.5D d) (
z)
z (inside the slab, total)
E
0
20
Again, a sanity check: for d = 0, the E-field due to the slab is downwards, at it should be, as is the E-field
due to the sheet. For d = D, at the bottom of the two, the slabs field turns positive:
D
(
z)
20
... which is the same equation that we found earlier. So we finally have the full set of equations:
E=

~ = D z (above the sheet and slab)


E
20

~ = (0.5D d) (
E
z)
z (inside the slab)
0
20
~ = D z (below the sheet and slab)
E
20
We can now plug in our values, and enter the answers as the z components, keeping all other components
zero. Keep the signs in mind!
(a) a distance h = 3 cm above (i.e. in the +z direction) the positively charged sheet.
The distance doesnt matter, but we plug in the values and find
Ez = 22588.2 V/m
> D so this makes sense. Also, with = D the answer is zero, which again makes sense.
(b) inside the slab at a distance d = 1 cm below (i.e. in the
z direction) the positively charged sheet.
Note that d < D so that this is also a distance 9 cm above the bottom of the slab.
We plug it in again, and find
Ez = 185223 V/m
Both E-fields should be downward at this location, so a strong downwards value makes sense.
(c) a distance H = 22 cm below (i.e. in the
z direction) the charged sheet. Note that H > D so that
this is also a distance 12 cm below the bottom of the slab.
Again, the distance doesnt matter, but we plug the values in:
Ez = 22588.2 V/m
The answer is the negative of the one we found above the sheet, as it should be.

14.4.3

Problem 3: RLC circuit

A circuit is composed of a capacitor C = 2F, two resistors both with resistance R = 67 Ohm, an inductor
L = 0.03 H, a switch S, and a battery V = 5 V. The internal resistance of the battery can be ignored.

260

Initially, the switch S is open as in the figure above and there is no charge on the capacitor C and no
current flowing through the inductor L. At t = 0 we close the switch.
Define the current through the inductor to be positive if it flows through the inductor and then through
the resistor and therefore down in the drawing. Similarly, define the current through the capacitor to be
positive if it flows down in the drawing.
What is the current through the inductor (in Amps) at t = 0 (i.e. just after the switch is closed)?
There are 6 questions, the above is the first; they want the current through both inductor and capacitor,
at t = 0, two times in between, and at t , for a total of 6 questions.
Now, the ones for t = 0 and t are easy, but the others might just be quite hard, instead.
L
However, I noticed that for the inductor, they ask for t = , and for the capacitor, for t = RC.
R
If we can treat the two as separate circuits, this would be almost trivial. Because they are in parallel with
the battery, which decides the voltage/EMF across L + R as V , and the voltage across C + R as V , I
believe that we indeed can treat them as separate RC and RL circuits.
Lets do the calculations as if they were separate. Since they are in parallel, the voltage across L + R
needs to be the same as the one across the C + R, which has a few possible implications. However, they
are also in parallel with the battery, which will always decide the voltage even if separate. With that in
mind, lets do the simple calculations for series RL and series RC circuits:
First, for the RC part:


V
t
I(t) = exp
R
RC
Plugging in the values, we find
5
0.074627 A
67
I(t = 1.34 104 ) = 0.0274537 A
I(t = 0) =

I(t ) = 0 A
The values make sense. The second answer should be about the first times 1/e (as it has decayed over
exactly one time constant), and it is.

261

For the RL part:


V
I(t) =
R




tL
1 exp
I(t = 0) = 0 A
R

I(t = 4.48 104 ) = 0.0471878 A


5
A
67
In this case, the second answer should be (1 1/e) times the final one, and it is.
I(t ) =

If these were two separate circuits, I would be very confident in these answers. As is, however, Im only
fairly confident in them.
However, considering that the 12th question on the exam, which is slightly similar, is ungraded in part
due to its difficulty (it was included as a special challenge), I would assume that this one is easier, and
so I submitted these answers, and they were marked as correct.

14.4.4

Problem 4: Plane electromagnetic wave

The electric and magnetic fields of a plane electromagnetic wave are given by the formulas below, where
z and are both in meters and t is in seconds. This wave is traveling through a medium whose index of
refraction is 1.2.
2
~ = 8.6
E
x sin( z + 5.91 1016 t) V/m

~ = B~0 sin( 2 z + 5.91 1016 t) Tesla


B

(a) In what direction does the wave propagate?


The direction of propagation is given by the sign inside the sin function. In this case, it is
z due to the
+z inside the sin.
(b) What is the magnitude and direction of B~0 (assuming B0 is positive)?
E0 = B0 v, where v is the propagation velocity. We can find v using the refractive index, as the definition
of n (here 1.2) is that
c
v
1.2v = c
c
v=
1.2
Therefore, since E0 = 8.6 V/m, and since we divide by v we flip the fraction upside down:
1.2 =

1.2
3.44 108 T
c
= v, that is, x B
=
The direction must be such that E B
z.
=
Via the right-hand rule, that means that B
y.
B0 = 8.6

(c) What is the wavelength of the wave (in meters)?

262

v
One way of finding the wavelength is by using f = v. This means that = .
f

f is the frequency, which can be found from using f =


. All in all, we have
2
2v
2.65786 108 m
=
f

14.4.5

Problem 5: Loop in a magnetic field

A rigid rectangular wire loop (above) of width W = 36.1 cm and length ` = 99 cm falls out of a region
with a magnetic field. The dashed line in the figure above is at z = 0 and z is positive downward as shown.
~ = 0.940 T out of the page for z < 0 (above the z = 0
The magnetic field is constant with magnitude |B|
line) and zero for z > 0 (below the z = 0 line). The loop has mass M = 0.810 kg and total resistance
R = 1.4 . You may ignore any self-magnetic field generated by the loop itself.
At t = 0, the rigid wire loop is at rest, the current I in the loop is 0, and the bottom of the loop is at
d
z = 0. At later times t, the loop carries current I(t) and is moving with a speed V (t) = z(t) downward,
dz
where z(t) is the distance to the bottom edge of the loop as shown on the figure. Use g = 9.81 m/s2 for
the acceleration due to gravity. (Note that downward acceleration is in the +
z direction).
In all of these questions, assume that z(t) < ` so that part of the loop is still in the region with magnetic
field.
(a) What is the total magnetic flux through the loop (in Tesla m2 ) when z(t) is 83 cm? Include only
~ (i.e. ignore the flux associated with the magnetic
the magnetic flux associated with the external field B
self-field generated by the current in the wire loop). Note that you do not need to calculate or know at
what time the loop is at this location.
Okay, lets see... We have the gravitational force pulling the loop downwards at all times.
In addition, there will be a magnetic force on the loop, due to the induced current.
The magnetic flux through the loop will constantly decrease, since it starts out 100% inside the magnetic
field, and ends up outside it.
The field is out of the page, so the induced current will be counterclockwise to attempt to keep the flux
through the loop constant.
Lets then look at the magnetic force on the three sides of the loop that are still in the magnetic field.
~ B,
~ where L is the directional vector along
The magnetic force on a current-carrying wire is given by I L
~ the external B-field.
which the current I flows, and B
For the top segment, the force is upwards. For the left side, it is to the left, and for the right side, to the
right. Those two cancel out by symmetry, so the magnetic force is
263

F~B = IW B(
z ) N (upwards)
There is then the downwards force by gravity. Since F~ = m~a and ~a = 9.81
z m/s,
F~g = 9.81 m/s2 0.81 kg = 7.95
zN
We know W and B, but need to calculate I in order to be able to find a numeric value for the upwards force.
First, lets calculate the (external) flux through the surface of the loop, which will also answer the first
question.
The magnetic flux B is given by the area inside the B-field, times the B-field strength:
B = BW (` z(t))
For z(t) = 83 cm, this is
B = (0.940 T)(36.1 cm)(99 cm 83 cm) = 0.0542944 T m2
(b) Using Faradays Law and Ohms Law, find the magnitude (in Amps) of the induced current I(t) in
the bar at the time when V = 9.00 m/sec. Note that you do not need to calculate or know at what time
the loop has this speed.
We need to calculate the time derivative of the flux, which via Faradays Law gives us the induced EMF.
We then divide that by the loop resistance, which will give us the magnituded of the current.
B = BW (` z(t))
B = BW ` BW z(t)
dB
= BW V (t)
dt
The negative of this is the induced EMF. Were asked for the magnitude, so we dont really care about
minus signs, but still. We divide this by the loop resistance to find the current:
BW V (t)
2.18147 A
R
... where the approximation is for V (t) = 9 m/s, as they asked for in part (b).
I(t) =

(c) Which way does the current flow around the loop, clockwise or counterclockwise?
Counterclockwise; we answered that long ago.
(d) What is the total magnetic force (in Newtons) on the rigid wire loop when V=9.00 m/sec? Again,
ignore any effects due to the self magnetic field.
Now that we know the current, we can calculate this.
|F~B | = IW B = (2.18147 A)(36.1/100 m)(0.940 T) = 0.74026 N
The direction is upwards, as stated earlier.
(e) What is the magnitude of the terminal speed (in m/sec) of the loop (i.e. the speed at which the loop
will be moving when it no longer accelerates)?

264

It stops accelerating when the net force is zero. That happens when the magnetic force upwards equals
the gravitational force downwards, in other words when
I(t)W B = 7.95
BW V (t)
W B = 7.95
R
B 2 W 2 V (t)
= 7.95
R
7.95R
V (t) = 2 2
B W
Plugging in the values on the right-hand side, we find that V (t) = 96.6552 m/s.
That looks far from a possible value, since the net acceleration must be at most 9.81 m/s2 (due to gravity)!
That is, it must have been accelerating for about 10 seconds or so, at which point it would clearly have
fallen out of the B-field long ago!
Still, I checked using dimensional analysis, and the result seems both sensibly calculated and in the correct
units, so I decided to use one of my three attempts, and I got green checkmarks for everything, including
the last part. Phew!
(I still believe the result to be impossible in practice, but there was a similar question on the previous
exam, where the E-field in a capacitor was way higher than possible in air without breakdown, due to the
random values assigned to each student.)

14.4.6

Problem 6: Charged particles in a magnetic field

Two charged particles initially are traveling in the positive x direction, each with the same speed v, and
enter a non-zero magnetic field at the origin O. The magnetic field for x > 0 is constant, uniform and
points out of the page. In the magnetic field, their trajectories both curve in the same direction, but
describe semi-circles with different radii. The radius R2 of the semi-circle traced out by particle 2 is bigger
R2
than the radius R1 of the semi-circle traced out by particle 1, and
= 4.25. Note that the drawing is
R1
not to scale, so the ratio of radii may not be represented accurately. Assume that the particles are sent
through at different times so that they do not interact with each other.

265

(a) Assume that the two particles have the same mass m, but have different charges, q1 and q2 . What is
q2
the ratio ?00
q1
~ would have sent them in
The two particles must be positively charged, or the magnetic force of q~v B
the other direction. That wasnt asked for, however!
The radius of a charged particles trajectory in a constant magnetic field is given by
R=

mv
qB

R2
= 4.25, and m, v and B are all the same for both particles, we can substitute in
R1
the values for R. We get a 4-level equation, which I write as two levels instead:

Since we know that

R2
mv mv
=
/
R1
q2 B q1 B
R2
1 1
= /
R1
q2 q1
R2
q1
=
R1
q2
q1
4.25 =
q2
q2
1
=
q1
4.25
Thus, q2 is smaller than q1 . That makes sense, because if it were larger, it would trace out a smaller radius
than R1 , due to the then larger magnetic force on it.
(b) How much energy did the particle with charge q2 gain traversing the region with magnetic field?
Well, thats an easy one. None whatsoever - the magnetic force cant change its energy, so the answer is
zero.
(c) Assume that instead of different charges as shown in the figure, the two particles have the same charge
m2
q, but have different mass m1 and m2 . What is the ratio
?
m1
R2
m2 v m1 v
=
/
R1
qB qB
R2
m2
=
R1
m1
m2
= 4.25
m1

14.4.7

Problem 7: Diffraction pattern

Light from a coherent monochromatic laser of wavelength = 4.30 105 m is incident on two slits, both
of width a, separated by a distance d. The slits are placed a distance L away from a screen where L  d
and L  a. The diffraction/interference intensity is plotted on the figure below as a function of sin()
(i.e. the X axis is sin() where varies from 90 to +90 ).

266

Based on the plot of intensity versus sin() in the figure above,


(a) what is a in m?
Lets see... This combined diffraction+interference pattern might be the weakest point of my knowledge
of the entire 8.02x material.
I recall that the interference pattern is the one with many small peaks, while the diffraction pattern is
the envelope. That is, the diffraction pattern (not really drawn as-is) has one large peak, from about
0.5 < sin < 0.5, with one secondary peak on each side of that.
For single-slit diffraction, we have the first minima at
sin =

(destructive interference, single-slit diffraction)


a
267


The first maximum is at the center, sin = = 0, and has a width of sin() , which is the first
a
minima.
For double-slit interference, we have one maximum at the center, as always, and other maxima at
n
(constructive interference, double-slit interference)
d
(2n + 1)
sin n =
(destructive interference, double-slit interference)
2d
So the diffraction causes the envelope, which has a width of (from 0 to the first minima) which looks
to be about sin = 0.43 or something like that. So we have
sin n =

a
4.30 105

=
= 104 m
a=
0.43
0.43
... which is incorrect! (The value for d below was correct, however.)
I tried again with sin 0.5, and got it correct! Thats a bit disappointing in a way, though: my method
was correct, but the estimation was off. Since its not marked explicitly on the graph, Im sure several
students will get this one wrong twice (there are two tries for this question) based on estimation errors alone.
0.43 =

As for the interference part of it all, we have the first-order maximum at approximately sin = 0.12 (I
hope the grader allows for a wide margin of error!), so we have
0.12 =

4.30 105

=
= 3.5833 104 m
0.12
0.12
I had another quick look in the book (since this is an open book exam; I only use it to check smaller stuff,
though), to find the horrible equation for the intensity plot. It is

2
sin(a sin /)
2
I = I0 cos (d sin /)
a sin /)
d=

I plotted I/I0 using this function, to see whether the plot was anything like this, and it looked fairly
similar... However I had as the x axis, while the book had sin , so an exact comparison wasnt possible.
Im not sure how to plot with sin as the x axis in Mathematica, so I skipped that.

14.4.8

Problem 8: Capacitor washers

A capacitor consists of two identical metal washers of inner radius a = 1.4 mm and outer radius b = 5.1
mm, separated by a distance d = 0.19 mm (note that b  d). The capacitor is initially uncharged at t = 0
and we then begin to charge it using wires outside the plates that are not shown in the figure. Suppose
at time t the charge on the lower washer is Q(t) = st with s = 19C/sec and that this charge is spread
uniformly over the area of the bottom washer.
The top washer has an identical, but negative, charge Q(t), also distributed uniformly. Throughout
this problem you may assume that the electric field is non-zero only inside the capacitor over the range
a < r < b, where r is the cylindrical radial distance.
You may also assume that, in the regions where the electric field is non-zero, it points exactly perpendicular
to the surface of the washers (i.e. upward as shown) and has the same magnitude at all locations (but the
magnitude varies with time).

268

(There is a note that there was an error in this problem up until 14:10 GMT, and that people who tried
to solve it prior to that, but failed, now need to use the new random values shown. This didnt apply to
me, as I started on this problem at 17:20 GMT.)
(a) What is the magnitude (in Volts/m) of the electric field for a < r < b at time t = 5sec?
Interesting! I wonder how accurate the solutions will be given all the strictly incorrect assumptions of a
constant/non-fringing E-field, etc. Still, without those assumptions, I wouldnt have a clue how to solve
this - probably with a computer EM simulator. I digress; lets look at the problem.
Given that the E-field is constant between the plates, and the plates have no mentioned thickness, its
clear that we are to use the result for the E-field due to a changed infinite plane here (that we derived
earlier), only that we use two of them, just as inside a regular circular or rectangular capacitor.
That is,

0
We need to find , though. First, lets find it in terms of Q, after which we will find it as a function of
time.
Q
= , as usual. What is a? The area of a ring is given by the area of a circle with radius b, minus the
A
area of a circle with radius a (the hole in the middle):
E=

A = b2 a2
So therefore,
=

b2

Q
a2

and
E=

Q
0 (b2 a2 )

As soon as we find Q(t) at t = 5s, we can calculate the answer to (a).


Q(t) = st = 19

C
t
sec

Q = 9.5 1011 C at t = 5s. We plug that in, and find


E 142007 V/m
That sounds high, but it turns out to be less than 30 volts, given the small separation between the two
washers.
269

Oh, that was the next question!


(b) What is the magnitude (in Volts) of the electrostatic potential difference between the washers at the
same time t = 5sec?

V = Ed
V = (142007 V/m)(0.19 103 m) 26.981 V
(c) What is the capacitance (in Farads) of this arrangement of conductors?
By definition,
C=

Q
V

As a symbolic answer,
C=

st
d s t
0 (b2 a2 )

C=

s t0 (b2 a2 )
dst

0 (b2 a2 )
d

Very nice. Another definition of capacitance is


C=

A0
d

... which is exactly what we found above!


We plug in the numbers, and find
C 3.52095 1012 F
Very tiny, as expected with almost no area, and no dielectric.
(d) What is the magnitude of the magnetic field (in Tesla) inside the capacitor at a radius r = 3.4 mm
and at time t = 5s?
Ah, thats a fairly painful calculation for a mere sub-question!
I could look back in my notes to find an equation for the B-field, but I will re-derive it as I have done with
most things so far.
Well need to use the Ampere-Maxwell law:


I
dE
~
~
B d` = 0 Ipen + 0
dt
We choose our Amperean loop to be a circle, concentric with the washers, of course. We center it exactly
between the plates (though that should not matter), and attach a flat surface to it.
Zero real current will penetrate that surface, since it is mid-air. There will however be a displacement
current, that is, a change in electric flux, as the capacitor is being charged.
The left-hand side of the equation should be easy, since we will assume the B-field to be constant along
the circle. We simplify the equation to
B(2r) = 0 0
270

dE
dt

0 0 dE
2r dt
Next up, we calculate the electric flux E . Lets not forget that there is a big hole in the center of the
capacitor, unlike previous times weve done a similar calculation!
B=

E = EA =

st
A
0 (b2 a2 )

What is A here, then? Again, lets be careful. We have a ring; the outer radius is r, the radius of the
Amperean loop, which is smaller than b (the outer radius of the capacitor) and larger than a (the inner
radius).
The inner radius is a. Thus A = r2 a2 , and
st
(r2 a2 )
0 (b2 a2 )
st
(r2 a2 )
E =
0 (b2 a2 )

E =

E =

st(r2 a2 )
0 (b2 a2 )

Next, we take the time derivative. The charge is Q = st, so the time derivative of that is I = s. s is given
to us in microcoulombs per second, which is the same as microamperes.
Alternatively, we can just take the derivative with respect to t of the above expression, and we get the
same result. That is,
s(r2 a2 )
dE
=
dt
0 (b2 a2 )
We can then calculate the B-field, since this term was the only thing missing:
B=

0 0 s(r2 a2 )
2r 0 (b2 a2 )

B=

0 s r 2 a2
2r b2 a2

B 4.46129 1010 (for r = 3.4 mm)


Time for a sanity check! If we use this method to calculate the B-field for r = b, and also use
0 I
(B-field for an infinite wire)
2r
... the results should be identical. We dont even need to put numbers in there, because when you set
r = b, it simplifies down to that instantly! Awesome.
B=

(e) What is the magnitude of the Poynting vector (in Watts/m2 ) at radius r = b and at time t = 5s?
~ is given by
The Poynting vector S
~
~
~ = EB
S
0
Making the substitutions (also, keep in mind that I = s),



1
Q
0 I
~
S=
0 0 (b2 a2 )
2r
271

~=
S

s2 t
20 2 r(b2 a2 )

~=
S

s2 t
20 2 b(b2 a2 )

But r = b (for this sub-question):

We can plug the numbers in to find


~ 84.2003 W/m2
|S|
Another sanity check: we have calculated values for E (constant) and B (at a different location), so if we
multiply those together and divide by 0 , we should find something not way far off. Doing that, we get
142007 4.46 1010
50.4 W/m2 . A bit off, but the B-field is greater further out, and so the Poynting
4107
vector will be greater there as well.
As for the direction, radially inward makes a lot of sense: energy is entering the capacitor.
~ B
~ term, which turns out
We can show that mathematically, however. The direction is given by the E
to be ~z
~ = ~r.
(e) What is the magnitude of the total energy per second (in Watts) flowing into or out of the outer side
of the capacitor (i.e. at a radius r = b) at time t = 5s?
We need to find the surface area (of an imaginary surface) of the capacitors outside. That is, the side
of a cylinder with radius b and height d.
A = 2bd
We multiply that by the Poynting vector, and find
s2 d t
dU
=
W (J/s)
dt
0 (b2 a2 )
As a number, we find
dU
5.12646 104 W (J/s)
dt

14.4.9

Problem 9: Circuit

Two identical parallel plate capacitors filled with air, with capacitances C1 = 4F and C2 = 5F, are
connected to a battery of voltage V = 10 V (see the circuit below). The capacitor plates are separated by
a distance d1 = 5 mm for C1 and d2 = 7 mm for C2 . Note that the two plate separations may be different
although shown as the same size in the drawing.

(a) What is the electric field (magnitude and direction) between the plates of each capacitor?

272

So they are identical, but differ in capacitance and plate separation? Hmm... Strange wording.
In fact, if we solve for the area, they differ in area too! How the heck are they identical, exactly?!
Ah well, I will assume that this is simply an oversight, and that the word identical should really be
removed...
Well then. We know that
1
0
2
E2 =
0
... but we do not know the charge density, nor do we know the charge.
We can also find it as
E1 =

V
d1
V
E2 =
d2
V is the same for both capacitors, as they are connected in parallel. Since there is a battery connected,
which decides V , we can just plug the numbers in at this point.
E1 =

E1 = 2000 V/m
E2 1428.57 V/m
Both E-fields are pointed downward, since the positive plate is on top.
We fill the entire air gap of C1 , the capacitor on the left, with a dielectric having = 4.0 while keeping
the battery connected.
(b) What now is the electric field (direction and magnitude) between the plates of each capacitor?
We must not forget (or miss) that the battery is still connected! The potential difference will still be V
across each capacitor; that simply cannot change!
The E-field inside is constant at all places inside, with or without a dielectric. Some equations that always
apply are
Q
V
Q
V =
C
V = Ed
0 A
C=

d
Because d is fixed and V is fixed, E must be fixed at
C=

E=

V
d

for both capacitors. That is


E1 =

V
= 2000 V/m
d1
273

V
1428.57 V/m
d2
... exactly as before. This is because the battery is connected, which forces V to stay unchanged.
E2 =

(c) Suppose that we disconnect the battery before filling the entire air gap of C1 , the capacitor on
the left, with a dielectric having = 4.0. When disconnecting the battery, we leave the two capacitors
connected to each other as shown. After the dielectric is put in (and the battery remains disconnected), what would the electric field be between the plates of each capacitor (direction and magnitude)?
What happens now is that V can change (but must still be the same for both capacitors), while Q on the capacitors can vary. The total change must still equal the total change that was there to begin with, however.
We know that
Q = CV
So
Q1 = C1 V = 4 105 C
Q2 = C2 V = 5 105 C
... before the dielectric is inserted! Thus, Qinitial = 9 105 C; this is the total charge on both capacitors,
which must be held constant once the battery is removed.
A new set of equations that must hold true now:
Q1 + Q2 = Qinitial
Q1
= C1
V
Q2
C2 =
V
= C1 , where C1 is the old, original capacitance.
C1new =

Because of the inserted dielectric, C1new

We can combine the above equations, and solve for V :


V (C1new + C2 ) = Qinitial
Qinitial
C1new + C2
We now have that in terms of variables we know. Finally, the electric field is given by
V =

E=

V
d

so that
E=
E=

Qinitial
)
d(C1new + C2
Qinitial
)
d(C1 + C2

Plugging in the numbers, keeping in mind that d varies between the capacitors:
E1 =

6000
857.143 V/m
7
274

30000
612.245 V/m
49
The potential difference between the plates is V 4.286 V after equilibrium has been reached.
E2 =

14.4.10

Problem 10: An LC circuit

(a) In the LC circuit shown in the figure, the current is in the direction shown and the charges on the
capacitor have the signs shown. At this time,

a) the current is increasing and the charge on the positive plate is increasing.
b) the current is increasing and the charge on the positive plate is decreasing.
c) the current is decreasing and the charge on the positive plate is increasing.
d) the current is decreasing and the charge on the positive plate is decreasing.
e) the current and the charge on the positive plate are constant in time.
f) none of the above
Well, since Im taking the time this exam (since these are probably the last physics problems Ill solve in a
few months) to re-derive everything, lets try to derive, with some help from Mathematica, the equations
governing the LC circuit.
Starting at the upper node, we go left (along the current) and apply Faradays law. First, the left-hand
side:
I
~ = dB
~ d`
E
dt
Q
dB
=
C
dt
Q
is positive, because we moved along the direction of the electric field. The inductor doesnt have an
C
electric field (we treat it as ideal), and only contributes to the right-hand side. Given that, by definition
B = LI
d(LI)
Q
=
C
dt
L is a constant, so we move it out:

275

Q
dI
= L
C
dt
We can write it all in terms of Q, by making the substitution I =

dQ
:
dt

Q
d2 Q
= L 2
C
dt
Thats about all we can do. We could rewrite it to have all terms on the left side, but since Im not going
to solve this differential equation manually, it hardly matters.
I solved it using the boundary conditions Q(0) = Q0 and Q0 (0) = 0, that is: initial charge Q0 on the
capacitor, and zero current at t = 0. The current through a capacitor is maximum while it is being
charged, but zero when the charge (and voltage across it) is at the maximum.
The solution is


t
Q(t) = Q0 cos
LC
Q
Q
Based on the definition C = , V = :
V
C


Q0
t
VC (t) =
cos
C
LC
Finally, the current is given by the time derivative of the charge:


t
Q0
I(t) =
sin
LC
LC
Using
Q0
I0 =
LC


t
I(t) = I0 sin
LC
Lets think about signs for a second. I recall getting this backwards previously, and I had a quick look in
dQ
the book (this is an open-book exam after all); indeed, they use I =
here. Lets see if that makes
dt
more sense.
In our equations, when Q0 has just reached its maximum and starts decreasing, that means that, using
the signs in the figure above, the current is clockwise, which is indeed negative as shown. Ill stick with
the convention I just found, as that seems less confusing than ignoring the arrows as drawn.
OK, so with these equations in mind, is the first question now easier to answer?
What do we know from the picture? The current is flowing into the positive plate (counterclockwise).
This must mean that the charge is increasing, since that would deposit positive charge on the top plate.
That leaves options (a), (c) and (f), none of the above.
Is the current increasing or decreasing? Weve established that the charge must be increasing (on the
positive plate). If we plot the two (just plot y = cos(x) for the charge and y = sin(x) for the current)
we see that when the charge is increasing (that is, is above y = 0 and has positive slope), in such a plot
between x = and x = 3
, the current is decreasing.
2
That also makes intuitive sense, if we know how these circuits work. As the charge is zero and starts
increasing, the current is at a maximum. The charge keeps increasing, and the current decreasing, until
the charge reaches a maximum.
276

Thus, the answer is (c).


For the following questions, assume that the inductance is 0.05 H, the capacitance is 4 F, and the maximum magnitude of the current in the circuit is 90 milliAmps.
(b) How many times per second will the magnitude of the current have this maximum value?
Lets see. First, they ask about the magnitude of the current, so this maximum happens more than once
per cycle!


t
I(t) sin
LC
The term multiplying the sine doesnt charge the period in any way, so we can ignore it for now. All we
want to know is how often this function is either +1 or 1, which is when the currents magnitude is at a
maximum.
This is twice per period, so we can calculate the frequency, and just double that.
The angular oscillation frequency is
0 =

1
LC

We can convert this to a regular frequency in hertz using f =

:
2

1
=
2
2 LC
We wanted twice this value though, so the answer is
f=

1
# maxima per second = 2f =
711.763 Hz
LC
(c) The energy stored in the capacitor oscillates up and down as a function of time. What is the magnitude of the difference (in Joules) between the largest and smallest amounts of energy ever stored in the
capacitor?
I dont like how that is phrased... It oscillates between 0 and a maximum, so I assume the answer is the
same as the maximum energy stored, then?
In an LC circuit, all energy sloshes back and forth between the inductor and capacitor. This means that
we dont have to calculate the capacitors energy, but we can instead calculate the inductors and use that
as the answer.
Doing so is easier, because they gave us I0 = 90 mA.
1
Umax = LI0 2 2.025 104 J
2
Lets verify that, though - this is the final exam!
1 Q2
.
We can calculate the maximum stored energy in the capacitor using U =
2C
The maximum charge can be found by using the given value of I0 .
Q0
I0 =
LC

Q0 = I0 LC 4.02492 105 C
Therefore,

277

Umax =

1 (4.02492 105 )2
2.025 104 J
2
4 106

Looks good!
(d) What is the time averaged power (in Watts) flowing into or out of the self inductor over 3 full cycles?
Define power into the inductor as positive.
In one cycle, a certain amount of energy will both enter and leave, which nets 0 energy per cycle. Thus
the time-average power over any amount of full cycles is zero.

14.4.11

Problem 11: Dielectric sphere

Consider a dielectric spherical shell with inner radius 1 cm and outer radius R = 15 cm. The spherical
shell is filled with dielectric with electric permittivity = 4.0 and there is a point charge q = 2.3 C
exactly at its center, which we consider the origin of our reference frame.
Calculate the direction and the magnitude of the electric field for the following points:
(a) What is the direction [and magnitude] of the electric field at the point whose distance from the origin
is 11 cm?
To clarify, there is a (very thick) shell (almost sphere, but there is a spherical void in the center) made
of a dielectric with the given value of , and a small spherical hole in the center, with a radius of only 1 cm.
First, using Coulombs law, the E-field due to the point change, in a vacuum, is
q
r (in vacuum)
40 r2
... where r is the unit vector pointing radially outwards from the charge. The above equation is only valid
inside a vacuum, however. At the point where they ask for the E-field, we are inside the dielectric.
A dielectric lowers the E-field below that of the vacuum field, by a factor . That is, inside the dielectric:
~ =
E

~ =
E

q
r (in the dielectric)
40 r2

Plugging in the numbers, we find


~ 427094 r V/m
E
(b) What is the direction [and magnitude] of the electric field at the point whose distance from the origin
is 15 cm?
Again, plugging in the numbers, we find E 229682 V/m... IF this is still inside the dielectric, otherwise
times that! Ugh.
I tried submitting that answer, and it came back wrong. I tried again with times the value (E 918728
V/m), and that was correct, so the intention was that 15 cm is exactly outside the dielectric, not inside
exactly on the edge.
(c) What is the total net charge (in Coulombs) contained in the dielectric (i.e. not counting the point
charge) inside a radius r of 11 cm?
This is the only question I missed on this exam, so this answer is written after looking at the staffs
answers.
My first try was zero, because charge cant flow freely inside the dielectric, but that was incorrect.
278

I then realized that induction occurs to create a negatively charged layer on the inside surface, and a
positive charged layer on the outside of the dielectric, so my second try was the negative of the charge at
the center, which was also incorrect, and left me with a wrong answer. Doh!
The correct answer is that the dielectric constant matters, such that


1
Qinduced = Qcenter 1

Since the centered charge is positive, the induced charge will, as mentioned above, be negative. In other
words,
Qinduced 1.725 106 C
(d) What is the total net charge (in Coulombs) contained in the dielectric (i.e. not counting the point
charge) inside a radius r of 15 cm?
This one is zero; the induced charge cancels out in the dielectric (charge cant come from nowhere; induced
negative charge means induced positive charge elsewhere).

14.4.12

Problem 12: Parallel RLC

SPECIAL CHALLENGE: Note that this problem is NOT worth any points! This problem is included as
a special challenge to allow students to test their detailed understanding on Faradays Law as applied to
circuits. You will be able to see whether your answers are correct but this problem is not worth any points.
Consider the following RLC circuit.

At t = 0 there is a charge Q0 on the capacitor.


(a) Faradays Law for the loop on the left side in the figure above can be written as
a1 Q + b1 I1 + c1 I2 = d1

dI1
dI2
+ e1
dt
dt

with a1 negative, and for the loop on the right side


a2 Q + b2 I1 + c2 I2 = d2
with b2 negative, where Q is the charge on the capacitor.

279

dI1
dI2
+ e2
dt
dt

Express all the coefficients above (a1 , b1 , c1 , d1 , e1 , a2 , b2 , c2 , d2 , e2 in terms of R, L, C and natural


constants.
This looks scary, but many of the coefficients will be zero, from the looks of it.
Still, scored question or not, having solved the rest of the exam (expect for 11c which I missed), I will of
course give this a try.
To get a1 negative, we go in the clockwise direction, as the arrow suggests. We find

Q
+ I1 R I2 R = 0
C

That is,
1
C
b1 = R

a1 =

c1 = R
d1 = 0
e1 = 0
For the right loop, we need to use Faradays law, since there is an inductor present. We also want to get
b2 negative, so we again go clockwise for that to happen. Starting at the bottom:
dB
dt
The last term, the change in magnetic flux, is given by the definition of inductance, and the current I2
through the inductor:
I1 R + I2 R =

I1 R + I2 R = L

dI2
dt

To get this result in term of the coefficients, we need


a2 = 0
b2 = R
c2 = R
d2 = 0
e2 = L
(b) What is

dQ
equal to? Your answer can include only one or both of I1 and I2 and numbers.
dt

The change in charge of the capacitor is equal to the current through it - whether it is positive or negative
is the only thing we really need to consider.
dQ
Say
> 0, so that the charge is increasing. The way I would see this is that the current is counterclockdt
wise, so that charge increases on the top plate, here marked as positive.
Therefore, the answer should be I1 (it is opposite of the marked current).
(c) The charge on the capacitor obeys the following differential equation:
d2 Q
dQ
+
a
+ bQ = 0
dt2
dt
280

Express a and b in terms of, if necessary, R, L and C.


Lets try to apply KCL for the top node, summing the current leaving (going downwards). Because the
voltage across all elements must be the same (they are in parallel), I call the voltage across each element
VC (t), i.e. the voltage across the capacitor.
Z
dVC (t) VC (t) 1 t
C
+
+
VC (t)dt = 0
dt
R
L 0
(assuming IL (0) = 0, or the integral needs to be from )
Q(t)
Because VC (t) =
, we can rewrite this:
C


Z
d Q(t)
Q(t) 1 t Q(t)
C
+
+
dt = 0
dt
C
RC
L 0 C
Z t
Q(t)
1
0
Q (t) +
+
Q(t)dt = 0
RC
LC 0
The units of each term works out to be amperes, so it looks like were still on the right track. However,
we dont want that integral, so we take the time derivative of both sides of the equation.
Q0 (t) Q(t)
+
=0
RC
LC
Awesome, we now have the equation in the form we need! We can rewrite the derivatives using Leibniz
notation to make it even more obvious what a and b are:
Q00 (t) +

d2 Q
1 dQ
1
+
+
Q(t) = 0
2
dt
RC dt
LC
So
1
RC
1
b=
LC
(d) Now consider an RLC circuit built connecting in series a resistor R0 , a capacitor C and an inductor
L; and a RLC circuit built connecting in parallel a resistor R, a capacitor C and an inductor L. Find the
expression of the resistance R0 in terms of R, C, L such that the charge on the capacitor obeys the same
equation derived in (c).
a=

Okay, so first off, we need to derive the differential equation for the charge on a capacitor in a series RLC
circuit.
I drew one on paper, in a simple rectangle with nothing on the left side, the resistor on top, capacitor on
the right, and inductor on the bottom - not that it matters, but knowing makes it easier to follow along.
I apply Faradays law, starting at the very left, going clockwise:
I
~ = dB
~ d`
E
dt
dI
Q(t)
+ 0 = L
C
dt
0
We can rewrite I(t) as Q (t), which also goes for the right-hand side:
I(t)R0 +

R0

dQ
1
d2 Q(t)
+ Q(t) = L
dt
C
dt2

281

dQ
1
d2 Q(t)
+ Q(t) + L
=0
dt
C
dt2
Divide by L throughout and sort, higher-order derivatives first:
R0

d2 Q(t) R0 dQ
1
+
Q(t) = 0
+
2
dt
L dt
LC
So the two equations are, for the parallel and series RLC circuits, respectively:
d2 Q(t)
1
1 dQ
+
Q(t) = 0
+
2
dt
RC dt
LC
d2 Q(t) R0 dQ
1
+
Q(t) = 0
+
2
dt
L dt
LC
1
What we need to do is choose R0 such that we get
multiplying the first derivative. Looks easy enough:
RC
L
R0 =
RC
... which concludes this problem, this exam, and this entire course!
I hope you enjoyed the course as much as I did, and thanks for reading!

282

You might also like